2000 feladat az elemi matematika köréből
 9789634931478 [PDF]

  • Commentary
  • decrypted from 502AE57A57225D66AEEAC4404C312A08 source file
  • 0 0 0
  • Gefällt Ihnen dieses papier und der download? Sie können Ihre eigene PDF-Datei in wenigen Minuten kostenlos online veröffentlichen! Anmelden
Datei wird geladen, bitte warten...
Zitiervorschau

2000 FELADAT AZ ELEMI MATEMATIKA KÖRÉBİL

Name: Order:

Jozsef Kolompar 26985340

Köszönjük a vásárlást a szerző és a kiadó, valamint a terjesztő nevében is. Vásárlásával támogatta, hogy Magyarországon az elektronikus könyvkiadás fejlődni tudjon, a digitális kereskedelemben kapható könyvek választéka egyre szélesebb legyen. Köszönjük, és reméljük webáruházunkban hamarosan viszontlátjuk.

Tehetségek példatára sorozat

A sorozat kötetei: Pogáts Ferenc: Varga Tamás matematikai versenyek 1. Pogáts Ferenc: Varga Tamás matematikai versenyek 2. Pogáts Ferenc – Fazakas Tünde: Varga Tamás matematikai versenyek 3. (Szerkesztı kollektíva): Bergengóc példatár 1. (Szerkesztı kollektíva): Bergengóc példatár 2.

RÓKA SÁNDOR

2000 FELADAT AZ ELEMI MATEMATIKA KÖRÉBİL Hatodik kiadás

Budapest, 2010

Kedves Olvasó! Önre gondoltunk, amikor a könyv elıkészítésén munkálkodtunk. Kapcsolatunkat szorosabbra főzhetjük, ha a kiadó honlapján, a www.typotex.hu címen feliratkozik hírlevelünkre, melybıl értesülhet új kiadványainkról, akcióinkról, programjainkról. A honlapon megismerkedhet teljes kínálatunkkal, egyes könyveinknél pedig új fejezeteket, bibliográfiát, hivatkozásokat találhat, illetve az esetlegesen elıforduló hibák jegyzékét is letöltheti. Kiadványaink egy része e-könyvként (is) kapható: www.interkonyv.hu Kiadja a Typotex kiadó, az 1795-ben alapított Magyar Könyvkiadók és Könyvterjesztık Egyesülésének tagja. Felelıs kiadó: Votisky Zsuzsa Felelıs szerkesztı: Fried Katalin Tördelés: Könyvmővek Bt. Borítóterv: Tóth Norbert Terjedelem: 26,5 (A/5) ív Készült a G-Print nyomdában Felelıs vezetı: Wilpert Gábor

Bevezető

Ez a feladatgyűjtemény sokéves munka eredménye. 1992-ben jelent meg az 1000 feladat az elemi matematika köréből c. könyvem, melynek bővített, átdolgozott kiadását, az 1500 feladat . . . -ot 1996-ban adta ki a TypoTEX. Az azóta eltelt évek alatt a feladatgyűjteményen elvégeztem a szükségesnek gondolt javításokat, kiegészítéseket, és új fejezeteket állítottam össze. Remélem, hogy előnyére vált a könyvnek ez az éveken át tartó bővítés és a korábbi feladatsorok csiszolgatása. Most vége az átdolgozásoknak; már évekkel ezelőtt is úgy terveztem, hogy a 2000 feladat az elemi matematika köréből lesz ennek a feladatgyűjteménynek az utolsó változata. Újabb átdolgozás, bővítés nem várható. A feladatokat zömmel a hazai és az orosz versenyfeladatok közül gyűjtöttem, felhasználtam a KöMaL (Középiskolai Matematikai és Fizikai Lapok) és a Kvant feladatait, és további sokféle más forrásra is támaszkodtam. A feladatgyűjtemény összeállításánál arra törekedtem, hogy szép és érdekes feladatok kerüljenek bemutatásra, egy-egy témakört minél gazdagabban tárjunk az olvasó elé; továbbá szándékom volt, hogy a feladatgyűjtemény segítse a versenyekre való felkészülést. Ezt a gyűjteményt használhatják 14–18 éves diákok a tanulmányi versenyekre és a KöMaL pontversenyére való felkészülésben. Megjegyezve azért, hogy noha vannak geometriai feladatok, de a hagyományos geometriai témák hiányoznak a feladatgyűjteményből. A középiskolás korosztályt tanító tanárok számára szakköri feladatgyűjteményként szolgálhat a könyv: egy-egy témakör feldolgozásához bőséges példaanyagot találunk. A 2000 feladat . . . használható segédkönyvként is a tanárjelöltek képzésében, az Elemi matematika szemináriumokon. A feladatokat témakörök szerint csoportosítottam, s legtöbbjükhöz megoldást adtam (esetleg csak a megoldás vázlatát, vagy ötletet, útmutatást). A feladat sorszáma mellett zárójelben feltüntetett szám (1-től 7-ig) a feladat nehézségét mutatja: kisebb szám könnyebb feladatot jelez. A nehézségi fokot egyértelműen megállapítani nyilván nem lehet, de a megadott skálázás némi segítséget adhat a feladat megoldójának. Érdemes egy-egy fejezet feldolgozásánál a feladatok sorrendjét követni, mert a korábban szereplő feladatok többnyire ötletet adnak és segítséget nyújtanak a megoldáshoz. Csábító lehet a feladat megismerése után rögtön elolvasni a megoldáshoz adott útmutatást, de ez némi akaraterővel halogatható. A feladattal barátkozni kell, néha napokig gondolkodni rajta. Közelebb juthatunk a feladat megoldásához, ha megpróbáljuk az állítást élesíteni vagy a feltételek változtatásával megfigyeljük, hogyan változik az állítás.

6

Bevezető

A problémamegoldásban jártas embereket az különbözteti meg a náluk kevésbé sikeres kezdőktől, hogy az előbbieknek nagyobb a tudásuk. Az orvosnak ismernie kell az anatómiát, a futballistának a saját sportágát, a szakácsnak az ételeket. Becslések szerint még a legtehetségesebb sakkozóból sem válhat nagymester addig, amíg legalább tíz évet el nem tölt sakkfeladványok megoldásával. Minden problémamegoldásnak van megtanulható része. Pólya György szerint: „A problémamegoldás csakúgy gyakorlat kérdése, mint az úszás, sízés vagy zongorázás. Megtanulni is csak utánzás és gyakorlás útján lehet . . . Aki úszni akar tanulni, annak vízbe kell ugrania, aki problémákat megoldani akar tanulni, annak problémák megoldását kell gyakorolnia.” Ahhoz, hogy valaki matematika versenyeken jó eredményeket érjen el, bizony sok munka szükséges. Nem elég a tehetség, érzék; a feladatmegoldást, matematikai gondolkodást tanulni is kell. A versenyekre lehet készülni, a versenyekre fel lehet készülni. Ez a készülés, ez az edzés feladatmegoldást jelent. A 2000 feladat . . . lehetőséget biztosít a problémák megoldásának gyakorlására. Aki egyszer megérezte az eredményes problémamegoldás örömét, egy életre megőrzi azt; és amit ez kifejleszt, a hajlékony gondolkodás, az az embert egész életén végigkísérő értékes képesség. A feladatok megismerésekor a figyelmes olvasó megtapasztalhatja a matematika örömét a meglepő tételekben, szép okoskodásokban, logikus gondolatmenetekben. Egy bizonyítás nem ritkán izgalmas, érdekfeszítő olvasmány. Hálás köszönettel tartozom a könyv lektorának, Hódi Endrének, aki alapos, körültekintő munkájával nagyon sok segítséget nyújtott. Az ő ösztönzésére jelentősen csökkent a megoldás nélküli feladatok száma; segített a hibák felderítésében; útmutatásai alapján igyekeztem a megoldásokat minél érthetőbben, világosabban leírni; a feladatok megfogalmazásában pedig, ahol kellett, figyelmeztetett a pontosságra. A lektor lelkiismeretes és gondos munkát végzett; nagyon sokat tett azért, hogy ez a könyv minél jobb legyen.

Feladatok

1. Fejtörők 1. [3] Az ábrán látható autóbusznak melyik az eleje?

2. [3] Az ábrán látható három egyfülű nyulat vágjuk ki papírból, majd helyezzük el a nyulakat úgy, hogy mindegyik nyúlnak két füle legyen.

3. [3] Az asztalra letettünk 10 db egyforma pénzérmét az ábrán látható módon. Ezekből 3 érmét helyezzünk máshová úgy, hogy egy más helyzetű, az eredetivel egybevágó szabályos háromszöget kapjunk.

4. [3] Rajzoljunk az ábrába a ceruza felemelése nélkül olyan töröttvonalat úgy, amely négy, végpontjaival sorban egymáshoz csatlakozó egyenesszakaszból áll, és mind a kilenc ponton áthalad.

Feladatok

8

5. [2] Egy vadász medvenyomokat követve 1 km-t haladt déli irányban, majd 1 km-t ment keletre, végül 1 km-t északra, és ekkor visszaért oda, ahonnan elindult. Milyen színű a medve? 6. [2] A tengeren négy hajó halad együtt, közel egymáshoz: bármely két hajó távolsága 3 km. A hajók között van teherszállító, olajszállító és utasszállító hajó. Milyen hajó a negyedik? 7. [3] Egy hajó hosszának, árbócmagasságának, a hajó kapitánya és kisfia életkorának (a számok egész számok) szorzata 323 935. Hány éves a kapitány? 8. [3] Az ábrán látható nyolcszöget – mint a második ábra mutatja – fel lehet osztani négy egybevágó részre. És hogyan lehet felosztani öt egybevágó részre?

9. [3] A felírt „egyenlőségben” egy számjegyet helyezzünk máshová úgy, hogy igaz egyenlőséghez jussunk.

101 − 102 = 1

10. [5] Egy alkalommal Félix kiment a táblához, és felírta a következőt: 1995 + +146 = 210 + 1117. Ez igaz egyenlőséggé változtatható úgy, hogy két számjegyet letörlünk, majd máshová írjuk. Hogyan? (Megengedett az, hogy egy számjegyet két számjegy közé „beékeljünk”.) 11. [4] Tegyük igazzá a 76 = 24 egyenlőséget úgy, hogy a számjegyeket szabadon mozgathatjuk. 12. [3] Elkészítettük az alábbi számkártyákat, és azokat az ábra szerint helyeztük el. Mozgassunk el két számkártyát úgy, hogy a két összeg egyenlő legyen.

1

3

2

4

7

5

+ 9 19

+ 8 20

1. Fejtörők

9

13. [5] Az ábrát gyufaszálakból készítettük. Helyezzünk át egy szál gyufát úgy, hogy igaz egyenlőséget kapjunk.

14. [3] Milyen számot kell írni az üres mezőkbe?

12 15 30 17 26 22 24 20 25 20 13 28 18 30 24 20 30 20 26 15. [2] Mi a sorozat következő eleme?

16. [2] Mi lehet a sorozat következő eleme? a) 2, 3, 5, 7, 11, 13, 17, 19, ? b) 1, 2, 6, 24, 120, 720, ? c) 1, 1, 2, 3, 5, 8, 13, 21, 34, 55, ? d) 100, 101, 103, 107, 115, 122, ? e) 1, 4, 9, 16, 25, 36, ? f) 77, 49, 36, 18, ? 17. [4] Mi lehet a sorozat következő eleme? 1000, 22, 20, 13, 12, . . . 18. [4] Fedezzük fel a szabályosságot a következő sorozatban: 111, 213, 141, 516, 171, 819, 202, 122, . . . 19. [5] Mi a sorozat következő eleme? 1, 11, 21, 1211, 111221, 312211, 13112221, . . . 20. [4] Vajon mi lesz a 461, 552, 663, 794, 846, . . . sorozat következő eleme? 21. [4] Vajon hogyan folytassuk a betűk sorozatát: NEKHNÖH . . . ? 22. [4] Mi lehet a három hiányzó betű: B, H , F , V , K, ?, ?, ?.

Feladatok

10

23. [4] Melyik szó következik a szavak sorozatában: segély, otthon, világ, birodalom, hadoszlop, érzék, . . . a) víz, b) zene, c) üdvözlet, d) mennyország. 24. [7] A számok között mindenhol ugyanaz az egyszerű összefüggés fedezhető fel. Vajon mi ez az összefüggés? 0 1

0 1

1

2 1

1 1 1

3

7

1 1

1

7 16

9 2

1 0

2 3

4

2

0

1 2

5

1

3 1

0

3

1

5

0 1

2

3

2 1

0

1

4

3

0 1

1

7

2 1

0

2

16

5

0 1

9

7

3 1

0

5

4

2

0 1

9

3

1

0 1

9 5

1 0

1 0

25. [2] Írjunk a 7 ∗ ∗ ∗ ∗ ∗ ∗ 9 tízes számrendszerbeli számban a ∗-ok helyére számjegyeket úgy, hogy bármely három szomszédos számjegy összege 20 legyen. 26. [3] A 231213 hatjegyű számban a két 1-es között 1 számjegy, a két 2-es között 2 számjegy, a két 3-as között 3 számjegy van. Írjunk fel olyan nyolcjegyű számot, amelyben 2 db 1, 2 db 2, 2 db 3 és 2 db 4-es számjegy van, és a két 1-es között 1 számjegy, a két 2-es között 2 számjegy, a két 3-as között 3 számjegy, a két 4-es között 4 számjegy legyen. 27. [3] Írjuk a 0, 0, 1, 1, 2, 2, 3, 3 számjegyeket egymás után olyan sorrendbe, hogy a 2 db 0 között 0, a 2 db 1-es között 1, a 2 db 2-es között 2, a 2 db 3-as között 3 számjegy legyen. 28. [3] A kis négyzetekbe írjuk be az 1, 2, 3, 7, 8, 9, 13, 14, 15 számokat úgy, hogy mindegyik sorban, mindegyik oszlopban és a két átlóban ugyanannyi legyen a számok összege.

1. Fejtörők

11

29. [3] Írjuk be a táblázat mezőibe az 1, 2, 3, 4, 5, 6, 7, 8, 9 számokat úgy, hogy mindegyik sorban és mindegyik oszlopban annyi legyen a számok összege, mint amennyit a sor mellett, ill. az oszlop alatt levő szám mutat.

6 16 23 14

12

19

30. [3] Írjuk be a táblázat mezőibe az 1, 2, 3, 4, 5, 6, 7, 8, 9 számokat úgy, hogy a számok szorzata minden sorban egyenlő legyen a sor mellé írt számmal, s hasonlóan minden oszlopban a számok szorzata legyen egyenlő az oszlop alá írt számmal.

20 108 168 42

31. [3] Írjunk a téglalapokba különböző pozitív egészeket úgy, hogy mindegyik az alatta levő két szám összege legyen (természetesen ez az alsó sorra nem vonatkozik), és a legfelső mezőben minél kisebb szám álljon.

32. [3] Válasszunk ki a 4 × 4-es táblázat mezőiből tízet úgy, hogy mindegyik sorban és mindegyik oszlopban páros legyen a kiválasztott mezők száma.

80 108

Feladatok

12

33. [3] A kör kerületét 12 egyenlő részre osztottuk. Erre a 12 helyre írjunk nullától különböző egész számokat úgy, hogy bármelyiktől indulva, minden harmadik számot véve, azok összege nulla legyen, és bárhonnan indulva, minden negyedik számot véve, azok összege is nulla legyen. (Tehát, ha kiválasztunk egy szabályos sokszöget, melynek csúcsai a 12 pont közül valók, ott a csúcsokban levő számok összege nulla.) 34. [3] A ∗ jelek helyére írjuk be a 0, 1, 2, 3, 4, 5, 6, 7, 8, 9 számjegyeket úgy, hogy helyes legyen az összeadás. (Minden számjegy egyszer szerepel egy összegben, és az összegben szereplő számok egyike sem kezdődik nullával.) Keressünk több megoldást.

35. [3] A ∗ jelek helyére írjuk be az 1, 2, 3, 4, 5, 6, 7, 8, 9 számjegyeket úgy, hogy helyes legyen az összeadás. (Minden számjegy egyszer szerepel egy összegben.) Keressünk több megoldást.

∗ ∗∗ + ∗∗∗ ∗∗∗∗ ∗∗∗ + ∗∗∗ ∗∗∗

36. [4] Válasszunk három számjegyet úgy, hogy azok különböző sorrendjével fel tudjunk írni három olyan háromjegyű számot, melyek közül kettőnek az összege lesz a harmadik szám. 37. [4] Válasszunk négy számjegyet úgy, hogy azok különböző sorrendjével fel tudjunk írni három olyan négyjegyű számot, melyek közül kettőnek az összege lesz a harmadik szám. 38. [4] Adjunk meg három olyan 9-jegyű számot, melyek közül az egyik a másik kettő összege, és amely számok mindegyikét az 1, 2, 3, 4, 5, 6, 7, 8, 9 számjegyek valamilyen sorrendjével írjuk fel. 39. [3] Határozzuk meg a törtkifejezés értékét, ha az azonos betűk azonos, a különböző betűk különböző számjegyeket jelölnek.

C·S·O·D·Á·L·A·T·O·S =? B·A·L·A·T·O·N

1. Fejtörők

13

40. [4] Az alábbi számításokban a számjegyeket betűkkel helyettesítettük; azonos számjegyeket azonos betűkkel, különböző számjegyeket különböző betűkkel. Milyen számokat rejtenek a betűk?

a)

b)

c)

S E ND + MORE MO NE Y

S AVE + MORE MO NE Y

S E ND MORE + G OL D MO NE Y

d)

e)

f)

USA +USSR PE ACE

THRE E +F OUR SE VE N

F ORTY T EN + T EN SI XTY

g)

h)

i)

SOK SOK KI S + KOS OKOS

CB AB CB CAA ADEA

L A P K E P KAR E K L P A O} K U L L O}

41. [4] Az alábbi szorzásokban csak néhány számjegyet ismerünk. Határozzuk meg az ismeretlen számjegyeket. a) b) c)

aa aa aa a7 aaaa

aa aa aaa aa 9aaa

aa 7a aa aaa aaa

Feladatok

14

42. [5] Az alábbi szorzásokban mindegyik számjegy prímszám. Határozzuk meg az ismeretlen számjegyeket. a)

b)

aaa aa aaaa aaaa aaaaa

aaa aaa aaaa aaaa aaaa aaaaaa

43. [5] Az alábbi szorzásokban az a betű páros, a b betű páratlan számjegyet jelöl. Határozzuk meg az ismeretlen számjegyeket. a)

b)

c)

aaa bbb aba aaaa abba baabba

bbb aaa aba aaba abba baaba

aab bb abb abab bbbbb

44. [3] A 2 : 3 : 4 : 5 : 6 = 5 kifejezésbe írjunk zárójeleket úgy, hogy az így kijelölt osztásokat elvégezve a felírt eredményt kapjuk. 45. [3] Az 1 : 2 : 3 : 4 : 5 : 6 : 7 : 8 : 9 : 10 = 7 kifejezésbe írjunk zárójeleket úgy, hogy az így kijelölt osztásokat elvégezve a felírt eredményt kapjuk. 46. [2] Írjunk az 1 2 3 4 5 6 7 8 9 = 100 kifejezésben a számjegyek közé + és − jeleket vagy semmit úgy, hogy igaz legyen az egyenlőség. Egy megoldás a következő: 123 − 4 − 5 − 6 − 7 +8 −9 = 100. Keressünk minél több megoldást. 47. [2] Írjunk a 9 8 7 6 5 4 3 2 1 = 100 kifejezésben a számjegyek közé + és − jeleket vagy semmit úgy, hogy igaz legyen az egyenlőség. Keressünk minél több megoldást. 48. [2] Írjunk a 9 8 7 6 5 4 3 2 1 = 99 kifejezésben a számjegyek közé + jeleket vagy semmit úgy, hogy igaz legyen az egyenlőség. Keressünk minél több megoldást. 49. [2] Az 1, 2, 3, . . . , 15 számokat osszuk szét öt csoportba úgy, hogy mindegyik csoportban három szám legyen, s a három szám közül a legnagyobb egyenlő legyen a másik kettő összegével. 50. [3] Az 1, 2, 3, . . . , 13 számokat osszuk szét három csoportba úgy, hogy egyik csoportban se szerepeljen két olyan szám, melyek összege is a csoportban van, és ne legyen ugyanabban a csoportban egy szám és a kétszerese.

1. Fejtörők

15

51. [3] A 10, 11, 12, . . . , 49 számokat osszuk szét két csoportba úgy, hogy egyik csoportban se szerepeljen két olyan szám, melyek összege is a csoportban van, és ne legyen ugyanabban a csoportban egy szám és a kétszerese. 52. [3] Válasszunk ki az 1, 2, 3, . . . , 100 számokból minél többet úgy, hogy a kiválasztott számok között ne legyen két olyan, melyek egyike osztója a másiknak. 53. [3] Válasszunk ki az 1, 2, 3, . . . , 100 számokból minél többet úgy, hogy a kiválasztott számok között ne legyen két olyan, melyek összege is a kiválasztott számok között van, és a kiválasztott számok között ne legyen olyan sem, melynek a kétszerese is a kiválasztottak között van. 54. [3] Egy zár, melyen három nyomógomb van (1-es, 2-es, 3-as), akkor nyílik, ha a gombokat egy előírt sorrendben nyomjuk meg közvetlenül egymás után. (A megfelelő három gombnyomást esetlegesen megelőző gombnyomások sorozatának nincs hatása a zár szerkezetére.) A zár biztosan kinyílik például a következő sorozatra: 123 132 213 231 312 321. Adjunk meg olyan 9 gombnyomásból álló sorozatot, mely biztosan kinyitja a zárat. 55. [2] Választunk egy háromjegyű számot és elosztjuk számjegyeinek összegével. Az így kapott hányados melyik háromjegyű szám esetén lesz a legkisebb, és mikor lesz a legnagyobb? 56. [1] Melyik az a legkisebb természetes szám, amelyben a számjegyek összege 1994? 57. [2] Melyik az a legkisebb természetes szám, amelyben a számjegyek szorzata 200? 58. [2] Melyik az a legnagyobb négyjegyű szám, amelyben nincs két azonos számjegy, és a számjegyek szorzata 216? 59. [3] Adjunk meg olyan pozitív egész számot, mely felírható 3 egymást követő egész szorzataként is, és felírható 6 egymást követő egész szorzataként is. 60. [3] Egy futóversenyen 12 rajtszámmal jelzett versenyző indult. Mi volt a beérkezés sorrendje, ha a rajtszám és a helyezési szám szorzata mindig 1-gyel nagyobb egy 13-mal osztható számnál? 61. [3] Az 1, 2, 3, 4, 5, 6 számokat felhasználva (egy szám többször is előfordulhat) egy négyjegyű számot írtunk fel. Többen megpróbálják kitalálni ezt a számot. Az első tipp: 4215. Két számot eltalált, de csak egy van jó helyi értéken. A második tipp: 2365. Ismét két számot talált el, de csak az egyik van jó helyi értéken. A harmadik tipp: 5525. Itt még a számokat sem találta el. Mi lehet a keresett szám? 62. [3] Rajzoljunk le 8 szakaszt úgy, hogy mindegyik három másikat metsszen. 63. [3] Egy háromszög és egy négyszög legfeljebb hány pontban metszi egymást, ha nincs közös oldalegyenesük?

16

Feladatok

64. [3] Vegyünk fel 6 pontot a síkon, és kössünk össze azokból néhányat szakaszokkal úgy, hogy mindegyik pontból 3 szakasz induljon, és a szakaszok ne keresztezzék egymást. 65. [3] Vegyünk fel 6 pontot a síkon, és kössünk össze azokból néhányat szakaszokkal úgy, hogy mindegyik pontból 4 szakasz induljon, és a szakaszok ne keresztezzék egymást. 66. [3] Vegyünk fel 7 pontot a síkon úgy, hogy ha azokat páronként összekötjük egyenesekkel, akkor összesen 14 különböző egyenest kapjunk. 67. [4] Adott néhány egyforma méretű tégla. Ezek segítségével csak egy vonalzót használva, hogyan lehetne megmérni a tégla testátlóját? 68. [4] Tudor egy 200-jegyű számra gondolt, amelynek számjegyösszegét megsúgta Vidornak. Vidor kiszámította a neki megsúgott szám számjegyösszegét, s ezt megsúgta Morgónak. Morgó meghatározta ezen szám számjegyösszegét, ami kétjegyű szám volt, s ezt megsúgta Hapcinak. Hapci is meghatározta a számjegyösszeget, s ezt megsúgta Kukának. Milyen számot súgott Vidor Morgónak? Milyen számot súgott Hapci Kukának? 69. [3] Két pohár közül az egyikben 1 dl bor, a másikban ugyanennyi víz van. Egy kanál bort átöntünk a vízbe, ezt jól megkavarjuk, majd innen egy kanál folyadékot (víz-bor keveréket) átöntünk a borba. A vízben lesz-e több bor, vagy a borban lesz-e több víz? 70. [3] Igaz-e, hogy nagyapáim dédapjai ugyanazok a személyek, mint dédapáim nagyapjai? 71. [3] A mesebeli róka a következő egyezséget kötötte egy legénnyel: ahányszor átmegy a hídon, a róka mindannyiszor megkétszerezi a pénzét, s ebből a pénzből 24 krajcár vámot kell fizetnie. A legény azt hitte, hogy ily módon sok pénzhez juthat, de nagyot csalódott. Amikor ugyanis harmadszor ment át a hídon, és kifizette a vámot, nem maradt egy krajcárja sem. Mennyi pénze volt a legénynek, mikor az egyezséget megkötötte? 72. [3] Öt játékos megegyezett, hogy a vesztes minden játszma után megkétszerezi a többi pénzét. Összesen öt játszmát játszottak, mindegyik játékos egyszer veszített. A játék végén mindegyiküknek 128 F t-ja volt. Mennyi pénze volt a játékosoknak külön-külön a játék megkezdése előtt? 73. [4] Egy szobában 10 szék van sorban egymás mellett. A székek kezdetben üresek. Időnként valaki bejön a szobába, leül egy üres székre, és ugyanekkor egyik szomszédja (ha van neki) föláll és kimegy. Legfeljebb hány szék lehet foglalt egyszerre a szobában?

1. Fejtörők

17

74. [3] Három ládikó mindegyikében 2-2 golyó van: fehér-fehér, piros-piros, fehérpiros, és mindegyik ládán felirat: F F , P P , F P , ami a ládában levő golyók színét jelzi. Azonban mindegyik felirat hamis. Valamelyik – általunk választott – ládikóból kivehetünk egy golyót, s ezek után meg kell mondani, hogy az egyes ládikókban milyen golyók vannak. Hogyan lehet ezt megtenni? 75. [3] Egy papírlapon az alábbi öt állítás olvasható. Ezen Ezen Ezen Ezen Ezen

a a a a a

lapon lapon lapon lapon lapon

1 2 3 4 5

állítás állítás állítás állítás állítás

hamis. hamis. hamis. hamis. hamis.

Az állítások közül melyik igaz, melyik hamis? 76. [3] Egy papírlapon az alábbi öt állítás olvasható. Ezen Ezen Ezen Ezen Ezen

a a a a a

lapon lapon lapon lapon lapon

legfeljebb legfeljebb legfeljebb legfeljebb legfeljebb

1 2 3 4 5

állítás állítás állítás állítás állítás

igaz. igaz. igaz. igaz. igaz.

Az állítások közül melyik igaz, melyik hamis? 77.

[3]

78.

[3]

A három ládikón levő állításból legfeljebb egy igaz.

Hol van az arany? A három ládikón levő állítások között van igaz és van hamis állítás.

Hol van az arany?

Feladatok

18

79. [2] Az asztalon kártyalapok vannak, melyek egyik oldalán betű, másik oldalán szám áll. Mi az előttünk levő négy kártyalapon az A, B, 1, 2 felírást látjuk. Legkevesebb hány lapot kell megfordítani, ha el akarjuk dönteni, hogy igaz-e a következő állítás: „Magánhangzó mögött páros szám áll.”? 80. [3] Az alábbi feladatokban szereplők mindegyike vagy az igazmondók, vagy a hazugok szektájába tartozik. a) Egy szobában ketten vannak, A és B. Azt mondja A: „Legalább egyikünk hazug.” Ki melyik szektába tartozik? b) Egy szobában ketten vannak, A és B. Azt mondja A: „Én hazug vagyok, de B nem az.” Ki melyik szektába tartozik? c) Egy szobában hárman vannak, A, B és C. A következők hangzanak el. A: „Mindhárman hazugok vagyunk.” B: „Pontosan egy igaz van köztünk.” Ki melyik szektába tartozik? d) Egy szobában hárman vannak, A, B és C. A következők hangzanak el. A: „Mindhárman hazugok vagyunk.” B: „Pontosan egy hazug van köztünk.” C melyik szektába tartozik? 81. [3] Egy vándor egy útelágazáshoz ér, egyik út a sivatagba, a másik Mekkába vezet. Az útelágazásnál ketten álldogálnak, egy igazmondó és egy hazug. Mit kell kérdeznie a vándornak, hogy megtudja a Mekkába vezető utat, ha csak egyetlen kérdést tehet fel, és nem tudja, hogy igazmondóval vagy hazuggal beszél-e? 82. [3] Két szomszédos város egyikében igazmondók, a másikban hazugok laknak. A városlakók természetesen eljárnak látogatóba a szomszédos városba. Mit kérdezzen egy turista, hogy megtudja, melyik városban tartózkodik (az igazmondókéban vagy a hazugokéban), ha csak egy kérdést tehet fel, és nem tudja, hogy igazmondót vagy hazugot kérdez? 83. [3] Hárman beszélgetnek: A, B és C. A azt mondja: „B hazudik.” B azt mondja: „C hazudik.” C azt mondja: „A és B hazudik.” Ki mond igazat és ki hazudik? 84. [4] Olvassuk el figyelmesen a következő 5 állítást: (A): A (B) állítás igaz. (B): Az (A), (B), (C), (D), (E) állítások közül legfeljebb az egyik igaz. (C): Az (A), (B), (C), (D), (E) állítások mindegyike igaz. (D): (E):

2. Páros vagy páratlan?

19

A (D) és (E) állításokat varázstintával írtuk, láthatatlanok azok számára, akik nem mindig mondanak igazat. Állapítsuk meg, hogy az (A), (B), (C), (D), (E) állítások közül melyik igaz, melyik hamis.

2. Páros vagy páratlan? 85. [1] El lehet-e osztani 100 nyulat 5 gyerek között úgy, hogy mindegyik gyereknek páratlan számú nyúl jusson? 86. [1] Meg lehet-e adni négy egész számot úgy, hogy összegük és szorzatuk is páratlan szám legyen? 87. [1] Három egész szám összege 1994. Lehet-e 1 a három szám szorzatának utolsó jegye? 88. [2] Szét lehet-e osztani az 1, 2, 3, . . . , 1994 számokat két csoportba úgy, hogy mindegyik csoportban páratlan legyen a számok összege? 89. [2] Adjunk meg néhány egész számot úgy, hogy szorzatuk és összegük is 9 legyen. 90. [2] Lehet-e 9 egész szám összege is, szorzata is 9? 91. [3] Lehet-e 10 egész szám összege is, szorzata is 10? 92. [2] Lehet-e 8 egész szám összege is, szorzata is 8? 93. [2] Lehet-e 9 egész szám szorzata 9, összege pedig 0? 94. [3] Lehet-e 10 egész szám szorzata 10, összege pedig 0? 95. [2] Lehet-e 8 egész szám szorzata 8, összege pedig 0? 96. [3] Az 1, 2, 3, . . . , 9 számok valamilyen más sorrendjét a1 , a2 , a3 , . . . , a9 jelöli. Lehet-e páratlan az (a1 − 1) · (a2 − 2) · (a3 − 3) · . . . · (a9 − 9) szorzat? 97. [3] Az a1 , a2 , a3 , . . . , a9 számok valamilyen más sorrendjét b1 , b2 , b3 , . . . , b9 jelöli. Lehet-e páratlan az (a1 − b1 ) · (a2 − b2 ) · (a3 − b3 ) · . . . · (a9 − b9 ) szorzat? 98. [3] Egy négyzet alapú tortát összesen 13 vízszintes és függőleges irányú vágással szeletekre vágtunk. Lehet-e páratlan szám a kapott szeletek száma? 99. [2] A ±1 ± 3 ± 5 ± 7 ± 9 ± 11 kifejezésben megválaszthatók-e úgy a + és − jelek, hogy ezt követően kiszámolva az összeget, a végeredmény 13 legyen? 100. [2] A ±1 ± 2 ± 3 ± · · · ± 100 kifejezésben megválaszthatók-e úgy a + és − jelek, hogy ezt követően kiszámolva az összeget, a végeredmény 1991 legyen? 101. [3] Miért nem lehet 100 különböző páratlan szám reciprokának összege 1? Lehet-e 100 különböző egész szám reciprokának összege 1?

Feladatok

20

102. [3] Lehet-e egész szám az első 100 prímszám reciprokának összege? 103. [3] Összeadtunk három egymást követő egész számot, majd összeadtuk a következő három számot is. Lehet-e 111 111 111 az így kapott két szám szorzata? Megjegyzés: A párosság vizsgálatával oldható meg a Diofantoszi egyenletek, az Invariáns tulajdonságok, a Feladatok a sakktáblán és a Különböző kombinatorikai feladatok c. fejezetek több feladata.

3. Párbaállítás 104. [1] Számoljuk ki a következő kifejezések értékét. a) 1 + 3 + 5 + · · · + 95 + 97 + 99 =? b)

1 + 2 + 3 + · · · + 99 + 100 + 101 =? 1 − 2 + 3 − · · · + 99 − 100 + 101

105. [3] Mennyi az a) ±1 ± 2 ± 3 ± 4 ± · · · ± 1993, illetve b) ±12 ± 22 ± 32 ± 42 ± · · · ± 19932 összeg legkisebb pozitív értéke? (A ± jel helyére a megfelelően választott + és − jelek valamelyikét írjuk be.) 106. [3] Mivel egyenlő az 1, 2, 3, . . . , 998, 999, 1000 számok számjegyeinek összege? 107. [1] Osszuk szét az 1, 2, 3, . . . , 100 számokat a) két, egyaránt ötven számból álló csoportba b) öt, egyaránt húsz-húsz számból álló csoportba úgy, hogy mindegyik csoportban ugyanannyi legyen a számok összege. 108. [2] Az autóbuszjegyen 4 vagy 5 lyuk lyukasztásával lehet többféle lyukkombinációt megvalósítani? 109. [2] Hány részre osztják a teret a kocka lapsíkjai? 110. [3] Hány részre osztják a teret a tetraéder lapsíkjai? 111. [3] Szabályos tetraéder minden élén át síkot fektetünk, mely két egybevágó részre osztja a tetraédert. Hány részre bontottuk fel így a tetraédert? 112. [2] 1992 asztaliteniszező között kieséses versenyt szervezünk. Minden forduló után mindegyik párból a győztes jut tovább. Ha egy fordulóban valamelyik játékosnak nem jut ellenfél, úgy ő automatikusan (erőnyerőként) tovább jut. Hány mérkőzést játszanak le, míg megtalálják a legjobb játékost?

3. Párbaállítás

21

113. [2] Egy 6 × 10-es négyzetrácsos papírt rácsegyenesek mentén két részre vágunk, majd az egyik darabot ismét csak rácsegyenesek mentén két részre vágjuk és így tovább: minden alkalommal valamelyik darabot két részre vágjuk. Legkevesebb hány vágásra van szükség, amíg a papirost 1 × 1-es négyzetekre daraboljuk? És legfeljebb? 114. [3] Mutassuk meg, hogy egy természetes szám (pozitív) osztóinak száma pontosan akkor páratlan, ha a szám négyzetszám. 115. [2] Miért nem lehet a 7 777 777 szám osztóinak száma 1991? 116. [3] A szultán születésnapján néhány rabot szabadon akar bocsátani. A 100 cellás börtönben 100 börtönőr van. Az 1. őr minden ajtót kinyit. A 2. őr minden 2. ajtót bezár. A 3. őr minden 3. ajtót kinyit, ha zárva volt, s bezár, ha nyitva volt. Hasonlóan nyit-zár a többi őr is. Mely cellák ajtaja marad nyitva? 117. [3] Az n számnak 100 osztója van. Szorozzuk össze a szám osztóit! Mi lesz a végeredmény? 118. [5] Mely n-re lesz n összes pozitív osztójának szorzata 2120 · 360 · 590 ? 119. [4] Az n természetes számot tökéletesnek nevezzük, ha egyenlő a nála kisebb pozitív osztóinak összegével (ilyen pl. a 6, a 28). Számoljuk ki az n tökéletes szám pozitív osztói reciprokának összegét. 120. [4] Egy n természetes szám pozitív osztóinak összegét elosztottuk az osztók reciprokainak összegével. Igaz-e, hogy a kapott hányados értéke n? √ [4] 121. Az n összetett szám legkisebb prímosztója p. Mutassuk meg, hogy p ≤ n. n 122. [4] Az n szám legkisebb prímosztója p, és összetett szám. Mutassuk meg, hogy p √ p ≤ 3 n. 123. [4] Mutassuk √ meg, hogy az n természetes szám pozitív osztóinak száma kisebb, mint 2 n. 124. [4] Mutassuk meg, hogy egy páros szám páros osztóinak összege nagyobb, mint a páratlan osztók összege. 125. [4] Mutassuk meg, hogy tetszőleges n > 2 természetes számnál kisebb, n-hez relatív prím számok száma páros. 126. [4] Mutassuk meg, hogy tetszőleges n > 2 természetes számnál kisebb, n-hez relatív prím természetes számok összege többszöröse n-nek. 127. [5] Mutassuk meg, hogy ha k > 1 egész szám, akkor a 4k-nál kisebb, 4k-hoz relatív prímek összege osztható 8k-val. 128. [4] Hány olyan ötjegyű szám van, melyben a számjegyek nemcsökkenő sorrendben követik egymást?

Feladatok

22

129. [4] Legfeljebb hány pontban metszik egymást egy n oldalú konvex sokszög átlói? 130. [4] Legfeljebb hány háromszöget fognak közre a háromszög belsejében egy n oldalú konvex sokszög átlói? 131. [3] Egy n-oldalú konvex sokszög csúcsai A1 , A2 , . . . , An . Tekintsük azokat a sokszögeket, melyek csúcsai A1 , A2 , . . . , An közül valók. Mely sokszögekből van több: azokból melyeknek egyik csúcsa A1 vagy azokból, amelyeknek A1 nem csúcsa? 132. [4] Hány téglalapot lehet kijelölni egy 6 × 10-es négyzetrácsban úgy, hogy oldalaik rácsegyenesek legyenek? 133. [4] Egy 27 db egybevágó kis kockából kirakott 3 × 3 × 3-as kockában hány olyan téglatest jelölhető ki, mely ezen kis kockákból áll? 134. [4] A számegyenesen egyesével lépkedünk az egész számokon; hol jobbra, hol balra lépünk. Hányféleképp juthatunk a 0-ból az 1-be a) 6, illetve b) 7 lépésben? 135. [4] A sakktábla bal alsó sarkából a jobb felső sarokba hányféle útvonalon juthat el a bástya, ha a bástyát csak előre és fölfelé lehet tolni? 136. [3] Igazoljuk, hogy 13 + 23 + 33 + · · · + 19903 osztható 1991-gyel. 137. [3] Számítsuk ki a

sin 0◦ + sin 1◦ + sin 2◦ + · · · + sin 90◦ tört értékét. cos 0◦ + cos 1◦ + cos 2◦ + · · · + cos 90◦

138. [3] sin2 1◦ + sin2 2◦ + sin2 3◦ + · · · + sin2 89◦ + sin2 90◦ =? 139. [4] Igazoljuk a lg tg 1◦ · lg tg 2◦ · lg tg 3◦ · . . . · lg tg 89◦ = lg tg 1◦ + lg tg 2◦ + lg tg 3◦ + · · · + lg tg 89◦ egyenlőséget. 140. [4] Mutassuk meg, hogy az 1, 2, . . . , 10k számokban levő számjegyek száma egyenlő az 1, 2, . . . , 10k+1 számokban levő 0-k számával. 141. [4] A 3 számot négy különböző módon bonthatjuk fel pozitív egészek összegére: 3; 2 + 1; 1 + 2; valamint 1 + 1 + 1. Az összeadásban a tagok sorrendje lényeges, így az 1 + 2 és a 2 + 1 különböző felbontásnak számít. Hány különböző módon bonthatjuk fel az n természetes számot? 142. [4] Egy 11 × 6 × 5 = 330 db egységkockából álló, fából készült téglatestet egy szú az egyik testátlója mentén végigrágott. Hány egységkockát károsított meg a szú?

4. Miért nem négyzetszám?

23

143. [4] Egy városban az autóbuszjegyeken a sorszámozás 000 000-tól 999 999-ig megy. Nevezzünk érdekesnek egy sorszámot, ha az első három számjegy összege megegyezik az utolsó három jegy összegével. Mutassuk meg, hogy az érdekes sorszámok száma megegyezik az olyan sorszámok számával, amelyekben a számjegyek összege 27. 144. [4] Adjuk össze azokat a 2000-nél kisebb természetes számokat, amelyekben a számjegyek összege páros. 145. [4] Csupa 1-esekből és 2-esekből álló n-jegyű számokat írunk egy táblára úgy, hogy bármely két szám legalább három helyen különbözzék. Bizonyítsuk be, hogy 2n legfeljebb db szám kerülhet a táblára. n+1 Megjegyzés: A párbaállítás ötletét használja a Területátalakítások és a Matematikai játékok c. fejezetek több feladatának megoldása, ill. a 1756., 1757. feladatok is.

4. Miért nem négyzetszám? Miért nem négyzetszámok a 146–187. feladatokban felírt számok? 146. [3] 117 + 116 + 115 + 114 + 113 + 112 + 11 + 1? 147. [3] 100! + 7? 148. [3] 1! + 2! + 3! + · · · + 100!? 149. [3] 3 + 32 + 33 + · · · + 3100 ? 150. [3] 1010 + 3? 151. [3] 1010 + 5? 152. [3] 10100 + 1050 + 1? 153. [3] abab alakú négyjegyű szám? 154. [3] ababab alakú hatjegyű szám? 155. [3] abcabc alakú hatjegyű szám? 156. [3] abc + bca + cab alakú háromjegyű számok összege? 157. [3] Az 1, 2, 3, 4, 5, 6 számjegyek valamilyen sorrendjével felírt hatjegyű szám? 158. [3] A 2, 3, 4, 5, 6 számjegyek valamilyen sorrendjével felírt ötjegyű szám? 159. [3] 1 db 1-es, 2 db 2-es, 3 db 3-as, . . . , 9 db 9-es számjegy valamilyen sorrendjével felírt 45-jegyű szám?

24

Feladatok

160. [3] 12 + 22 + 32 + · · · + 562 ? 161. [3] 30 db 1-es és néhány 0 segítségével felírt szám? 162. [4] 600 db 6-os és néhány 0 segítségével felírt szám? 163. [3] Az 1, 2, 3, . . . , 1991 számok valamilyen sorrendben történő, egymás után írásával kapott szám? 164. [3] 4 egyforma, 0-tól különböző számjegyre végződő szám? 165. [3] Az a szám, amelynek utolsó két számjegye páratlan? 166. [3] 111 . . . 111 (100 db 1-es)? 167. [3] 444 . . . 444 (100 db 4-es)? 168. [3] 144 . . . 444 (99 db 4-es)? 169. [3] 19948 + 7? 170. [3] 2 egymást követő pozitív egész szám szorzata? 171. [3] 3 egymást követő pozitív egész szám szorzata? 172. [4] 4 egymást követő pozitív egész szám szorzata? 173. [3] 2 páratlan négyzetszám összege? 174. [3] 3 egymást követő négyzetszám összege? 175. [3] 4 egymást követő négyzetszám összege? 176. [3] 5 (ill. 6, 7, 8, 9, 10) egymást követő négyzetszám összege? 177. [3] p1 p2 p3 . . . pk − 1, ahol p1 , p2 , p3 , . . . , pk az első k db (k > 1) prímszámot jelöli? 178. [3] p1 p2 p3 . . . pk + 1, ahol p1 , p2 , p3 , . . . , pk az első k db (k ≥ 1) prímszámot jelöli? 179. [3] Az első 99 prímszám négyzetének összege? 180. [5] Egy pozitív négyzetszám és valamelyik osztójának összege? 181. [5] Egy pozitív négyzetszám és valamely osztója kétszeresének összege? 182. [3] x 2 + y és y 2 + x számok mindegyike, ahol x és y pozitív egészek? 183. [3] x 2 + 4y és y 2 + 4x számok mindegyike, ahol x és y pozitív egészek? 184. [3] n4 + 2n3 + 2n2 + 2n + 1, ahol n pozitív egész számot jelöl? 185. [3] n3 + 2n2 + 2n + 1, ahol n pozitív egész számot jelöl? 186. [5] 11571990 + 341990 ? 187. [5] 2n + 3n , ahol n pozitív egész számot jelöl?

5. Négyzetszámok

25

5. Négyzetszámok 188. [1] Lehet-e két négyzetszám szorzata is, hányadosa is négyzetszám? 189. [2] Van-e különböző négyzetszámokból álló, végtelen sok elemű számtani sorozat? 190. [2] 190 246 849 és 190 302 025 két egymás utáni páratlan szám négyzete. Számítsuk ki a köztük levő páros szám négyzetét. 191. [3] Mely pozitív egész n-ekre lesz n2 + n + 41 négyzetszám? 192. [3] Az alább felsorolt számok közül az egyik négyzetszám, a többi nem. Melyik a négyzetszám: a) 3 669 517 136 205 224, b) 1 898 732 825 398 318, c) 4 751 006 864 295 101, d) 5 901 643 220 186 100, e) 7 538 062 944 751 882, f) 2 512 339 789 576 516? 193. [3] Miért nem lehet egy négyzetszám jegyeinek összege 1991? 194. [2] Van-e olyan négyzetszám (köbszám stb.), melynek első négy számjegye 1991 (ebben a sorrendben)? 195. [4] Adott egy 99 jegyű, 9-esekből álló szám. Írhatunk-e mögé 100 számjegyet úgy, hogy a kapott 199-jegyű szám négyzetszám legyen? 196. [2] Van-e minden n-re n-jegyű négyzetszám? 197. [2] Mutassuk meg, hogy négyzetszám 16-tal osztva mindig négyzetszám maradékot ad. 198. [4] Mutassuk meg, hogy ha egy pozitív egészekből álló, végtelen sok elemű számtani sorozat elemei közt van egy négyzetszám (köbszám stb.), akkor végtelen sok négyzetszám (köbszám stb.) van. 199. [3] Adjunk meg különböző pozitív egészekből álló végtelen számtani sorozatot, amelynek elemei közt nincs négyzetszám. 200. [4] Adjunk meg végtelen sok természetes számot úgy, hogy közülük akárhányat is választunk, azok összege nem négyzetszám. 201. [4] Mutassuk meg, hogy végtelen sok olyan természetes szám van, mely nem állítható elő a) két négyzetszám, illetve b) két köbszám összegeként.

26

Feladatok

202. [3] Mutassuk meg, hogy 10 bármely pozitív egész kitevős hatványa előállítható két négyzetszám összegeként. 203. [3] Mutassuk meg, ha az a, b, c pozitív egész számok olyanok, hogy a · b és b · c is négyzetszám, akkor a · c értéke is négyzetszám. 204. [3] Megoldhatók-e az egész számok körében a következő egyenletek: a) x 2 + y 2 = 1987, b) x 2 + y 2 = 1986, c) x 2 + y 2 = 1989, d) x 2 − y 2 = 1989, e) x 2 − y 2 = 1990? 205. [3] Mutassuk meg, hogy ha x is, y is a) a 2 + b2 , illetve b) a 2 + 2b2 alakú, akkor xy is ilyen alakú. 206. [4] Mutassuk meg, hogy x akkor és csak akkor a 2 + b2 alakú, ha a) 4x, illetve b) 2x is ilyen alakban írható. 207. [4] Mutassuk meg, hogy x akkor és csak akkor a 2 +2b2 alakú, ha 3x is ilyen alakban írható. 208. [5] Tekintsük az a 2 + 5b2 alakú számok M halmazát, ahol a és b nemnegatív egész számok. a) Bizonyítsuk be, hogy ha x és y eleme az M halmaznak, akkor xy is eleme az M halmaznak. b) Alapszámoknak nevezzük azokat az M-beli számokat, melyek nagyobbak 1-nél és nem oszthatók M-beli számokkal (1-et és önmagukat kivéve). Van-e olyan M-beli szám, mely két különböző módon állítható elő alapszámok szorzataként? c) Bizonyítsuk be, hogy végtelen sok alapszám van. 209. [5] Bizonyítsuk be, hogy ha két egész szám összege a 2 + 3b2 alakú, ahol a és b egész számok, akkor a köbösszegük is ilyen. 210. [6] Nevezzük az n számot érdekesnek, ha előáll 3x 2 + 32y 2 alakban, ahol x és y egész számok. Mutassuk meg, hogy ha n érdekes szám, akkor 97n is érdekes szám.

5. Négyzetszámok

27

211. [5] Az x, y, z pozitív egész számok legnagyobb közös osztója 1, továbbá

1 1 1 + = . x y z

Bizonyítsuk be, hogy x + y négyzetszám. 212.

[5]

Bizonyítsuk be, hogy ha az x, y egészekre teljesül a 2x 2 +x = 3y 2 +y egyenlőség, akkor 2x + 2y + 1 és 3x + 3y + 1 négyzetszámok.

213. [5] Legyen N olyan 16-jegyű pozitív egész szám, amelynek jegyei között a 0, 1, 4, 9 nem fordul elő. Bizonyítsuk be, hogy N-nek van néhány olyan egymást követő számjegye, amelyek szorzata négyzetszám. Igaz-e a feladat állítása a fenti típusú 15-jegyű számokra is? 214. [4] Mutassuk meg, hogy 12 egymást követő egész szám négyzetének összege nem osztható ezeknek a számoknak az összegével. 215. [4] Milyen pozitív egész n-ekre lesz négyzetszám a) n2 − n + 2, b) n3 − n + 2, c) n4 − n + 2, d) n5 − n + 2? 216. [4] Mutassuk meg, hogy a következő számok négyzetszámok. a) 999 . . . 9 000 . . . 0 25 m) 1 777 . . . 7 92 888 . . . 8 921   n

n

n

b) 999 . . . 9 8 000 . . . 0 1   n

n

n

c) 111 . . . 1 222 . . . 2 5   n

n+1

n

n

r) 255 999 . . . 9 68 000 . . . 0 1

n

t) 783 999 . . . 9 44 000 . . . 0 1

n

n

n−1 n

n

l) 1 777 . . . 7 6 888 . . . 8 9     n

n

v) 111 . . . 1 ·1 000 . . . 0 5 + 1   n

n

k) 224 999 . . . 9 1 000 . . . 0 9     n−2

u) 575 999 . . . 9 52 000 . . . 0 1 n−1

j) 7 111 . . . 1 2 888 . . . 8 9     n

n

n−1

i) 1 777 . . . 7 9 555 . . . 5 6    

n

s) 143 999 . . . 9 76 000 . . . 0 1

n

n

n

n−1

h) 2 777 . . . 7 888 . . . 8 9     n−1

n

n−1

g) 5 444 . . . 4 7 555 . . . 5 6     n

n

q) 63 999 . . . 9 84 000 . . . 0 1

n−1

f) 444 . . . 4 6 222 . . . 2 4   n

n

p) 15 999 . . . 9 2 000 . . . 0 1

n−1

e) 111 . . . 1 555 . . . 5 6   n

n

o) 7 111 . . . 1 0 222 . . . 2 5 n−2

d) 444 . . . 4 888 . . . 8 9    

n−1

n) 3 999 . . . 9 6 000 . . . 0 1

n−1

w) 444 . . . 4 −11 · 444 . . . 4 +9     2n

n

Feladatok

28

217. [4] Igazoljuk az alábbi összefüggéseket. . . . 3 = 11 . . . 1 0 99  . . . 9 88 . . . 8 9 a) 33  . . . 3 · 33       m

n

m−n

n−1

n−1

. . . 6 = 44 . . . 4 3 99  . . . 9 55 . . . 5 6 b) 66 . . . 6 · 66         m

n

m−n

n−1

n−1

c) 99  . . . 9 · 99 . . . 9 = 99 . . . 9 8 99  . . . 9 00 . . . 0 1       m

n

m−n

n−1

n−1

. . . 6 = 22 . . . 2 1 99  . . . 9 77 . . . 7 8 d) 33 . . . 3 · 66         m

n

m−n

n−1

n−1

e) 33  . . . 3 · 99 . . . 9 = 33 . . . 3 2 99  . . . 9 66 . . . 6 7       m

n

m−n

n−1

n−1

f) 66  . . . 6 · 99 . . . 9 = 66 . . . 6 5 99  . . . 9 33 . . . 3 4       m

n

m−n

n−1

n−1

218. [4] Igazoljuk az alábbi összefüggéseket. a) (666 . . . 6)2 + 888 . . . 8 = 444 . . . 4          n 2n n  b) 111 . . . 1 − 222 . . . 2 = 333 . . . 3       2n

219.

[4]

n

n

Az A számot úgy képezzük, hogy a 16 számjegyei közé 2k + 1 db 7-est írunk; a B számot pedig úgy kapjuk, hogy a 32 számjegyei közé k db 5-öst írunk. Mutassuk meg, hogy A − B négyzetszám!

6. Két szomszédos egész szám szorzata 220. [2] Lehet-e 1! + 2! + 3! + ... + 100! két egymást követő egész szám szorzata? 221. [3] Lehet-e 3100 + 1 két egymást követő egész szám szorzata? 222. [3] Lehet-e 100! + 4 két egymást követő egész szám szorzata? 223. [3] Lehet-e 2(6n + 1) két egymást követő egész szám szorzata? 224. [3] Mutassuk meg, hogy bármely egész n-re 3n3 + 2n2 + n + 1 nem lehet két szomszédos egész szorzata. 1 225. [3] Mutassuk meg, hogy minden egész n-re (n4 + 2n3 + 3n2 + 2n) két szomszédos 4 egész szorzata. 226. [3] Mutassuk meg, hogy minden egész n-re 2(9n + 9n−1 + · · · + 9 + 1) két szomszédos egész szorzata.

7. Diofantoszi egyenletek

29

227. [3] Bizonyítsuk be, hogy bármely nemnegatív n egészre n2 + 7n + 8 nem lehet két szomszédos egész szorzata. Mi a válasz negatív egész n számok esetén? 228. [3] Mely egész n-ekre lesz n2 + 2n + 12 két szomszédos egész szorzata? 229. [4] Mely egész n-ekre lesz 15-tel osztható n2 + n + 2? 230. [4] Igazoljuk, hogy n2 + n + 1 egyetlen egész n-re sem többszöröse 1990-nek. 231. [4] Oldjuk meg a 6x = y 2 + y − 2 egyenletet az egész számok körében. 232. [4] A 2, 3, 4, 5, 6, 7, 8, 9, 0 számok valamilyen sorrendjével fel lehet-e írni olyan kilencjegyű számot, mely két szomszédos egész szorzata? 233. [4] Mutassuk meg, hogy 9n + 2 pontosan akkor két szomszédos egész szorzata, ha n is ilyen. 234. [4] Mutassuk meg, hogy bármely egész n-re és természetes k-ra 2n3k + 4nk + 10 nem lehet két szomszédos egész szorzata. 235. [4] Mutassuk meg, hogy a következő számok mindegyike két szomszédos egész szorzata. a) 8999 . . . 91000 . . . 02 (a számban ugyanannyi 9 és 0 van) b) 111 . . . 1222 . . . 2 (a számban ugyanannyi 1 és 2 van)

7. Diofantoszi egyenletek Mutassuk meg, hogy a következő egyenletek nem oldhatók meg a pozitív egészek körében. (236–269. feladatok) 236. [2] ab(a + b) = 1993. 237. [2] a 2 + b2 + a + b = 1993. 238. [2] nk+1 = (n + 1)k . 239. [2] 1992x + 1993y = 1994z . 240. [2] x 7 + 45x − 1993 = 0. 241. [2] n4 + n3 + n2 + n = 3100 . 242. [2] a 5 b = ab5 + 1993. 243. [2] a 2 − 2ab = 1994. 244. [2] 6x − 15y = 14. 245. [3] a 2 − b2 + c2 = 100,

5 | abc.

30

246. [3] a 2 − 3b = 17. 247. [3] 3a 2 − 4b2 = 13. 248. [3] a 3 − a = 3b2 + 1. 249. [3] a 2 + 4a − 8b = 11. 250. [3] a 3 + 2 = 4b(b + 1). 251. [3] a 2 − 2b2 = 11. 252. [3] a 2 − 2b2 + 8c = 3. 253. [4] a 2 + b2 + c2 = 73 . 254. [4] a 3 + b3 + c3 = 9abc + 4. 255. [4] 19a 3 − 17b3 = 50. 256. [3] a 2 + b2 = 13013. 257. [4] a 3 + b3 = 1991. 258. [4] a 3 + b3 + c3 = 1993. 259. [4] a 3 + b3 + c3 = 19872 . 4 260. [4] a14 + a24 + · · · + a14 = 1599.

261. [4] 19a 3 − 84b2 = 1984. 262. [4] a 2 − 2b2 + 363c2 = 77. 263. [4] a 2 + 9b2 − 2c2 = 3. 264. [3] 5n + 11n = 12n . 265. [3] x 2 + y 2 = 2x + 2y − 3. 266. [3] x 4 − 2x 2 y + 3y 2 + 2 = 0. 267. [4] a 2 − 2ab + 2b2 − 4b3 = 0. 1 1 1 + = 1. 268. [3] 2 + a ab b2 1 1 1 1 3 269. [3] 2 + 2 + 2 + 2 = , ahol a, b, c, d különböző egész számok. a b c d 2

Feladatok

7. Diofantoszi egyenletek

31

Oldjuk meg a következő egyenleteket az egész számok körében. (270–297. feladatok) 270. [2] ab = a + b. 271. [2] ab + a + b = 6. 272. [2] ab + 2a + 3b = 36. 273. [2] abc + ab + bc + ca + a + b + c = 1000. 1 1 1 274. [2] + = . a b 7 1 1 1 1 275. [2] + − = . a b ab 7 276. [2] 2a 3 + ab − 7 = 0. 277. [2] ab + 3a − 5b + 3 = 0. 278. [2] a 2 − b2 = 100. 279. [2] a 2 − 4b2 = 116. 280. [2] 2a 2 + 5ab − 12b2 = 28. 281. [3] a(b + 1)2 = 243b. 282. [3] a 2 = b2 + 2b + 13. 283. [3] a 2 = b3 + 1. 284. [3] a 2 + b2 + c2 = ab + bc + ca. 285. [4] a 2 − ab + b2 = a + b. 286. [3] a 3 + b3 − 3a 2 + 6b2 + 3a + 12b + 6 = 0. 287. [2] (n! + 1) · (m! + 1) = (n + m)!, n ≥ 1, m ≥ 1. 288. [3] 1! + 2! + 3! + ... + n! = m2 , n ≥ 0. 289. [3] abc = a! + b! + c!, a > 0, b > 0, c > 0. 290. [3] a + b + c = 1, a 3 + b3 + c3 = 1 és abc = −16. 291. [4] x 3 − 5x 2 + 8x − 6 = 0. 292. [4] a 3 + (a + 1)3 = (a + 2)3 . 293. [4] a 3 + (a + 1)3 + (a + 2)3 = (a + 3)3 . 294. [3] abc = a + b + c, a > 0, b > 0, c > 0. 295. [3] abc = ab + bc + ca, a > 0, b > 0, c > 0.

32

Feladatok

296. [3] abc = 2(ab + bc + ca), a > 0, b > 0, c > 0. 1 1 1 1 = 2, a > 0, b > 0, c > 0. 297. [3] + + + a b c abc 298. [2] Egy téglalap kerületének és területének mérőszáma megegyezik, oldalainak mérőszáma egész szám. Mekkora a területe? 299. [2] Melyek azok a kétjegyű számok, amelyek 17-tel nagyobbak számjegyeik szorzatánál? 300. [3] Három prímszám szorzata ötszöröse összegüknek. Melyek ezek a prímek? 301. [4] Bizonyítsuk be, hogy sem 1994, sem 1995 nem lehet az egész együtthatós ax 2 + +bx + c = 0 másodfokú egyenlet diszkriminánsa. Lehet-e 1996? 302. [4] Mutassuk meg, hogy ha az a, b, c nemnegatív egészekre 28a + 30b + 31c = 365 teljesül, akkor a + b + c = 12. Oldjuk meg az egyenletet. 303. [4] Mutassuk meg, hogy az [x]+[2x]+[4x]+[8x]+[16x]+[32x] = 12345 egyenletnek nincs megoldása. 304. [5] Megoldható-e az egész számok körében az x 2 + 3xy − 2y 2 = 122 egyenlet? 305. [5] Oldjuk meg az egész számok körében az a + b + c = 7, a 3 + b3 + c3 = 1 egyenletrendszert. 306. [5] Oldjuk meg az egész számok körében az a + b + c = 31, a 2 + b2 + c2 = 325 egyenletrendszert. 307. [5] Mutassuk meg, hogy az a 2 + b2 = c2 egyenletnek végtelen sok különböző megoldása van az egész számok körében, melyre (a, b, c) = 1. 308. [5] Mutassuk meg, hogy az a 2 + b2 = c3 egyenletnek végtelen sok különböző megoldása van az egész számok körében, melyre (a, b, c) = 1. 309. [5] Mutassuk meg, hogy az a 2 + b2 = c4 egyenletnek végtelen sok különböző megoldása van az egész számok körében, melyre (a, b, c) = 1. 310. [4] Mutassuk meg, hogy az a 2 = 2b2 egyenlet nem oldható meg a nullától különböző egészek körében. 311. [4] Mutassuk meg, hogy az a 2 = 7b2 egyenlet nem oldható meg a nullától különböző egészek körében. 312. [4] Mutassuk meg, hogy a 6(a 2 + b2 ) = c2 + d 2 egyenlet nem oldható meg a nullától különböző egészek körében. 313. [4] Mutassuk meg, hogy a 7(a 2 + b2 ) = c2 + d 2 egyenlet nem oldható meg a nullától különböző egészek körében. 314. [4] Mutassuk meg, hogy az a 2 + b2 = 3c2 egyenlet nem oldható meg a nullától különböző egészek körében.

8. Prímszámok

33

315. [4] Mutassuk meg, hogy az a 3 + 3b3 + 9c3 = 9abc egyenlet nem oldható meg a nullától különböző egészek körében. 316. [4] Mutassuk meg, hogy az a 3 − 2b3 − 4c3 = 0 egyenlet nem oldható meg a nullától különböző egészek körében. 317. [4] Mutassuk meg, hogy az a 2 + b2 + c2 = a 2 b2 egyenlet nem oldható meg a nullától különböző egészek körében. 318. [4] Mutassuk meg, hogy az a 2 +b2 +c2 = 2abc egyenlet nem oldható meg a nullától különböző egészek körében. 319. [4] Mutassuk meg, hogy az a 2 + b2 + c2 + d 2 = 2abcd egyenlet nem oldható meg a nullától különböző egészek körében. 320. [6] Mutassuk meg, hogy az a 4 + b4 = c4 egyenlet nem oldható meg a nullától különböző egészek körében. 321. [5] Mutassuk meg, hogy az a 2 + b2 + c2 + a + b + c = 1 egyenlet nem oldható meg a racionális számok körében. 322. [5] Van-e az a 2 + ab + b2 = 2 egyenletnek racionális számokból álló megoldása?

8. Prímszámok 323. [1] Páros-e vagy páratlan az első 100 prímszám összege? 324. [1] Adjunk meg két egész számot, melyek összege is, szorzata is prím. 325. [1] Három egész szám számtani sorozatot alkot, s a három szám szorzata prímszám. Melyek ezek a számok? 326. [1] Adjunk meg két olyan p és q prímszámot, melyekre igaz, hogy p + q és p − q is prím. 327. [1] Miért nem lehet két prím összege 1991? 328. [1] Három prímszám összege 1992. E három prímszám közül melyik a legkisebb? 329. [2] Van-e három olyan prímszám, amelyek összege 1234, szorzata 87654321? 330. [4] Két prímszám különbsége 100. A tízes számrendszerbeli alakjukat egymás után írva, egy újabb prímszámot kapunk. Melyek ezek a számok? 331. [1] Van-e hat egymást követő prímszám, melyek összege is prím? 332. [1] Miért nincs olyan prímszám, mely után pontosan 10 összetett szám következik? 333. [2] Lehet-e 15 egymást követő egész szám összege prímszám?

Feladatok

34

334. [2] Lehet-e 16 egymást követő egész szám összege prímszám? 335. [2] Lehet-e 3 × 3-as bűvös négyzetet készíteni az első kilenc prímszámból? 336. [2] Oldjuk meg a prímszámok körében a 2x + 3y + 6z = 78 egyenletet. 337. [3] Oldjuk meg a prímszámok körében a pq + 1 = r egyenletet. 338. [3] Oldjuk meg a prímszámok körében a p2 − 2q 2 = 1 egyenletet. 339. [3] Oldjuk meg a prímszámok körében a p2 + q 3 = r 4 egyenletet. 340. [2] Egy apa és két különböző korú kisgyermekének életkora ugyanannak a prímszámnak pozitív egész kitevős hatványai. Egy évvel ezelőtt mindhármuk életkora prímszám volt. Hány évesek most? 341. [3] Egy háromjegyű szám jegyeinek összege egy prím négyzete, a jegyek szorzata egy prím köbe. Melyik ez a szám? 342. [3] Van-e olyan n egész szám, melyre 2n − 1 és 2n + 1 is prímszám? 343. [3] Legyen p > 3 prímszám. Mutassuk meg, hogy 3 | p2 − 1, valamint 24 | p2 − 1. 344. [3] Lehetnek-e az n + 5, n + 7 és n + 15 számok egyszerre prímszámok, ahol n egész számot jelöl? 345. [3] Mely p prímszámra lesz a) p + 5 és p + 10, b) p + 10 és p + 14, c) 8p − 1 és 8p + 1, d) 8p2 − 1 és 8p2 + 1, e) p2 + 2, f) 14p2 + 1, illetve g) 2p + 1, 3p + 2, 4p + 3 egyszerre prímszám?

és 6p + 1

346. [3] Mutassuk meg, hogy ha p és p2 + 8 prímszámok, akkor p2 + p + 1 is prímszám. 347. [3] Mutassuk meg, hogy ha p és 8p2 +1 prímszámok, akkor 8p2 +2p+1 is prímszám. 348. [4] Mely p és q prímszámokra lesz p + q és p2 + q 2 − q is prímszám? 349. [4] Mely p és q prímszámokra lesz pq + q p is prímszám? 350. [4] Mely p és q prímszámokra lesz pq − 1 és pq + 1 is prímszám? 351. [4] Mely p prímszámra lesz 4p + 1 négyzetszám? 352. [4] Legyen p olyan prím, amelyre p2 + p + 1 és p2 − p + 1 is prím. Bizonyítsuk be, hogy ha p4 + p3 + p2 + p + 1 nem prímszám, akkor négyzetszám.

8. Prímszámok

35

353. [4] Határozzuk meg azokat a p prímeket, amelyekre 4p2 +1 és 6p2 +1 is prímszám. 354. [3] Mutassuk meg, hogy ha a p, q és r, s számpárok ikerprímek, és mindegyik nagyobb, mint 3, akkor pr −qs osztható 12-vel. (Két prím iker, ha különbségük 2.) 355. [4] Mutassuk meg, hogy két egymás utáni páratlan prím összege legalább három (nem feltétlenül különböző) prím szorzata. 356. [3] Lehetnek-e valamely n természetes számra prímszámok a következő kifejezések: a) n4 + 2n3 + 2n2 + 2n + 1, b) 6n + 3n + 2n+1 + 2, c) 32n+1 − 22n+1 − 6n , d) 8n2 + 10n + 3, e) 25n4 + 9n2 + 1, f) 4n3 + 6n2 + 4n + 1, g) |n4 + n3 − 2n2 − 3n − 3|? 357. [4] Adjunk meg olyan, 1-nél nagyobb egész számot, mely nem osztható a 2, 3, 5, 7, . . . , p prímek egyikével sem. 358. [4] A következő polinomok néhány egymást követő természetes számra prímszámot adnak értékül: n2 + n + 41, n = 0, 1, 2, . . . , 39 (Euler, 1772), n2 − 79n + 1601, n = 0, 1, 2, . . . , 79, n2 − 5n + 47, n = 0, 1, 2, . . . , 42, 2n2 − 2n + 19, n = 0, 1, 2, . . . , 18, 3n2 − 3n + 23, n = 0, 1, 2, . . . , 22, 8n2 − 326n + 2659, n = 0, 1, 2, . . . , 39, 36n2 − 810n + 2753, n = 0, 1, 2, . . . , 44. Keressünk további ilyen polinomokat — számítógéppel. 359. [4] Keressünk — számítógéppel — különböző prímszámokból álló 3 × 3-as bűvös négyzetet. 360. [4] Miért nincs olyan egész együtthatós polinom, mely az x változó minden egész értékére prímszámot vesz fel értékül? 361. [4] Miért nincs csupa különböző prímszámból álló végtelen számtani sorozat? 362. [3] Keressünk 200-nál kisebb prímekből álló — minél több elemű — számtani sorozatot. 363. [3] Keressünk 100-nál kisebb, páronként relatív prímekből álló — minél több elemű — számtani sorozatot.

Feladatok

36

364. [4] Adjunk meg olyan, természetes számokból álló végtelen számtani sorozatot, amelynek egyik eleme sem állítható elő két prím összegeként. 365. [3] Igaz-e, hogy bármely természetes szám egy jegyének megváltoztatásával prímszámmá alakítható? 366. [4] Tudjuk, hogy az a, b, c, d természetes számokra ab = cd. Mutassuk meg, hogy az a 2000 + b2000 + c2000 + d 2000 szám összetett szám. 367. [4] Legyen p 1-nél nagyobb egész szám. Bizonyítsuk be, hogy p pontosan akkor prímszám, ha p bármely négy pozitív egész szám összegére való felbontásában semelyik két tag szorzata sem egyenlő a másik két tag szorzatával. 368. [6] Mutassuk meg, hogy a prímszámok száma végtelen.

9. Oszthatósági feladatok 369. [2] Igazoljuk a következő oszthatóságokat: a) 9 | 1033 + 8, b) 6 | 1010 + 14, c) 72 | 1020 + 8. 370. [2] Határozzuk meg a felírt szám hiányzó számjegyeit úgy, hogy teljesüljön az oszthatóság. Keressük meg az összes megoldást. a) 36 | 52x2y, b) 72 | x378y, c) 45 | 24x68y, d) 48 | 24x68y, e) 99 | 62xy427. 371. [2] Írjunk az 1994 elé is, után is egy-egy számjegyet úgy, hogy a kapott hatjegyű szám osztható legyen a) 88-cal; b) 99-cel. 372. [1] Igazak-e a következő állítások: a) Ha egy szám osztható 6-tal és 8-cal, akkor osztható 48-cal is. b) Ha egy szorzat osztható 6-tal, akkor valamelyik tényezője osztható 6-tal.

9. Oszthatósági feladatok

37

373. [2] Mutassuk meg, hogy a következő számok összetett számok. a) 106 − 57 , b) 10100 − 7, c) 420 − 1, d) 1 000 027, e) 1000 . . . 001 (1991 db 0), f) 111 . . . 111 (1989 db 1-es), g) 111 . . . 111 (1990 db 1-es), h) 111 . . . 121 . . . 111 (a 2 előtt is, után is ugyanannyi db 1-es), i) 10300 − 2 · 10100 + 1, j) 1! + 2! + 3! + · · · + 100!. 374. [3] Mutassuk meg, hogy 347 777 743 összetett szám. 375. [3] Mutassuk meg, hogy a 49 + 610 + 320 szám összetett szám. 376. [3] Mutassuk meg, hogy a 210 + 512 szám összetett szám. 377. [4] Mutassuk meg, hogy az 53 · 83 · 109 + 40 · 66 · 96 szám összetett szám. 378. [4] Mutassuk meg, hogy a 989 · 1001 · 1007 + 320 szám összetett szám. 379. [4] Mutassuk meg, hogy az 5123 + 6753 + 7203 szám összetett szám. 380. [4] Az a és b természetes számokra teljesül a 34a = 43b összefüggés. Mutassuk meg, hogy ekkor a + b összetett szám. 381. [3] Mutassuk meg, hogy minden, 0-tól különböző n természetes számhoz van olyan m természetes szám, melyre n · m + 1 összetett szám. 382. [4] Mutassuk meg, hogy 19·8n +17 az n minden nemnegatív egész értékére összetett szám. 383. [1] Melyik az a legkisebb természetes szám, mely osztható az 1, 2, 3, 4, 5, 6, 7, 8, 9, 10 számok mindegyikével? 384. [1] Mutassuk meg, hogy 9 egymást követő egész szám összege osztható 9-cel. 385. [1] Mutassuk meg, hogy 10 egymást követő egész szám összege sohasem osztható 10-zel. 386. [1] Öt egymást követő egész számot összeszorzunk. Milyen jegyre végződik az így kapott szám? 387. [1] Öt egymást követő páratlan számot összeszorzunk. Milyen jegyre végződik az így kapott szám? 388. [1] Négy egymást követő pozitív egész szorzata 3024. Melyek ezek a számok?

Feladatok

38

389. [2] Adjuk meg 15-nek az első három olyan pozitív többszörösét, melyben a számjegyek összege 15. 390. [3] Adjuk meg 45 legkisebb olyan pozitív többszörösét, melyben csak 0 és 8-as számjegyek vannak. 391. [2] Adjuk meg 1991 legkisebb olyan pozitív többszörösét, mely 1992-re végződik. 392. [2] Miért nem lehet egy természetes szám számjegyeinek szorzata 111? 393. [3] Miért nem lehet egyszerre egész

n+1 15

és

n+8 , n ∈ N? 21

394. [3] Miért nem lehet az 1, 2, 3, . . . , 10 számokat két csoportra osztani úgy, hogy a két csoportban levő számok a) összege vagy b) szorzata egyenlő legyen? 395. [3] Miért nem lehet 5 egymást követő természetes számot két csoportba osztani úgy, hogy a két csoportban levő számok szorzata egyenlő legyen? 396. [5] Miért nem lehet 6 egymást követő természetes számot két csoportba osztani úgy, hogy a két csoportban levő számok szorzata egyenlő legyen? 397. [2] Írjuk fel az 1, 2, 3, 4, 5, 6 számjegyek valamilyen sorrendjével a legnagyobb, 12-vel osztható számot. 398. [2] Írjuk fel azt a legkisebb 9-jegyű számot, melynek első 2 jegyéből álló szám osztható 2-vel, az első 3 jegyből álló szám osztható 3-mal, . . . , az első 8 jegyből álló szám osztható 8-cal, és maga a szám osztható 9-cel. 399. [3] Írjunk fel olyan, az 1, 2, . . . , 6 számjegyek valamilyen sorrendjéből álló 6-jegyű számot, melynek első 2 jegyéből álló szám osztható 2-vel, az első 3 jegyből álló szám osztható 3-mal, az első 4 jegyből álló szám osztható 4-gyel, az első 5 jegyből álló szám osztható 5-tel, és maga a szám osztható 6-tal. 400. [4] Írjuk fel azt az 1, 2, . . . , 9 számjegyek valamilyen sorrendjéből álló 9-jegyű számot, melynek első 2 jegyéből álló szám osztható 2-vel, az első 3 jegyből álló szám osztható 3-mal, . . . , az első 8 jegyből álló szám osztható 8-cal, és maga a szám osztható 9-cel. 401. [3] Írjuk fel azt az 1, 2, . . . , 9 számjegyek valamilyen sorrendjéből álló legkisebb 9-jegyű számot, mely osztható 99-cel. 402. [5] Határozzuk meg a 999-cel osztható, 9-es számjegyet nem tartalmazó legkisebb pozitív egész számot. 403. [4] Adjunk meg olyan 7-jegyű számot, melynek számjegyei különbözők és a szám osztható mindegyik számjegyével.

9. Oszthatósági feladatok

39

404. [4] Határozzuk meg azokat a 4-jegyű, 9-re végződő számokat, amelyek oszthatók számjegyeik mindegyikével. 405. [4] Keressük meg azt a legnagyobb számot, amelyre teljesül, hogy az első n számjegyéből álló n-jegyű szám osztható az n-edik prímszámmal, n = 1, 2, 3, . . . , N esetén, ahol N a keresett szám számjegyeinek száma. 406. [5] Melyik az az ötjegyű szám, amelyik egyenlő számjegyei szorzatának 45-szörösével? 407. [5] Melyik az a legkisebb, 7-esekből és 3-asokból álló szám, melyben a számjegyek összege s maga a szám is osztható 7-tel és 3-mal? 408. [4] Írjunk fel 6 db 2-es és 2 db 1-es segítségével 7-tel osztható 8-jegyű számot. 409. [4] Melyik az a legkisebb, 28-cal osztható pozitív egész szám, amelynek a 10-es számrendszerbeli alakja 28-ra végződik, és számjegyeinek összege 28? 410. [4] Melyik az a legkisebb, 56-tal osztható pozitív egész szám, amelynek a 10-es számrendszerbeli alakja 56-ra végződik, és számjegyeinek összege 56? 411. [4] Adjunk meg két olyan 6-jegyű számot, melyeket egymás után írva olyan 12jegyű számot kapunk, amely osztható a két 6-jegyű számmal. 412. [4] Meg lehet-e adni két ötjegyű számot úgy, hogy azok mindegyik számjegye páratlan legyen, és a két szám szorzataként kapott számban is minden számjegy páratlan? 413. [4] Határozzuk meg a legkisebb pozitív egész számot, amely legalább 600-szor akkora, mint bármelyik prímosztója. 414. [5] Egy ötjegyű számot nevezzünk felbonthatatlannak, ha a szám nem áll elő két háromjegyű szám szorzataként. Legfeljebb hány egymást követő felbonthatatlan szám van? 415. [5] 49 pozitív egész szám összege 999. Határozzuk meg ezen számok legnagyobb közös osztójának lehetséges legnagyobb értékét. 416. [5] 10 pozitív egész szám összege 1001. Határozzuk meg ezen számok legnagyobb közös osztójának lehetséges legnagyobb értékét. Bizonyítsuk be a következő oszthatóságokat. (417–440. feladatok) 417. [2] 5 | 119 + 118 + 117 + · · · + 11 + 1. 418. [3] 100 | 1110 − 1. 419. [2] 200 | 1993 − 199. 420. [2] 13 | 270 + 370 . 421. [2] 13 | 31974 + 51974 .

Feladatok

40

422. [3] 1976 | 19751975 − 19771977 + 1978. 1977

423. [3] 19771977 + 1 | 1977(1977 424.

[3]

18 | 17 + 19 19

17

)

+ 1.

(Hogyan lehetne általánosítani?).

425. [4] 1897 | 2903n − 803n − 464n + 261n , 426.

[4]

n ∈ N.

1946 | 1492 − 1770 − 1863 + 2141 , n

n

n

n

427. [4] 1957 | 17212n − 732n − 5212n + 2122n , 428.

[4]

1958 | 2225 − 2082 + 1815 , n

n

n ∈ N. n ∈ N.

n ∈ N , n ≥ 1.

n

429. [4] 1995 | 2801n − 2696n − 2269n + 169n ,

n ∈ N.

430.

[4]

5 | 1 + 2 + 3 + 4 pontosan akkor, ha 4 - n, n ∈ N .

431.

[3]

18 | 22n + 24n − 10, n > 0, n ∈ N .

n

n

n

n

432. [3] 64 | 32n+3 − 24n + 37, n ∈ N . 433. [3] 11 | 32n+2 + 26n+1 , n ∈ N . 434. [3] 16 | 5n − 4n − 1, n ∈ N . 435. [3] 36 | 7n − 6n − 1, n ∈ N (Hogyan lehetne általánosítani?). 436. [3] 4003 | 1 · 2 · . . . · 2001 + 2002 · 2003 · . . . · 4002. 437. [7] 71 | 61! + 1. 438. [7] 71 | 63! + 1. 439. [4] 7 | 22225555 + 55552222 . 440. [4] 19922 + 1993 | 19921993 − 1992. 441. [3] Mutassuk meg, hogy ha 2n + 1 prímszám, akkor n = 2k (n, k ∈ N ). 442. [3] Mutassuk meg, hogy ha 2n − 1 prímszám, akkor n is prímszám (n ∈ N , n > 1). 443. [4] Mutassuk meg, hogy ha m páratlan, akkor (2m − 1, 2n + 1) = 1. n

m

444. [4] Mutassuk meg, hogy (22 + 1, 22 + 1) = 1. 445. [5] Mutassuk meg, hogy ha m > 2, akkor 2n + 1 sohasem osztható (2m − 1)-gyel. 2n − 2 22 −1 − 2 egész szám, akkor is egész szám. n 2n − 1 n

446. [5] Mutassuk meg, hogy ha

447. [6] Legyen p > 3 prímszám és n =

22p − 1 . Mutassuk meg, hogy n|2n − 2. 3

448. [3] Adott néhány egész szám, melyek összege osztható 6-tal. Mutassuk meg, hogy ekkor a számok köbeinek összege is osztható 6-tal.

9. Oszthatósági feladatok

41

449. [4] Mutassuk meg, hogy a következő oszthatóságok sohasem teljesülnek. a) n + 4 | n2 + 8n + 15, n ∈ N , b) 121 | n2 + 3n + 5, n ∈ N . 450. [4] Igazoljuk, hogy x 5 + 3x 4 y − 5x 3 y 2 − 15x 2 y 3 + 4xy 4 + 12y 5 semmilyen x, y értékre nem veszi fel értékül a 33-at. 451. [3] Határozzuk meg 1989 + 8189 utolsó két számjegyét. 452. [2] Igaz-e, hogy ha egy szám számjegyeinek összege osztható 27-tel, akkor a szám is osztható 27-tel? 453. [4] Tudjuk, hogy az a, b, c egész számokra a + b + c = (a − b)(b − c)(c − a). Mutassuk meg, hogy ekkor a + b + c osztható 27-tel. 454. [3] Mutassuk meg, hogy 111 . . . 111 (81 db 1-es) osztható 81-gyel. 455. [4] Osztható-e 1980-nal a 19 20 21 22 23 . . . 78 79 80 szám? 456. [3] Mutassuk meg, hogy ha egy szám osztható 99-cel, akkor a szám jegyeinek összege legalább 18. 457. [3] Mutassuk meg, hogy a 37, 537, 5537, 55537, 555537, . . . sorozatban végtelen sok összetett szám van. 458. [3] Mutassuk meg, hogy a 13, 149 13, 149 149 13, 149 149 149 13, . . . sorozatban végtelen sok összetett szám van. 459. [4] Mutassuk meg, hogy a 7, 37, 337, 3337, 33 337, . . . sorozatban végtelen sok összetett szám van. 460. [4] Mutassuk meg, hogy a 31, 331, 3331, 33 331, . . . sorozatban végtelen sok összetett szám van. √   461. [6] Mutassuk meg, hogy a 2k · 2 k = 1, 2, . . . számok között végtelen sok összetett szám van. 462. [3] Mutassuk meg, hogy ha egy tetszőleges háromjegyű számot a szám után újra leírunk, akkor az így kapott abcabc alakú hatjegyű szám osztható 13-mal. 463. [3] Mutassuk meg, hogy ha 27 | abc, akkor 27 | bca. 464. [4] Felírtunk egy kör kerületére 1989 db számjegyet. Tegyük fel, hogy egy számjegytől kezdve egy irányban végigolvasva, a kapott 1989-jegyű szám osztható 27-tel. Mutassuk meg, hogy ez esetben bárhonnan kezdve az olvasást, a kapott szám osztható lesz 27-tel. 465. [3] Mutassuk meg, hogy egy 41-gyel osztható ötjegyű szám első számjegyét áttéve utolsó számjegynek, az így kapott ötjegyű szám is osztható lesz 41-gyel.

Feladatok

42

466. [4] Mutassuk meg, hogy egy 7-tel osztható hatjegyű szám utolsó jegyét elsőnek írva, az így kapott hatjegyű szám is osztható lesz 7-tel. 467. [4] Bizonyítsuk be, hogy ha egy nyolcjegyű szám osztható 101-gyel, akkor a szám jegyeinek valamely más sorrendjével felírt nyolcjegyű szám is osztható lesz 101-gyel. 468. [3] Mutassuk meg, hogy ha 13 | 2a + b és 13 | 5a − 4b, akkor 13 | a − 6b. 469. [3] Mutassuk meg, hogy ha 7 | 100a + b, akkor 7 | a + 4b. 470. [3] Mutassuk meg, hogy 7 | 10a + b pontosan akkor, ha 7 | a − 2b. 471. [3] Mutassuk meg, hogy ha 11 | 3a + 4b, akkor 11 | a + 5b. 472. [3] Mutassuk meg, hogy ha 17 | 5a + 2b, akkor 17 | 9a + 7b. 473. [2] Mely n egészekre lesz egész szám n + 11 , n−9 3n + 5 b) , n+3 a)

c)

n2 + 1 , n+1

d)

n2 + 1992 ? n + 1992

474. [3] Mely n egész szám esetén lehet az

n + 11 törtet egyszerűsíteni? n−9

475. [3] Bizonyítsuk be, hogy a következő törteket egyetlen n egész szám esetén sem lehet egyszerűsíteni. 3n + 1 , 5n + 2 12n + 1 b) , 30n + 2 21n + 4 c) , 14n + 3 a)

d)

n3 + 2n . n4 + 3n2 + 1

10. Különféle számelméleti feladatok

43

476. [3] Az a) 5n + 6 és 8n + 7; b) 7n + 1 és 8n + 3; c) n + 7 és 2n + 3; d) n2 + 1 és (n + 1)2 + 1 számoknak bizonyos n természetes számok esetén van 1-nél nagyobb közös osztója. Mi lehet ez a közös osztó? 477. [3] Mutassuk meg, hogy az 19931994 + 19941993 és az 1993 · 1994 számok relatív prímek. 478. [3] Egy 6-ra végződő szám utolsó jegyét elhagyjuk és ezt a jegyet a szám első jegye elé írjuk. Az így kapott szám négyszerese az eredetinek. Melyik ez a szám? 479. [3] Egy 2-re végződő szám utolsó jegyét elhagyjuk és ezt a jegyet a szám első jegye elé írjuk. Az így kapott szám kétszerese az eredetinek. Melyik ez a szám? 480. [3] Van-e olyan 3-ra végződő szám, melynek utolsó jegyét letörölve, s a szám elé írva olyan számot kapunk, mely az eredeti számnak 3-szorosa? 481. [3] Egy 1-essel kezdődő szám első jegyét elhagyjuk és ezt a jegyet a szám utolsó jegye után írjuk. Az így kapott szám háromszorosa az eredetinek. Melyik ez a szám? 482. [3] Van-e olyan n egész szám, mely elé, ha alkalmasan választott számjegyet írunk, az n szám 58-szorosát kapjuk? Vajon van-e olyan n egész szám, mely elé alkalmasan választott számjegyet írva, az n szám 57-szeresét kapjuk?

10. Különféle számelméleti feladatok 483. [1] Lehet-e három egymást követő egész szám a) összege 1991, illetve b) szorzata 1990? 484. [3] Mutassuk meg, hogy három egymást követő pozitív egész szám szorzata nem lehet köbszám. 485. [3] Bizonyítsuk be, hogy négy egymást követő egész szám mindegyike nem lehet hatványszám. 486. [3] Mutassuk meg, hogy hat egymást követő, 6-nál nagyobb egész között a) van négy összetett szám; b) van olyan, mely relatív prím a többihez.

44

Feladatok

487. [3] Mutassuk meg, hogy 12 egymást követő egész szám között van olyan, mely kisebb, mint valódi osztóinak összege. 488. [3] Néhány egymást követő egész szám összege 100. Melyek ezek a számok? 489. [2] Lehet-e a 0, 1, 2, . . . , 9 számok alkalmas sorrendjével olyan 10-jegyű számot felírni, amely szám 2-nek pozitív egész kitevőjű hatványa? 490. [4] Kártyalapokra felírtuk 11 111-től 99 999-ig az egész számokat (minden lapra egy számot). Ezeket a lapokat valamilyen sorrendben egymás után helyezve a kapott 444 445-jegyű szám lehet-e 2-nek pozitív egész kitevőjű hatványa? 491. [4] Lehet-e egymást követő pozitív egész számok összege 2100 ? 492. [4] Végződhet-e 2 valamelyik hatványa négy egyforma számjegyre? 493. [3] Hány számjegyből áll a 2100 tízes számrendszerben felírt alakja? 494. [4] Valaki a tízes számrendszerben pontosan kiszámította 21996 és 51996 értékét. Hány számjegy van a két számban összesen? 495. [3] Van-e olyan kétjegyű szám, melynek minden hatványa ugyanerre a két jegyre végződik? 496. [3] Van-e olyan 3-jegyű szám, melynek minden hatványa ugyanerre a három jegyre végződik? 497. [2] 13 különböző pozitív egész szám összege 92. Melyek ezek a számok? 498. [3] Melyik az a csupa különböző jegyből álló legnagyobb természetes szám, amelyben bármely három számjegy összege nem osztható 19-cel? 499. [3] Egy matematikaórán a tanár felírt egy számot a táblára. Az egyik diák így szólt: „A szám osztható 31-gyel”. A második: „A szám 30-cal is osztható.” Egy harmadik diák szerint a szám osztható 29-cel, a negyedik szerint 28-cal, és így tovább, végül a harmincadik diák azt mondta, hogy a szám osztható 2vel. A tanár ezek után közölte, hogy csak két állítás nem volt igaz, s ez a kettő egymás után hangzott el. Melyik volt a két téves állítás? 500. [4] A tanár egy 50 000-nél kisebb természetes számot írt fel a táblára. Az első tanuló szerint a szám osztható 2-vel, a második diák szerint osztható 3-mal, . . . , a 12. tanuló szerint osztható 13-mal. Két egymás után megszólaló diák kivételével mindenki igazat mondott. A tanár melyik számot írta fel a táblára? 501. [3] Egy 80 tagú számsorozatról tudjuk, hogy bármely közbülső tagja egyenlő szomszédainak szorzatával. Továbbá az első 40 tag szorzata is 8, valamint összes tagjának szorzata is 8. Határozzuk meg a sorozat első és második elemét. 502. [5] Mutassuk meg, hogy a Fibonacci-sorozat elemei között nem találunk 7k alakú számot.

10. Különféle számelméleti feladatok

45

503. [5] Az a1 , a2 , . . . , an , . . . sorozat elemei a 0, 1, 2 számok. Tudjuk, hogy a sorozat nem periodikus. Legyen bn = 0, ha an = 0, különben pedig bn = 1. Legyen cn = 1, ha an = 2, különben pedig cn = 0. Mutassuk meg, hogy a bn és cn sorozat egyidejűleg nem lehet periodikus. 504. [4] Mutassuk meg, hogy az a1 = 2, an+1 = an2 − an + 1, n = 1, 2, 3, . . . sorozat elemei páronként relatív prímek. 505. [3] Van-e olyan abcd négyjegyű szám, melyre abcd − dcba = 1008? 506. [3] Lehet-e egyenlő egy kétjegyű szám a számjegyeinek szorzatával? 507. [2] Egy számot nevezzünk szépnek, ha egyenlő valódi osztóinak szorzatával (ilyen pl. a 10). Melyik a 10. szép szám? 508. [3] Miért nem lehet köbszám 10150 + 5 · 1050 + 1? 509. [3] Jelöljön n pozitív egész számot. Mutassuk meg, hogy az n + 1 és 8n + 1 számok nem lehetnek egyszerre köbszámok. 510. [3] Mutassuk meg, hogy ha a, b, c olyan pozitív egészek, melyekre ab, bc, ca számok mindegyike köbszám, akkor az a, b, c számok is köbszámok. 511. [3] A 0, 1, 2, 3, 4, 5, 6, 7, 8, 9 számjegyek pontosan egyszeri felhasználásával számokat írtunk fel. Lehet-e 100 ezeknek a számoknak az összege? 512. [3] 7 rabló a zsákmányolt aranyat úgy osztja el, hogy névsor szerint vesznek belőle annyit, amennyi az ott levő aranyak számának számjegyösszege (pl. ha a soron következő zsivány előtt 156 db arany van, akkor ő 1 + 5 + 6 = 12 db-ot vesz el). Két teljes kör után az arany elfogy. Mindenkinek ugyanannyi jutott, csak a bandavezérnek lett több. Hányadik a névsorban a főnök, s mennyi arany jutott neki? 513. [3] Két padon 6-6 gyerek ül. Valamennyien különböző életkorúak (az életkorok egész számok), és az egyik padon ülő gyerekek életkorának összege és szorzata is megegyezik a másik padon ülők életkorának összegével, ill. szorzatával. A legidősebb gyerek 16 éves. Hány évesek azok a gyerekek, akik vele egy padon ülnek? 514. [4] — Nem tudod — kérdezte a lottóhúzás napján egy matematikus a kollégáját —, hogy milyen számokat húztak ki? — Képzeld — felelte az —, van köztük olyan szám, amellyel bármely két kihúzott szám összege osztható! — Mi ez a szám? — Ha megmondanám, kitalálnád a nyerőszámokat. — Legalább azt mondd meg, páros-e vagy páratlan-e ez a szám? — kérte a matematikus, majd a válasz után felkiáltott: — Ötösöm van! Mi volt az öt nyerőszám, ha a telitalálatot elért matematikus csak egy szelvénnyel játszott? (Feltesszük, hogy mindketten jól okoskodtak és persze igazat mondtak.)

46

Feladatok

515. [4] Kovács úr Seholsincs utcában lakik egy házban, amelynek száma 13 és 1300 közötti szám. Horváth úr kíváncsi Kovács úr házának számára, ezért kérdezgetni kezdi. Horváth: „Nagyobb, mint 500?” Kovács válaszol, de hazudik. (Horváth ezt nem tudja.) Horváth: „Négyzetszám?” Kovács válaszol, de hazudik. (Horváth ezt nem tudja.) Horváth: „Köbszám?” Kovács válaszol és igazat mond. Horváth: „Ha most megmondod, hogy a szám utolsó előtti számjegye 1-es vagy sem, akkor meg tudom mondani, hogy hol laksz.” Kovács válaszol. Erre Horváth megmondja az általa helyesnek tartott házszámot, amire Kovács azt mondja: „Tévedtél.” Mi Kovács úr házszáma? 516. [5] Gyermekeim években kifejezett életkorának szorzata 1664. A legfiatalabb legalább fele annyi idős, mint a legidősebb. Én 50 éves vagyok. Hány gyermekem van? 517. [5] X-né dicsekszik a munkatársainak: „Mind a két fiamnak ma van a születésnapja. Egyik sincs még 10 éves. Találjátok ki, hogy mennyi idősek! Anna, neked megsúgom az életkoruk szorzatát.” (Megsúgja.) Anna: „Ebből még nem lehet meghatározni.” „Akkor Borinak az életkoruk hányadosát súgom meg.” (Megsúgja.) Bori: „Még most sem egyértelmű.” „Igazatok van, Borinak a korkülönbséget kellett volna megmondanom, akkor kitalálhatta volna.” Hány évesek X-né fiai? 518. [5] Egy reggel a pap azt mondja a sekrestyésnek: — Ma találkoztam három emberrel. Az években kifejezett életkoruk szorzata egyenlő 2450-nel, összege pedig kétszerese az ön életkorának. Milyen idősek azok az emberek? Délután a sekrestyés bevallja, hogy nem tud válaszolni a kérdésre. Erre a pap kisegíti: — Megjegyzem, hogy a három ember közül az egyik idősebb nálam. Hány éves a pap? 519. [5] „Hányan laknak ebben a házikóban?” — tette fel a kérdést a népszámlálási összeíró. „Hárman.” „Hány évesek?” „Életkoruk szorzata 225, éveik összege azonos a házszámmal.” Az összeíró megnézte a házszámot, majd így felelt: „Nekem ez elég is. Ön a legidősebb?” „Igen” — hangzott a válasz. Határozzuk meg a lakosok életkorát. 520. [5] Egy matematikai kongresszus szünetében történt. Mikor az egyik résztvevő professzortól azt kérdezték kollégái, hány gyereke van és milyen idősek, így felelt: — Három fiam van; a véletlen úgy hozta, hogy mind a háromnak épp ma van a születésnapja. Ha években kifejezett életkorukat összeszorzom, 36-ot kapok; ha viszont összeadom ugyanezt a három számot, akkor pontosan annyit, ahányadika ma van. Kisvártatva így hangzott a viszontválasz:

10. Különféle számelméleti feladatok

47

— Ebből még nem tudhatjuk, hogy hány évesek a gyerekek. — Igaz is, elfelejtettem megmondani: amikor a legkisebb gyerek születését vártuk, a két idősebbet elküldtük vidékre a nagyszüleikhez. — Köszönjük, most már tudjuk mind a három gyerek életkorát. Állapítsuk meg mi is, hány évesek a gyerekek, és azt is, hogy a hónapnak hányadik napján hangzott el ez a beszélgetés. 521. [5] András és Tamás is kiválaszt magának egy-egy pozitív egész számot, s ezt a számot megsúgják Petinek. Peti közli velük, hogy a két szám különbsége 1995. Erre András kijelenti, hogy ő nem tudja megmondani a másik számot, majd Tamás is ugyanezt mondja. Ezek után András kijelenti, hogy most már tudja, milyen számot választott Tamás; de ha mindketten 1-gyel nagyobb számra gondoltak volna, akkor még nem tudta volna kitalálni. Mi volt a két szám? 522. [5] Peti két egymást követő pozitív egész számot választ, ezeket külön-külön felírja egy-egy cédulára. Egyiket Andrásnak, másikat Tamásnak adja oda (akik tudják, hogy a két szám szomszédos), mindketten megnézték, hogy milyen szám van a papírjukra írva, de ezt nem közölték a másikkal. Ezután a következő tartalmas párbeszéd zajlott le közöttük: András: „Én nem tudom, hogy nálad milyen szám van.” Tamás: „Én sem tudom, hogy nálad milyen szám van.” András: „Én nem tudom, hogy nálad milyen szám van.” Tamás: „Én sem tudom, hogy nálad milyen szám van.” . . . András összesen tízszer mondta el, hogy nem tudja, milyen szám van Tamásnál, és Tamás is tízszer válaszolja azt, hogy ő nem tudja, milyen szám van Andrásnál. Ám a tizenegyedik alkalommal András azt mondta: „Most már tudom, hogy milyen szám van nálad.” Milyen szám van Tamásnál? 523. [7] Egy tanár így szól két kedves tanítványához, A-hoz, B-hez: „Gondoltam két, 1nél nagyobb, nem feltétlenül különböző egész számot. A-nak megsúgom a két szám szorzatát, B-nek a két szám összegét. Ez az összeg kisebb 100-nál. Anélkül, hogy egymással beszélnétek, próbáljátok meg kitalálni holnapig a gondolt számokat.” A következő napon a tanítványok így szólnak a tanárhoz: A: „Nem jöttem rá, hogy melyek a gondolt számok.” B: „Én tudtam, hogy nem tudod meghatározni a gondolt számokat.” A: „Akkor már tudom, hogy melyek a gondolt számok.” B: „Már én is tudom.” Melyek voltak a gondolt számok? 524. [3] Határozzuk meg a legkisebb x pozitív számot, amelyre [x] · {x} ≥ 3.

48

Feladatok

525. [3] Mely n-re lesz n pozitív osztóinak összege a) n + 3, b) n + 6? 526. [3] Melyek azok a háromjegyű számok, amelyek pozitív osztóinak száma 5? 527. [3] Melyik az a legkisebb természetes szám, melynek a) 10 pozitív osztója, illetve b) 12 pozitív osztója van? 528. [4] Melyek azok a kétjegyű természetes számok, melyeknek legtöbb osztója van? 529. [4] Tamás felírta egymás után 1-től 2000-ig az egész számokat. Az így kapott N = = 123 . . . 19992000 számról azt állítja, hogy annak 25 323 osztója van. Andris szerint 25 322 osztója van. Bizonyítsuk be, hogy mindketten tévedtek. 530. [4] Melyik az a legkisebb pozitív egész szám, melynek fele négyzetszám, harmada köbszám? 531. [3] Van-e olyan pozitív egész szám, melynek fele négyzetszám, negyede negyedik hatvány? 532. [4] Melyik az a legkisebb pozitív egész szám, melynek kétszerese négyzetszám, háromszorosa köbszám, ötszöröse ötödik hatvány? 533. [3] Van-e olyan n pozitív egész szám, amelyre az n! szám pontosan 11 db nullára végződik? 534. [3] Hány nullára végződik a 100! szám? 535. [4] Határozzuk meg (2n )! prímtényezős alakjában 2 kitevőjét. 100! 536. [4] Mi lesz a 100 tört nevezője az egyszerűsítések után? 6

1000 [3] 537. Osztható-e 7-tel ? 500 538. [4] Van-e olyan n pozitív egész szám, melyre n! osztható 2n -nel? 539. [2] Hány olyan szám van az első 1000 természetes szám között, mely sem 2-vel, sem 3-mal nem osztható? 540. [2] Hány olyan szám van az első 1000 természetes szám között, mely relatív prím a 2, 3, 5 számok mindegyikéhez? 541. [2] Hány olyan szám van az első 1000 természetes szám között mely a 2, 3, 5 számok közül a) legalább az egyikkel, b) pontosan eggyel, c) legfeljebb kettővel, d) pontosan kettővel osztható?

10. Különféle számelméleti feladatok

49

542. [2] 327-et és 539-et elosztjuk ugyanazzal a háromjegyű számmal, s mind a kétszer ugyanaz a maradék. Mennyi ez a maradék? 543. [1] Egy számot 5-tel osztva 2, 6-tal osztva 5 a maradék. Mennyi a maradék, ha 30-cal osztunk? 544. [1] Melyik az a legkisebb, 7-tel osztható természetes szám, mely 2-vel, 3-mal, 4-gyel, 5-tel vagy 6-tal osztva mindig 1 maradékot ad? 545. [1] Melyik az a legkisebb természetes szám, melyet ha 2-vel, 3-mal, 4-gyel, 5-tel, ill. 6-tal osztunk, rendre az 1, 2, 3, 4, ill. 5 maradékot kapjuk? 546. [2] Melyik az a legkisebb természetes szám, melyet ha 5-tel, 7-tel, 9-cel osztunk, rendre a 3, 4, ill. 5 maradékot kapjuk? 547. [2] Mutassuk meg, hogy minden, 10-nél nagyobb egész szám előállítható prímszámok összegeként. 548. [2] Mutassuk meg, hogy minden, 10-nél nagyobb egész szám előállítható összetett számok összegeként. 549. [3] Melyek azok a számok, amelyek előállíthatók két összetett szám összegeként? 550. [4] Melyek azok a számok, amelyek előállíthatók két, 1-nél nagyobb, egymáshoz relatív prím szám összegeként? 551. [4] Melyek azok a számok, amelyek előállnak három, 1-nél nagyobb, páronként relatív prím szám összegeként? 552. [5] Adva van 13 racionális szám, melyek közül bármelyiket hagyjuk is el, a maradék 12 beosztható két hatos csoportba úgy, hogy az egyes csoportokban levő számok összege megegyezik. Mutassuk meg, hogy mind a 13 szám egyenlő. 553. [6] Adott 10 pozitív egész, amelyek egyikének sincs 20-nál nagyobb prímosztója. Mutassuk meg, hogy kiválasztható közülük néhány (esetleg csak egy, esetleg az összes) úgy, hogy a szorzatuk négyzetszám. 554. [6] Adott 11 pozitív egész, amelyek egyikének sincs 30-nál nagyobb prímosztója. Mutassuk meg, hogy kiválasztható közülük néhány (esetleg csak egy, esetleg az összes) úgy, hogy a szorzatuk négyzetszám. 555. [6] Adott 1994 db pozitív egész szám, melyek szorzatának 1993 különböző prímosztója van. Mutassuk meg, hogy a számok közül néhánynak a szorzata négyzetszám. 556. [6] Adott 48 pozitív egész szám, melyek szorzatának 10 különböző prímosztója van. Mutassuk meg, hogy a számok között van négy olyan, melyek szorzata négyzetszám.

Feladatok

50

557. [6] Adott 1985 pozitív egész szám, melyeknek egyikének sincs 26-nál nagyobb prímosztója. Mutassuk meg, hogy a számok között van négy olyan, melyek szorzata negyedik hatvány. 558. [7] Adott 2k + 1 db pozitív egész szám, melyek szorzatának k különböző prímosztója van. Mutassuk meg, hogy a számok között van néhány olyan, melyek szorzata köbszám. 559. [7] Adott 3k + 1 db pozitív egész szám, melyek szorzatának k különböző prímosztója van. Mutassuk meg, hogy a számok között van néhány olyan, melyek szorzata negyedik hatvány.

11. Számok reciprokainak összege Jelölések: Legyen a1 , a2 , a3 , . . . , egy számsorozat. Gyakran használjuk az olyan összegeket, mint a1 + a2 + a3 + · · · + an . Ez az összeg tömörebben is n felírható a következő jelöléssel: ak , ill. ak . (Ezt így olvassuk: szumma k=1

1≤k≤n

k egyenlő egytől n-ig ak .) Az a1 ·a2 ·a3 ·. . .·an szorzatot szokás a következő — rövidebb — módon is írni: n ak , ill. ak . (Ezt így olvassuk: produktum k egyenlő egytől n-ig ak .) k=1

1≤k≤n

560. [4] Mutassuk meg, hogy a

n 1 k=1

561. [3] Mutassuk meg, hogy a

k

összeg értéke nem lehet egész szám.

n 1 összeg értéke nem lehet egész szám. 2 k k=1

1 1 1 562. [4] Mutassuk meg, hogy az + +···+ összeg értéke nem lehet egész n n+1 n+m szám. 1 1 1 563. [5] Mutassuk meg, hogy az + + · · · + összeg értéke nem lehet egész 3 5 2n + 1 szám. 1 564. [6] Mutassuk meg, hogy a összeg értéke nem lehet egész szám. ab 1≤a N, akkor < 2. a i=1 i 698. [6] Mutassuk meg, hogy ha az 1 ≤ a1 < a2 < · · · < am ≤ N számok közül √ bármely két különböző számra [ai , aj ] ≤ N, akkor m ≤ 2 N . 699. [5] Mutassuk meg, hogy ha az 1 ≤ a1 < a2 < · · · < an ≤ 2n számok közül  2n  bármely két különböző számra [ai , aj ] > 2n, akkor a1 > . 3 700. [4] Mutassuk meg, hogy az 1 ≤ a1 < a2 < · · · < am számok legkisebb közös többszöröse legalább m · a1 . 701. [5] Mutassuk meg, hogy ha az N ≥ a1 > a2 > · · · > am > 0 egész számok közül bármely két különböző számra [ai , aj ] ≤ N, akkor iai ≤ N. 702. [6] Az 1 < a1 < a2 < · · · < ak ≤ n számokra igaz, hogy egyik ai sem osztója a többi szorzatának. Mutassuk meg, hogy k ≤ π(n), ahol π(n) a prímek száma n-ig. 703. [7] [Erdős–Szekeres tétel.] Mutassuk meg, hogy a különböző valós számokból álló a1 , a2 , . . . , amn+1 sorozatnak vagy létezik egy m-nél hosszabb csökkenő részsorozata, vagy van egy n-nél hosszabb növekvő részsorozata.

60

Feladatok

704. [5] Az a1 , a2 , . . . , ak pozitív egész számok nem nagyobbak n-nél, s az ai + aj összegek mind különbözők. Jelölje f (n) az ilyen tulajdonságú sorozatok elemei számának (tehát k-nak) a maximumát. Mutassuk meg, hogy √ a) f (n) < 2n + 1; √ b) f (n) ≥ 3 n. 705. [4] Bizonyítsuk be, hogy 101 különböző, 99-nél nem nagyobb abszolútértékű egész szám közt van olyan három, melyek összege 0. 706. [4] Száz szám összege 0. Bizonyítsuk be, hogy a közülük kiválasztható számpárok között legalább 99 van, amelyben a két tag összege nem negatív. 707. [5] n darab valós szám összege 99. Tudjuk, hogy a számok abszolút értéke legfeljebb 3. Bizonyítsuk be, hogy kiválasztható közülük néhány olyan, amelyek összege 32 és 34 közé esik. 708. [4] Adott 50 szám, melyek összege 100. Bizonyítsuk be, hogy a számok közül kiválasztható 3 úgy, hogy az összegük legalább 6 legyen. 709. [4] Adott hét különböző pozitív egész szám, melyek összege 100. Bizonyítsuk be, hogy van köztük három, melyek összege legalább 50. 710. [4] Adott 101 szám. Bárhogyan választunk ki közülük 50-et, ezek összege kisebb, mint a ki nem választottak összege. Bizonyítsuk be, hogy a számok mind pozitívak. 711. [4] a1 , a2 , . . . , a16 száznál nem nagyobb, különböző pozitív egész számok. Bizonyítsuk be, hogy van köztük négy különböző, amelyekre ai + aj = ak + al . 712. [4] Legyen a1 , a2 , . . . természetes számokból álló, szigorúan monoton növekvő sorozat. Bármely természetes szám vagy eleme a sorozatnak, vagy egyértelműen előállítható a sorozat két különböző tagjának összegeként. Mutassuk meg, hogy an < n2 .

16. Számkonstrukciók 713. [3] Melyik az a legnagyobb ötjegyű szám, amelynek minden számjegye nagyobb, mint a mögötte álló számjegyek összege? 714. [3] Melyik az a legnagyobb egész szám, amelyben a harmadik jeggyel kezdődően minden számjegy az előző két számjegy összege? 715. [3] Melyik az a legnagyobb hatjegyű szám, amelyben a harmadik jeggyel kezdődően minden számjegy az előző két számjegy szorzata?

16. Számkonstrukciók

61

716. [3] Melyik az a legnagyobb hatjegyű szám, amelynek minden számjegye legalább akkora, mint a mögötte álló számjegyek szorzata? 717. [3] Van-e olyan háromjegyű szám, amelyet számjegyei összegével akár növelünk, akár csökkentünk, csupa egyenlő jeggyel írt számot kapunk? 718. [3] Az összes számjegyet pontosan egyszer felhasználva írjunk fel öt olyan számot, melyek közül az egyik kétszerese, a másik háromszorosa, a harmadik négyszerese, a negyedik ötszöröse a legkisebbnek. 719. [3] Az összes számjegyet pontosan egyszer felhasználva írjunk fel három olyan számot, melyek közül az egyik háromszorosa, a másik ötszöröse a legkisebbnek. 720. [3] Az összes, 0-tól különböző számjegyet pontosan egyszer felhasználva írjunk fel három olyan számot, melyek közül az egyik háromszorosa, a másik ötszöröse a legkisebbnek. 721. [4] Az összes, 0-tól különböző számjegyet pontosan egyszer felhasználva írjunk fel három olyan számot, melyek úgy aránylanak egymáshoz, mint 1 : 2 : 3. 722. [3] Az összes számjegyet pontosan egyszer felhasználva írjunk fel három olyan számot, melyek úgy aránylanak egymáshoz, mint 2 : 3 : 4. 723. [3] Keressük meg azt a számot, melynek köbe és a negyedik hatványa együtt mind a tíz számjegyet tartalmazza, és mindegyik számjegy csak egyszer szerepel a két szám valamelyikében. 724. [5] Keressünk olyan számot, amely és a négyzete az összes, 0-tól különböző számjegyet tartalmazza, és mindegyik számjegy csak egyszer szerepel a két szám valamelyikében. 725. [4] Az összes, 0-tól különböző számjegyet pontosan egyszer felhasználva írjunk fel négy olyan számot, melyek közül kettőnek-kettőnek ugyanannyi a szorzata (pl.: 174 · 32 = 96 · 58 = 5568). Keressünk több megoldást. 726. [4] Írjunk fel két olyan számot, melyek szorzatukkal együtt mind a tíz számjegyet pontosan egyszer tartalmazzák (pl.: 39 · 402 = 15 678). Keressünk több megoldást. 727. [4] Írjunk fel két olyan számot, melyek szorzatukkal együtt az összes, 0-tól különböző számjegyet pontosan egyszer tartalmazzák (pl.: 48 · 159 = 7632). Keressünk több megoldást. 728. [4] Írjunk fel két olyan számot, melyek ugyanazokat a számjegyeket tartalmazzák, mint a két szám szorzata (pl.: 21 · 87 = 1827). Keressünk több megoldást. 729. [4] Keressük meg azokat a háromjegyű számokat, melyek egyenlők számjegyeik köbének összegével.

62

Feladatok

730. [4] Keressük meg azokat a négyjegyű számokat, melyek egyenlők számjegyeik negyedik hatványának összegével. 731. [4] Keressük meg azokat az ötjegyű számokat, melyek egyenlők számjegyeik ötödik hatványának összegével. 732. [3] Keressük meg a legnagyobb olyan 8-jegyű számot, melyben bármely számjegy (a két szélső kivételével) kisebb két szomszédjának számtani közepénél. 733. [4] Keressünk olyan hatjegyű számot, melyre az jellemző, hogy akár 2-vel, akár 3mal, akár 4-gyel, akár 5-tel, akár pedig 6-tal szorozzuk meg, mindegyik esetben olyan számot kapunk eredményül, amely az eredeti szám számjegyeinek más sorrendjével áll elő. 734. [3] Egy számot nevezzünk szerencsésnek, ha jegyei két csoportba oszthatók úgy, hogy a két csoportban a számok összege ugyanannyi (pl. 32 843 szerencsés, hiszen 8 + 2 = 3 + 4 + 3). Melyik a legkisebb szerencsés szám, melynek a szomszédja is szerencsés? Melyik a legkisebb négyjegyű szerencsés szám, melynek a szomszédja is szerencsés? 735. [2] Megadható-e néhány természetes szám úgy, hogy összegük is, szorzatuk is 203 legyen? 736. [3] Adjunk meg 100 (ill. 200) egész számot, melyek összege 0, szorzata 100 (ill. 200). 737. [3] Adjunk meg 1000 darab pozitív egész számot úgy, hogy azok összege ugyanannyi legyen, mint a szorzatuk. 738. [3] Készítsünk 3 × 3-as, 5 × 5-ös bűvös négyzeteket. 739. [3] Egy 3 × 3-as táblázat mezőibe írjunk különböző egész számokat úgy, hogy minden sorban és minden oszlopban ugyanannyi legyen a számok szorzata. 740. [4] Az 1, 2, 3, . . . , 29, 30 számok közül válasszunk ki alkalmasan kilenc különbözőt, és rendezzük el egy 3 × 3-as táblázat mezőibe úgy, hogy minden sorban és minden oszlopban 270 legyen a számok szorzata. 741. [5] Az 1, 2, 3, . . . , 39, 40 számok közül válasszunk ki alkalmasan kilenc különbözőt, és rendezzük el egy 3 × 3-as táblázat mezőibe úgy, hogy minden sorban, minden oszlopban és mindkét átlóban ugyanannyi legyen a számok szorzata. 742. [4] Egy 3 × 3-as táblázat mezőibe írjunk különböző egész számokat úgy, hogy minden sorban, minden oszlopban és a két átlóban ugyanannyi legyen a számok szorzata. Mutassuk meg, hogy ekkor ez a szorzat köbszám lesz. 743. [5] Az 1, 2, 3, . . . , 29, 30 számok közül válasszunk ki alkalmasan 16 különbözőt és rendezzük el egy 4×4-es táblázat mezőibe úgy, hogy minden sorban és minden oszlopban ugyanannyi legyen a számok szorzata.

16. Számkonstrukciók

63

744. [3] Írjunk egy 3 × 4-es táblázat mezőibe számjegyeket úgy, hogy a sorokban álló négyjegyű számok, és az oszlopokban álló háromjegyű számok mindegyike osztható legyen 92-vel. 745. [3] Írjunk egy 4 × 4-es táblázat mezőibe nullától különböző egész számokat úgy, hogy a táblázat bármely 2 × 2-es, 3 × 3-as részének, ill. magának a 4 × 4-es táblázatnak a csúcsaiban álló négy szám összege nulla legyen. 746. [4] Töltsük ki az 5 × 5-ös táblázat üres mezőit pozitív egész számokkal úgy, hogy mindegyik sorban, mindegyik oszlopban egy-egy számtani sorozat egymást követő elemei álljanak. 74 186 103 0 747. [5] Mutassuk meg, hogy egy 2n × 2n-es táblázat mezői kitölthetők a −1, 0, 1 számokkal úgy, hogy a 4n db sor- és oszlopösszeg között ne legyenek egyenlők. 748. [5] Mutassuk meg, hogy ha egy (2n + 1) × (2n + 1)-es táblázat mezőibe a −1, 0, 1 számokat írjuk, akkor a 4n + 2 db sor- és oszlopösszeg között lesznek egyenlők. 749. [3] Az 1, 2, 3, . . . , 20 számokat írjuk fel körbe egy kör kerületén úgy, hogy az egymás mellett levő számok különbségének abszolútértéke mindenhol más legyen. 750. [5] Az 1, 2, 3, . . . , 12 számokat írjuk fel körbe egy kör kerületén úgy, hogy bármely, egymás után álló a, b, c számra b2 − ac osztható legyen 13-mal. 751. [3] Az 1, 2, 3, . . . , 3n számokat osszuk szét n db, 3–3 számból álló csoportba úgy, hogy mindegyik csoportban más-más legyen a számok összege. 752. [4] Az 1, 2, 3, . . . , 60 számok közül válasszunk ki minél többet úgy, hogy a kiválasztott számok között bármely kettőnek a különbsége más-más legyen. 753. [4] Igaz-e a következő állítás: bármely hat természetes számból vagy kiválasztható három, páronként relatív prím, vagy három olyan választható ki, melyeknek van 1-nél nagyobb közös osztója? 754. [5] Igaz-e a következő állítás: 17 egymást követő egész szám között mindig van olyan, mely a többihez relatív prím? 755. [3] Az első 1983 pozitív egész közül válasszunk ki minél több számot úgy, hogy semelyik két kiválasztott szám szorzata ne szerepeljen a kiválasztott számok között. 756. [3] Igaz-e a következő állítás: ha az 1, 2, 3, . . . , 50 számokból kiválasztunk 30-at, akkor mindig lesz köztük két olyan, amelyeknek az egyike a másik kétszerese?

64

Feladatok

757. [4] Osszuk szét a természetes számokat két halmazba úgy, hogy egyikben se legyen végtelen sok elemű mértani sorozat. 758. [4] Osszuk szét a természetes számokat két halmazba úgy, hogy egyikben se legyen végtelen sok elemű számtani sorozat. 759. [4] Osszuk szét a természetes számokat két halmazba úgy, hogy az egyikben ne legyen végtelen sok elemű számtani sorozat, a másikban pedig ne legyen három elemű számtani sorozat. 760. [4] Adjunk meg néhány, páronként különböző differenciájú számtani sorozatot úgy, hogy bármelyik természetes szám eleme legyen legalább az egyik sorozatnak. (A differenciák 1-nél nagyobb egészek.) 761. [4] Képezzünk 5 számból álló számtani sorozatot a 0, 1, 2, . . . , 9 számjegyek mindegyikének egyszeri felhasználásával. 762. [4] Adjunk meg a) 3, illetve b) 5 pozitív egész számból álló számtani sorozatot úgy, hogy a számok szorzata négyzetszám legyen. 763. [4] Adjunk meg végtelen sok szomszédos számpárt úgy, hogy mindegyiküknek legyen 1-nél nagyobb négyzetszám osztója. 764. [4] Adjunk meg négy szomszédos természetes számot úgy, hogy mindegyiküknek legyen 1-nél nagyobb négyzetszám osztója. 765. [5] Van-e 21 olyan, egymás után következő pozitív egész szám, hogy a számok mindegyike osztható a 2, 3, 5, 7, 11, 13 prímek közül legalább eggyel? 766. [7] Adjunk meg végtelen sok olyan m, n számpárt, melyekre az m és n számoknak ugyanazok a prímosztói, továbbá az m + 1 és az n + 1 számoknak is ugyanazok a prímosztói. 767. [3] Adjunk meg három olyan, 1-nél nagyobb egész számot, hogy bármely kettő szorzatához 1-et adva, az így kapott szám osztható legyen a harmadik számmal. 768. [3] Adjunk meg négy különböző pozitív egész számot úgy, hogy bármely három szorzatához 1-et adva, az így kapott szám osztható legyen a negyedik számmal. 769. [3] Adjunk meg négy különböző egész számot úgy, hogy bármely három összege osztható legyen a negyedikkel. 770. [4] Osszuk szét az 1, 2, 3, 4, . . . , 14, 15, 16 számokat két 8-as csoportba úgy, hogy ha az egyik csoportban kiszámítjuk bármelyik két (különböző) szám összegét, akkor pontosan ugyanazokat a számokat kapjuk eredményül (és mindegyiket ugyanannyiszor), mint ha a másik csoportban végezzük el ugyanezeket az összeadásokat.

16. Számkonstrukciók

65

771. [2] Adjunk meg négy különböző pozitív egész számot úgy, hogy képezve belőlük a páronkénti összegeket, hat egymást követő számot kapjunk. 772. [2] Adjunk meg négy különböző pozitív egész számot úgy, hogy képezve belőlük a páronkénti különbségeket, hat egymást követő számot kapjunk. 773. [5] Adjunk meg n db páronként relatív prím számot úgy, hogy közülük akárhánynak az összege összetett szám legyen. 774. [3] Adjunk meg n db különböző természetes számot úgy, hogy közülük bármelyik szám osztója legyen a többi szorzatának. 775. [5] Adjunk meg 8 természetes számot úgy, hogy egyik sem osztója valamely másiknak, de bármelyik négyzete osztható a többi szám bármelyikével. 776. [5] Az 1, 2, 3, . . . , 1994 számokból válasszuk ki a legtöbb számot úgy, hogy ne legyen köztük két olyan szám, melyek összege osztható lenne a két szám különbségével. 777. [6] Adjunk meg 100 különböző természetes számot úgy, hogy bármely kettő összege osztható legyen különbségükkel. 778. [6] Mutassuk meg, hogy a 0, 1, 2, . . . , 3k − 1 számok közül megadható 2k db különböző szám úgy, hogy közülük semelyik ne legyen valamely másik kettőnek a számtani közepe. 779. [3] Adjuk meg az {1, 2, 3, 4, 5, 6} halmaznak hat olyan részhalmazát, hogy közülük bármely kettőnek egy közös eleme legyen. 780. [3] Adjuk meg a természetes számoknak három olyan végtelen részhalmazát úgy, hogy a részhalmazok közül bármely kettőnek végtelen sok közös eleme legyen, ám a három részhalmaznak ne legyen közös eleme. 781. [6] Van-e nemnegatív egész számokból álló két olyan végtelen A és B halmaz, hogy bármely pozitív egész szám pontosan egyféleképpen írható fel egy A-hoz és egy B-hez tartozó szám összegeként? 782. [6] Van-e nemnegatív egész számokból álló három olyan végtelen A, B és C halmaz, hogy bármely pozitív egész szám pontosan egyféleképpen írható fel egy A-hoz, egy B-hez és egy C-hez tartozó szám összegeként? 783. [6] A nemnegatív egészek halmazát osszuk fel végtelen sok, közös elem nélküli részhalmazra úgy, hogy mindegyiknek végtelen sok eleme legyen, és a részhalmazok hasonlók egymáshoz, azaz egyik részhalmazból megkapjuk bármely másikat, ha egy alkalmasan választott egész számot hozzáadunk a részhalmaz elemeihez („eltoljuk” a részhalmazt). 784. [3] Osszuk két csoportba az 1, 2, 3, 4, 5, 6, 7, 8 számokat úgy, hogy egyik csoportban se legyen két olyan szám, melyek összegének fele is ugyanabban a csoportban lenne.

Feladatok

66

785. [3] Osszuk két csoportba az első 1000 négyzetszámot úgy, hogy a két csoportban a négyzetszámok összege egyenlő legyen. 786. [5] Osszuk az első 27 négyzetszámot két csoportba úgy, hogy azokban egyenlő legyen a számok összege. 787. [4] Osszuk három csoportba az első 81 négyzetszámot úgy, hogy a három csoportban a négyzetszámok összege egyenlő legyen, és mindegyik csoportban 27–27 szám legyen. 788. [3] Keressünk olyan n-jegyű számot, amelynek első számjegye egyenlő a számjegyei között található 0-k számával, második számjegye az 1-esek számával, harmadik számjegye a 2-esek számával és így tovább. 789. [3] Írjunk a pontok helyébe számokat úgy, hogy a keretben igaz állítás legyen. Ebben a keretben a 0 számjegyből pontosan . . . darab van, az 1-ből . . . darab, a 2-ből . . . darab, a 3-ból . . . darab, a 4-ből . . . darab, az 5-ből . . . darab, a 6-ból . . . darab, a 7-ből . . . darab, a 8-ból . . . darab, s végül a 9-ből . . . darab. 790. [3] Adjunk meg olyan x1 , x2 , . . . , x10 számokat, melyekre teljesülnek a következő feltételek: x1 a [0;1] intervallumba esik; x1 és x2 közül az egyik a [0;1] intervallum egyik felébe, a másik a másik felébe esik; x1 , x2 , x3 közül az egyik a [0;1] intervallum egyik harmadába, a másik egy másik harmadába, a harmadik a harmadik harmadába esik és így tovább.

17. Melyik szám a nagyobb? 791. [1] 2300 vagy 3200 ? 792. [1] 2100 vagy 1030 ? 793. [1] 321 vagy 231 ? 794. [2] 202303 vagy 303202 ? 795. [2] 9920 vagy 999910 ? 796. [2] 920 vagy 2713 ? 797. [2] 3500 vagy 7300 ? 798. [2] 199110 vagy 199010 + 19909 ? 799. [3] 792 vagy 891 ?

17. Melyik szám a nagyobb?

67

800. [2] 242 vagy 327 ? 801. [3] 2100 + 3100 vagy 4100 ? 802. [2] 803. [2] 804. [2] 805. [2] 806. [2] 807. [1]

999 1000 vagy ? 1000 1001 210 − 1 220 − 1 vagy ? 10 2 220 434343 434343434343 vagy ? 575757 575757575757 22222221 33333331 vagy ? 22222223 33333334 222221 444443 vagy ? 333332 666665 3 300000001 vagy ? 5 500000001

808. [3] 2 809. [3] 7

√ 3 √ 5



vagy 3 2 ? √

vagy 5 7 ?

810. [2] 3111 vagy 1714 ? 811. [2] 12723 vagy 51318 ? 812. [2] 111979 vagy 371320 ? 100

813. [4] 23

150

vagy 32 ?

814. [3] 2 log12 145 vagy

√ 15?

815. [3] log4 9 vagy log9 25? 816. [4] log5 6 vagy log6 7? 817. [4] lg2 11 vagy lg 12? √ 818. [4] sin 1 vagy log3 7? 819. [4] 1, 011000 vagy 1000? 820. [4] Mutassuk meg, hogy 479 < 2100 + 3100 < 480 .

Feladatok

68

18. Egyenletek és egyenletrendszerek 821. [3] Miért nem lehet megoldani az alábbi egyenleteket a valós számok körében? √ √ a) x − 2 + 3 − x = x − 4. √ √ b) x 2 + 1 + x 2 + 2 = 2. 2 c) x 2 + 2 = 2. x 2 d) 2sin x = sin x. e) sin x · cos x = sin 60◦ .

π f) sin x · sin x + = 1. 3 1 g) sin x · sin(x + π) = . 3 h) x 10 − x 7 + x 2 − x + 1 = 0. Oldjuk meg az alábbi egyenleteket a valós számok halmazán. (822–881. feladatok) √ 822. [1] (x + 2) x − 1 = 0. 823. [1] x + 824. [1]

1 1 = 1991 + . x − 1991 x − 1991

x2 − 1 = 0. x3 − 1 2

2

825. [2] (x 3 − 1) + (x 4 − 1) = 0. 826. [3] 2x + 3x = 35. 827. [3] 2x + 3x = 5x .    828. [3] x 2 − 9 + x 2 − 16 + x 2 − 25 = 7.    35 829. [3] x 2 − 9 + x 2 − 16 + x 2 − 25 = . x 830. [3] 3x + 27 = 18x. 831. [3] 4x = 1 + 2x. 832. [3] 8x (3x + 1) = 4. √ x √ x 833. [4] (2 + 2) + (2 − 2) = 2−x . 834. [4] log1991 (x − 3) + log1992 (x − 3) = 3 − lg(x 5 − 24). √ √ √ √ √ √ 835. [4] sin x + sin 3x + sin 5x = cos x + cos 3x + cos 5x.

18. Egyenletek és egyenletrendszerek

836. [3] cos x = y 2 − 2y + 2.   √ √ 837. [3] (2x + 3) 2x + 3 = (6 − x) 6 − x. 838. [3] 839. [3] 840. [3] 841. [3] 842. [4] 843. [4] 844. [4]

x − 5 x − 4 x − 3 x − 100 x − 101 x − 102 + + = + + . 100 101 102 5 4 3 x−a x−b x−c + + = 3, (a + b)(b + c)(c + a) = 0. b+c c+a a+b 3x x−a x−b x−c + + = , (a + b)(b + c)(c + a) = 0. b+c c+a a+b a+b+c x − bc x − ca x − ab + + = a + b + c, (a + b)(b + c)(c + a) = 0. b+c c+a a+b x x(x − 1) x(x − 1)(x − 2) · · · (x − n + 1) 1− + − · · · + (−1)n · = 0. 1 1·2 n! √ √ x − 2 + 3 − x = x 2 − 5x + 7.   x 2 + x − 1 + x − x 2 + 1 = x 2 − x + 2.

845. [3] x 2 + y 2 = 2x + 2y − 2. 846. [3] 8(x 4 + y 4 ) − 4(x 2 + y 2 ) + 1 = 0.  √ 36 4 +√ = 28 − 4 x − 2 − y − 1. 847. [4] √ y−1 x−2 848. [3] (x + y)2 = (x + 1)(y − 1). √ √ 849. [4] 3x 2 + 6x + 7 + 5x 2 + 10x + 14 = 4 − 2x − x 2 .  850. [3] x 2 + 2y 2 − 2yz = 100 . 2xy − z2 = 100 851. [4] (x 2 − a 2 )2 = 4ax + 1. 852. [3] x 2 + y 2 + z2 + t 2 = x(y + z + t). ⎫ 853. [3] x 2 + 10y + 41 = 0 ⎪ ⎬ y 2 − 2z − 23 = 0 . ⎪ ⎭ z2 − 6x + 17 = 0 ⎫ 854. [3] x 2 − yz = y − x ⎪ ⎬ y 2 − zx = z − y . ⎪ ⎭ z2 − xy = x − z

69

Feladatok

70

855. [3]

856. [4] 857. [4]

⎫ x 2 − xy − xz + z2 = 0 ⎪ ⎬ x 2 − xz − yz + 3y 2 = 2 . ⎪ ⎭ y 2 + xy + yz − z2 = 2 2x 2 2y 2 2z2 = y, = z, = x. 1 + x2 1 + y2 1 + z2







1 1 1 1 1 1 1 1 x= y+ ,y= z+ ,z= t+ ,t= x+ . 2 y 2 z 2 t 2 x

858. [3]

2 xy = , x+y 3

yz 6 = , y+z 5

zx 3 = . z+x 4

859. [4]

xyz = 2, x+y

xyz 6 = , y+z 5 

xyz 3 = . z+x 2

860. [3] x 2 + xy = 10 y 2 + xy = 15

.

1 . 3 ⎫ 1⎪ x3 − y2 − y = ⎪ ⎪ ⎪ 3⎪ ⎪ ⎬ 1 3 2 . y −z −z = 3⎪ ⎪ ⎪ ⎪ 1⎪ ⎭ z3 − x 2 − x = ⎪ 3

861. [4] x 3 − x 2 − x = 862. [5]

863. [3] (x − y + z)2 = x 2 − y 2 + z2 . √ √ √ √ 864. [4] x − y + z = x − y + z. 865. [4] x 3 − 3x 2 + 3x + 7 = 0. 866. [4] 4x 3 + 18x 2 + 27x + 10 = 0. 867. [4] x 4 + 4x 3 + 6x 2 + 4x − 2 = 0. 868. [4] 9x 3 + 3x 2 + 3x + 1 = 0. 869. [4] 3x 4 + 4x 3 − 6x 2 + 4x − 1 = 0. 870. [4] x 4 − x 2 + 2x − 1 = 0. 871. [4] x 4 + 4x 3 + 10x 2 + 12x − 16 = 0. √ √ 3 872. [4] 2 − x + x − 1 − 1 = 0. √ √ 3 3 873. [4] 1 + x + 1 − x = 2.

18. Egyenletek és egyenletrendszerek

874. [4]

√ 3

x−1+

x5

√ 3

71

√ 3 x − 2 = 2x − 3.

875.

[4]

x

876.

[4]

[x ] + [x 2 ] + [x] = {x} − 1.

= 5.

3

877. [4] [x] = 1 + {x 2 − 1}. 878. [4] x 3 − [x] = 3.   879. [4] lg x = [x] − x .    √ √ 880. [4] x x x x . . . = 1994.  881.

[4]

x=

 √ −3 + 4 −3 + 4 −3 + 4x.

882. [4] Tudjuk, hogy a 2 + b2 + c2 = ab + bc + ca. Mutassuk meg, hogy a = b = c. Oldjuk meg a 4x + 9x + 25x = 6x + 10x + 15x egyenletet. 883. [3] Mutassuk meg, hogy sin2n x + cos2n x ≤ 1, n ∈ N , n > 0. 884. [4] Oldjuk meg a következő egyenleteket. a) sin10 x + cos10 x = 1, b) sin9 x + cos11 x = 1, c)

sin10 x − cos10 x = 1,

d)

sin9 x − cos11 x = 1.

885. [4] Oldjuk meg az x3 + y3 = 1



x4 + y4 = 1 egyenletrendszert. 886. [4] Mutassuk meg, hogy az x 3 + 4x 2 + 6x + c = 0 egyenletnek semmilyen c valós szám esetén sem lehet három különböző valós gyöke. √ 3 [5] 887. Oldjuk meg az (x 3 − 6)3 = 6 + x + 6 egyenletet. 888. [5] Oldjuk meg az (x 2 + x − 2)3 + x 2 − 2 = x 3 egyenletet. 889. [3] Az x 2 + px + q = 0 egyenlet gyökei p és q. Határozzuk meg p és q értékét. 890. [3] Igazoljuk, hogy az x 10 + px 7 + q = 0 egyenletnek nincs egész gyöke, ha p és q páratlan egészek.

Feladatok

72

19. Egyenlőtlenségek 891. [1] Néhány szám összege 1. Lehet-e négyzetösszegük kisebb, mint

1 ? 100

892. [1] Néhány szám összege 1. Lehet-e négyzetösszegük nagyobb, mint 100? 893. [3] 100 pozitív szám összege 1. Közülük minden lehetséges módon kiválasztunk 50-et és kiszámoljuk azok szorzatát. Mutassuk meg, hogy az 50 tényezős szorzatok összege kisebb 1-nél.  894. [2] Határozzuk meg a 0, 999 . . . 9 (60 db 9-es) szám első 60 tizedesjegyét. √ √ √ √ 895. [3] Mutassuk meg, hogy 1 + 2 + 3 + · · · + n > n, (n > 1). √ 1 1 1 1 896. [3] Mutassuk meg, hogy √ + √ + √ + · · · + √ > n, (n > 1). n 3 1 2 1 1 1 1 1 897. [3] Mutassuk meg, hogy + + + ···+ > , (n > 1). n+1 n+2 n+3 2n 2 1 1 1 1 1 898. [3] Mutassuk meg, hogy + + + · · · + 2 > 1 − , (n > 1). n+1 n+2 n+3 n n 1 1 1 1 889. [3] Mutassuk meg, hogy 1 − < √ +√ + ···+ √ < 1. 2 2 2 n n +1 n +2 n +n 900. [3] Mutassuk meg, hogy 901. [3] Mutassuk meg, hogy 902. [3] Mutassuk meg, hogy 903. [4] Mutassuk meg, hogy 904. [4] Mutassuk meg, hogy

905. [3] Mutassuk meg, hogy

1 1 1 1 + + + ··· + > 1. 51 52 53 200 1 1 1 1 1 + + + + ···+ > 4. 2 3 4 64 1 1 1 1 1 < + + + ···+ < 2. 5 6 7 17 1 1 1 1 1+ + + + ··· + < 3. 1! 2! 3! n! 1 1 1 1 1 + 2 + 2 + 2 + · · · + 2 < 2. 2 3 4 n    √ 6 + 6 + 6 + 6 < 3.

906. [3] Mutassuk meg, hogy      √ √ √ 12 + 12 + 12 + 20 + 20 + 20 + 30 + 30 + 30 < 15.

19. Egyenlőtlenségek

73

907. [4]

       √ Bizonyítsuk be, hogy 100 + 99 + 98 + · · · + 2 + 1 < 11.

908. [4]

       √ 2 Bizonyítsuk be, hogy 1 + n + n + n3 + · · · + nn < n 

909.

[5]

910. [7]

Bizonyítsuk be, hogy



1+

2+

(n > 1).

 √ 9 3 + ···+ n < . 5

       √ Mutassuk meg, hogy 2 3 4 . . . n < 3, ahol n = 2, 3, 4, . . .

911. [3] Mutassuk meg, hogy ha a és b pozitív számok, akkor

912. [3]

a+b a b ≤ + . 1+a+b 1+a 1+b 1 1 1 Legyen a > b > c. Mutassuk meg, hogy + + > 0. a−b b−c c−a

913. [3] Legyen a1 ≤ a2 ≤ . . . ≤ a10 . Mutassuk meg, hogy a1 + a2 + · · · + a10 a1 + a2 + · · · + a6 ≤ . 6 10 914. [4] Legyen a > b > c. Mutassuk meg, hogy a 2 (b − c) + b2 (c − a) + c2 (a − b) > 0. 915. [4] Mutassuk meg, hogy tetszőleges a és b pozitív valós számokra teljesül az a 3 + b3 ≥ a 2 b + ab2 egyenlőtlenség. 916. [4] Mutassuk meg, hogy tetszőleges a és b valós számokra teljesül az a 4 + b4 ≥ ab(a 2 + b2 ) egyenlőtlenség. 917.

[4]

Mutassuk meg, hogy tetszőleges a és b pozitív valós számokra teljesül az a 2 + b2 ≤ egyenlőtlenség.

a 3 b3 + b a

Feladatok

74

918. [4] Mutassuk meg, hogy tetszőleges a és b valós számokra teljesül az a + b a 2 + b2 a 3 + b3 · ≤ 2 2 2

egyenlőtlenség.

919. [4] Mutassuk meg, hogy tetszőleges a és b valós számokra teljesül az a + b a 3 + b3 a 4 + b4 · ≤ 2 2 2

egyenlőtlenség. 920.

[4]

Mutassuk meg, hogy tetszőleges a és b valós számokra teljesül az

egyenlőtlenség. 921.

[4]

a + b a 2 + b2 a 3 + b3 a 6 + b6 · · ≤ 2 2 2 2

Határozzuk meg a következő kifejezések minimumát. a) x 2 + y 2 + 2x − 4y + 6, b) x 6 + 2x 4 + 2x 3 + x 2 + 2x − 1, c) 2x 2 − 8xy + 17y 2 − 16x − 4y + 2062.

Igazoljuk az alábbi egyenlőtlenségeket. (922–933. feladatok) 922. [3] a 2 + b2 + c2 ≥ ab + bc + ca. 923. [4] a 2 + b2 + c2 +

3 ≥ a + b + c. 4

924. [4] a 4 + b4 + 2 ≥ 4ab. 925. [4] a 4 + 1 > a. 926. [4] a 2 + b2 + 1 ≥ ab + a + b. 927. [4]

a2 + b2 + c2 ≥ ab − ac + 2bc. 4

928. [4] a 2 + b2 + c2 + 4 ≥ ab + 3b + 2c. 929. [4] a 2 + b2 + c2 + d 2 ≥ ab + ac + ad. 930. [4] a 2 + b2 + c2 + d 2 − ab − bc − cd − d +

2 ≥ 0. 5

931. [4] a 2 + b2 + c2 + d 2 + e2 ≥ a(b + c + d + e). A 921–933. feladatokban x, y, a, b, c, d, e tetszőleges valós számokat jelent, ahol a kifejezés értelmezhető.

19. Egyenlőtlenségek

932. [6]

c b a a 2 b2 c2 + 2 + 2 ≥ + + , ahol abc = 0. 2 b c a b a c

933. [6]

a 4 b4 a b a 2 b2 + 4 + + ≥ 2 + 2 + 2, ahol a > 0, b > 0. 4 b a b a b a

75

A számtani és mértani közepek közötti a1 + a2 + · · · + an √ ≥ n a1 · a2 · . . . · an a1 ≥ 0, a2 ≥ 0, . . . , an ≥ 0 n egyenlőtlenség segítségével igazoljuk a következő egyenlőtlenségeket. Az egyenlőtlenségekben szereplő számok mindegyike pozitív. (934–996. feladatok) 1 ≥ 2, a a2 + 2 b) √ ≥ 2, a2 + 1 a2 + 3 > 2, c) √ a2 + 2 1 d) a 2 + 2 ≥ 1, a +1 a+b b+c c+a e) + + ≥ 6, c a b a b c f) + + ≥ 3. b c a 1 1 4 16 64 + + + ≥ . a b c d a+b+c+d

934. [3] a) a +

935. [4]

936. [3] (a + b)(b + c)(c + a) ≥ 8abc. 937. [3] (a + 1)(b + 1)(a + c)(b + c) ≥ 16abc. 938. [4]

bc ca a+b+c ab + + ≤ . a+b b+c c+a 2

939. [4] (a 2 + b2 )c + (b2 + c2 )a + (c2 + a 2 )b ≥ 6abc. 940. [4] ab(a + b) + bc(b + c) + ca(c + a) ≥ 6abc. 941. [4] a 2 (1 + b2 ) + b2 (1 + c2 ) + c2 (1 + a 2 ) ≥ 6abc. 942. [4] (a + b + c)(ab + bc + ca) ≥ 9abc. 943. [4]

ab bc ca + + ≥ a + b + c. c a b

944. [4] abc ≥ (a + b − c)(b + c − a)(c + a − b). √ √ 945. [4] a a 2 + c2 + b b2 + c2 ≤ a 2 + b2 + c2 .

Feladatok

76

946. [3] (a 2 + a + 1)(b2 + b + 1)(c2 + c + 1)(d 2 + d + 1) ≥ 81abcd. √ √ √ 947. [4] ab + bc + ca ≥ a bc + b ca + c ab. √ √ √ 948. [4] a + b + c ≥ ab + bc + ca.    a b c [4] + + > 2. 949. b+c c+a a+b 950. [4]

b c 3 a + + ≥ . b+c c+a a+b 2

951. [4]

a2 b2 c2 a+b+c + + ≥ . b+c c+a a+b 2

952. [4] a + b + c ≤

a 2 + b2 b2 + c2 c2 + a 2 a 3 b3 c3 + + ≤ + + . 2c 2a 2b bc ca ab

953. [4] (a + b + c)2 ≥ 3(ab + bc + ca). 954. [4] (ab)2 + (bc)2 + (ca)2 ≥ abc(a + b + c). 955. [4] (ab + bc + ca)2 ≥ 3abc(a + b + c). 956. [4] 3(a 2 + b2 + c2 ) ≥ (a + b + c)2 . 957. [4] ab5 + bc5 + ca 5 ≥ abc(a 2b + b2 c + c2 a). 958. [4] a 4 + b4 + c4 ≥ abc(a + b + c). 959. [4] a 4 + b4 + 8 ≥ 8ab. 960. [4] a 6 + b6 + c6 ≥ a 5 b + b5 c + c5 a. 961. [4] a 7 + b7 + c7 ≥ a 2 b2 c2 (a + b + c). 962. [4] a 8 + b8 + c8 ≥ a 2 b2 c2 (ab + bc + ca). 963. [4] a 3 b + b3 c + c3 a ≥ abc(a + b + c). √ √ √ √ 964. [4] (a + b) c + (b + c) a + (c + a) b ≥ 6 abc. 965. [4] (a 2 + b2 )(a 4 + b4 ) ≥ (a 3 + b3 )2 .

b  c  a 966. [4] a + b+ c+ ≥ 8. ac ba cb 967. [4] (a + b + c)(a 2 + b2 + c2 ) ≥ 9abc. 968. [4] 969. [4]

2 2 2 9 + + ≥ . b+c c+a a+b a+b+c 2 2 2 1 1 1 + + ≤ + + . a+b b+c c+a a b c

19. Egyenlőtlenségek

77

970. [4] ab(a + b) + bc(b + c) + ca(c + a) ≤ 2(a 3 + b3 + c3 ). 971. [4] c2 (a + b) + a 2 (b + c) + b2 (c + a) ≤ 2(a 3 + b3 + c3 ). 972. [4] (a + b + c)(ab + bc + ca) ≤ 3(a 3 + b3 + c3 ). 973. [4] (a + b + c)(a 2 + b2 + c2 ) ≤ 3(a 3 + b3 + c3 ). 974. [4] a 3 + b3 + c3 ≥ a 2 b + b2 c + c2 a. 975. [4]

a 2 b2 c2 + + ≥ a + b + c. b c a

976. [4] a 2 cd + b2 da + c2 ab + d 2 bc ≥ 4abcd. 977. [4]

a 3 + b3 + c3 b3 + c3 + d 3 c3 + d 3 + a 3 d 3 + a 3 + b3 + + + ≥ a 2 + b2 + c2 + d 2 . a+b+c b+c+d c+d +a d +a+b

978. [4] 8(a 3 + b3 + c3 ) ≥ 3(a + b)(b + c)(c + a). √ √ √ 979. [4] 2a + 1 + 2b + 1 + 2c + 1 < 4, ha a + b + c = 1. √ √ √ 980. [4] 4a + 1 + 4b + 1 + 4c + 1 < 5, ha a + b + c = 1. √ √ √ 981. [4] 6a + 1 + 6b + 1 + 6c + 1 < 9, ha a + b + c = 2.

1  1  1 982. [4] 1 + 1+ 1+ ≥ 64, ha a + b + c = 1. a b c √ √ √ 983. [4] a + b + c < 2, ha a + b + c = 1. √ √ √ 3 50 984. [4] a + 1 + 2a − 3 + 50 − 3a < 12, ha ≤ a ≤ . 2 3 985. [4] a 2 + b2 + c2 ≥ 14, ha a + 2b + 3c ≥ 14. 1 986. [4] log230 2 + log230 3 + log230 5 > . 3  √ √ 987. [4] (a + c)(b + d) ≥ ab + cd. 988. [3] a + b + c ≥ 3, ha abc = 1. 989. [3] a 2 + b2 + c2 ≥

1 1 1 + + , ha abc = 1. a b c

990. [4] a 4 + b4 + c4 ≥ a + b + c, ha abc = 1. √ √ √ 991. [4] ab + bc + ca ≥ a + b + c, ha abc = 1. √ √ 1 1 1 √ 992. [4] + + ≥ a + b + c, ha abc = 1. a b c 993. [4] (a + 2b)(b + 2c)(c + 2a) ≥ 27, ha abc = 1. 994. [4] (a + 1)(b + 1)(c + 1) ≥ 8, ha abc = 1.

Feladatok

78

995. [4] a 2 + b2 + c2 + d 2 + ab + bc + cd + ac + bd + ad ≥ 10, ha abcd = 1. 996. [4] (1 + a1 ) · (1 + a2 ) · . . . · (1 + an ) ≥ 2n , ha a1 · a2 · . . . · an = 1. 997. [4] p és q olyan pozitív valós számok, amelyek reciprokainak összege 1. 1 1 1 1 1 1 Igazoljuk, hogy ≤ + < és 1 ≤ + . 3 p(p + 1) q(q + 1) 2 p(p − 1) q(q − 1) 998. [4] Egy egységoldalú négyzetben n db kis négyzetet helyeztünk el úgy, hogy semelyik kettőnek nincs közös belső√pontja. Igazoljuk, hogy a négyzetek oldalhosszúságának összege legföljebb n. 999. [4] Egy egységnyi élű kockában n db kis kockát helyeztünk el úgy, hogy semelyik kettőnek nincs közös belső pontja. Igazoljuk, hogy a kockák élének összege legfeljebb n2/3 . √ a 2 + b2 1000. [4] Mutassuk meg, hogy ha a > b és ab = 1, akkor ≥ 2 2. a−b 1001. [3] Legyen a > 0, b > 0. Mutassuk meg, hogy

a 2 − b2 a 2 + b2 > . a−b a+b

1002. [3] Bizonyítsuk be a következő egyenlőtlenségeket. 1 1 a) + > 2, log2 π logπ 2 1 1 + > 2. b) log2 π log5 π 1003. [4] Mutassuk meg, hogy log4 5 + log5 6 + log6 7 + log7 8 > 4,4. 1004. [3] Mutassuk meg, hogy sin 40◦ + cos 40◦ > 1. √ √ 1005. [3] Mutassuk meg, hogy sin 40◦ + cos 40◦ > 1.

n+1 n 1006. [4] Mutassuk meg, hogy n! ≤ , n ∈ N. 2 1007. [4] Mutassuk meg, hogy ha a ≥ 0, b ≥ 0, n ∈ N , akkor 1008.

[5]

1009. [5]

√ a + bn n+1 a · bn . ≥ n+1



n+1 1 n 1 Mutassuk meg, hogy 1 + < 1+ , n ≥ 1, n ∈ N . n n+1

1 n Mutassuk meg, hogy 1 + < 4, n ≥ 1, n ∈ N . n

1010. [5] Mutassuk meg, hogy a pozitív a, b számokra mindig teljesül a √ √ √ 3 5 2 a + 3 b ≥ 5 ab egyenlőtlenség.

19. Egyenlőtlenségek

79

1011. [5] Mutassuk meg, hogy a pozitíva, b, c számokra mindig teljesül az  b 3 c a + + >2 b c a egyenlőtlenség. 1012. [5] Mutassuk meg, hogy nincsenek olyan pozitív a, b, c számok, melyek egyszerre kielégítenék az alábbi egyenlőtlenségeket: a(1 − b) >

1 , 4

b(1 − c) >

1 , 4

c(1 − a) >

1 . 4

1013. [5] Mutassuk meg, hogy nincsenek olyan pozitív a, b, c számok, melyek egyszerre kielégítenék az alábbi egyenlőtlenségeket: a+

1 < 2, b

b+

1 < 2, c

c+

1 < 2. a

1014. [5] x > 0 értékekre határozzuk meg a következő függvények legkisebb értékét. a a) f (x) = x + , a > 0, x x3 + x + 2 b) f (x) = . x 1 1015. [5] Az f (x) = (1 − x)3 (1 + 3x) függvénynek mennyi a maximuma a − < x < 1 3 intervallumon? 1016. [5] Az f (x) = (1 + x)3 (1 − x) függvénynek mennyi a maximuma a 0 ≤ x ≤ 1 intervallumon? 1017. [6] Az x 4 − 4x 3 + ax 2 − bx + 1 = 0 egyenletnek négy pozitív valós gyöke van. Határozzuk meg a és b értékét. 1018. [5] Határozzuk meg az alábbi függvények maximumát. √ √ a) f (x) = x + 7 + 11 − x, ha −7 ≤ x ≤ 11.  √ 2 b) f (x) = 4 cos2 x + 1 + 4 sin x + 3. 1019. [4] Határozzuk meg azt a legkisebb n természetes számot, amelyre az (a 2 + b2 + c2 )2 ≤ n(a 4 + b4 + c4 ) egyenlőtlenség igaz minden a, b, c valós számra. 1020. [5] Határozzuk meg k legnagyobb értékét úgy, hogy az a 4 + b4 + c4 + abc(a + b + c) ≥ k(ab + bc + ca)2 igaz legyen minden a, b, c valós számra. 1021. [4] Igazoljuk az alábbi egyenlőtlenséget.







1 1 1 1 1+ · 1+ · 1+ · ...· 1 + |y − z + t|, |z| > |x − y + t|,

|y| > |x − z + t|, |t| > |x − y + z|.

1026. [5] Igazoljuk a következő egyenlőtlenséget.  √ n n! < n+1 (n + 1)!, n = 1, 2, 3, . . . 1027. [5] Igazoljuk a következő egyenlőtlenséget. √ √ n+1 n n> n + 1, n = 3, 4, 5, . . . 1994n [4] 1028. Az tört értéke mely pozitív egész n esetén lesz maximális? n! 19n + 94n 1029. [4] A tört értéke mely pozitív egész n esetén lesz maximális? n! n2 1030. [4] Az tört értéke mely pozitív egész n esetén lesz maximális? 1, 001n 1031. [5] Mutassuk meg, hogy ha a1 , a2 , . . . , an pozitív valós számok, akkor n 1  ak  1+ 1 + 2 ≥ (n + 1)2 . ak k k=1 1032. [5] Mutassuk meg, hogy minden n pozitív egészre

√ 1 1 1 1 n n( n + 1 − 1) ≤ 1 + + + · · · + ≤ 1 + n 1 − √ . n 2 3 n n 1 1 1 1 1033. [6] Legyen Sn = 1 + + + + · · · + . Mutassuk meg, hogy Sp + Sq − Spq ≤ 1, 2 3 4 n ahol p és q tetszőleges pozitív egészek.

20. Különféle algebrai feladatok

81

20. Különféle algebrai feladatok 1034. [1] Adottak az 1 − x, 2 − x, 3 − x, . . . , 100 − x számok. a) Számoljuk ki a szorzatukat, ha x = 77. b) Számoljuk ki az összegüket, ha x = 50,5. c) Mutassuk meg, hogy szorzatuk x = 0-ra és x = 101-re ugyanaz! 1035. [1] Számoljunk ügyesen! 4372 − 3632 a) =? 5372 − 4632 b) 96 · 104 =? c) Bontsuk prímtényezőire 899-et. d) Igazoljuk a 77782 − 22232 = 55 555 555 egyenlőséget. 1036. [2] Alakítsuk a kifejezést minél több egész együtthatós polinom szorzatává. a) x 4 + x 3 + 2x 2 + x + 1, b) x 3 + 2x 2 + 2x + 1, c) x 4 + 2x 3 + 2x 2 + 2x + 1, d) x 4 + 2x 3 + 3x 2 + 2x + 1, e) x 4 + x 2 + 1, f) x 10 + x 8 + x 6 + x 4 + x 2 + 1, g) x 4 + 4, h) x 4 − 7x 2 + 1, i) x 5 + x 4 + 1. 1037. [4] Alakítsuk szorzattá az x 8 − x 7 y + x 6 y 2 − x 5 y 3 + x 4 y 4 − x 3 y 5 + x 2 y 6 − xy 7 + y 8 kifejezést. 1038. [4] Bizonyítsuk be, hogy minden x racionális számhoz van olyan y racionális szám, amelyekre 2x 3 + 2y 3 − 3x 2 − 3y 2 + 1 = 0. Számítsuk ki a következő (teleszkopikus) összegeket, szorzatokat. (1039–1051. feladatok) 1039. [2] 1002 − 992 + 982 − 972 + · · · + 22 − 12 . 1 1 1 1 1040. [2] + + + ··· + . 1·2 2·3 3·4 99 · 100 A fejezet feladataiban x, y, z, a, b, c, . . . tetszőleges valós számokat jelent, ahol az adott kifejezések értelmezhetők.

Feladatok

82

1041. [3]

3 5 7 2n + 1 + + + ···+ 2 . 12 · 22 22 · 32 32 · 42 n · (n + 1)2

1042. [3] 1 · 1! + 2 · 2! + 3 · 3! + · · · + n · n!. 2 3 n 1 + + + ··· + . 1043. [3] 2! 3! 4! (n + 1)! 1044. [4]

3 4 5 n+2 + + + ···+ . 1! + 2! + 3! 2! + 3! + 4! 3! + 4! + 5! n! + (n + 1)! + (n + 2)!

1045. [3] √ 1046. [4]

1

1 1 1 √ +√ √ + ···+ √ √ . √ +√ 1+ 2 2+ 3 3+ 4 n+ n+1 1

1 1 1 √ + √ √ + ···+ √ . √ + √ √ (n + 1) n + n n + 1 2 1+1 2 3 2+2 3 4 3+3 4 √

1047. [5] tg α + 2 tg 2α + 4 tg 4α + · · · + 2n tg 2n α. 1048. [5]

1 1 1 + + ···+ . cos α · cos 2α cos 2α · cos 3α cos nα · cos(n + 1)α

1049. [5]

1 1 1 1 + + + ··· + . sin 2α sin 4α sin 8α sin 2n α





1 1 1 · 1+ · ...· 1 + . 1+ 2 3 100





1 1 1 1 − 2 · 1 − 2 · ...· 1 − . 2 3 1002

1050. [2] 1051. [2]

1 1 1 1 √ < 2 minden pozitív 1052. [4] Mutassuk meg, hogy √ + √ + √ + · · · + (n + 1) n 2 1 3 2 4 3 n egészre. √ √ √ √ n 2 6 12 n(n + 1) [3] 1053. Mutassuk meg, hogy + + + ··· + < . 3 5 7 2n + 1 2 1054. [4] Tudjuk, hogy 0 < x1 < 1 és xk+1 = xk − xk2 , k = 1, 2, . . . Mutassuk meg, hogy x12 + x22 + x32 + · · · + xn2 < 1. 1055. [4] x1 = 0, 001; xn+1 = xn − xn2 ; n = 1, 2, 3, . . . Mutassuk meg, hogy x1001 < < 0, 0005. 1056. [5] Számoljuk ki a következő tört értékét. √ √ (4 + 15)3/2 + (4 − 15)3/2 √ √ (6 + 35)3/2 − (6 − 35)3/2

20. Különféle algebrai feladatok

1057. [5] Számoljuk ki a következő tört értékét.



1 1 44 + · . . . · 204 + 24 + 4 4



1 1 14 + 34 + · . . . · 194 + 4 4

83

1 4

1 4

1058. [3] Számoljuk ki a következő kifejezés értékét.

1 · 2 · 4 + 2 · 4 · 8 + 3 · 6 · 12 + · · · + 100 · 200 · 400 1/3 =? 1 · 3 · 9 + 2 · 6 · 18 + 3 · 9 · 27 + · · · + 100 · 300 · 900 1059. [2] Ismertek a következő azonosságok: 0 (x + y)3 = x 3 + y 3 + 3xy(x + y)(x 2 + xy + y 2 ) , 1

(x + y)5 = x 5 + y 5 + 5xy(x + y)(x 2 + xy + y 2 ) , 2

(x + y)7 = x 7 + y 7 + 7xy(x + y)(x 2 + xy + y 2 ) . Vajon igaz-e az ezekkel analóg 3 (x + y)9 = x 9 + y 9 + 9xy(x + y)(x 2 + xy + y 2 ) azonosság? 1060. [4] Igazoljuk az alábbi azonosságokat: a) (a 2 +b2 +c2 )(x 2 +y 2 +z2 ) = (ax +by +cz)2 +(bz−cy)2 +(cx −az)2 +(ay −bx)2 b) (a + b)(b + c)(c + a) = (a + b + c)(ab + bc + ca) − abc = = ab(a + b) + bc(b + c) + ca(c + a) + 2abc c) (a − b)(b − c)(c − a) = a(b2 − c2 ) + b(c2 − a 2 ) + c(a 2 − b2 ) d) a 3 + b3 + c3 = (a + b + c)3 − 3(a + b)(b + c)(c + a) e) a(b + c)2 + b(c + a)2 + c(a + b)2 − 4abc = (a + b)(b + c)(c + a) f) (a 2 + b2 + c2 )2 = (ab + bc + ca)2 + (a 2 − bc)2 + (b2 − ca)2 + (c2 − ab)2 g) (a 2 + b2 )(x 2 + y 2 ) = (ax + by)2 + (ay − bx)2 h) (a 2 + b2 + c2 + d 2 )(x 2 + y 2 + z2 + t 2 ) = = (ax + by + cz + dt)2 + (bx − ay + dz − ct)2 + (cx − dy − az + bt)2 + +(dx + cy − bz − at)2 i) (a − b)3 + (b − c)3 + (c − a)3 = 3(a − b)(b − c)(c − a) j) (a + b)3 + (a + c)3 + (a + d)3 + (b + c)3 + (b + d)3 + (c + d)3 = = 3(a + b + c + d)(a 2 + b2 + c2 + d 2 ) k) (a + b + c)2 + (−a + b + c)2 + (a − b + c)2 + (a + b − c)2 = 4(a 2 + b2 + c2 ) = = (2a)2 + (2b)2 + (2c)2 l) (a + b + c)3 = (−a + b + c)3 + (a − b + c)3 + (a + b − c)3 + 24abc m) (a + b + c)5 = (−a + b + c)5 + (a − b + c)5 + (a + b − c)5 + 80abc(a 2 + b2 + c2 ) n) (9a 4 )3 + (1 − 9a 3 )3 + (3a − 9a 4 )3 = 1

Feladatok

84

o) p) q) r) s)

(1 + 6a 3 )3 + (1 − 6a 3 )3 + (−6a 2 )3 = 2 [a(a 3 + 2b3 )]3 + [b(2a 3 + b3 )]3 + (3a 2 b2 )3 = (a 6 + 7a 3 b3 + b6 )2 (3a 2 +5ab−5b2)3 +(4a 2 −4ab+6b2)3 +(5a 2 −5ab−3b2)3 = (6a 2 −4ab+4b2)3 (75b5 −a 5 )5 +(a 5 +25b5 )5 +(a 5 −25b5)5 +(10a 3 b2 )5 +(50ab4)5 = (a 5 +75b5 )5 [a(a 2 − 3b2 )]2 + [b(3a 2 − b2 )]2 = (a 2 + b2 )3

1061. [4] Mutassuk meg, hogy ha a + b + c = 0, akkor a 3 + b3 + c3 a 2 + b2 + c2 a 5 + b5 + c5 · = 3 2 5 a 5 + b5 + c5 a 2 + b2 + c2 a 7 + b7 + c7 a 3 + b3 + c3 a 4 + b4 + c4 · = =2· · b) 5 2 7 3 4

7 7 7 3 3 3 5 5 5 2 a +b +c a +b +c a +b +c · = c) 7 3 5 a)

1062. [2] Tudjuk, hogy x + ex = y + ey . Következik-e ebből, hogy sin x = sin y? 1063. [2] Legyen a + b + c = 0. Mutassuk meg, hogy ab + bc + ca ≤ 0. 1064. [4] Legyen (a + b + c)2 = 3(a 2 + b2 + c2 ). Igazoljuk, hogy a = b = c. a b c = = . Mutassuk meg, hogy a = b = c. b c a 1 1 1 Legyen abc = 1 és a + b + c = + + . Mutassuk meg, hogy ekkor az a, b, a b c c számok egyike 1-gyel egyenlő.

1065. [4] Legyen 1066. [4]

1 1 1 1067. [4] Legyen abc = 1 és a + b + c > + + . Bizonyítsuk be, hogy ekkor az a, b, a b c c számok egyike 1-nél nagyobb, a másik kettő pedig 1-nél kisebb. 1068. [4] Legyen a + b + c > 0, ab + bc + ca > 0, abc > 0. Mutassuk meg, hogy ekkor az a, b, c számok mindegyike pozitív. 1 1 1 + + > 0. Mutassuk meg, hogy ekkor a, ab bc ca b, c azonos előjelű számokat jelöl.

1069. [4] Legyen ab + bc + ca > 0 és

1 1070. [3] Mutassuk meg, hogy ha x + y + z − 2(xy + yz + zx) + 4xyz = , akkor az x, y, 2 1 z számok valamelyikének értéke . 2 1 1 1 1071. [4] Mutassuk meg, hogy ha x + = y + = z+ , akkor x = y = z vagy x 2 y 2 z2 = 1. y z x 1072. [4] Mutassuk meg, hogy ha x + y + z = 0 és xy + yz + zx = 0, akkor x = y = z.

20. Különféle algebrai feladatok

85

1 1 1 + + = 0 egyenletrendszernek x y z nincs megoldása a valós számok körében.

1073. [4] Bizonyítsuk be, hogy az x + y + z = 0,

1074. [4] Legyen

1 1 1 + + = 0. Mutassuk meg, hogy x y z y+z z+x x+y + + + 3 = 0. x y z

1075. [4] Tudjuk, hogy

y z x + + = 1. y+z z+x x+y

Mutassuk meg, hogy x2 y2 z2 + + = 0. y+z z+x x+y 1076. [5] Igazoljuk az alábbi azonosságokat: 1 1 1 + + = 0, (a − b)(a − c) (b − a)(b − c) (c − a)(c − b) a b c + + = 0, (a − b)(a − c) (b − a)(b − c) (c − a)(c − b) a2 b2 c2 + + = 1, (a − b)(a − c) (b − a)(b − c) (c − a)(c − b) a3 b3 c3 + + = a + b + c. (a − b)(a − c) (b − a)(b − c) (c − a)(c − b) (Az azonosságok általánosíthatók 3 helyett több változóra.) 1077. [4] a) Legyen b olyan valós szám, amelyre b3 = b + 1. Mutassuk meg, hogy ekkor b5 = b4 + 1. b) Legyen b olyan valós szám, amelyre b5 = b4 +1. Mutassuk meg, hogy ekkor b3 = b + 1.

1 2 1 1078. [4] Tudjuk, hogy x + = 3. Mivel egyenlő x 3 + 3 ? x x 1 1 1079. [4] Az x > 0 számra x 2 + 2 = 7 teljesül. Mutassuk meg, hogy x 5 + 5 is egész x x szám. 1080. [4] Az a, b, c valós számok kielégítik az a +b+c = 0, a 2 +b2 +c2 = 1 egyenleteket. Számítsuk ki az a 4 + b4 + c4 értékét. 1081. [4] Az a, b, c egész számokra a + b + c = 0 teljesül. Mutassuk meg, hogy  2 a 4 + b4 + c4 négyzetszám.

Feladatok

86

  2  1082. [5] Egy háromszög a, b, c oldalaira 2 a 8 + b8 + c8 = a 4 + b4 + c4 teljesül. Mutassuk meg, hogy a háromszög derékszögű. 1083. [4] Az x 2 +ax +b+1 = 0 egyenlet gyökeinek négyzete pozitív egész szám. Mutassuk meg, hogy a 2 + b2 összetett szám! 1084. [3] Tudjuk, hogy x + y = a, x 2 + y 2 = b, x 3 + y 3 = c. Mutassuk meg, hogy a 3 − − 3ab + 2c = 0. 1085. [4] Fejezzük ki a, b, c segítségével xyz-t, ha x + y + z = a, x 2 + y 2 + z2 = b, x 3 + y 3 + z3 = c. 1086. [4] Oldjuk meg a következő egyenletrendszert a valós számok körében. ⎫ x + y + z =3 ⎪ ⎬ x 2 + y 2 + z2 =5 ⎪ ⎭ x 4 + y 4 + z4 =17 1087. [4] Mutassuk meg, hogy az x 4 − 5x 3 − 4x 2 − 7x + 4 = 0 egyenletnek nincs negatív gyöke. 1088. [3] Mutassuk meg, hogy nincs olyan a és b valós szám, melyekre

√ 2 √ a + b · 3 = 99 999 + 111 111 · 3 teljesülne. 1089.

[3]

Mutassuk meg, hogy nincs olyan m és n pozitív egész szám, melyekre

√ m √ n 5+3· 2 = 3+5· 2 teljesülne.

1090.

[3]

Mutassuk meg, hogy nincsenek olyan a, b, c, d egész számok, melyekre

teljesülne.

a+b·

√ 2 √ 2 √ 2 + c+d · 2 =5+4· 2

√ 1993 1091. [4] Mutassuk meg, hogy a 26 + 5 számban a tizedesvessző utáni első 1993 jegy mindegyike nulla.  30  √ [5] értéke páratlan szám. 1092. Mutassuk meg, hogy 45 + 1975

20. Különféle algebrai feladatok

87

√ 19 √ 82 1093. [4] Milyen számjegy áll 15 + 220 + 15 + 220 tizedestört alakjában a tizedesvessző előtt? 1094. [7] Állapítsuk meg, hogy mely számjegyek állanak közvetlenül a tizedesvesszőtől

√ √ 1980 balra és jobbra a 2+ 3 szám tízes számrendszerben felírt alakjában. 1095. [3] Az (m + 1)x 2 + 2mx + m − 1 = 0 egyenletnek az m valós értékű paraméter mely értékeire lesz két valós gyöke? 1096. [3] Az x 2 + 5x + c = 0 egyenletnek van egy pozitív és egy negatív gyöke. Lehet-e c pozitív? 1097. [4] Az ax 2 + bx + c kifejezés minden x-re pozitív. Lehet-e c negatív? 1098. [4] Az ax 2 + bx + c = 0 egyenletnek nincs valós gyöke, és a + b + c < 0. Lehet-e c pozitív? 1099. [4] Az a, b, c valós számokra (a + b + c)c < 0 teljesül. Mutassuk meg, hogy b2 > > 4ac. 1100. [4] Vannak-e olyan a, b, c valós számok, melyekre az ábrán látható parabolák képlete ax 2 + bx + c, cx 2 + + ax + b, bx 2 + cx + a?

y

x 1101. [4] Vannak-e olyan a, b, c valós számok, melyekre az ábrán látható parabolák képlete ax 2 + bx + c, cx 2 + + ax + b, bx 2 + cx + a?

y

x 1102. [2] Mutassuk meg, hogy az ax 2 + bx + c = 0 egyenletnek van racionális gyöke, ha a, b és c 1-gyel, 2-vel, −3-mal egyenlők valamilyen sorrendben. 1103. [4] Mutassuk meg, hogy az ax 2 + bx + c = 0 egyenletnek nincs racionális gyöke, ha a, b és c páratlan egész számok. 1104. [5] Az ax 2 + bx + c = 0 egész együtthatós egyenletnek két különböző, 1-nél kisebb pozitív gyöke van. Határozzuk meg a legkisebb pozitív értékét.

88

Feladatok

1105. [3] Az ax 2 + bx + c kifejezés minden egész x-re egész szám. Mutassuk meg, hogy 2a, a + b, c is egész szám. 1106. [4] Az egész együtthatós ax 3 +bx 2 +cx +d polinom minden egész x-re 5-tel osztható értéket vesz fel. Mutassuk meg, hogy az együtthatók mindegyike osztható 5-tel. 1107. [4] Tudjuk, hogy 2a + 3b + 6c = 0. Mutassuk meg, hogy az ax 2 + bx + c = 0 egyenletnek van 0 és 1 közé eső gyöke. 1108. [4] Mutassuk meg, hogy a következő egyenleteknek van két különböző valós gyöke. (A paraméterek valós számok.) a) (x − a)(x − b) + (x − b)(x − c) + (x − c)(x − a) = 0, ahol a > b > c; b) (x − a)(x − c) + p(x − b)(x − d) = 0, ahol a > b > c > d és p tetszőleges valós szám, p = −1; c) a(x − b)(x − c) + b(x − c)(x − a) + c(x − a)(x − b) = 0, ahol a > b > c és a + b + c = 0; d) (a + b)(x − a)(x − b) + (b + c)(x − b)(x − c) + (c + a)(x − c)(x − a) = 0, ahol a 2 < b2 < c2 és a + b + c = 0; e) (ab + bc + ca)x 2 − (a 2 + b2 + 2c2 )x + c2 = 0, ahol ab + bc + ca = 0; f) ax 2 + bx + c = 0, ahol a = 0, ac + bc + c2 < 0. 1109. [4] Igazoljuk a következő egyenlőtlenséget.     a12 + b12 + a22 + b22 +· · ·+ an2 + bn2 ≥ (a1 + a2 + · · · + an )2 + (b1 + b2 + · · · + bn )2 1110. [5] Igazoljuk a√következő √ egyenlőtlenséget. √ √ a 2 + b2 + b2 + c2 + c2 + a 2 ≥ 2(a + b + c) 1111. [5] Mutassuk meg, hogy ha a, b, c pozitív valós számok, akkor mindig teljesül a következő egyenlőtlenség.   √ a 2 + b2 − ab + b2 + c2 − bc ≥ a 2 + c2 + ac számok. 1112. [5] Adva vannak az a, b, c pozitív  Határozzuk meg, hogy az √ f (x) = a 2 + x 2 + (b − x)2 + c2 függvény x mely értékére veszi fel legkisebb értékét.  √ 1113. [5] Határozzuk meg az f (x) = 2x 2 + 2x + 1 + 2x 2 − 2x + 1 függvény minimumát.   √ 17 [5] 2 2 1114. Mutassuk meg, hogy 3x + 2x + 1 + 3x − 4x + 2 ≥ . 3   √ 1115. [5] Mutassuk meg, hogy (a + c)2 + b2 + (a − c)2 + b2 ≥ 2 a 2 + b2 .

20. Különféle algebrai feladatok

89

1116. [5] Az x, y, z pozitív valós számokra teljesülnek a következő egyenletek: ⎫ y2 ⎪ 2 ⎪ =25 ⎪ x + xy + ⎪ ⎪ 3 ⎬ 2 y 2 + z =9 ⎪ ⎪ ⎪ 3 ⎪ ⎪ ⎭ 2 2 z + zx + x =16 Számoljuk ki xy + 2yz + 3zx értékét. 1117. [4] Oldjuk meg a következő egyenletrendszert a valós számok halmazán. ⎫ 2x + x 2 y =y ⎪ ⎬ 2y + y 2 z =z ⎪ ⎭ 2z + z2 x =x 1118. [4] Oldjuk meg a

1119. [4]

8x(2x 2 − 1)(8x 4 − 8x 2 + 1) = 1 egyenletet a valós számok halmazán.     Mutassuk meg, hogy a 1 − a 2 + 1 − b2 + 1 − c2 ≤ 9 − (a + b + c)2 egyenlőtlenség teljesül minden olyan, valós számokból álló a, b, c számhármasra, amelyre értelmezve vannak a benne szereplő kifejezések.

1120. [5] Mutassuk meg, hogy ha 0 ≤ a, b, c ≤ 1, akkor    √ a(1 − b)(1 − c) + b(1 − a)(1 − c) + c(1 − a)(1 − b) ≤ 1 + abc. 1121. [5] Mutassuk meg, ha x és y 1-nél kisebb pozitív számok, akkor x−y < 1. 1 + xy 1122. [5] Mutassuk meg, hogy négy különböző valós számból mindig kiválasztható kettő, x és y úgy, hogy x−y 0≤ ≤ 1. 1 + xy 1123. [5] Mutassuk meg, hogy öt különböző valós számból mindig kiválasztható kettő, x és y úgy, hogy 1 + xy > 1. x−y 1124. [5] Mutassuk meg, hogy 13 különböző valós számból mindig kiválasztható kettő, x és y úgy, hogy  √ √ x−y 2− 3 0< < √ (= 2 − 3). 1 + xy 2+ 3

Feladatok

90

1125. [4] Mutassuk meg, hogy a + b − 2ab < 1, ha 0 < a < 1, 0 < b < 1. 1126. [5] Mutassuk meg, hogy a(1 − b) + b(1 − c) + c(1 − a) < 1, ahol 0 < a < 1, 0 < b < 1, 0 < c < 1. 1127. [5] Mutassuk meg, hogy minden n pozitív egészre √  √  √  √          n + 3 n + 4 n + · · · + n n = log2 n + log3 n + log4 n + · · · + logn n . 1128. [4] Mutassuk meg, hogy nincsenek olyan p(x) és q(x) polinomok, melyekre sin x = p(x) teljesülne minden valós x számra. = q(x) 1129. [5] Mutassuk meg, hogy nincsenek olyan p(x) és q(x) polinomok, melyekre 1 + 1 1 1 p(n) + + + ···+ = teljesülne minden n pozitív egész számra. 2 3 n q(n) 1130. [4] Számítsuk ki az S =

∞ 3k + 1 k=1

3k

összeg értékét!

1131. [3] a1 , a2 , a3 , . . . , a1998 , a1999 , a2000 jelöli az 1, 2, 3, . . . , 1998, 1999, 2000 számok valamilyen sorrendjét. Legfeljebb mekkora lehet az |a1 − a2 | + |a2 − a3 | + |a3 − a4 | + · · · + |a1999 − a2000 | + |a2000 − a1 | összeg értéke? 1132. [3] Mutassuk meg, hogy ha az     1 − x + x 2 − x 3 + · · · − x 99 + x 100 · 1 + x + x 2 + x 3 + · · · + x 99 + x 100 szorzatot polinommá alakítjuk, akkor olyan polinomot kapunk, melyben x-nek csak páros kitevőjű hatványai fordulnak elő. 1133. [4] Mutassuk meg, hogy a cos 32x + a31 cos 31x + · · · + a2 cos 2x + a1 cos x kifejezés pozitív és negatív értékeket is felvesz. 1134. [3] A következő módon adott függvényeknek egy sorozata: 1 , fn (x) = f0 (fn−1 (x)), n = 1, 2, 3, . . . . f0 (x) = 1−x f1994 (1994) =?

21. Függvényegyenletek Keressük meg az összes olyan, a valós számokon értelmezett folytonos f (x) függvényt, mely kielégíti a felírt függvényegyenletet. (1135–1178. feladatok) 1135. [3] f (x 1990 ) = x 1991 .

21. Függvényegyenletek

91

1136. [4] 2f (x) + 3f (1 − x) = 4x − 1. 1137. [4] f (x) − f (−x) = x 2 .

  1138. [4] (x + 1) · f (1 − x) + (x − 1) · f (x + 1) = 2x x 2 − 1 . 1139. [4] f (1 − x) + 2f (1 + x) = x + 3.   1140. [4] f (x 2 + x + 1) + 2f x 2 − x + 1 = 3x 2 − x + 6. 1141. [4] f (x − 1) − f (1 − x) = x.

1 1142. [4] 2f (x) − 3f = x 2 , x = 0. x

1 [4] + 2f (x) = x − 1, x = 0. 1143. x · f x

1 [4] 1144. f (x) + x + f (1 − x) = 1. 2 1145. [4] 2f (x) + f (1 − x) = x 2 .

x−1 [5] 1146. f (x) + f = 1 + x, x = 0, x = 1. x

1 1147. [4] f + f (1 − x) = x, x = 0, x = 1. x 1148. [4] x 2 · f (x) + f (1 − x) = 2x − x 4 . 1149. [4] (1 − x) · f (x) + 2x · f (1 − x) = −3x 2 + 4x + 1.

1 x−1 [4] + f (x) = − x + 1, x = 0, x = 1. 1150. f x x 1151. [4] f (x + y) − f (x − y) = 4xy. 1152. [4] f (xy) + x + y = xy + f (x) + f (y). 1153. [4] f (x) · f (y) = f (x + y − xy), f (1) = 0. 1154. [4] f (x) · f (y) = f (x − y). 1155. [4] f (x + 1) ≤ x ≤ f (x) + 1. 1156. [4] f (x) − x ≤ x 2 ≤ f (x − 1) + x. 1157. [4] f (x) ≤ x és f (x + y) ≤ f (x) + f (y). 1158. [4] f (x + y) =

f (x) + f (y) . 1 + f (x) · f (y)

1159. [4] (x + y) · f (xy) = f (x) + f (y).

92

Feladatok

1160. [4] f (x) · f (y) = f (x + xy). 1161. [4] f (x) · f (y) + f (x) + f (y) + 1 = xy. 1162. [4] f (x) · f (y) + x · f (y) = 2y · f (x). 1163. [4] f (x) · f (y) − xy = f (x) + f (y) − 1. 1164. [4] x · f (y) + y · f (x) = (x + y) · f (x) · f (y). 1165. [4] f (x) · f (y) − xy = x · f (x) + y · f (y) − x 2 − y 2 + 1. 1166. [5] f (x + y) + 2f (x − y) = 3f (x) − y. 1167. [5] 3f (x) − 2f (x − y) − f (x + y) = 0, f (0) = 0. 1168. [5] f (x + y) + f (x − y) = 2f (x) · cos y. 1169. [5] f (x − y) = f (x) · f (y) + f (x) · cos y + f (y) · cos x − sin x · sin y. 1170. [5] f (x + y) = f (x) cos y + f (y) cos x. 1171. [6] f (x + y) = f (x) + f (y). 1172. [6] f (x + y) = f (x) · f (y). 1173. [6] f (x · y) = f (x) + f (y), ha x > 0, y > 0. 1174. [6] f (x · y) = f (x) · f (y), ha x > 0, y > 0. 1175. [6] f (x + y) = f (x) + f (y) + 1.

x + y  f (x) + f (y) 1176. [6] f = . 2 2 1177. [7] f (x · y) = x · f (y) + y · f (x), ha x > 0, y > 0. 1178. [6] f (x · y) = f (x) · f (y) − f (x + y) + 1, f (1) = 2. 1179. [5] Van-e olyan f (x) és g(x) függvény, hogy minden x-re és y-ra f (x) · g(y) = x + y + 1? 1180. [5] Van-e olyan f (x) és g(x) függvény, hogy minden x-re és y-ra f (x) · g(y) = x 2 y 2 + 1? 1181. [5] Van-e olyan f (x) és g(x) függvény, hogy minden x-re és y-ra x 2 + xy + y 2 = f (x) + g(y)? 1182. [5] Van-e olyan, minden valós x-re értelmezett h(x) függvény, hogy minden x-re h(f (x)) + h(g(x)) = g(f (x)), ahol f (x) = x 2 + x + 2 és g(x) = x 2 − x + 2? 1183. [5] Van-e megoldása az f (g(x)) = g(f (x)) egyenletnek, ha f (x) = x 2 + 1 és g(x) = = 3 − 3x − x 2 ?

21. Függvényegyenletek

93

1184. [5] Adott folytonos f (x) függvény esetén az f (x) = x egyenletnek nincs megoldása. Van-e megoldása az f (f (x)) = x egyenletnek? 1185. [7] Van-e olyan, minden valós x-re értelmezett f (x) függvény, mely minden x-re kielégíti az a) f (f (x)) = x 2 − 2, illetve b) f (f (x)) = −x 3 függvényegyenletet? 1186. [5] Legyen f (x) = x 2 − 2. Bizonyítsuk be, hogy az f (f (f (x))) = x egyenletnek nyolc különböző megoldása van. 1187. [4] Az f (x) függvény folytonos, és f (x) = f (2x) teljesül minden valós x-re. Határozzuk meg az összes ilyen függvényt. 1188. [4] Az f (x, y, z) valós számokon értelmezett folytonos függvény változóinak minden értékére kielégíti az f (x, y, z) = 2 · f (y, z, x) függvényegyenletet. Határozzuk meg az összes ilyen függvényt. 1189. [5] Legyenek f (x) és g(x) minden valós x-re értelmezett függvények, amelyekre teljesül, hogy f (x − y) = f (x) · g(y) − g(x) · f (y), g(x − y) = g(x) · g(y) − f (x) · f (y). Számítsuk ki f (0) és g(0) értékét. 1190. [5] Legyenek f (x) és g(x) minden valós x-re értelmezett függvények, amelyekre teljesül, hogy f (x + y) = f (x) · g(y) + g(x) · f (y), g(x + y) = g(x) · g(y) − f (x) · f (y). Számítsuk ki f (0) és g(0) értékét. 1191. [7] Legyenek f (x) és g(x) minden valós x-re értelmezett, nem konstans, differenciálható függvények, amelyekre teljesül, hogy f (x + y) = f (x) · g(y) + g(x) · f (y), g(x + y) = g(x) · g(y) − f (x) · f (y), valamint g (0) = 0. Mutassuk meg, hogy (f (x))2 + (g(x))2 = 1 minden valós x-re. Melyek azok a pozitív egész számokon értelmezett valós függvények, amelyek kielégítik a következő függvényegyenleteket? (1192–1197. feladatok) 1192. [5] nf (n)−1 = (n − 1)f (n−1) . 1193. [5] f (n + m) = f (n) + f (m) + nm és f (1) = 1. 1194. [5] f (n + m) = f (n) + f (m) + nm + 1 és f (1) = 1. 1195. [5] f (n · m) = f (n) · f (m), és f (n) > f (m), ha n > m, továbbá f (2) = 2 és f (n) minden értéke egész szám.

Feladatok

94

1196. [5] f (n + m) + f (n − m) = f (3n), ha n ≥ m, továbbá a függvény a 0 helyen is értelmezve van. 1197. [5] f (k · m) + f (k · n) − f (k) · f (m · n) ≥ 1, továbbá a függvény a 0 helyen is értelmezve van. 1198. [5] A pozitív egészeken értelmezett f (n) függvényről a következőket tudjuk: f (n) = n − 10, ha n > 100; továbbá f (n) = (f (n + 11)), ha n ≤ 100. Határozzuk meg f (18) értékét. 1199. [5] A pozitív egészeken értelmezett f (n) függvényről a  következőket tudjuk:  f (f (n)) = 4n − 3, minden pozitív egész n-re; továbbá f 2k = 2k+1 − 1 minden nemnegatív egész k-ra. Határozzuk meg f (1985) értékét. 1200. [5] Az f (x) folytonos függvényre f (x) · f (f (x)) = 1 és f (1000) = 999. f (500) =? Határozzuk meg az összes olyan f (x) polinomot, mely minden valós x-re kielégíti a következő függvényegyenleteket. (1201–1205. feladatok) 1201. [5] (x − 1) · f (x + 1) = (x + 2) · f (x). 1202. [5] x · f (x − 1) = (x − 2) · f (x). 1203. [5] x · f (x − 1) = (x + 1) · f (x). 1204. [5] x · f (x − 1) = (x − 12) · f (x) és f (12) = 12!. 1205. [5] f (x) =

1 [f (x + 1) + f (x − 1)] és f (0) = 0. 2

1206. [5] Az f (n) n-edfokú polinom olyan, hogy f (k) = k/(k + 1) a k = 0, 1, 2, . . . , n értékekre. Határozzuk meg f (n + 1) értékét.     [5] 1207. Az f (x) és g(x) polinomok között fennáll az f x 2 − x + 1 = g x 2 + x + 1 egyenlőség minden x valós számra. Mutassuk meg, hogy mindkét polinom azonosan konstans. 1208. [4] Bizonyítsuk be, hogy minden valós függvény előállítható egy páros és egy páratlan függvény összegeként. 1209. [5] Mutassuk meg, hogy az f (x) = x 2 függvény nem állítható elő két periodikus függvény összegeként. Mit állíthatunk az f (x) = x 3 függvény esetében? 1210. [7] Van-e olyan, a valós számokon értelmezett folytonos f (x) függvény, melynek racionális helyen irracionális, irracionális helyen racionális az értéke? 1211. [6] Van-e olyan, a valós számokon értelmezett folytonos f (x) függvény, melyre f (x) akkor és csak akkor racionális, ha f (x + 1) irracionális?

22. Vektorok a geometriában

95

Mutassuk meg, hogy az alábbi függvényegyenleteket kiegyenlítő függvények periodikusak. (1212–1215. feladatok) f (x) − 1 , ahol a ∈ R konstans, f (x) = 1. f (x) + 1 √ f (x + 1) + f (x − 1) = 2 · f (x).

1212. [5] f (x + a) = 1213. [5]

1214. [5] f (x + a) · f (x) = b, ahol ab = 0, a, b ∈ R konstansok. 1215. [5] f (x + a) =

f (x) · cos φ − sin φ π , ahol φ = , f (x) · sin φ + cos φ = 0. f (x) · sin φ + cos φ n

Mutassuk meg, hogy a következő függvények nem periodikusak. (1216–1219. feladatok) 1216. [5] a) f (x) = cos x 2 , illetve b) f (x) = sin x 2 . √ 1217. [5] a) f (x) = cos x, illetve √ b) f (x) = sin x. 1218. [5] a) f (x) = cos x + cos ax, a ∈ R\Q , illetve b) f (x) = sin x + sin ax, a ∈ R\Q . √ 1219. [5] f (x) = cos x · cos 2x.

22. Vektorok a geometriában 1220. [3] Egy szabályos sokszög csúcsai A1 , A2 , . . . , An , köré írt körének középpontja O, X pedig egy tetszőleges pont. Mutassuk meg, hogy a) O-ból a csúcsokba mutató vektorok összege nullvektor; −→ b) X-ből a csúcsokba mutató vektorok összege n · XO. 1221. [3] Egy 1994-oldalú szabályos sokszög belsejében adott az A és a B pont. A-ból → a csúcsokba mutató vektorok összege legyen − a , B-ből a csúcsokba mutató − → − → → vektorok összege legyen b . Mikor mondhatjuk, hogy |− a | < | b |? 1222. [4] Mutassuk meg, hogy az n-oldalú szabályos sokszög csúcsaiba lehet nullától különböző számokat írni úgy, hogy bármely szabályos sokszög esetén, melynek csúcsai ezek közül a csúcsok közül valók, a csúcsokban álló számok összege mindig nulla legyen. 1223. [4] Egy szabályos sokszög csúcsai A1 , A2 , . . . , An , köré írt körének középpontja O, a kör sugara r, és X egy tetszőleges pont, d = |OX|. Mutassuk meg, hogy |A1 X|2 + |A2 X|2 + · · · + |An X|2 = n(r 2 + d 2 ).

Feladatok

96

1224. [4] Az ABC szabályos háromszög köré írt körnek egy tetszőleges pontja P . Mutassuk meg, hogy a P A2 +P B 2 +P C 2 összeg értéke P választásától függetlenül állandó. 1225. [6] Az ABC szabályos háromszög köré írt körnek egy tetszőleges pontja P . Mutassuk meg, hogy a P A4 +P B 4 +P C 4 összeg értéke P választásától függetlenül állandó. 1226. [6] Egy n oldalú szabályos sokszög csúcsai A1 , A2 , . . . , An ; a sokszög köré írt kör tetszőleges pontja P . Mutassuk meg, hogy a |P A1 |4 + |P A2 |4 + · · · + |P An |4 összeg értéke a P pont választásától független, állandó érték. 1227. [4] Az ABCD rombusz egyik szöge 60◦ , beírt körének egy tetszőleges pontja P . Mutassuk meg, hogy a P A2 + P B 2 + P C 2 + P D2 összeg értéke P választásától függetlenül állandó. 1228. [4] Az a, b, c oldalú ABC háromszög A csúcsát B-re, B csúcsát C-re, C csúcsát A-ra tükrözzük. Az így kapott három pont egy x, y, z oldalú háromszög csúcsait alkotja. Mutassuk meg, hogy x 2 + y 2 + z2 = 7 a 2 + b2 + c2 . 1229. [4] Az a, b, c oldalú háromszög súlyvonalainak hossza: sa , sb , sc . Mutassuk meg,  3 2 a + b2 + c2 . hogy sa2 + sb2 + sc2 = 4 1230. [4] Az ABC háromszög súlypontja S. Mutassuk meg, hogy   |AB|2 + |BC|2 + |CA|2 = 3 |SA|2 + |SB|2 + |SC|2 . 1231. [4] Az ABCD tetraéder súlypontja S. Mutassuk meg, hogy |AB|2 + |AC|2 + |AD|2 + |BC|2 + |BD|2 + |CD|2 =   = 4 |SA|2 + |SB|2 + |SC|2 + |SD|2 . 1232. [4] Az ABC háromszög súlypontja S, P tetszőleges pont. Mutassuk meg, hogy |P A|2 + |P B|2 + |P C|2 = 3|P S|2 + |SA|2 + |SB|2 + |SC|2 . 1233. [4] Adott az ABC háromszög. Mely P pontra lesz a P A2 + P B 2 + P C 2 összeg értéke minimális, ha a) P egy adott egyenesen helyezkedik el; b) P a sík tetszőleges pontja lehet? 1234. [5] Adottak a síkon az A1 , A2 , . . . , An pontok. A sík mely P pontjára lesz minimális a P A21 + P A22 + · · · + P A2n összeg értéke? 1235. [4] Az ABC háromszög súlypontja S, körülírt körének sugara r. Tudjuk, hogy SA2 + SB 2 + SC 2 = 3r 2 . Határozzuk meg a háromszög szögeit.

22. Vektorok a geometriában

97

1236. [4] Az ABCD konvex négyszög átlóinak felezőpontja M és N. Mutassuk meg, hogy AB 2 + BC 2 + CD2 + DA2 = AC 2 + BD2 + 4MN 2 . 1237. [4] Az ABCD húrnégyszög AC átlójának felezőpontja a BD átlón van. Mutassuk meg, hogy AB 2 + BC 2 + CD2 + DA2 = 2BD2 . 1238. [4] Az ABCD húrnégyszög köré írt kör sugara r. Igazoljuk, ha AB 2 + CD2 = 4r 2 , akkor a négyszög átlói merőlegesek. 1239. [4] Mutassuk meg, hogy egy négyszög átlói akkor és csak akkor merőlegesek egymásra, ha a szemben fekvő oldalak négyzetének összege megegyezik. 1240. [4] Mutassuk meg, hogy egy paralelogramma átlóinak szorzata nagyobb, mint két szomszédos oldala négyzetének különbsége. 1241. [6] Adott a térben négy pont: A, B, C, D. Mutassuk meg, hogy AC 2 + BD2 + AD2 + BC 2 ≥ AB 2 + CD2 . Mikor áll fenn az egyenlőség? 1242. [5] Az O középpontú, egységsugarú körön adottak az A1 , A2 , . . . , An pontok úgy, → −−→ −−→ −−→ − hogy OA1 + OA2 + · · · + OAn = 0 . Mutassuk meg, hogy tetszőleges P pontra |P A1 | + |P A2 | + · · · + |P An | ≥ n. 1243. [4] Az ABC háromszög oldalai a, b, c, körülírt körének középpontja O, sugara r, magasságpontja M, és d = |OM|. Mutassuk meg, hogy −→ −→ −→ −−→ a) OA + OB + OC = OM;   b) d 2 = 9r 2 − a 2 + b2 + c2 . 1244. [4] Az ABC háromszög oldalai a, b, c, körülírt körének sugara r. Mutassuk meg, hogy a háromszög pontosan akkor hegyesszögű, derékszögű, illetve tompaszögű, ha a 2 + b2 + c2 − 8r 2 pozitív, nulla, illetve negatív. 1245. [4] Az egységsugarú, O középpontú körbe írtuk a tompaszögű ABC háromszöget. −→ −→ −→ Mutassuk meg, hogy |OA + OB + OC| > 1. 1246. [4] Egy háromszög oldalai a, b, c, körülírt körének középpontja O, sugara r, súly 1 2 pontja S. Mutassuk meg, hogy OS 2 = r 2 − a + b2 + c2 . 9 1247. [4] Mutassuk meg, hogy az ABC háromszögben az A és B csúcsból induló súlyvonalak akkor és csak akkor merőlegesek egymásra, ha a 2 + b2 = 5c2 , ahol a, b, c a háromszög A, B, ill. C csúcsával szemközti oldalai. 1248. [4] Mutassuk meg, hogy egy háromszög pontosan akkor derékszögű, ha súlyvonalaira (megfelelően választva azokat) fennáll az sa2 + sb2 = 5sc2 összefüggés. 1249. [4] Az ABC háromszögben az A és B csúcsból induló súlyvonalak merőlegesek egymásra. Bizonyítsuk be, hogy a háromszög C csúcsánál levő γ szögre 4 cos γ ≥ . 5

98

Feladatok

1250. [4] Határozzuk meg egy egyenlő szárú háromszög szárszögének cosinusát, ha az alap végpontjaiból induló súlyvonalak merőlegesek egymásra. 1251. [4] Egy háromszög szögei α, β, γ . Mutassuk meg, hogy 3 a) cos α + cos β + cos γ ≤ ; 2 3 b) cos 2α + cos 2β + cos 2γ ≥ − . 2 1252. [4] Mennyi az alábbi kifejezések minimuma, maximuma? a) 3 sin x + 4 cos x; b) 6 sin x · cos y + 2 sin x · sin y + 3 cos x. 1253. [4] Adott nyolc valós szám: a, b, c, d, e, f , g, h. Mutassuk meg, hogy az ac + + bd, ae + bf , ag + bh, ce + df , cg + dh, eg + f h számokból legalább az egyik nemnegatív. 1254. [6] Adott a síkon négy vektor, a, b, c, d, melyekre a + b + c + d = 0. Mutassuk meg, hogy |a| + |b| + |c| + |d| ≥ |a + d| + |b + d| + |c + d|. 1255. [6] Mutassuk meg, hogy a tetszőlegesen választott a, b, c vektorokra teljesül az |a| + |b| + |c| + |a + b + c| ≥ |a + b| + |b + c| + |c + a| egyenlőtlenség. (Hlawka-féle egyenlőtlenség.) 1256. [4] Egy tetszőleges P pontot tükrözzünk először egy A pontra, majd a tükörképet egy B pontra, az így nyert képet pedig egy C pontra; tovább folytatva újra Ara, B-re és végül C-re. Bizonyítsuk be, hogy a hatodik tükrözés visszaviszi a P pontot az eredeti helyzetbe. 1257. [3] Egy tetszőleges P pontot tükrözünk egy paralelogramma egy csúcsára, majd az eredményt egy vele szomszédos csúcsra, és így körbe a paralelogramma minden csúcsára. Bizonyítsuk be, hogy a negyedik tükrözés visszavisz az eredeti pontba. 1258. [3] Egy konvex hatszög másodszomszédos oldalainak felezőpontjait összekötve két háromszöget kapunk. Igazoljuk, hogy ezek súlypontjai egybeesnek. 1259. [3] Mutassuk meg, hogy egy konvex négyszög szemközti oldalfelező pontjait összekötő szakaszok és az átlók felezőpontját összekötő szakasz egy közös pontban metszi egymást. 1260. [3] Mutassuk meg, hogy egy tetraéder szemközti éleinek felezőpontját összekötő szakaszok metszik egymást. 1261. [3] Egy háromszög oldalaira kifelé paralelogrammákat rajzoltunk, s összekötöttük a szomszédos külső csúcsokat. Mutassuk meg, hogy az így kapott hatszög másodszomszédos oldalaiból szerkeszthető háromszög. 1262. [3] Mutassuk meg, hogy egy háromszög súlyvonalaiból mindig szerkeszthető háromszög.

22. Vektorok a geometriában

99

1263. [3] A1 B1 C1 D1 és A2 B2 C2 D2 azonos körüljárású paralelogrammák. Mutassuk meg, hogy az A1 A2 , B1 B2 , C1 C2 , D1 D2 szakaszok felezőpontjai egy paralelogramma csúcsai. 1264. [4] Legyenek A1 B1 C1 és A2 B2 C2 azonos körüljárású szabályos háromszögek. Mutassuk meg, hogy az A1 A2 , B1 B2 , C1 C2 szakaszok felezőpontjai egy szabályos háromszög csúcsai. 1265. [4] Legyenek A1 B1 C1 és A2 B2 C2 azonos körüljárású egyenlő szárú derékszögű háromszögek! Mutassuk meg, hogy az A1 A2 , B1 B2 , C1 C2 szakaszok felezőpontjai egy egyenlő szárú derékszögű háromszög csúcsai. 1266. [4] Az ABCD konvex négyszög AB oldalának felezőpontja E, DC oldalának felezőpontja F . Mutassuk meg, hogy az ED, EC, FA, FB szakaszok felezőpontjai egy paralelogramma csúcsai. 1267. [4] Mutassuk meg, hogy a háromszög magasságpontja, súlypontja és köré írt körének középpontja egy egyenesre illeszkedik. (Euler-egyenes) 1268. [3] Az ABCD, AEF G, ADF H , F KLE, BKLC négyszögek paralelogrammák. Mutassuk meg, hogy AF H G is paralelogramma. 1269. [3] Az OAB háromszög OA és AB oldalát osszuk fel három egyenlő részre, az Ahoz közelebbi harmadolópontok F és E. Az OE és BF szakaszok a háromszög −→ −→ −→ belsejében a P pontban metszik egymást. Írjuk fel az OP vektort az OA és OB vektorok segítségével. 1270. [4] Az ABC háromszög kerületén pozitív körüljárás szerint haladva válasszuk ki mindegyik oldal első harmadolópontját, ezek: A1 , B1 , C1 . Most az A1 B1 C1 háromszögre ismételjük meg ezt az eljárást! Igazoljuk, hogy az így kapott A2 B2 C2 háromszög hasonló az ABC háromszöghöz, amelyből kiindultunk. 1271. [3] Egy paralelogramma oldalaira kifelé négyzeteket írtunk. Bizonyítsuk be, hogy ezek középpontjai egy négyzet csúcsai. 1272. [3] Egy egyenlő szárú trapéz oldalaira kifelé téglalapokat szerkesztünk; ezeknek a téglalapoknak a másik oldala egyenlő a trapéz szemközti oldalával. Mutassuk meg, hogy a téglalapok középpontjai egy négyzet csúcsai. 1273. [5] Konvex négyszög oldalai fölé építsünk négyzeteket az adott oldalhosszal! Mutassuk meg, hogy a szemközti négyzetek középpontját összekötő szakaszok egyenlő hosszúak és merőlegesek egymásra. 1274. [3] Az ABC háromszög AB és AC oldalaira kifelé szabályos háromszöget rajzolunk, az ABP és ACR háromszögeket. Az AP , AR és BC szakaszok felezőpontjai M, N és D. Bizonyítsuk be, hogy MND szabályos háromszög. 1275. [3] Az ABCD paralelogramma BC és CD oldalaira kifelé a BCM és CDN szabályos háromszögeket rajzoljuk. Mutassuk meg, hogy az AMN háromszög szabályos.

100

Feladatok

1276. [4] Rajzoljunk egy szakasz mint oldal fölé szabályos háromszöget. Majd osszuk a szakaszt két tetszőleges részre, és mindegyik rész mint oldal fölé ismét rajzoljunk szabályos háromszöget, de az elsővel ellentétes félsíkban. Bizonyítsuk be, hogy a három szabályos háromszög középpontja egy újabb szabályos háromszög csúcsait alkotja. 1277. [3] Az ABC háromszög AB és AC oldalaira kifelé szerkesztett négyzetek középpontjai O1 ill. O2 , és a BC oldal felezőpontja D. Bizonyítsuk be, hogy O1 D és O2 D egyenlő hosszú és merőleges egymásra. 1278. [4] Az ABC háromszög AB és AC oldalai fölé kifelé az AC1 C2 B és AB1 B2 C négyzeteket írjuk. Bizonyítandó, hogy a B2 C2 szakasz felezőpontja a BC átmérőjű körön fekszik. 1279. [4] Rajzoljunk egy háromszög oldalai fölé kifelé négyzeteket. Bizonyítsuk be, hogy a négyzeteknek a háromszögcsúcsoktól különböző csúcsai olyan hatszöget határoznak meg, amelynek minden második oldala a háromszög egy-egy súlyvonalának kétszerese. 1280. [5] Az ABC háromszög oldalaira kifelé írunk két egyenlő szárú derékszögű háromszöget, a CAN és a CBM háromszögeket, ahol a derékszög A-nál és B-nél van. Az AM és BN szakaszok metszéspontja legyen H . Bizonyítsuk be, hogy CH ⊥AB. 1281. [5] Az r sugarú körben a közös pont nélküli AB, CD és EF húrok hossza legyen r. A BC, DE, FA húrok felezőpontjai P , R, S. Mutassuk meg, hogy P RS szabályos háromszög. Hogyan lehetne általánosítani a feladatot? 1282. [6] Egy háromszög oldalaira kifelé szabályos háromszögeket írunk. Bizonyítsuk be, hogy ezek középpontjai egy szabályos háromszög csúcsai. 1283. [7] Legyenek ABC, AA1 A2 , BB1 B2 , CC1 C2 egyező körüljárású szabályos háromszögek; az A2 B1 , B2 C1 , C2 A1 szakaszok felezőpontjai X, Y , Z. Mutassuk meg, hogy XY Z szabályos háromszög.

23. Területátalakítások

101

23. Területátalakítások A következő feladatokban mutassuk meg, hogy a vonalkázott részek területe egyenlő a jelöletlen részek területével. (1284–1292. feladatok) 1284. [2] A paralelogramma egy belső pontját kötöttük össze a csúcsokkal.

1285. [2] Konvex négyszögben oldalfelező pontokat kötöttünk össze a csúcsokkal.

1286. [2] Konvex négyszög szemközti oldalfelező pontjait kötöttük össze.

1287. [4] Egy konvex négyszög minden oldalát 8 egyenlő részre osztjuk, majd a szemközti oldalak megfelelő osztópontjait összekötjük. Az így kapott 64 darab kis négyszöget sakktáblaszerűen kiszíneztük.

102

1288. [2] A hatszög szemközti oldalai párhuzamosak és egyenlők.

1289. [2] Az egyenesek párhuzamosak a négyzet oldalaival, illetve átlóival.

1290. [2] Szabályos csillagötszög.

Feladatok

23. Területátalakítások

103

1291. [2] Szabályos nyolcszög.

1292. [3] Szabályos háromszög belső pontját a csúcsokkal kötöttük össze, és merőlegeseket állítottunk az oldalakra.

A következő feladatokban mutassuk meg, hogy a vonalkázott részek területe egyenlő a pöttyözött részek területével. (1293–1300. feladatok) 1293. [2] Egy paralelogramma átlójának egyik pontján át párhuzamosakat húztunk az oldalakkal.

1294. [2] Trapézban az átlókat rajzoltuk meg.

104

1295. [2] Trapézban a párhuzamos oldalak egy-egy tetszőleges pontját a szemközti csúcsokkal kötöttük össze.

1296. [2] Szabályos hatszögben két szomszédos csúcsot egy-egy oldalfelező ponttal kötöttünk össze az ábra szerint.

1297. [2] Téglalapban oldalfelezőpontokat csúcsokkal kötöttünk össze az ábra szerint.

1298. [3] A téglalap két szomszédos oldalának egyegy tetszőleges pontját a szemközti csúcsokkal kötöttük össze.

1299. [3] Konvex négyszögben a két szemben levő oldal felezőpontját a szemközti oldalak csúcspontjaival kötöttük össze.

Feladatok

23. Területátalakítások

105

1300. [4] Paralelogramma két belső pontját a csúcsokkal kötöttük össze.

A következő feladatokban mutassuk meg, hogy a vonalkázott részek területe egyenlő a pöttyözött részek területével. (1301–1311. feladatok) 1301. [3] Konvex négyszögben szomszédos oldalfelező pontokat kötöttünk össze.

1302. [3] Téglalapot egyik átlója két háromszögre bont. Az ezek egyikébe írt kör középpontján át párhuzamosakat húztunk az oldalakkal.

1303. [3] Egy körlemez negyedét határoló körív harmadolópontjaiból merőlegeseket bocsátottunk az egyik határoló sugárra, továbbá az egyik harmadolópontot összekötöttük a kör középpontjával az ábra szerint.

106

1304. [3] Egy körlemez negyedében két félkörív.

1305. [3] Egy körlemez negyedében félkört rajzoltunk és berajzoltuk a szögfelezőt.

1306. [3] Az ábrán két négyzetet látunk, melyeknek van egy közös csúcsa.

1307. [3] Paralelogramma csúcsait oldalfelező pontokkal kötöttük össze az ábra szerint.

Feladatok

23. Területátalakítások

107

1308. [4] Konvex négyszög csúcsait oldalfelező pontokkal kötöttük össze az ábra szerint.

1309. [2] A két négyzet oldalai párhuzamosak egymással.

1310. [3] Egyenlő szárú trapézban átlók és a csúcsokra illeszkedő magasságvonalak az ábra szerint.

1311. [5] Konvex hatszög szemközti oldalai párhuzamosak. (A két ábrán ugyanaz a hatszög látható.)

108

Feladatok

A következő feladatokban az egész síkidom területének hányad része a vonalkázott terület? (1312–1328. feladatok) 1312. [2] Négyzetben félkörív és negyedkörívek az ábra szerint.

1313. [2] Négyzetben negyedkörívek az ábra szerint.

1314. [3] Rombusz oldalainak harmadolópontjait kötöttük össze az ábra szerint.

23. Területátalakítások

1315. [3] Szabályos háromszögben oldalak harmadolópontjait kötöttük össze az ábra szerint.

1316. [3] Négyzet oldalainak harmadolópontjait kötöttük össze az ábra szerint.

1317. [3] Négyzetben szakaszfelező pontokat kötöttünk össze az ábra szerint.

109

110

Feladatok

1318. [3] Négyzetben oldalfelező pontokat kötöttünk össze egy-egy csúccsal az ábra szerint.

1319. [3] Egy négyzetbe öt egybevágó négyzetből álló keresztet írtunk az ábra szerint úgy, hogy a kereszt négy csúcsa a négyzet egyegy oldalfelező pontjába esik.

1320. [3] A téglalap egyik oldala kétszerese a másiknak. Oldalfelező pontokat és csúcsokat kötöttünk össze az ábra szerint.

23. Területátalakítások

111

1321. [3] Konvex négyszögben harmadolópontokat és csúcsokat kötöttünk össze az ábra szerint.

1322. [4] Konvex négyszögben harmadolópontokat kötöttünk össze az ábra szerint.

1323. [3] Hegyesszögű háromszög oldalfelező pontjaiból merőlegeseket bocsátottunk a szemközti oldalakra.

112

Feladatok

1324. [3] A téglalap két átellenes csúcsát a szemközti oldalfelező pontokkal kötöttük össze.

1325. [3] Paralelogramma egy csúcsát a szemközti oldalfelező pontokkal kötöttük össze, és megrajzoltuk a paralelogrammának azt az átlóját, amelyik nem illeszkedik a kiszemelt csúcsára.

1326. [3] A háromszög egy-egy oldalának felező-, egyik harmadoló- és negyedelőpontját kötöttük össze.

1327. [5] Egy háromszög oldalainak harmadolópontjait a csúcsokkal kötöttük össze az ábra szerint.

23. Területátalakítások

113

1328. [4] Egy négyzetben oldalfelező pontokat kötöttünk össze a csúcsokkal, ill. berajzoltunk egy átlót.

1329. [3] Egy háromszög, négyszög, szabályos hatszög oldalait saját hosszukkal, ill. annak kétszeresével meghosszabbítottuk adott körüljárási irányt követve. Hányszorosa az így kapott síkidom területe az eredetinek?

1330. [3] Hogyan kell az ADEF G töröttvonalat felvenni, hogy az ABC háromszöget öt egyenlő területű háromszögre bontsuk?

C

G F E D A

B

1331. [3] Egy négyzet belsejében felveszünk két pontot, s azokat összekötjük a négyzet csúcsaival. Így a négyzetet kilenc részre osztjuk. Lehetséges-e, hogy e részek mindegyikének ugyanannyi a területe? 1332. [3] Félkörbe írt téglalapok közül melyiknek legnagyobb a területe? 1333. [3] Az ABCD téglalap belsejében felveszünk egy M pontot. Mutassuk meg, hogy a téglalap t területére igaz a következő egyenlőtlenség: t ≤ |AM| · |BM| + |CM| · |DM|.

Feladatok

114

1334. [4] Bizonyítsuk be, hogy a T területű ABCD négyszögre és a belsejében levő P pontra pontosan akkor áll fenn a 2T = P A2 + P B 2 + P C 2 + P D2 egyenlőség, ha a négyszög négyzet, és P a négyzet középpontja. 1335. [4] Háromszög egyik oldalán adott egy pont. Szerkesszünk e ponton át olyan egyenest, mely felezi a háromszög területét. 1336. [4] Konvex négyszög egyik csúcsán át szerkesszünk olyan egyenest, mely felezi a négyszög területét.

24. Geometriai konstrukciók 1337. [2] Vághat-e két egyenes egy négyszöget 6 részre? 1338. [2] Az ábrákon látható síkidomokat vágjuk szét 4 db egybevágó részre. 2

1 1

1 1

4

1 1

1

2

2

a)

b)

1339. [2] Hogyan lehet az ábrán látható, 5 négyzetből álló alakzatokat 3 darabra vágni úgy, hogy azokat összerakva egy nagy négyzetet kapjunk?

1

4

2

2 6

2

c)

d) a)

1340. [2] Az ábrán látható síkidomot a vonalak mentén vágjuk két részre úgy, hogy azokból négyzetet lehessen összeállítani.

b)

24. Geometriai konstrukciók

115

1341. [2] Az ábrákon látható téglalapok mindegyikét a vonalak mentén vágjuk két részre úgy, hogy a két részből négyzetet lehessen összeállítani. a) b) (8

18-as)

(9

16-os)

1342. [3] Vegyünk három négyzetet úgy, hogy közülük egyet, esetleg kettőt két részre vágva ezekből egy nagyobb négyzetet tudjunk összerakni. 1343. [2] Egy mennyezetre 12 lámpát függesztenek fel úgy, hogy 6 egyenesen legyenek és minden egyenesen 4 lámpa helyezkedjék el. Hogyan lehet ezt megvalósítani? 1344. [3] Helyezzünk el 7 pontot és 6 egyenest a síkon úgy, hogy mindegyik egyenesen 3 pont legyen. 1345. [3] Helyezzünk el 8 pontot és 7 egyenest a síkon úgy, hogy mindegyik egyenesen 3 pont legyen. 1346. [3] Helyezzünk el 9 pontot és 9 egyenest a síkon úgy, hogy mindegyik egyenesen 3 pont legyen, és minden pontra 3 egyenes illeszkedjék. 1347. [4] Helyezzünk el 10 pontot és 10 egyenest a síkon úgy, hogy mindegyik egyenesen 3 pont legyen, és minden pontra 3 egyenes illeszkedjék. 1348. [3] Helyezzünk el 10 pontot és 5 egyenest a síkon úgy, hogy mindegyik egyenesen 4 pont legyen. 1349. [3] Az asztalra letettünk néhány egyforma méretű kockát. Ha ezeket elölről, ill. oldalról nézzük, akkor a következőket látjuk:

Hány kocka lehet az asztalon? Legkevesebb és legfeljebb hány kocka lehet? 1350.

[3]

Lehet-e olyan, a kockától különböző testet készíteni, melynek mindegyik oldallapja négyzet?

116

Feladatok

1351. [5] Egy fából készült nem üreges testet felülről, ill. egyik oldalról nézve az ábrán feltüntetett képet látjuk (a testre semmi sincs rajzolva). Mit látunk, ha elölről nézzük a testet? 1352. [4] Adjunk meg 6 pontot a síkon úgy, hogy közülük bármelyiktől három másik pont legyen 1 egység távolságra. 1353. [7] Megadható-e a síkon 7 pont úgy, hogy közülük bármely három között legyen kettő, melyek távolsága 1 egység? 1354. [3] Van-e olyan konvex hatszög, melynek minden oldala nagyobb, mint 1 egység, de egyik átlója sem hosszabb 2 egységnél? 1355. [4] Igaz-e a következő állítás: Ha a síkon 12 pont közül bármelyik kettő távolsága legfeljebb 3 egység, akkor kiválasztható közülük négy pont úgy, hogy ezek közül bármely kettő távolsága legfeljebb 2 egység? 1356. [4] Adjunk meg a) 5, illetve b) n pontot a síkon úgy, hogy közülük bármely három pont egy tompaszögű háromszög csúcsait alkossa. 1357. [3] Adjunk meg 6 pontot a síkon úgy, hogy közülük bármely három pont egy egyenlőszárú háromszög csúcsait alkossa. 1358. [4] 120◦ -os szárszögű egyenlőszárú háromszöget daraboljunk fel 5 egyenlőszárú háromszögre. 1359. [4] Szabályos háromszöget daraboljunk fel 5 egyenlőszárú háromszögre. 1360. [4] Szabályos háromszöget daraboljunk fel 7 egyenlőszárú háromszögre. 1361. [3] Háromszöget daraboljunk fel 4 egyenlőszárú háromszögre. 1362. [3] Háromszöget daraboljunk fel 6, 7, ill. 8, hozzá hasonló háromszögre. 1363. [4] Van-e olyan háromszög, mely feldarabolható a) három darab egybevágó, az eredetihez hasonló háromszögre? b) öt darab egybevágó, az eredetihez hasonló háromszögre? 1364. [4] Felosztható-e az egyenlő szárú derékszögű háromszög hozzá hasonló, páronként nem egybevágó háromszögekre? 1365. [4] Mutassuk meg, hogy bármely háromszöget négy darabra lehet vágni úgy, hogy azokból két, az eredetihez hasonló háromszög állítható össze.

24. Geometriai konstrukciók

117

1366. [4] A 2 és 3 befogójú derékszögű háromszöget daraboljuk fel 13 db, az eredetihez hasonló, egymással egybevágó háromszögre. 1367. [4] Egyenlő szárú háromszögből vágjunk ki 3 db egybevágó háromszöget úgy, hogy ezek területe (külön-külön) nagyobb legyen, mint az eredeti háromszög területének negyede. 1368. [3] Négyzetet daraboljunk fel 6, 7, ill. 8 db négyzetre. 1369. [4] Négyzetet daraboljunk fel hegyesszögű háromszögekre. Mutassuk meg, hogy a négyzetet nem lehet feldarabolni nyolcnál kevesebb hegyesszögű háromszögre. 1370. [4] Téglalapot daraboljunk fel 5, 6, 7, ill. 8 téglalapra úgy, hogy közülük semelyik két szomszédos se alkosson együtt téglalapot. 1371. [5] Daraboljunk fel egy négyzetet konvex ötszögekre. 1372. [5] Alkalmasan választott paralelogrammát daraboljunk fel 3 egyenlő szárú háromszögre. 1373. [5] Fel lehet-e osztani egy négyzetet két egybevágó sokszögre, ahol a sokszög oldalainak száma a) 7; b) 8? 1374. [6] Bontsunk fel egy körlapot egybevágó részekre úgy, hogy legyen olyan rész, amely nem tartalmazza az origót (még a határán sem). 1375. [5] Azt mondjuk, hogy az A háromszög a B háromszögbe van írva, ha az A csúcsai a B oldalegyeneseire illeszkednek és a két háromszögnek nincs közös oldalegyenese. Létezik-e három olyan háromszög, A, B és C, melyek ciklikusan egymásba vannak írva, azaz az A a B-be, B a C-be, és C az A-ba? 1376. [5] Adjunk meg két egymásba írt ötszöget. (Lásd az előző feladatot!) 1377. [5] Adjunk meg három, ciklikusan egymásba írt négyszöget. (Lásd az előző feladatokat!) 1378. [6] Adjunk meg két egymásba írt tetraédert. (Egy tetraéder egy másikba van írva, ha csúcsai illeszkednek a másik lapsíkjaira, egy lapsíkra csak egy csúcs.) 1379. [3] Adjunk meg olyan konvex testet, melyen a lapok, élek, csúcsok száma rendre 6, 10, 6. 1380. [4] Van-e olyan (nem feltétlenül konvex) test, melynek ugyanannyi lapja, éle, csúcsa van, mint a kockának, de nincs négyszöglapja? 1381. [4] Helyezzünk el a síkon végtelen sok kört úgy, hogy bármely egyenes legfeljebb két kört metsszen. 1382. [4] Le lehet-e fedni a síkot véges sok parabolatartománnyal? 1383. [5] Pontszerű fényforrás eltakarható-e 4 gömbbel?

118

Feladatok

1384. [5] A síkon adott egy P pont. El lehet-e helyezni a síkon a) 4 kört, illetve b) 5 kört úgy, hogy egyik se tartalmazza a P pontot, és bármely P -ből induló félegyenesnek legyen közös pontja legalább két körrel? 1385. [5] A síkon adott egy P pont. El lehet-e helyezni a síkon a) 6 kört, illetve b) 7 kört úgy, hogy egyik sem tartalmazza a P pontot, és bármely P -ből induló félegyenesnek legyen közös pontja legalább három körrel? 1386. [5] Lehet-e egy egyenessel metszeni egy a) 1990, illetve egy b) 1991 oldalú sokszög minden oldalát? 1387. [5] El lehet-e helyezni az asztalon a) 1990, illetve b) 1991 egyforma pénzérmét úgy, hogy mindegyik három másikat érintsen?

25. Invariáns tulajdonságok 1388. [3] 24 papírlap közül néhányat 10 részre vágtak, majd az így kapott részek közül néhányat ismét 10 részre vágtak szét és így tovább. Lehetséges-e, hogy ezt néhányszor megismételve 1991 papírdarabot kapjunk? 1389. [3] Egy táblára felírtuk az 1, 2, 3, 4, 5, 6, 7, 8, 9, 10 számokat. Egy-egy alkalommal két számot letörlünk és helyettük azok különbségét írjuk fel. Ezt kilencszer elvégezve előfordulhat-e, hogy a megmaradó szám a nulla? 1390. [3] A táblán 50 darab 0 és 50 darab 1-es számjegy van. Egy alkalommal két számjegyet törlünk le. Ha ezek egyenlők, akkor 1-est, ha különbözők, akkor egy 0-t írunk a táblára. Milyen számjegy marad a táblán a kilencvenkilencedik törlés után? 1391. [3] Béla egy papírlapot tetszőleges módon felosztott hat részre. Az így kapott részek közül az egyik részt felosztotta 11 részre, majd a kapott részek egyikét ismét 6 részre, és így folytatta, arra sem ügyelve, hogy a 6, illetve 11 részre osztást váltogatva végezze. Bizonyos számú osztás után megszámolta a kapott részeket, és azt állította, hogy 1993 részt kapott. Helyesen számolt-e Béla? 1392. [3] A táblára felírtuk az 1, 2, 3, . . . , 20 számokat. Egy-egy alkalommal letörlünk két számot, a-t és b-t, s helyettük felírjuk az a + b − 1 számot. Ezt az eljárást 19-szer elvégezve milyen szám maradhat a táblán?

25. Invariáns tulajdonságok

119

1393. [4] A táblára felírtuk az 1, 2, 3, . . . , 20 számokat. Egy-egy alkalommal letörlünk két számot, a-t és b-t, s helyettük felírjuk az ab + a + b számot. Ezt az eljárást 19-szer elvégezve milyen szám maradhat a táblán? 1394. [3] Egy vázában 75 fehér és 150 fekete babszem lapul. A váza mellett van egy halom fekete bab. A babszemeket a következőképp szedegetjük a vázából: találomra kiveszünk két babszemet; ha van köztük fekete, akkor azt kitesszük a halomba, a másikat (akár fehér, akár fekete) visszadobjuk a vázába; ha azonban mindkét babszem fehér, akkor eldobjuk őket, s a halomból egy fekete szemet dobunk a vázába. Így a vázában levő babszemek száma egyesével csökken. Ha már csak egy szem bab van a vázában, akkor mit mondhatunk annak színéről? 1395. [3] A Csodakert fáin 25 banán és 30 narancs van. Egy-egy alkalommal két gyümölcsöt veszünk le: ha egyformákat vettünk le, akkor egy narancs nő helyettük; ha különbözőket vettünk le, akkor egy banán nő. Utolsónak milyen gyümölcs marad? 1396. [3] Kezdetben egy 3 × 3-as táblázat minden mezőjén 0 áll, majd egy-egy lépésben a tábla valamely 2 × 2-es részén a számok mindegyikét 1-gyel növeljük. Ilyen lépésekkel megkaphatjuk-e a mellékelt kitöltést? 4 9 5 10 18 12 6 13 7 1397. [3] Két kupacban gyufák vannak. Egy-egy alkalommal valamelyik kupacból elveszünk néhány szálat, s a másik kupacba kétszer annyit helyezünk. Elérhető-e, hogy mindkét kupacban ugyanannyi gyufaszál legyen, ha kezdetben az egyik kupacban 1, a másikban 2 szál volt? 1398. [4] Egy háromszög csúcsaiba gyufákat helyeztünk. Egy-egy alkalommal bármelyik csúcsból vehetünk el gyufákat, de a másik két csúcs mindegyikébe kétszer annyit kell helyeznünk. Kezdéskor az egyik csúcsban 1 szál volt, a többiben egy sem. Elérhető-e, hogy mindegyik csúcsban ugyanannyi gyufa legyen? 1399. [4] Egy négyzet csúcsaiba gyufákat helyeztünk. Egy-egy alkalommal bármelyik csúcsból vehetünk el gyufákat, de az egyik, ezzel szomszédos csúcsba kétszer annyi gyufát kell helyezni. Kezdéskor a csúcsokban valamilyen körüljárást tekintve a gyufák száma: 1, 0, 0, 0. A fentebb közölt eljárást ismételve elérhető-e, hogy a csúcsokban levő gyufák száma 1, 9, 8, 9 legyen? 1400. [5] Az asztalon egy kupacban 1001 kavics van. Egy lépésben valamely, 1-nél több kavicsot tartalmazó kupacból kidobunk egy kavicsot, a maradékot két részre osztjuk. Elérhető-e ilyen lépésekkel, hogy az asztalon 3–3 kavicsból álló kupacok legyenek?

120

Feladatok

1401. [4] Egy szigeten 13 szürke, 15 barna, 17 zöld kaméleon él. Ha két különböző színű kaméleon találkozik, akkor annyira megijednek egymástól, hogy mindketten a harmadik színre változtatják bőrüket. Két azonos színű kaméleon nem ijed meg egymástól, így találkozáskor nem változtatják meg színüket. Lehetséges-e, hogy egy idő múlva minden kaméleon ugyanolyan színű legyen? Ha kezdetben 13 szürke, 25 barna és 17 zöld kaméleon él a szigeten, akkor elérhető-e, hogy mindegyik kaméleon ugyanolyan színű legyen. Milyen szín lehet ez? 1402. [4] Egy táblára 6 db 0, 7 db 1-es és 8 db 2-es számot írtunk fel. Egy-egy alkalommal két különböző számot letörlünk, s helyettük a harmadikat írjuk fel (pl. ha 0-t és 2-est törlünk le, akkor 1-est írunk fel). Néhány ilyen törlés-felírás után csak egy szám marad a táblán. Mi lehet ez a szám? 1403. [4] Egy táblára az 1, 2, 3, . . . , 1988 számokat írtuk fel. Egy-egy alkalommal két számot letörlünk, s helyettük összegük 13-as osztási maradékát írjuk fel. Néhány ilyen törlés-felírás után csak egy szám marad a táblán. Mi lehet ez a szám? √ √ 1404. [5] A táblára a következő számok vannak felírva: 7 + 2, 5 2 − 1, 9. Bármely lépésben√kiválaszthatunk √ a táblán két számot, mondjuk a-t és b-t; és azokat (a + b)/ 2-vel és (a−b)/ 2-vel helyettesítjük. √Elérhető-e, hogy valahány lépés √ után a táblán valamilyen sorrendben 8 + 2, 5 2, 7 számok álljanak? 1405. [5] A táblára felírtunk néhány, nullától különböző számot, majd egy-egy lépésben valamely két számot, A-t és B-t letöröljük; s helyettük két új számot írunk fel, B A az A + és B − számokat. Néhány ilyen lépés után megkaphatjuk-e újra az 2 2 eredetileg felírt számokat? 1406. [3] Egy négyzet csúcsaiba számokat írtunk. Egy-egy alkalommal két szomszédos csúcs mindegyikében 1-gyel növeljük az ott levő számokat. Elérhető-e, hogy mindegyik csúcsban ugyanaz a szám álljon, ha kezdetben a) az egyik csúcsban 1, a többiben 0 van? b) két szemközti csúcsban 1, a többiben 0 van? 1407. [4] Egy kocka csúcsaiba számokat írtunk. Egy-egy alkalommal valamelyik él két végén álló számot 1-gyel növelhetjük. Ezt az eljárást néhányszor megismételve elérhető-e, hogy minden csúcsban ugyanaz a szám álljon, ha a kezdő állapotban a) az egyik csúcsban 1-es, a többiben 0 van? b) az egyik oldallap két szemközti csúcsában 1-es, a többiben 0 van? 1408. [4] Egy asztalt hatan ülnek körbe, s egyikük előtt 6 tányér van. Egy-egy alkalommal bárki elvehet az előtte levő tányérok közül kettőt, és azokat valamelyik szomszédjának vagy szomszédainak adja át. Ezt néhányszor megismételve elérhetjük-e, hogy mindenki előtt 1 tányér legyen?

25. Invariáns tulajdonságok

121

1409. [4] Egy asztalon sorban 6 tányér van. Ubul mindkét kezével megragad egy-egy tányért a hat közül, és mindkettőt egy hellyel jobbra vagy balra helyezi (ha már van ott tányér, akkor annak tetejére). El tudja-e érni, hogy az összes tányér egy oszlopba kerüljön? 1410. [4] Egy asztal körül hatan ülnek, és közülük két személy előtt egy-egy tányér van, a többiek előtt nincs tányér. E két személy között egy ember ül. Egy lépésben valamely két szomszédos személy elé egy-egy újabb tányért helyezünk. Elérhető-e néhány ilyen lépéssel, hogy mindenki előtt ugyanannyi tányér legyen? 1411. [4] Egy házaspár négy másik házaspárt hív meg vacsorára. Egy kör alakú asztalhoz felváltva ülnek le a nők és a férfiak. Mindenki előtt egy pohár van, s ezek egyszer csak elkezdenek vándorolni. Minden percben egy pohár egyet balra, egy másik pohár egyet jobbra lép. Elérhető-e, hogy egy idő múlva minden pohár a házigazda előtt legyen? 1412. [4] Egy körvonalon 44 fa helyezkedik el. Mindegyik fán ül egy majom. Egy-egy perc elteltével valamelyik két majom átugrik a szomszédos fára, az egyik az óramutató járásával azonos, a másik ellentétes irányba. Lehetséges-e, hogy egy idő múlva mindegyik majom ugyanazon a fán ül? 1413. [4] Egy tisztáson 14 fa áll körben, és mindegyiken egy-egy mókus ül. Megengedjük, hogy egyszerre két mókus valamelyik szomszédos fára ugorjon át. Lehetséges-e, hogy egy idő múlva mindannyian ugyanarra a fára kerüljenek? 1414. [6] Van egy N tagú társaság. A társaság minden tagja legfeljebb 19 másik tagot ismer. Feladatunk az, hogy a társaságot 2 részre bontsuk, a „7-esek” és a „11esek” klubjára úgy, hogy a „7-esek” klubjukon belül legfeljebb 7, a „11-esek” klubjukon belül legfeljebb 11 másik tagot ismerjenek. Egy matematikus a következő eljárást ajánlja: először osszuk tetszőlegesen két részre a társaságot, majd „fokozatosan”, lépésenként javítsuk ki az esetleges hibákat: ha valaki egy adott lépésben a „7-esek” klubjához tartozik és ebben a klubban legalább 8 ismerőse van, tegyük át a „11-esek” klubjába, ill. fordítva, ha valakinek a „11-esek” klubjában legalább 12 ismerőse van, tegyük át a „7-esek” közé. Így minden csere után megváltozik a klubok összetétele. Igaz-e, hogy ez az eljárás véges számú lépésben biztosan célra vezet? 1415. [6] Egy 10 × 10-es tábla 9 mezője gyomos. Egy parcella (mező) elgyomosodik, ha annak legalább két oldalszomszédja gyomos. Lehetséges-e, hogy előbb-utóbb az egész tábla gyomos lesz? 1416. [6] Egy tornaórán n gyerek áll egy sorban a tanárral szemben. Adott jelre mindegyikük 90◦ -kal elfordul: néhányuk balra, mások jobbra. Ezután vezényszóra mindazok, akik valamelyik szomszédjukkal szemben állnak, hátraarcot csinálnak. Bizonyítsuk be, hogy az (n + 1)-edik vezényszóra már senki sem mozdul meg.

Feladatok

122

1417. [5] Bal oldalon egy sáska, középen egy szöcske, jobb oldalon egy tücsök ül egy hosszú, egyenes árokban. Időnként valamelyik átugorja egyik szomszédját. Előfordulhat-e, hogy 1999 ugrás után újra a kiinduló sorrendben ülnek, ha végig csak az árokban (egy egyenes mentén) ugrálnak? 1418. [4] Egy asztalon 6 pohár áll. Egy-egy alkalommal 5 poharat megfordítunk. E művelet ismétlésével elérhetjük-e, hogy mindegyik pohár meg legyen fordítva? 1419. [4] Adott egy 10 elemű sorozat, mely +1 és −1 számokból áll. A sorozat öt elemének megváltoztathatjuk az előjelét. Ezt a változtatást többször megtehetjük. Elérhető-e bizonyos számú lépés után, hogy olyan sorozatot kapjunk, mely az eredetitől csak egy helyen különbözik? Mi a válasz akkor, ha egy-egy alkalommal négy elem előjelét változtatjuk meg? 1420. [4] Egy táblára 10 db + és 15 db − jelet írtunk fel. Egy-egy alkalommal két jelet letörlünk, s helyettük + jelet írunk fel, ha a két jel egyforma volt, különben pedig − jelet. 24 ilyen törlés-felírás után már csak egy jel marad a táblán. Melyik jel lehet az? 1421. [5] Az ábrán látható táblázat bármelyik sorában vagy oszlopában, vagy valamelyik átlóval párhuzamosan elhelyezkedő mezőkben ellentétesre változtatjuk az előjeleket. Ezt a műveletet többször megismételve elérhetjük-e, hogy mindegyik mezőben + jel legyen?

+

+



+

+

+

+

+

+

+

+

+

+

+

+

+

1422. [5] Egy 8 × 8-as táblázat mindegyik mezőjébe egy-egy egész számot írtunk. Megengedett a következő művelet: a táblázat bármely 3 × 3-as vagy 4 × 4-es részén a számok mindegyikéhez 1-et adunk. Ilyen műveletekkel elérhető-e a táblázat bármilyen kezdeti kitöltése esetén, hogy a benne levő mindegyik szám osztható legyen 3-mal? 1423. [4] Szabályos 12-szög egyik csúcsában mínusz jel, a többiben plusz jel áll. Egy-egy alkalommal a) 6, b) 5, c) 4, illetve d) 3 szomszédos csúcsban változtatjuk meg az előjeleket. Az eljárást megismételve elérhetjük-e, hogy a mínusz jel az egyik szomszédos csúcsba költözzék át, s a többi 11 csúcsban plusz jel álljon? 1424. [4] Adott egy négyzet három csúcsa. Új pontot úgy kapunk, hogy a már meglevő pontot másik (ugyancsak meglevő) pontra tükrözzük. Ezt az eljárást néhányszor megismételve megkaphatjuk-e a négyzet hiányzó negyedik csúcsát?

25. Invariáns tulajdonságok

123

1425. [4] Adott a síkon négy pont: (0; 0), (1; 1), (3; 0), (2; −1). Új pontot úgy kapunk, hogy a már meglevő pontot másik (ugyancsak meglevő) pontra tükrözzük. Ezt az eljárást néhányszor megismételve megkaphatjuk-e a (0; 0), (0; 1), (1; 0), (1; 1) csúcspontokból álló négyzetet? 1426. [4] Az 1, 9, 9, 1, 0, 9, . . . sorozatot úgy képezzük az ötödik elemétől kezdődően, hogy az azt megelőző négy számot összeadjuk, s az összeg utolsó jegye lesz ez az elem. Szerepelhet-e a sorozatban valahol egymás után az 1, 2, 3, 4 részsorozat? 1427. [6] Az 1, 0, 1, 0, 1, 0, 3, . . . sorozatot úgy képezzük a hetedik elemétől kezdve, hogy az azt megelőző hat számot összeadjuk, s az összeg utolsó jegye lesz ez az elem. Szerepelhet-e ebben a sorozatban valahol egymás után a 0, 1, 0, 1, 0, 1 részsorozat? 1428. [4] Az (a, b) számpárból a (b, a), (a + b, b), (a − b, b) számpárok bármelyikét származtathatjuk. Ezeket a műveleteket a meglévő számokkal megismételve eljuthatunk-e a (12, 21) számpárból a a) (19, 91), illetve a b) (15, 51) számpárhoz?

a b [4] 1429. Az (a, b) számpárból származtathatjuk az (a + 1, b + 1), , (utóbbit, 2 2 ha a és b is páros) számpárok bármelyikét. Két, már korábban származtatott (a, b) és (b, c) párból kaphatjuk az (a, c) számpárt is. Ezekkel a műveletekkel eljuthatunk-e az (5, 19) számpárból az a) (1, 50), illetve az b) (1, 100) számpárhoz? 1430. [4] Az 1, 2, 3, . . . , n számok kezdetben ebben a sorrendben állnak. Egy művelet két szám felcserélését jelenti. Lehet-e 15 cserével újra az eredeti sorrendhez jutni? 1431. [5] Egy sorban 11 szék áll, melyek az 1, 2, 3, . . . , 11 számokkal ebben a sorrendben vannak megszámozva. Megengedett művelet négy egymás után álló szék sorrendjének megfordítása (pl. 4, 5, 9, 8-ból 8, 9, 5, 4 lesz). Ilyen műveletekkel felcserélhető-e két szomszédos szék úgy, hogy a többi sorrendje változatlan maradjon? Mi a válasz akkor, ha 11 helyett 12 szék van, vagy 4 helyett 3 szék sorrendje fordítható meg? Mi a válasz akkor, ha a székek egy asztal körül körben vannak elhelyezve? 1432. [6] Az ábrán látható 4 × 4-es keretben 15 számozott lapocska van. Az üres helyre mindig oda tudjuk tolni bármelyik lapszomszédot. Ilyen tologatással elérhetjük-e, hogy mindegyik lap visszakerüljön arra a helyre, ahol a kiinduló állapotban volt, és csak az 1-es és a 2-es jelű legyen felcserélve?

1

2

3

4

5

6

7

8

9

10 11 12

13 14 15

124

Feladatok

1433. [7] Képzeljünk el egy 4 × 4 × 4-es kockát, mely 64 egybevágó kis kockából van kirakva. Ezek egyikét kivesszük, s a többi kockát megszámozzuk 1-től 63-ig, s a sorszámot ráírjuk a kis kockák mind a hat oldallapjára. A játék elején a kockák valamilyen sorban vannak. Tegyük fel, hogy az üres helyre mindig oda tudjuk tolni bármelyik lapszomszédját. A kérdés ezek után, hogy el tudunk-e érni ilyen tologatásokkal olyan helyzetet, amelyben minden kis kocka ugyanazon a helyen van, mint a kiinduló állásnál (a belül lévő, sehonnan sem láthatók is!), csak éppen az 1-es és a 2-es jelű fel van cserélve. 1434. [5] Egyszerű megmutatni, hogy egy háromszög átdarabolható téglalappá; ekkor az átdarabolás során a részeket elforgatjuk. Felosztható-e a háromszög olyan részekre, melyeket megfelelően eltolva téglalapot kapunk? 1435. [4] Egy táblára az x 2 +10x +20 polinomot írtuk. Valaki egy-egy alkalommal a polinom egyik tetszőlegesen választott együtthatóját 1-gyel növeli vagy csökkenti. Egy idő múlva az x 2 + 20x + 10 polinom kerül a táblára. Mutassuk meg, hogy közben a táblán szerepelt olyan polinom, melynek volt egész gyöke. 1436. [4] Egy konkáv sokszögön a következő műveletet végezzük el. Kiválasztunk két, nem szomszédos csúcsot, legyenek ezek A és B. Ha a sokszög az AB egyenes egyik oldalán van, akkor határvonalának azt a részét, amely A és B közé esik, tükrözzük az AB felezőpontjára. Bizonyítsuk be, hogy elég sokszor ismételve ezt az eljárást, konvex sokszöget kapunk. 1437. [4] Egy n×m-es táblázat mezőibe számokat írtunk. Egy-egy alkalommal bármelyik sorban vagy oszlopban megváltoztathatjuk a számok előjelét. Mutassuk meg: ilyen műveletekkel elérhető, hogy minden sorban és minden oszlopban az ott álló számok összege ne legyen negatív! 1438. [5] Egy körvonalra felírtunk néhány számot. Ha néhány, egymás után elhelyezkedő a, b, c, d számra (a − d)(b − c) < 0, akkor b és c helyet cserél. Igazoljuk, hogy ezt az eljárást csak néhányszor lehet végrehajtani. 1439. [7] Egy szabályos ötszög csúcsaihoz egy-egy egész számot rendelünk úgy, hogy összegük pozitív legyen. Megengedett a következő művelet. Ha három szomszédos csúcs X, Y , Z és a hozzájuk rendelt számok x, y, z és y < 0, akkor az x, y, z számok helyére ugyanilyen sorrendben az x + y, −y, z + y számokat írjuk. Ezt a műveletet ismételgetjük addig, amíg csak található negatív y. Döntsük el, vajon minden esetben véget ér-e az eljárás véges sok lépés után. 1440. [5] A számnégyeseken értelmezzük a következő műveletet: (a, b, c, d) → (a − b, b − c, c − d, d − a). Mutassuk meg, hogy ha kezdetben a négy szám között van legalább két különböző, akkor ezt az eljárást egymás után elég sokszor végrehajtva, a négy szám között lesz olyan, mely nagyobb, mint 1991.

25. Invariáns tulajdonságok

125

1441. [5] A végtelen négyzetrács néhány mezőjét fehérről feketére festettük. Ezt követően percenként minden mező átváltozik arra a színre, amelyikből több van két oldalszomszédja és a fölötte levő mező színei közt. Bizonyítsuk be, hogy egy idő múlva minden mező fehér színű lesz. 1442. [5] A végtelen négyzetrács néhány mezőjét fehérről feketére festettük. Ezt követően percenként minden mező átváltozik a másik színre, ha olyan színű a jobb oldali szomszédja és a fölötte levő mező (különben megőrzi eredeti színét). Bizonyítsuk be, hogy egy idő múlva minden mező fehér színű lesz. 1443. [5] Az erdőben lakó törpék mindegyikének háza vagy fehér, vagy kék színű. Megállapodnak abban, hogy ha találnak maguk közt olyat, akinek háza más színű, mint barátai többségének háza, akkor az átfesti erre a színre házikóját. Minden nap keresnek egy-egy ilyen törpét. Mutassuk meg, hogy néhány nap elteltével már senkinek sem kell átfestenie a házikóját. 1444. [5] A törpék városában járvány tört ki, néhányan megfáztak, és influenzásak lettek. Ezután a járvány fertőzéssel terjedt: ha egy egészséges törpe meglátogatta beteg barátait, akkor másnapra ő is beteg lett. A törpék egy napig betegek, utána pedig egy napig immunisak (aznap nem fertőződhetnek meg). Minden egészséges törpe naponta meglátogatja beteg barátait. Bizonyítsuk be, hogy ha valamelyik törpe a járvány első napján be volt oltva (vagyis azon a napon immunis volt), akkor a járvány akár örökké is tarthat, különben biztosan véget ér. 1445. [5] Egy 5 × 5-ös telket 25 darab 1 × 1-es parcellára osztottunk. Van 25 manónk, akik közül mindegyik legfeljebb három másikat utál (az utálat kölcsönös). Bizonyítsuk be, hogy szétoszthatók a manók a parcellákba úgy, hogy egyetlenegy manó se utálja a szomszédait. (Két parcella szomszédos, ha van közös oldaluk.) 1446. [5] A parlamentben egy-egy képviselőnek legfeljebb három ellenfele van (az ellenségesség kölcsönös). Mutassuk meg, hogy a képviselők két csoportba oszthatók úgy, hogy bárkinek a saját csoportjában legfeljebb egy ellenfele van. 1447. [5] Egy 2n fős társaságban mindenkinek legfeljebb n − 1 ellensége van (az ellenségesség kölcsönös). Mutassuk meg, hogy a társaság leültethető egy kerek asztalhoz úgy, hogy az egymás mellett ülők nem ellenségek. 1448. [5] A síkon adott n pont, néhány közülük szakasszal összekötve. Bármely pontból legfeljebb 11 szakasz indul. Bizonyítsuk be, hogy a pontok 4 színnel színezhetők úgy, hogy az azonos színű pontokat összekötő szakaszok száma nem több n-nél. 1449. [5] Adott a síkon 2n pont, melyek közül semelyik három sincs egy egyenesen. A pontok fele kék, fele piros színű. Mutassuk meg, hogy rajzolható e pontok között n db olyan szakasz, amelyek mindegyike egy piros és egy kék pontot köt össze, s a szakaszoknak nincs közös pontja.

126

Feladatok

1450. [5] Adott a síkon n db pont, melyek közül semelyik három sincs egy egyenesen. Mutassuk meg, hogy van olyan, önmagát nem metsző zárt görbevonal, melyre mindegyik pont illeszkedik. 1451. [5] Egy 3 × 3-as táblázat mezőiben +1 vagy −1 áll. Egy-egy alkalommal minden mezőbe az élekkel szomszédos mezőkben álló számok szorzatát írjuk, s így jutunk a régi táblázatból egy új kitöltéséhez. Igazoljuk, hogy az eljárást néhányszor megismételve, minden mezőben +1 fog állni. 1452. [3] Egy körre tetszőlegesen 4 db 1-es és 5 db 0 számot írtunk. Egy-egy alkalommal két szomszédos egyenlő szám közé 0-t, két szomszédos különböző szám közé 1-est írunk, majd letöröljük az előző kilenc számot. Ezt az eljárást néhányszor megismételve, kaphatunk-e kilenc 0 számot? 1453. [5] Legyen (a1 , a2 , . . . , an ) olyan szám-n-es, ahol az ai számok értéke +1 vagy −1. Tekintsük a következő műveletet: (a1 , a2 , . . . , an ) → (a1 a2 , a2 a3 , . . . , an−1 an , an a1 ). Mutassuk meg, hogy ha n = 4, akkor az eljárást újra és újra megismételve a kapott számnégyesen, egy idő múlva csupa +1-ből álló számnégyest kapunk. 1454. [5] Tekintsük a következő műveletet: (a1 , a2 , . . . , an ) → (|a1 − a2 |, |a2 − a3 |, . . . , |an−1 − an |, |an − a1 |), ahol az ai számok nem negatív egész számok. Mutassuk meg, hogy ha n = 4, akkor az eljárást újra és újra megismételve a kapott számnégyesen, egy idő múlva csupa 0-ból álló számnégyest kapunk.

26. Feladatok a sakktáblán 1455. [3] Miért nem lehet egy 8 × 8-as sakktábla bal alsó sarkából a huszárt eljuttatni a jobb felső sarokba úgy, hogy mire odaér, addig a sakktábla minden mezőjét pontosan egyszer érinti? 1456. [3] A sakktábla bal alsó sarkát kivágtuk. A jobb felső sarokból indulva be lehete járni ezt a táblát egy huszárral úgy, hogy minden mezőt pontosan egyszer érintünk? 1457. [3] Egy 8 × 8-as sakktábla egyik mezője hiányzik. Be lehet-e járni ezt a táblát egy huszárral úgy, hogy minden mezőt pontosan egyszer érintünk, s utolsónak olyan mezőre érkezünk, mely szomszédos azzal a mezővel, amelyről elindultunk? 1458. [3] A huszár n lépést tett meg a sakktáblán és visszajutott a kiindulási mezőre. Mutassuk meg, hogy n páros szám. 1459. [3] Egy 8 × 8-as sakktábla bal alsó sarkából el lehet-e jutni egy huszárral a jobb felső sarokba úgy, hogy közben minden sorba pontosan egyszer lépünk?

26. Feladatok a sakktáblán

127

1460. [4] Bizonyítsuk be, hogy a 4×5-ös sakktáblát bejárhatja a huszár úgy, hogy minden mezőre pontosan egyszer lép. 1461. [4] Bejárható-e a 4 × n-es sakktábla egy huszárral úgy, hogy az minden mezőre pontosan egyszer lépjen, és utolsó lépésével éppen visszaérjen a kiindulási mezőre? 1462. [4] Bejárható-e a 7 × n-es sakktábla (2, 3) huszárral úgy, hogy minden mezőre pontosan egyszer lépünk? (A (2, 3) huszár olyan figura, mely L alakban 2-t vízszintesen, 3-at függőlegesen lép vagy fordítva.) 1463. [4] Egy n × n-es sakktáblán egy bábu léphet a szomszédos mezőre vagy jobbra, vagy felfelé, vagy átlósan balra lefelé. Bejárható-e vele a tábla úgy, hogy minden mezőre egyszer lép, s útja azon a mezőn ér véget, mely jobb oldali szomszédja annak a mezőnek, amelyről elindult? 1464. [4] A 8 × 8-as sakktábla bal alsó 3 × 3-as sarkában 9 dámafigura áll. El lehet-e juttatni ezt a 9 bábut a) a bal felső, illetve b) a jobb felső 3 × 3-as sarokba, ha csak úgy lehet lépni, hogy egy másik figurát vízszintesen, függőlegesen vagy átlósan átugorva üres mezőre érkezünk? 1465. [4] Legfeljebb hány huszárt helyezhetünk el a 8 × 8-as sakktáblán úgy, hogy egyik se üsse a másikat? 1466. [4] Legfeljebb hány huszárt helyezhetünk el az 5×5-ös sakktáblán úgy, hogy egyik se üsse a másikat? 1467. [4] Helyezzünk el minél kevesebb huszárt a 3 × 3-as, 4 × 4-es, . . . , 8 × 8-as, 9 × 9-es, 10 × 10-es sakktáblán úgy, hogy azok ütés alatt tartsák a nem foglalt mezőket. 1468. [4] Legfeljebb hány királyt helyezhetünk el a 8 × 8-as sakktáblán úgy, hogy egyik se üsse a másikat? 1469. [3] Helyezzünk el a 8 × 8-as sakktáblán minél több királynőt úgy, hogy semelyik kettő se üsse egymást. 1470. [3] Helyezzünk el a 8 × 8-as sakktáblán minél kevesebb királynőt úgy, hogy azok a tábla minden mezőjét ütés alatt tartsák. 1471. [4] Legkevesebb hány királyt kell a 8×8-as sakktáblán elhelyezni, ha minden mezőt ütés alatt akarunk tartani? 1472. [4] Legfeljebb hány királynő helyezhető el a 8 × 8-as sakktáblán úgy, hogy mindegyik legfeljebb egy másikat üssön?

128

Feladatok

1473. [5] Helyezzünk el a 8×8-as sakktáblán 32 huszárt úgy, hogy mindegyik két másikat tartson ütés alatt. 1474. [5] A 3 × 3-as sakktábla két felső sarkában fekete, a két alsó sarokban fehér huszárok állnak. Elérhető-e néhány lépéssel, hogy az azonos színű huszárok a szemközti sarkokban legyenek? 1475. [5] A 3 × 3-as sakktábla két felső sarkában fekete, a két alsó sarokban fehér huszárok állnak. Elérhető-e néhány lépéssel, hogy a világos huszárok helyet cseréljenek a sötétekkel? Mennyi a lépések minimális száma? 1476. [4] A 8 × 8-as sakktábla mezőibe sorfolytonosan beírtuk 1-től 64-ig az egész számokat. Kiválasztunk 8 számot úgy, hogy minden sorból és minden oszlopból veszünk. Milyen határok között mozoghat e számok összege? 1477. [4] A 8 × 8-as sakktáblán elhelyeztünk 8 bástyát úgy, hogy nem ütik egymást. Mutassuk meg, hogy a fekete mezőkön álló bástyák száma páros. 1478. [4] Egy 5 × 5-ös sakktábla minden mezőjén áll egy bogár. Egy-egy perc elteltével mindegyik bogár átmászik valamelyik oldalszomszédos mezőre. Igaz-e, hogy minden órában van olyan perc, amikor valamelyik mező üresen áll? 1479. [3] Egy 5 × 5-ös táblázatba lehet-e számokat írni úgy, hogy a számok összege minden sorban pozitív, minden oszlopban pedig negatív? 1480. [3] Egy 5 × 5-ös táblázatba lehet-e számokat írni úgy, hogy e számok összege pozitív, ám a táblázat bármely 2 × 2-es részében a számok összege negatív? 1481. [3] Egy 5 × 5-ös táblázatba lehet-e számokat írni úgy, hogy a számok szorzata mindegyik sorban 20, míg mindegyik oszlopban 30? 1482. [3] Egy 5 × 5-ös táblázat mindegyik sorába beírjuk valamilyen sorrendben az 1, 2, 3, 4, 5 számokat úgy, hogy a kapott kitöltés szimmetrikus a főátlóra. Mutassuk meg, hogy ekkor a főátlóban ott áll mind az öt szám. 1483. [5] Egy n × n-es táblázat mezőibe egész számokat írtunk úgy, hogy közös oldalú mezőkbe írt számok között legfeljebb 1 a különbség. Mutassuk meg, hogy van n olyan szám, amely a táblázatban legalább -szer szerepel. 2 1484. [4] Egy 8 × 8-as sakktábla mezőit valahogyan megszámoztuk az 1-től 64-ig terjedő számokkal. Mutassuk meg, hogy van két olyan, oldalban szomszédos mező, melyen álló számok különbségének abszolútértéke legalább 5. 1485. [5] Egy 8 × 8-as sakktábla mezőit valahogyan megszámoztuk az 1-től 64-ig terjedő számokkal. Nevezzük két szomszédos mező távolságának a rajtuk álló számok különbségének abszolútértékét. (Két mező szomszédos, ha van közös csúcsuk.) Mi az a legnagyobb szám, amelynél nagyobb távolság a sakktábla bármely számozásakor fellép a szomszédos mezők között?

26. Feladatok a sakktáblán

129

1486. [7] Egy n × n-es táblázat (mátrix) elemei különböző egész számok. Bizonyítsuk be, hogy van két szomszédos (oszlopban vagy sorban egymás mellett álló) mező, melyeken álló számok különbségének abszolútértéke legalább n. 1487. [5] Egy 10 × 10-es táblázatba beírunk 10-10 db 0, 1, 2, . . . , 9 számot. a) Meg lehet-e ezt tenni úgy, hogy minden sorban és oszlopban legfeljebb 4 különböző szám szerepeljen? b) Bizonyítsuk be, hogy van olyan sor vagy oszlop, amelyben legalább 4 különböző szám van. 1488. [5] A végtelen négyzetrács mezőibe be szeretnénk írni a pozitív egész számokat, mégpedig úgy, hogy az n szám éppen n-szer forduljon elő (azaz 1 darab 1-es, 2 db 2-es stb. szerepeljen); továbbá tetszőleges két, közös oldalú mezőbe került szám különbségének abszolútértéke kisebb legyen egy előre megadott k-nál. Melyik az a legkisebb egész k, amelyre a kitöltést el lehet végezni? 1489. [5] Ki lehet-e tölteni egy 1000×1000-es táblázat mezőit az 1-től 200 000-ig terjedő egészekkel úgy, hogy minden számot pontosan ötször használunk fel, minden mezőben egy szám áll, és az n-edik sor m-edik mezőjén az n · m szorzatnál kisebb vagy egyenlő szám áll? 1490. [4] Egy 10 × 20-as táblázat soraiban, oszlopaiban álló számok mindenhol számtani sorozatot alkotnak. A négy sarokban levő szám összege 30. Mennyi az összes szám összege? 1491. [4] A négyzetrácsos mezőn kiválasztottunk 100 mezőt. Mutassuk meg, hogy ezek között van 25 olyan, melyek közül semelyik kettőnek sincs közös pontja. 1492. [4] Egy 29 × 29-es négyzetrácsos papírból kivágtunk 99 olyan 2 × 2-es négyzetet, amelyek csúcsai rácspontok. Bizonyítsuk be, hogy még egy századikat is ki tudunk vágni. 1493. [5] Egy 7 × 7-es sakktáblán legfeljebb hány mezőt lehet kijelölni úgy, hogy ne legyen négy olyan közöttük, amelyek egy, a tábla oldalaival párhuzamos állású téglalap négy csúcsát alkotnák? 1494. [4] Az ábrán feketével jelzett 15 kör helyén érmék vannak. Az a célunk, hogy valamennyi érme átkerüljön a fehérrel jelzett körök helyére. Egy lépésben bármely érmével vízszintesen vagy függőlegesen átugorhatunk egy szomszédos érmét, ha annak túlsó oldalán éppen nincs érme. Legalább hány lépésre van szükség a cseréhez?

Feladatok

130

1495. [5] Ismert játék a szoliter. A játéktáblán 33 lyuk van, s kezdetben a középső kivételével mindegyikben van egy tüske. A játékot egy személy játssza; úgy lépked, hogy egy tüskével vízszintes vagy függőleges irányban átugrik egy másikat, s ekkor egy üres lyukba érkezik. Az átugrott tüskét leveszi a tábláról. A cél az, hogy a játék végén egyetlen tüske maradjon a táblán. Mutassuk meg, hogy ha sikerül elérni a kitűzött állapotot; akkor az egyedül maradt tüske vagy a tábla közepén áll, vagy a négy oldal valamelyikének középső mezőjén. 1496. [3] A 8 × 8-as sakktábla bal alsó és jobb felső sarkát kivágtuk. Le lehet-e fedni ezt a táblát hézagmentesen és átfedés nélkül 1 × 2-es dominókkal? A sakktábla mely két mezőjének hiánya esetén valósítható meg ez a lefedés? 1497. [4] A 8 × 8-as sakktábla egyik sarokmezője hiányzik. Le lehet-e fedni ezt a táblát 1 × 3-as dominókkal? A sakktábla mely mezőjének hiánya esetén valósítható meg ez a lefedés? 1498. [4] Egy 10 × 10-es sakktáblát le lehet-e fedni 1 × 4-es dominókkal? 1499. [4] Egy 10 × 10-es sakktáblát le lehet-e fedni

alakú dominókkal?

1500.

[4]

Lefedhető-e a 8 ×8-as sakktábla 15 db

alakú és 1 db

alakú dominóval?

1501.

[4]

Lefedhető-e a 8 × 8-as sakktábla 15 db

alakú és 1 db

alakú dominóval?

1502.

[4]

Lefedhető-e a 8 × 8-as sakktábla 1 db dominóval?

1503. [4] Lefedhető-e a 6 × 6-os sakktábla 9 db 1504.

[5]

Egy téglalap lefedhető db alakú és néhány

és néhány

, ill.

alakú

alakú dominóval?

alakú és alakú dominókkal is. Lefedhető-e 1 alakú dominóval is?

1505. [3] Lehet-e téglalapot összerakni a következő öt alakzatból? 1506. [4] Egy 9×14-es táblát le lehet-e fedni 10 db 3×2-es és 11 db 2 × 3-as dominóval? (Itt a dominók nem forgathatók el, míg a többi feladatban megtehetjük ezt.) 1507. [5] Ki lehet-e rakni egy 101 × 101-es táblát 2 × 2-es és 3 × 3-as dominókkal? 1508. [3] Egy 55 × 39-es táblát lefedhetünk-e 5 × 11-es dominókkal? 1509. [5] A 11 × 12-es sakktábla lefedhető 20 db dominóval, ha 1 × 6-os és 1 × 7-es dominókból válogatunk (12 db 1 × 7-es és 8 db 1 × 6-os). Le tudjuk-e fedni ezt a táblát 19 dominóval is, ha most is csak 1 × 6-os és 1 × 7-es dominókat használhatunk?

26. Feladatok a sakktáblán

131

1510. [5] Hány 6×1-es dominót lehet elhelyezni átfedés nélkül egy 9×10-es sakktáblán? (A dominókat a tábla oldalaival párhuzamosan helyezhetjük el.) 1511. [6] Bizonyítsuk be, hogy egy n×m-es tábla akkor és csak akkor fedhető le (k×1)-es dominókkal, ha k|m vagy k|n. 1512. [7] Egy téglalapot le tudunk fedni egyrétűen olyan téglalapokkal, amelyeknek valamelyik oldala egész szám. Bizonyítsuk be, hogy az eredeti téglalapnak is egész az egyik oldala. 1513. [5] 1 × 2-es dominókkal lefedtünk egy a) 6 × 6-os, illetve egy b) 4 × 100-as sakktáblát. Mutassuk meg, hogy van olyan vízszintes vagy függőleges rácsegyenes, melyet nem keresztez dominó! 1514. [4] A 8 × 8-as sakktábla összes mezőjét egy kivételével átfesthetjük-e fehérre, ha egy lépésben valamely 2 × 2-es rész mezőinek színét ellentétesre változtatjuk? (Mit mondhatunk 9 × 9-es sakktábla esetén?) 1515. [4] A 8 × 8-as sakktábla összes mezőjét egy kivételével átfesthetjük-e fehérre, ha egy lépésben valamely sor és valamely oszlop mezőinek színét ellentétesre változtatjuk? 1516. [5] A 8 × 8-as sakktáblán egy-egy lépésben felcserélhetek egymással két sort vagy két oszlopot. Elérhető-e ilyen lépésekkel, hogy a sakktábla egyik fele fehér, a másik fele fekete legyen? 1517. [5] A 8 × 8-as sakktábla mindegyik mezőjén van egy kocka, melynek valamelyik oldala fekete, a többi fehér. Elérhető-e mindig, hogy a kockák fekete oldala legyen felül, ha egy-egy alkalommal valamelyik sorban vagy valamelyik oszlopban levő kockákat fordíthatjuk el közös tengelyük mentén? (A kockák lapjai egybevágók a sakktábla mezőivel.) 1518. [4] Kirakható-e egy 7 × 9 × 11-es tégla 3 × 3 × 1-es téglákból? 1519. [6] Kirakható-e egy 6 × 6 × 6-os kocka 1 × 2 × 4-es téglákból? 1520. [6] Egy téglatest alakú ládát meg tudunk tölteni 1 × 2 × 4-es téglákkal. Mutassuk meg: ekkor a láda úgy is kitölthető, hogy az ugyanolyan hosszú téglaélek párhuzamosak. 1521. [5] A 3 × 3 × 3-as kockát az oldallapokkal párhuzamos síkokkal 27 db egybevágó kockára vágtuk. Levehetjük-e ezeket egymás után úgy, hogy mindegyik lapszomszédos az előzőleg elvett kockával, s a középső megmarad? 1522. [6] Rajzolható-e a Rubik-kocka minden lapjának minden egyes kis négyzetén egyegy átló úgy, hogy egy önmagát nem metsző zárt töröttvonalhoz jussunk?

132

Feladatok

1523. [5] Egy 6 × 6 × 6-os kockába legfeljebb hány 1 × 1 × 4-es tégla helyezhető el? (A téglák élei párhuzamosak a kocka éleivel.) 1524. [4] Kirakható-e egy 3 × 3 × 3-as kocka 6 db 2 × 2 × 1-es és 3 db 1 × 1 × 1-es téglából? 1525. [4] Mely k esetén lehet egy k × k × k-as kocka egységkockáit fekete és fehér színekkel színezni úgy, hogy bármelyik kocka pontosan két ugyanolyan színű kockával legyen szomszédos? (Két kocka szomszédos, ha van közös oldallapjuk.) 1526. [4] Lehet-e bábukat rakni a 8 × 8-as táblára úgy, hogy mind a 30 átlóban páratlan legyen a bábuk száma? 1527. [4] Legfeljebb hány bábut lehet felrakni a 8×8-as táblára úgy, hogy minden sorban, minden oszlopban és minden átlóban páros legyen az ott álló bábuk száma? 1528. [4] Egy 17×17-es táblázat mezőibe lehet-e számokat írni úgy, hogy nem mindegyik nulla, és minden mezőben olyan szám áll, mely az oldalszomszédos mezőkben álló számok összege? 1529. [4] Egy 20 × 30-as sakktáblán áthaladó egyenes metszheti-e a tábla 50 mezőjét?

27. Skatulyaelv 1530. [1] Egy ládában négyfajta alma van, minden fajtából egyenlő mennyiségű, összesen 100 darab. Hány almát kell kivenni véletlenszerűen, hogy valamelyik fajtából biztosan legyen 10 alma? 1531. [1] Egy zacskóban 80 cukor van: 20 piros, 20 fekete, 20 zöld, 20 sárga. Egy bekötött szemű gyereknek legalább hány cukrot kell kiemelnie ahhoz, hogy biztosan legyen közöttük a) valamelyik színből 4 darab? b) mindegyik színből 4 darab? 1532. [1] Egy zsákban 11 piros, 8 fehér és 6 fekete golyó van. Hány golyót kell kivenni véletlenszerűen, hogy biztosan legyen közte a) fehér vagy fekete? b) fehér és fekete? c) két különböző szín? d) valamelyik színből mind? e) két színből mindegyik? f) valamelyik színből legalább három?

27. Skatulyaelv

133

1533. [1] Egy zsákban 20 db fekete és 20 db fehér, egyforma méretű zokni van. Hány darabot kell találomra kivenni, hogy biztosan legyen közte a) 1 pár? b) 1 pár fehér? c) 2 pár? d) 2 ugyanolyan színű pár? e) 2 különböző színű pár? f) 2 pár fehér? g) 5 pár? h) 3 pár fehér és 2 pár fekete? 1534. [1] Egy zsákban 10 fehér, 20 fekete, 30 barna, egyforma méretű zokni van. Hány darabot kell kivenni véletlenszerűen, hogy biztosan legyen közte a) 1 pár? b) 1 pár fehér? c) 1 pár fekete? d) 1 pár barna? e) 2 pár? f) 5 pár? g) 3 pár fehér? h) 3 pár fekete? i) 2 pár fehér és 1 pár barna? j) 2 pár fehér, 3 pár fekete, 5 pár barna? k) k pár? 1535. [1] Egy zsákban 10 pár fekete és 10 pár barna, ugyanolyan méretű kesztyű van. Hány darabot kell találomra kivenni, hogy biztosan legyen köztük egy pár (azonos színű) kesztyű? 1536. [1] Legalább mekkora létszámú az az osztály, ahol biztosan van két olyan diák, akinek ugyanannyi foga van? 1537. [2] Legalább hány lakosa van annak az országnak, ahol biztosan van két olyan lakos, akiknek ugyanolyan a fogazata? (Ugyanazon a helyen hiányoznak, ill. vannak fogai.) 1538. [1] Igaz-e, hogy egy 37-es létszámú osztályban biztosan van négy diák, akik ugyanabban a hónapban ünneplik születésnapjukat?

134

Feladatok

1539. [3] Egy dobozban végtelen sok cédula van. Minden cédulára felírtunk egy természetes számot. Tudjuk, hogy bármiképpen veszünk is ki a dobozból két cédulát, a rajtuk levő számok különbsége legfeljebb egymillió. Bizonyítsuk be, hogy van olyan szám, mely végtelen sok cédulán szerepel. Igaz-e az állítás akkor is, ha csak azt tudjuk, hogy bárhogyan választva végtelen sok cédulát, mindig lesz köztük kettő, melyekre felírt számok különbsége legfeljebb egymillió? 1540. [3] Legalább hány, 13 tippes totóoszlop megfelelő kitöltésével biztosítható, hogy valamelyik oszlopban legyen legalább 5 találatunk? 1541. [3] 8 × 8-as sakktáblán 31 bábut helyeztünk el. Mutassuk meg, hogy a táblának van alakú része, ahol egy bábu sem áll. olyan 1542. [4] Egy gulyában két falu 65 tehene legel: vörösek, fehérek, feketék és tarkák. Igazoljuk, hogy ha nincs öt különböző korú, azonos színű tehén a gulyában, akkor található három azonos színű és egyidős tehén ugyanabból a faluból. 1543. [3] Egységsugarú körlapon 7 pontot helyeztünk el. Igazoljuk, hogy van közöttük kettő, melyek távolsága nem nagyobb 1-nél. 1544. [4] Egységsugarú körlapon 6 pontot helyeztünk el. Igazoljuk, hogy van közöttük kettő, melyek távolsága nem nagyobb 1-nél. 1545. [6] Egységsugarú körlapon 8 pontot helyeztünk el. Igazoljuk, hogy van közöttük kettő, melyek távolsága kisebb 1-nél. 1546. [3] Egy 1 m oldalú szabályos háromszöglapon 10 pontot helyeztünk el. Mutassuk meg, hogy van közöttük két olyan pont, melyek távolsága kisebb 34 cm-nél. 1547. [3] Egy 20 × 15-ös téglalapban felvettünk 26 db pontot. Mutassuk meg, hogy a pontok között van kettő, melyek távolsága legfeljebb 5. 1548. [3] Egy 70 × 70-es négyzetben felvettünk 50 db pontot. Mutassuk meg, hogy a pontok között van kettő, melyek távolsága kisebb 15-nél. 1549. [3] Egy 8×8-as négyzetben felvettünk 260 db pontot. Mutassuk meg, hogy a pontok között van kettő, melyek távolsága kisebb 1-nél. 1550. [3] Egy 35 × 42-es téglalapban felvettünk 100 db pontot. Mutassuk meg, hogy van olyan 6 × 5-ös téglalap, melybe legalább 2 pont került. 1551. [3] Egy 80 × 90-es téglalapban felvettünk 365 db pontot. Mutassuk meg, hogy van olyan 5 × 8-as téglalap, melybe legalább 3 pont került. 1552. [3] Adott egy négyzet, melynek átlója 2 egység. Melyik az a legkisebb d szám, amelyre igaz, hogy bármiképpen veszünk is fel öt pontot a négyzet belsejében vagy kerületén, mindig kiválasztható közülük két olyan pont, amelyek távolsága legfeljebb d?

27. Skatulyaelv

135

1553. [3] Adott egy 2 egység oldalú négyzet. El lehet-e helyezni a négyzet belsejében vagy határán 9 pontot úgy, hogy bármely két pont távolsága legalább 1 legyen? Mi a válasz 10 pont esetén? 1554. [4] Egy 3 × 4-es téglalapban felvettünk 6 db pontot. √ Mutassuk meg, hogy a pontok között van kettő, melyek távolsága legfeljebb 5. √ 1555. [5] A 6 3 és 18 egység befogójú derékszögű háromszög belsejében adott 865 pont. Mutassuk meg, hogy a pontok közül kiválasztható 3, amelyeket lefed egy 1 egység átmérőjű zárt félkörlemez. 1556. [5] Egy egységnyi átfogójú derékszögű háromszögben, amelynek egyik szöge 30◦ , adott 25 pont. Bizonyítsuk be, hogy van a pontok között 3, amelyek lefedhetők egy 5/17 átmérőjű körrel. √ [6] 1557. Egy derékszögű háromszög befogói 1 és 3 egységnyi hosszúak. A háromszög belsejében adott √ 20 pont. Bizonyítsuk be, hogy kiválasztható közülük 3, amely lefedhető egy 1/ 3 átmérőjű zárt félkörlemezzel. 1558. [4] Egy szabályos háromszög alakú, 1 m oldalhosszúságú céltáblát 7 találat ért. Bizonyítsuk be, hogy ezek között van két olyan, amelyek távolsága kisebb 40 cm-nél. 1559. [4] Egy 8 cm oldalú négyzetben adott 33 pont. Mutassuk meg, hogy ezek között van 3 olyan, melyek által meghatározott háromszög területe legfeljebb 2 cm2 . 1560. [4] Egységnégyzetben adott 51 pont. Igazoljuk, hogy van köztük 3 olyan, melyek 1 egy egység sugarú körben vannak. 7 1561. [4] Egy négyzet alakú, 1 m2 -es céltáblát 49 találat ért. Bizonyítsuk be, hogy van 4 olyan találati pont, melyek közül bármely kettőnek a távolsága kisebb, mint 36 cm. 1562. [6] Az 1 egység oldalú négyzetbe elhelyeztünk a) 101; b) 59 pontot úgy, hogy nincs három, amely egy egyenesbe esik. Mutassuk meg, hogy ezen pontok közül kiválasztható három úgy, hogy az általuk alkotott háromszög 1 területe legfeljebb területegység. 100 1563. [4] Egységnyi élű kocka belsejében felvettünk 400 különböző pontot. Mutassuk 4 meg, hogy van köztük 4 olyan, melyek benne vannak egy sugarú gömbben. 23 1564. [4] Egy 10 egység sugarú körben felvettünk 450 pontot. Igazoljuk, hogy ezek között van kettő, melyek távolsága kisebb, mint 1 egység. 1565. [5] Egy 16 egység sugarú körlemezen elhelyeztünk 650 pontot. Rendelkezésünkre áll egy körgyűrű, melynek külső sugara 3 egység, belső sugara 2 egység. Mutassuk meg, hogy ez elhelyezhető úgy, hogy a pontokból legalább 10-et lefedjen.

136

Feladatok

1566. [4] Egy kör belsejében tetszőlegesen kijelölünk 17 pontot. Bizonyítsuk be, hogy mindig található közülük kettő, amelyek távolsága kisebb a kör sugarának 2/3 részénél. 1567. [4] Egy egység sugarú kör kerületét egy szabályos n-szög és egy szabályos (n + 1)szög csúcsai ívekre darabolják. Bizonyítsuk be, hogy van olyan ív, melynek π hossza legfeljebb . n(n + 1) 1568. [5] Az n-oldalú szabályos sokszög csúcsai A1 , A2 , A3 , . . . , An , és adott a belsejében egy O pont.

1 Mutassuk meg, hogy van olyan i, j , melyekre 1 − π ≤ Ai OAj  ≤ π. n 1569. [4] Az egységnyi élű kocka minden pontját kiszínezzük 3 szín valamelyikével. Igaz-e, hogy mindig van két egyszínű pont, melyek távolsága legalább 1,4? 1570. [5] Egy szabályos háromszöget lefedtünk 5 egybevágó szabályos háromszöggel. Bizonyítsuk be, hogy megfelelő elhelyezéssel ezek közül már négy háromszöggel is lefedhető az eredeti háromszög. 1571. [4] 9 egyenes mindegyike metszi egy adott négyzet két szemközti oldalát, s a négyzet területét 2:3 arányban osztja. Mutassuk meg, hogy van közöttük 3 egyenes, mely egy ponton megy át. 1572. [4] Egység sugarú körvonalon elhelyeztünk 4 pontot. √ Mutassuk meg, hogy van közöttük kettő, melyek távolsága nem nagyobb 2-nél. Mutassuk meg, hogy van 1573. [4] Egység sugarú gömb felületén elhelyeztünk 5 pontot. √ közöttük kettő, melyek távolsága nem nagyobb 2-nél. 1574. [4] Egység oldalú négyzet belsejébe köröket írtunk, melyek sugarainak összege 0,51. Bizonyítsuk be, hogy van olyan, a négyzet valamelyik oldalával párhuzamos egyenes, mely legalább két kört metsz. 1575. [4] Egység oldalú négyzet belsejébe köröket írtunk, melyek kerületének összege 10 egység. Bizonyítsuk be, hogy van olyan egyenes, amely legalább négy kört metsz. 1576. [5] Egy 3 egység sugarú gömb belsejében elhelyeztünk néhány kisebb gömböt (ezek metszhetik is egymást), melyek sugarainak összege 25 egység. Mutassuk meg, hogy van olyan sík, amely legalább kilenc kisebb gömböt metsz. 1577. [5] Egy egységnyi oldalhosszúságú négyzet belsejébe egy 1000 egységnél hoszszabb, önmagát nem metsző töröttvonalat rajzoltunk (a töröttvonal egymáshoz végpontjaikkal csatlakozó egyenesszakaszokból áll). Bizonyítsuk be, hogy van olyan, a négyzet valamely oldalával párhuzamos egyenes, mely a töröttvonalat legalább 501 pontban metszi.

27. Skatulyaelv

137

1578. [5] Adott a síkban egy egyenes, egy n cm sugarú kör (n pozitív egész szám) és a körben 4n db 1 cm-es szakasz. Bizonyítsuk be, hogy húzható az adott egyenessel párhuzamosan vagy rá merőlegesen olyan húr, amelynek legalább két szakasszal van közös pontja. 1579. [4] Megjelöltük a 8×8-as sakktábla mezőinek középpontjait Feloszthatjuk-e a sakktáblát 13 egyenessel részekre úgy, hogy egyik részben se legyen két megjelölt pont? 1580. [4] A síkon adott 25 pont úgy, hogy közülük bárhogyan választva 3 pontot, azok közt van kettő, melyek távolsága kisebb, mint egy egység. Mutassuk meg, hogy van olyan 1 egység sugarú kör, mely lefed e pontokból legalább 13-at. 1581. [5] Adott a síkon 1000 pont: P1 , P2 , P3 , . . . , P1000 és egy 1 egység sugarú kör. 1000 |P Pi | ≥ 1000. Mutassuk meg, hogy van a körnek olyan P pontja, melyre i=1

1582.

[3]

A sík minden pontját kékre vagy pirosra színeztük. Igazoljuk, hogy van két azonos színű pont, melyek távolsága 1 egység.

1583. [3] A sík minden pontját kékre vagy pirosra színeztük. Mutassuk meg, hogy van két különböző színű pont, melyek távolsága 1 egység. 1584. [7] A sík minden pontját három szín valamelyikével színeztük. Mutassuk meg, hogy van két azonos színű pont, melyek távolsága 1 egység. 1585. [4] A sík minden pontját kékre vagy pirosra színeztük. Mutassuk meg, hogy van olyan szabályos háromszög, melynek csúcsai azonos színűek. 1586. [3] Egy 100-oldalú szabályos sokszög csúcsaiból 76 piros, 24 kék. Mutassuk meg, hogy van olyan négyzet, melynek csúcsai e 100 csúcs közül valók, és a csúcsok azonos színűek. 1587. [4] Egy szabályos tízszög csúcsai közül 5-öt pirosra festettünk, a többit kékre. Mutassuk meg, hogy mindig van olyan háromszög, amelyiknek minden csúcsa piros; és amelyikhez található vele egybevágó, csupa kék csúcsú háromszög. 1588. [5] Egy szabályos 13-szög csúcsait két színnel színeztük. Bizonyítsuk be, hogy van három egyszínű csúcs, melyek egy egyenlő szárú háromszög csúcsait alkotják. 1589. [5] Egy szabályos 13-szög csúcsait két színnel színeztük. Mutassuk meg, hogy van két egybevágó háromszög, melynek csúcsai a 13 pontból valók, és az egymásnak megfelelő csúcsok azonos színűek. 1590. [5] A négyzetrács rácspontjait 6 színnel színeztük. Bizonyítsuk be, hogy van olyan téglalap, melynek csúcsai azonos színűek, s oldalai rácsegyenesekre illeszkednek.

138

Feladatok

1591. [4] Négyzetrácson kijelöltünk öt rácspontot és azokat páronként összekötöttük. Mutassuk meg, hogy e szakaszok valamelyike a végpontokon kívül is tartalmaz további rácspontot. 1592. [4] Egy konvex poliédernek legalább 9 csúcsa van, a csúcsok koordinátái egész számok. Mutassuk meg, hogy található a poliéderben olyan, a csúcsoktól különböző pont, amelynek koordinátái szintén egész számok. 1593. [4] Mutassuk meg, hogy ha a síkbeli négyzetrácson adott 9 rácspont úgy, hogy semelyik három nem esik egy egyenesre, akkor az általuk meghatározott háromszögek között van olyan, amelyiknek a súlypontja is rácspont. 1594. [4] A sík minden pontját pirosra vagy kékre színeztük. Bizonyítsuk be, hogy létezik olyan háromszög, amelynek csúcsai és súlypontja azonos színűek. 1595. [3] Mutassuk meg, hogy öt, 10-nél nagyobb prímszám közül mindig kiválasztható kettő, melyek különbsége osztható 10-zel. 1596. [3] Bizonyítsuk be, hogy hét négyzetszám között mindig van kettő, melyek különbsége osztható 10-zel. 1597. [3] Mutassuk meg, hogy három négyzetszám között mindig van kettő, melyek különbsége osztható 3-mal. 1598. [3] Igazoljuk, hogy három négyzetszám között mindig van kettő, melyek különbsége osztható 4-gyel. 1599. [3] Mutassuk meg, hogy hét egész szám között mindig van kettő, melyek különbsége osztható 6-tal. 1600. [3] Mutassuk meg, hogy hét egész szám között mindig van kettő, melyek összege vagy különbsége osztható 11-gyel. 1601. [3] Bizonyítsuk be, hogy bármely n természetes számhoz végtelen sok 2-nek pozitív egész kitevős hatványa tartozik úgy, hogy közülük bármely kettő különbsége osztható n-nel. 1602. [4] Mutassuk meg, hogy 1991-nek van olyan többszöröse, melynek minden számjegye 1. 1603. [4] Mutassuk meg, hogy n egész szám között mindig van néhány szám, melyek összege osztható n-nel. 1604. [3] Igazoljuk, hogy 3 egész szám között mindig van kettő, melyek összege osztható 2-vel. 1605. [4] Mutassuk meg, hogy 5 egész szám között mindig van három, melyek összege osztható 3-mal. 1606. [5] Mutassuk meg, hogy 7 egész szám között mindig van négy, melyek összege osztható 4-gyel.

27. Skatulyaelv

139

1607. [6] Adottak az a1 , a2 , . . . , a37 , b1 , b2 , . . . , b37 , c1 , c2 , . . . , c37 egész számok. Mutassuk meg, hogy van olyan k, l, m index, melyekre 3 | ak + al + am , 3 | bk + bl + bm , 3 | ck + cl + cm . 1608. [4] Legfeljebb hány különböző prímszámot lehet megadni úgy, hogy közülük bármely három összege prímszám legyen? 1609. [3] Mutassuk meg, hogy a 2 pozitív egész kitevőjű hatványainak utolsó két jegyéből álló sorozat periodikus. 1610. [4] Bizonyítsuk be, hogy a Fibonacci-sorozat elemeinek utolsó három jegyéből álló sorozat periodikus. 1611. [4] Legyen a0 pozitív egész, s an az an−1 szám jegyei köbének összege, ahol n = = 1, 2, . . . . Mutassuk meg, hogy az így kapott sorozat periodikus. 1612. [5] Igazoljuk, hogy a 3-nak van olyan pozitív egész kitevőjű hatványa mely a 0001 számjegyekre végződik. 1613. [6] Mutassuk meg, hogy bármely n páratlan számhoz van olyan k egész szám, hogy n | 2k − 1. 1614. [6] Mutassuk meg, hogy ha (m, n) = 1, akkor van olyan k, (m, n, k ∈ n | mk − 1.

N

) hogy

1615. [3] Kiválasztunk az 1, 2, 3, . . . , 100 számokból 27 számot. Mutassuk meg, hogy ezek közt van kettő, melyek nem relatív prímek. 1616. [4] Legyen p = 3 prímszám n-edik hatványa 20-jegyű szám! Mutassuk meg, hogy ebben a számban van olyan jegy, mely legalább háromszor fordul elő. 1617. [7] Mutassuk meg, hogy ha a és b relatív prím egészek, akkor a 2 + b2 minden pozitív osztója előáll két négyzetszám összegeként. 1618. [6] Mutassuk meg, hogy ha A √és n relatív √ prímek, akkor vannak olyan x és y egész számok, melyekre |x| ≤ n, |y| ≤ n és n | Ax − y. 1619. [4] Mutassuk meg, hogy a sík 50 egyenese közül kiválasztható 8 úgy, hogy azok vagy páronként metszik egymást, vagy páronként nem metszik egymást (párhuzamosak). 1620. [6] Igazoljuk, hogy az egyenesen kijelölt n2 +1 intervallum közül kiválasztható n+1 intervallum úgy, hogy azok között vagy egyik sem tartalmaz valamely másikat, vagy bármely két intervallum egyike tartalmazza a másikat. 1621. [4] Mutassuk meg, hogy a π, 2π, 3π, . . . , 11π számok közt van kettő, melyek végtelen sok tizedesjegyben megegyeznek egymással. 1622. [2] Mutassuk meg, hogy π tizedesjegyei közt van három egymást követő számjegy, melyek így együtt végtelen sokszor fordulnak elő.

140

Feladatok

1623. [4] Mutassuk meg, hogy egy konvex poliédernek mindig van két azonos oldalszámú lapja. 1624. [4] Mutassuk meg, hogy véges, egyszerű gráfban van két azonos fokszámú csúcs. 1625. [4] Bizonyítsuk be, hogy egy 6 tagú társaságnak mindig van vagy három olyan tagja, akik kölcsönösen ismerik egymást, vagy három olyan, akik kölcsönösen nem ismerik egymást. 1626. [5] Mutassuk meg, hogy 6 irracionális szám között mindig van három olyan, hogy közülük bármely kettő összege irracionális. 1627. [5] Mutassuk meg, hogy bármely 5 irracionális szám között mindig van három olyan, hogy közülük bármely kettő összege irracionális. 1628. [5] 17 tudós mindegyike levelezik a többivel angol, német vagy francia nyelven. Mutassuk meg, hogy van közöttük három, akik egymás közt ugyanazt a nyelvet használják. 1629. [5] 66 színész közül bármely kettő játszott egymással vagy közös színházi előadáson, vagy szinkronban, vagy filmen, vagy tv-játékban, de mindenki mindenkivel pontosan csak egyféle produkcióban. Bizonyítsuk be, hogy van köztük három olyan színész, akik ugyanabban a produkcióban vettek részt. 1630. [5] Egy sakkmester 11 hetes túrára készül. A túrán mindegyik nap lejátszik legalább egy partit, de 12-nél többet nem játszik le egy hét alatt. Mutassuk meg, hogy lesznek olyan, egymást követő napok, melyekben összesen 21 partit játszik le. 1631. [5] Egy sorba leírtunk 1000 természetes számot. Mindegyik természetes szám alá írjuk le, hogy hányszor fordult elő a sorban. Az így kapott sor alá ugyanilyen módon készítsünk újabb sort úgy, hogy csak az előző sort vegyük figyelembe. Igaz-e, hogy ezt az eljárást ismételve egy idő után csupa egyforma sort fogunk kapni? 1632. [4] Egy 2n × 2n-es sakktáblára 3n bábut tettünk fel. Mutassuk meg, hogy el lehet hagyni n sort és n oszlopot úgy, hogy a megmaradó n×n-es sakktáblán egyetlen bábu se maradjon. 1633. [4] Egy 5 × 5-ös táblázat mezőibe 1 és −1 számokat írunk tetszőlegesen, és kiszámítjuk soronként is, oszloponként is az ott álló számok összegét. Mutassuk meg, hogy a kapott összegek között lesz két azonos. 1634. [4] Egy n × n-es táblázat mezőibe tetszőlegesen beírjuk az 1, 2, 3 számokat, és kiszámítjuk soronként, oszloponként és a két átlóban az ott levő számok összegét. Igazoljuk, hogy a kapott összegek között lesz két azonos. 1635. [5] Egy 100×100-as táblázat mezőibe egészeket írunk úgy, hogy két oldalszomszédos mezőben álló szám különbségének abszolút értéke legfeljebb 20. Mutassuk meg, hogy legalább három mezőben ugyanaz a szám áll.

28. Matematikai játékok

141

28. Matematikai játékok 1636. [3] Egy téglalap alakú asztalra két játékos felváltva helyez 1 Ft-os érméket (az érmék nem fedhetik egymást). Az a győztes, aki utoljára tud tenni. A kezdő nyerhet. Hogyan? 1637. [3] Egy szabályos 2n oldalú sokszögben ketten felváltva átlókat húznak be úgy, hogy azok a sokszög belsejében nem metszhetik egymást. Az veszít, aki nem tud lépni. A kezdő nyerhet. Hogyan? Mi a válasz, ha az oldalak száma 2n + 1? 1638. [4] Egy szabályos 2n oldalú sokszög csúcsait felváltva pirosra és kékre színeztük. Két játékos felváltva átlókat rajzol a sokszögbe úgy, hogy azok azonos színű csúcsokat kötnek össze és a sokszög belsejében nem metszhetik egymást. Az a vesztes, aki nem tud lépni. Melyik játékosnak van nyerő stratégiája? 1639. [4] Egy szabályos n oldalú sokszög csúcsait valamilyen körüljárást követve megszámoztuk az 1, 2, 3, . . . , n számokkal. Két játékos felváltva átlókat rajzol a sokszögbe úgy, hogy azok azonos párosságú csúcsokat kötnek össze és a sokszög belsejében nem metszhetik egymást. Az a vesztes, aki nem tud lépni. Melyik játékosnak van nyerő stratégiája? 1640. [3] Sorban egymás után felállítottunk n db kuglibábut. Két játékos felváltva felborít egy bábut vagy két szomszédos bábut. Az veszít, aki nem tud lépni. A kezdő nyerhet. Hogyan? 1641. [3] Körben felállítottunk n db kuglibábut. Két játékos felváltva felborít egy bábut vagy két szomszédos bábut. Az veszít, aki nem tud lépni. Melyik játékosnak van nyerő stratégiája? 1642. [3] Két játékos az előttük levő 2 piros, 2 fehér, 2 fekete, 2 sárga korongból felváltva helyez egy-egy korongot egy kocka csúcsaira. A kezdő játékos akkor nyer, ha a kockának lesz olyan éle, melynek két végén azonos színű korongok vannak; ha nincs ilyen él, akkor a második játékos nyer. Kinek van nyerő stratégiája? 1643. [3] Két játékos felváltva színezi egy kocka 3–3 élét pirossal, illetve feketével. Az a győztes, aki a kocka valamely lapjának mind a négy élét saját színével színezte ki. Melyik játékosnak van nyerő stratégiája? 1644. [3] 60 és 70 kavics van egy-egy kupacban. Két játékos felváltva vesz valamelyik kupacból legalább egyet, de abból az összeset is elveheti. Az győz, aki utolsóként vesz. Melyik játékosnak van nyerő stratégiája? 1645. [4] Három kupacban kavicsok vannak: 1, 65 és 117 darab. A játék ugyanaz, mint az előző feladatban. Melyik játékosnak van nyerő stratégiája?

142

Feladatok

1646. [4] Az asztalon két kupacban kavicsok vannak. Ketten felváltva vesznek a kavicsokból: vagy valamelyik kupacból 1 kavicsot, vagy mindkét kupacból 1–1 kavicsot. Az nyeri a játékot, aki az utolsó kavicsot veszi fel. Melyik játékosnak van nyerő stratégiája? 1647. [4] Két játékos felváltva helyez egy-egy huszárt (lovat) a sakktáblára. Csak olyan mezőre lehet tenni a bábut, amely nem áll ütés alatt. Az a játékos veszít, aki nem tud lépni. Melyik játékosnak van nyerő stratégiája? 1648. [4] Két játékos felváltva helyez egy-egy királyt a 9 × 9-es sakktáblára. Csak olyan mezőre lehet tenni a bábut, amely nem áll ütés alatt. Az a játékos veszít, aki nem tud lépni. Melyik játékosnak van nyerő stratégiája? 1649. [4] Két játékos felváltva helyez egy-egy 1 × 2-es dominót a 10 × 10-es sakktáblára. A dominókat úgy kell letenni, hogy két szomszédos üres mezőt fedjenek le. Az a játékos veszít, aki nem tud lépni. Melyik játékosnak van nyerő stratégiája? 1650. [4] Két játékos felváltva festi feketére egy egységkockákból épített téglatest valamely, egységkockákból álló oszlopát. (Az oszlop lehet függőleges vagy vízszintes helyzetű is.) Csak olyan oszlop választható, melynek még nincs befestett kockája. Az a játékos veszít, aki nem tud lépni. Melyik játékosnak van nyerő stratégiája, ha a téglatest méretei: a) 4 × 4 × 4; b) 4 × 4 × 3; c) 4 × 3 × 3? 1651. [4] Két játékos felváltva ír egy-egy számot egy 9×9-es táblázatba. A kezdő játékos 0-t, a második 1-est ír. A játék végén megszámolják, hogy hány sorban és oszlopban van többségben a 0 szám, ill. az 1-es. Az a játékos nyer, aki több sorban és oszlopban szerez többséget. Melyik játékosnak van nyerő stratégiája? 1652. [3] Négyzet alakban leraktunk az asztalra a) 5 × 5, illetve b) 6 × 6 kavicsot. Két játékos felváltva vesz belőlük egy darabot vagy két „oldalszomszédosat”. Az győz, aki utolsóként vesz. Melyik játékosnak van nyerő stratégiája? 1653. [4] Két kupacban cukorkák vannak, mindegyikben 9 darab. Két játékos felváltva a következőt teszi: valamelyik kupacból 1 cukorkát áttesz a másikba és az egyik kupacból kivesz magának két cukorkát. Az a játékos veszít, aki nem tud lépni. Melyik játékosnak van nyerő stratégiája? 1654. [5] Két kupacban kavicsok vannak, mindegyikben 9–9 darab. Ketten felváltva vesznek a kavicsokból: vagy az egyik kupacból 1 vagy 2 kavicsot, vagy mindkét kupacból 1–1 kavicsot. Az nyeri a játékot, aki az utolsó kavicsot veszi fel. Melyik játékosnak van nyerő stratégiája?

28. Matematikai játékok

143

1655. [4] Ketten felváltva futókat helyeznek a 8 × 8-as sakktáblára. Az a játékos veszít, aki nem tud úgy futót helyezni a táblára, hogy az ne álljon ütés alatt. Melyik játékosnak van nyerő stratégiája? 1656. [4] A 10 × 10-es tábla mezőit ketten felváltva színezik. Egy lépésben a játékos kiszínezheti a tábla egy 1 × 1-es, egy 1 × 2-es vagy egy 1 × 3-as részét, ha annak egyik mezője sincs még kiszínezve. Az a játékos nyer, aki a tábla utolsó mezőjét színezi ki. Melyik játékosnak van nyerő stratégiája? 1657. [3] Egy 8 × 8-as sakktábla bal alsó sarkából indul egy bábu. Két játékos felváltva lépteti vagy vízszintesen jobbra néhány mezőt, vagy felfelé valamennyi mezőt. Az a játékos nyer, aki a jobb felső sarokba lép. Melyik játékosnak van nyerő stratégiája? 1658. [4] (Mérgezett csoki) Van egy n × k-s csokitáblánk, melynek bal felső mezője mérgezett. Két játékos felváltva tör a táblából úgy, hogy valamelyik mezőt kiválasztja, s azt meg az egész tőle jobbra és lefelé eső szeletet letöri. Az veszít, aki kénytelen elvenni a mérgezett mezőt. Mutassuk meg, hogy a kezdő megnyerheti a játékot, ha a tábla a) n × n-es, illetve b) 2 × n-es. 1659. [4] Egy n × m-es csokoládétáblából ketten felváltva „rácsegyenesek” mentén tördelnek le darabokat. Az nyer, aki először tud 1 × 1-es darabot letörni. Kinek van nyerő stratégiája? 1660. [4] Egy 10×5-ös csokoládétáblát, ill. annak a tördelések során kapott részeit ketten felváltva tördelik „rácsegyenesek” mentén. Az nyer, aki először tud 1 × 1-es darabot letörni. Kinek van nyerő stratégiája? 1661. [4] Egy 10×5-ös csokoládétáblát, ill. annak a tördelések során kapott részeit ketten felváltva tördelik „rácsegyenesek” mentén. Az veszít, aki először kénytelen 1 × × 1-es darabot letörni. Kinek van nyerő stratégiája? 1662. [5] Adott a síkon a) 5, illetve b) 6 pont, melyek közül semelyik három sincs egy egyenesen. Ketten felváltva kötnek össze két, még össze nem kötött pontot szakasszal, és saját színükkel színezik azt. Az a játékos veszít, akinek hamarabb lesz saját háromszöge, azaz három olyan pont, melyek közti három szakasz mindegyikét ő színezte. Melyik játékosnak van nyerő stratégiája? 1663. [4] Adott a síkon 6 pont, melyek közül semelyik 3 nincs egy egyenesen. Ketten felváltva kötnek össze két, még össze nem kötött pontot szakasszal. Az a játékos veszít, aki olyan háromszöget hoz létre, melynek minden csúcsa az adott 6 pont közül való. Igazoljuk, hogy a kezdő megnyerheti a játékot, ha ügyesen játszik.

144

Feladatok

1664. [4] Két játékos a négyzetrács rácspontjaiból felváltva jelöl meg egy-egy pontot. Arra kell törekedniük, hogy a megjelölt pontok együtt egy konvex sokszög csúcsai legyenek. Aki már nem tud úgy választani rácspontot, hogy ez teljesüljön, elveszti a játékot. Melyik játékosnak van nyerő stratégiája? 1665. [3] Két játékos felváltva kiválasztja a táblán levő szám valamely, nullától különböző számjegyét, s azt levonja a számból. A régi számot letörli, s felírja a különbséget. Az a játékos nyer, aki különbségként nullát kap. Melyik játékosnak van nyerő stratégiája, ha kezdetben 1991 volt a táblán? 1666. [3] Az asztalon 27 db gyufaszál van, s ketten felváltva vesznek 1, 2 vagy 3 szálat. Az a játékos nyer, aki utolsóként vesz. Melyik játékosnak van nyerő stratégiája? 1667. [4] Az asztalon 25 db gyufaszál van, s ketten felváltva vesznek 1, 2 vagy 3 szálat, egészen addig, míg a gyufák el nem fogynak. Az a játékos nyer, aki utoljára vesz el egyszerre két szál gyufát. Melyik játékosnak van nyerő stratégiája? 1668. [3] Az asztalon 27 db gyufaszál van, s ketten felváltva vesznek 2 vagy 3 szálat, utolsó lépésként 1 is elvehető. Az a játékos nyer, aki utolsóként vesz. Melyik játékosnak van nyerő stratégiája? 1669. [3] Az asztalon 40 db gyufaszál van, s ketten felváltva vesznek 2, 3, 4 vagy 5 szálat. Az a játékos nyer, aki utolsóként vesz. Melyik játékosnak van nyerő stratégiája? 1670. [3] Az asztalon 40 db gyufaszál van, s ketten felváltva vesznek 2, 3, 4 vagy 5 szálat. Az a játékos veszít, aki utolsóként vesz. Melyik játékosnak van nyerő stratégiája? 1671. [4] Az asztalon 1991 db gyufaszál van, s ketten felváltva vesznek pn szálat, ahol p prím, n nemnegatív egész (pl. 8, 25, 7, 1 stb. szálat). Az nyer, aki utolsóként vesz. A kezdő nyerhet. Hogyan? 1672. [3] Egy kupacban 101 db kavics van. Ketten felváltva vesznek a kavicsokból 1, 2, 3, 4 vagy 5 darabot. A játék véget ér, amikor a kupacból elfogynak a kavicsok; ekkor a kezdő játékos a db, a második b db kavicsot gyűjtött magának. A kezdő játékos akkor nyer, ha a és b relatív prímek, különben a második játékos a győztes. Melyik játékosnak van nyerő stratégiája? 1673. [4] Ketten felváltva vesznek az asztalon levő n db gyufaszálból 2k db-ot (1, 2, 4, . . . ). Az a játékos nyer, aki utolsóként vesz. Melyik játékosnak van nyerő stratégiája? 1674. [4] Az asztalon n db gyufaszál van, s ketten felváltva vesznek. k db-ot akkor vehetnek el, ha k relatív prím a kupacban levő gyufaszálak számához. Az nyer, aki utolsóként vesz. Kinek van nyerő stratégiája? 1675. [5] Ketten felváltva húznak páratlan sok golyóból tetszés szerinti számú golyót úgy, hogy minden húzáskor legfeljebb az eredeti golyószám felét veszik el. A játékot az nyeri, akinek a golyók elfogytával páros sok golyója van. Van-e nyerő stratégiája valamelyik játékosnak?

28. Matematikai játékok

145

1676. [6] Két játékos felváltva vesz n db kavicsból. Az első nem veheti el kezdéskor az összeset. A soron következő játékos legalább egyet, legfeljebb az előző által elvettek számának kétszeresét veheti el. Az nyer, aki az utolsó kavicsot veszi el. Melyik játékosnak van nyerő stratégiája? 1677. [5] Ketten felváltva húznak páratlan sok golyóból 1, 2 vagy 3 golyót. A játékot az nyeri, akinek a golyók elfogytával páros sok golyója van. Kinek van nyerő stratégiája? 1678. [3] Két játékos felváltva osztja az előttük levő kupacok valamelyikét két részre. Az győz, aki utolsónak lép. Melyik játékosnak van nyerő stratégiája, ha kezdéskor az asztalon egy kupac volt 1991 db kaviccsal? 1679. [5] Két játékos felváltva osztja az előttük levő kupacok mindegyikét — melyben legalább két kavics van — két részre. Kezdetben az asztalon a) egy kupac volt 31 kaviccsal. b) egy kupac volt 100 kaviccsal. Az győz, aki utolsónak lép. Kinek van nyerő stratégiája? 1680. [4] Két játékos játszik. Az első mond egy egész számot, mely 1-nél nagyobb és 10-nél kisebb. A második játékos megszorozza ezt a számot egy, az előbbi feltételeknek eleget tevő számmal. Ezt a szorzatot most az előző szorozza egy, a megadott számkörből való számmal és így tovább. A játékot az nyeri, aki szorzatával először lépi túl az 1000-et. Melyik játékosnak van nyerő stratégiája? 1681. [4] Ketten felváltva mondanak számokat. A kezdő elsőként 1-et mond. A soron következő játékos az előzőleg elhangzott számnál legalább 1-gyel nagyobbat mond, de nagyobb számot nem mondhat, mint az előzőleg elhangzott szám és e szám jegyeinek összege. Az nyer, aki kimondja a 100-at. Kinek van nyerő stratégiája? 1682. [4] Egy 8 × 8-as sakktábla bal alsó sarkából indul egy bábu. Két játékos felváltva tolja egy-egy mezővel jobbra, felfelé vagy jobbra átlósan felfelé. Az nyer, aki a jobb felső mezőre lép. A kezdő játékos nyerhet. Hogyan? 1683. [5] A sötét király az n × m-es sakktábla bal felső sarkán áll. Ketten a következő játékot játsszák: felváltva lépnek a királlyal, de mindig csak olyan (csúcsban vagy oldalban szomszédos) mezőre, ahol addig még nem járt. Az veszít, aki nem tud lépni. Kinek van nyerő stratégiája? 1684. [4] A 8 × 8-as sakktábla bal fölső sarkában egy bábu áll, amely vízszintesen jobbra léphet legfeljebb 4 mezőt, vagy függőlegesen lefelé legfeljebb 3 mezőt. András és Balázs felváltva lépnek a bábuval. Kinek van nyerő stratégiája, ha a) az nyer, b) az veszít, aki a tábla jobb alsó mezőjére lép?

146

Feladatok

1685. [4] Egy 8 × 8-as sakktábla bal alsó sarkában áll a királynő. Ezután ketten felváltva lépnek a királynővel. Lépni a sakk szabályai szerint lehet, és minden lépésnek közelebb kell vinnie a jobb felső sarokhoz. Az a játékos veszít, aki a jobb felső sarokba kénytelen tolni a bábut. Melyik játékosnak van nyerő stratégiája? 1686. [5] Mutassuk meg, hogy véges sok lépésben győzelemmel véget érő kétszemélyes játékban mindig van nyerő stratégiája valamelyik játékosnak. 1687. [4] Egy táblára m darab 1-est és n darab 2-est írtunk fel. Két játékos felváltva letöröl a tábláról két számot és felír helyettük egy számot: ha két azonos számot töröl le, akkor egy 2-est; ha különbözőket, akkor egy 1-est ír helyettük a táblára. Néhány ilyen lépés után a táblán csak egy szám marad. Ha ez a szám 1-es, akkor a kezdő játékos; ha pedig 2-es, akkor a második játékos nyer. Melyik játékosnak van nyerő stratégiája? 1688. [5] Két játékos felváltva ír a táblára egy adott n számnál nem nagyobb pozitív egészeket. Az veszít, aki olyan számot ír, mely egy már korábban felírt számnak osztója. Mutassuk meg, hogy a kezdő játékosnak mindig van nyerő stratégiája. Mi ez a stratégia, ha n = 10? 1689. [5] Két játékos felváltva írja a táblára egy adott n szám osztóit. Az veszít, aki olyan számot ír, mely egy már korábban felírt számnak osztója. Mutassuk meg, hogy a kezdő játékosnak mindig van nyerő stratégiája. Mi ez a stratégia, ha n = 72? 1690. [4] A táblára két számot írtunk, a 25 és 36 számokat. Ketten felváltva írnak fel egyegy újabb számot. Olyan számot lehet felírni, amely a táblán levő valamely két szám különbsége, és még nem szerepel a táblán. Az a játékos veszít, aki nem tud lépni. Melyik játékosnak van nyerő stratégiája? 1691. [3] A táblára ez van felírva: . . . x 3 + . . . x 2 + . . . x + · · · = 0. Ketten felváltva írják a kipontozott helyekre az együtthatókat: valamelyik még üres helyre egy szabadon választott valós számot. A második játékos célja, hogy a kapott egyenletnek legyen egész gyöke. Elérheti-e biztosan ezt a második játékos? 1692. [4] A táblára ez van felírva: x 3 + . . . x 2 + . . . x + · · · = 0. Ketten felváltva írják a kipontozott helyekre az együtthatókat: valamelyik még üres helyre egy szabadon választott egész számot. Az első játékos célja, hogy a kapott egyenletnek mindhárom gyöke egész szám legyen. Elérheti-e biztosan ezt az első játékos? 1693. [4] A táblára ez van felírva: x 3 + . . . x 2 + . . . x + · · · = 0. Ketten felváltva írják a kipontozott helyekre az együtthatókat: valamelyik még üres helyre egy szabadon választott valós számot. Az első játékos célja, hogy a kapott egyenletnek pontosan egy valós gyöke legyen. Elérheti-e biztosan ezt az első játékos?

28. Matematikai játékok

147

1694. [3] A táblára ez van felírva: . . . x 2 + . . . x + · · · = 0. Az első játékos választ három valós számot, a második játékos pedig beírja ezeket (általa eldöntött sorrendben) a kipontozott helyekre együtthatóknak. Az első játékos célja, hogy a kapott egyenletnek két különböző racionális gyöke legyen. Elérheti-e biztosan ezt az első játékos? 1695. [3] Két játékos felváltva ír az egyenletrendszerben a csillagok helyére szabadon választott számokat együtthatónak. Mutassuk meg, hogy a kezdő játékos elérheti azt, hogy az egyenletrendszernek legyen nullától különböző gyöke. ⎫ x + y + z = 0 ⎪ ⎬ x + y + z = 0 ⎪ ⎭ x + y + z = 0 1696. [5] Bizonyítsuk be, hogy a mérgezett csoki játékban mindig a kezdőnek van nyerő stratégiája. (Lásd az 1658. feladatot.) 1697. [5] A dupla sakk ugyanaz a játék, mint a hagyományos sakk, csak egy-egy alkalommal két lépést léphet a játékos. Mutassuk meg, hogy a kezdőnek van nem vesztő stratégiája. 1698. [5] Mutassuk meg, hogy az amőba játékban a kezdőnek van nem vesztő stratégiája. (Az amőba játékot ketten játszák egy négyzetrácsos papíron. A játékosok felváltva írják egy általuk választott üres négyzetbe a jelüket: egyikük O-t, másikuk X-et. Az a játékos nyer, akinek hamarabb lesz jeleiből öt egymás mellett vízszintesen, függőlegesen vagy átlós irányban.) 1699. [5] 5 × 5-ös táblázat mezőibe a kezdő játékos mindig csak 1 jelet, a második pedig 2 jelet tesz. Az nyer, akinek először van öt saját jele vízszintesen vagy függőlegesen. A második nyerhet. Hogyan? 1700. [5] Egy 10 × 10-es táblázat mezőibe két játékos felváltva írhat egy keresztet vagy egy karikát, minden alkalommal amelyiket jónak látja. Az nyer, aki eléri, hogy sorban egymás mellett három azonos jel álljon. Kinek van nyerő stratégiája? 1701. [5] Egy 11 × 11-es sakktábla középső mezőjében áll egy bábu. A bábut két játékos felváltva eltolhatja egy másik mezőre, ahol még nem állt eddig a bábu; s mindig nagyobb távolságra kell tolni a bábut, mint előző alkalommal (két mező közti távolságon a köztük levő vízszintes és függőleges szakaszokból álló legrövidebb útvonal hosszát értjük). Az veszít, aki nem tud lépni. Kinek van nyerő stratégiája? 1702. [5] n × n-es sakktábla bal alsó sarkában áll egy bábu, melyet ketten felváltva léptetnek. Lépni csak olyan oldalszomszédos mezőre lehet, melyen még nem járt a bábu. Az veszít, aki nem tud lépni. Bizonyítsuk be, hogy ha n páros, akkor a kezdő nyerhet; ha n páratlan, akkor a második. Mi a helyzet, ha a bábu a sarokmezővel oldalszomszédos mezőről indul?

148

Feladatok

1703. [5] Két játékos közül az egyik a sakktábla valamelyik mezőjére egy huszárt helyez. Ezután a játékosok felváltva lépnek a sakkjáték szabálya szerint; de tilos olyan mezőre lépni, ahol már korábban is járt a huszár. Az veszít, aki már nem tud lépni. Kinek van nyerő stratégiája, ha a sakktábla mérete a) 8 × 8 b) m × n, ahol m ≥ n ≥ 3? 1704. [5] Egy 5 × 5-ös táblán az ábra szerint fehér és fekete bábuk állnak. A kezdő játékos fehérrel, a második feketével lép. Lépni csak oldalszomszédos üres mezőre lehet. Az nyer, aki utolsóként lép. Kinek van nyerő stratégiája?

1705. [3] Két játékos felváltva foglal le egész számokat. A kezdő akkor nyer, ha számai közt lesz három egymás utáni, különben a második győz. Kinek van nyerő stratégiája? 1706. [3] Két játékos felváltva írja egy n-jegyű szám számjegyeit. A kezdő játékos nyer, ha a kapott szám osztható 9-cel, különben a második a győztes. Melyik játékosnak van nyerő stratégiája? 1707. [5] Két játékos felváltva írja egy 2n-jegyű szám számjegyeit. E jegyek csak a 6, 7, 8 és 9 lehetnek. A kezdő játékos akkor nyer, ha a kapott szám nem osztható 9-cel, különben a második a győztes. Melyik játékosnak van nyerő stratégiája? 1708. [5] Két játékos felváltva írja egy 2n-jegyű szám számjegyeit. E jegyek csak az 1, 2, 3, 4 és 5 lehetnek. A második nyer, ha a kapott szám osztható 9-cel, különben a kezdő a győztes. Melyik játékosnak van nyerő stratégiája? 1709. [5] Két játékos a következő játékot játssza. Egy papírra felírják a pozitív egész számokat 1-től 100-ig egymás mellé, majd a számok közti 99 helyre felváltva írják az összeadás, a kivonás vagy a szorzás jelét. A játék végén kiszámolják az így kialakult számítás eredményét, és ha az páros, akkor a kezdő nyer. Kinek van nyerő stratégiája? Kinek van nyerő stratégiája akkor, ha a kezdő páratlan végeredmény esetén nyeri meg a játékot? 1710. [4] Egy asztalra sorban 7 korongot helyeztünk el. A korongok egyik oldala fehér, a másik fekete, és az asztalon levő korongok egy része fehér oldalával, a többi pedig fekete oldalával van felfelé. Ketten játszanak. A soron következő mindig kiválaszt egy fehér oldalával felfelé álló korongot; s azt, és az összes tőle jobbra levőt átfordítja a másik oldalára. Az veszít, aki nem tud lépni, azaz mindegyik korongnak a fekete oldala van felül. A kezdőnek vagy a másodiknak van-e nyerő stratégiája, és mi a nyerő stratégia?

29. Különféle kombinatorikai feladatok

149

1711. [5] Egy 2n + 1 fokú lépcső alsó n fokának mindegyikén egy-egy kő van. András és Béla felváltva rakosgatják a köveket a következő szabályok szerint: András bármely követ áthelyezheti felfelé az első szabad lépcsőfokra; Béla pedig bármelyik követ eggyel lejjebb teheti, ha az a lépcsőfok üres. Meg tudja-e Béla akadályozni, hogy András a legfelső fokra rakjon követ, ha András kezdi a játékot? 1712. [4] Az asztalon fekszik kilenc kártya, rajtuk az 1, 2, . . . , 9 számok, mindegyik kártyán egy szám van. Ezek közül két játékos felváltva választ magának egy-egy kártyát. Az a játékos nyer, akinek elsőként lesz három olyan kártyája, melyeken a számok összege 15. Melyik játékosnak van nyerő stratégiája? 1713. [5] Az 1, 2, . . . , 101 számokból ketten felváltva vesznek el 9 számot. 11 lépés után két szám marad. A kezdő nyer, ha különbségük abszolút értéke 55. A kezdő megnyerheti-e biztosan a játékot? 1714. [5] Az 1, 2, . . . , 27 számokból ketten felváltva vesznek egy-egy számot addig, míg 2 db szám marad. Ha összegük osztható 5-tel, akkor a kezdő nyert. Kinek van nyerő stratégiája? 1715. [5] A 0, 1, 2, . . . , 1024 számokból ketten felváltva vesznek: az első alkalommal 512 db számot, a második alkalommal 256 db számot, a harmadik alkalommal 128 db számot vesznek, . . . . A tizedik elvétel után 2 db szám marad. A kezdő nyer, ha ezek különbségének abszolút értéke legalább m. Mely m-nél előnyös ez a játék a kezdő számára?

29. Különféle kombinatorikai feladatok 1716. [3] Hét ember találkozott, egyesek kezet is fogtak egymással. Lehetséges-e, hogy mindenki pontosan háromszor fogott kezet? 1717. [3] Egy tíztagú társaságban mindenkitől megkérdezték, hogy hány ismerőse van a jelenlevők között. A válaszok: 6, 3, 4, 5, 7, 2, 3, 5, 6, 8. Mutassuk meg, hogy valaki tévedett. (Az ismeretségek mindig kölcsönösek.) 1718. [3] Felvehető-e a síkon 5 szakasz úgy, hogy mindegyik három másikat messen? 1719. [3] Négyzetrácsos papíron ki lehet-e jelölni 25 mezőt úgy, hogy mindegyik páratlan sok másikkal legyen oldalszomszédos? 1720. [3] Lehet-e konvex testet építeni 9 db háromszöglapból? 1721. [3] Lehet-e egy szabályos hétszög oldalait és átlóit hat színnel színezni úgy, hogy minden csúcsból induljon mindenféle színű él?

150

Feladatok

1722. [3] Miért nem lehet beírni egy szabályos 45-szög csúcsaiba a 0, 1, 2, . . . , 9 számjegyeket úgy, hogy bármely, különböző számjegyekből álló pár előforduljon valamely oldal két végén? 1723. [3] Lehet-e egész számokat írni a körökbe úgy, hogy az öt vonal mindegyikén a négy szám összege páratlan legyen? 1724. [3] Helyezzük el az ábra kis köreibe az 1, 2, 3, 4, 5, 6, 8, 9, 10, 12 számokat úgy, hogy az egy egyenesen elhelyezkedő 4-4 szám összege ugyanannyi legyen. 1725. [5] Lehet-e az ábra kis köreibe beírni az 1, 2, 3, 4, 5, 6, 7, 8, 9, 10 számokat úgy, hogy az egy egyenesen elhelyezkedő 4-4 szám összege ugyanannyi legyen? 1726. [3] Elhelyezhetők-e a szabályos nyolcszög csúcsaiba az 1, 2, . . . , 8 számok úgy, hogy bármely három szomszédos csúcsban levő szám összege 13-nál nagyobb legyen? 1727. [3] Felírhatjuk-e egy kocka éleire az 1, 2, . . . , 12 számokat úgy, hogy az egy-egy csúcsba befutó három élen levő szám összege ugyanannyi legyen? 1728. [3] Lehet-e különböző számokat írni egy tetraéder 4 lapjára úgy, hogy ha kiszámoljuk mindegyik csúcsnál a csúcsra illeszkedő lapokon álló számok összegét, akkor mindig ugyanazt az eredményt kapjuk? 1729. [3] Lehet-e különböző számokat írni egy tetraéder 6 élére úgy, hogy ha kiszámoljuk mindegyik csúcsnál a csúcsra illeszkedő éleken álló számok összegét, akkor mindig ugyanazt az eredményt kapjuk? 1730. [3] Egy kör alakú asztalnál 77-en ülnek; mindenki gondol egy egész számra, majd mindenki felírja egy cédulára két szomszédja számának összegét. Bizonyítsuk be, hogy nem állhat minden cédulán 1991. 1731. [3] Mutassuk meg, hogy ha az 1, 2, 3, . . . , 9 számokat három csoportba osztjuk, akkor valamelyikben a számok szorzata legalább 72. 1732. [3] Mutassuk meg, hogy az 1, 2, 3, . . . , 30 számokat nem lehet tíz csoportba osztani úgy, hogy minden csoportban három szám legyen, s a három szám közül a legnagyobb egyenlő legyen a másik kettő összegével. 1733. [3] Szét lehet-e osztani az 1, 2, 3, . . . , 21 számokat néhány csoportba úgy, hogy minden csoportban a legnagyobb szám egyenlő legyen a többi összegével?

29. Különféle kombinatorikai feladatok

151

1734. [3] Van-e öt olyan egész szám, melyekből képezve az összes kéttagú összeget, eredményül tíz egymást követő számot kapunk? 1735. [4] 11 érme közül 5 hamis. A hamis érme 1 grammal különbözik a valóditól. Egy érmét kiválasztva, arról egy kétkarú mérleg és mérősúlyok segítségével egy méréssel döntsük el, hogy az érme valódi-e vagy sem. 1736. [4] Egy 25 × 25-ös táblázat mezőibe a +1 és −1 számokat írjuk. Kiszámoljuk minden sorban és minden oszlopban a számok a) összegét, illetve b) szorzatát. Lehet-e az így kapott 50 szám összege nulla? 1737. [4] Adottak az x1 , x2 , . . . , xn számok, melyek értékei +1 vagy −1, továbbá x1 x2 + + x2 x3 + · · · + xn−1 xn + xn x1 = 0. Mutassuk meg, hogy 4 | n. 1738. [4] Egy 101 csúcsú poliéder éleit megjelöltük a +1 és −1 számokkal. Mutassuk meg, hogy van a poliédernek olyan csúcsa, amelybe befutó élekre írt számok szorzata +1. 1739. [4] Igaz-e, hogy bármiképpen is írjunk egy 100 csúcsú konvex poliéder csúcsaira +1 és −1 számokat, lesz a testnek olyan csúcsa, amelybe befutó élek másik végén álló számok szorzata +1? 1740. [4] Bizonyítsuk be, hogy egy 100 csúcsú konvex poliéder élei megszámozhatók a +1 és −1 számokkal úgy, hogy minden egyes csúcsba befutó élekre írt számok szorzata −1. 1741. [4] Egy kocka csúcsaiba a +1 és −1 számokat írtuk, majd minden lapra a négy csúcsban levő szám szorzatát. Lehet-e nulla a csúcsokon és a lapokon levő számok összege? 1742. [4] Le lehet-e fedni egy a) 4 × 100-as, illetve egy b) 5 × n-es táblát +1 −1 dominókkal úgy, hogy minden sorban és minden oszlopban +1 legyen az ott álló számok szorzata? 1743. [4] Egy kör kerületére 50 számot írtak fel, ezek mindegyike +1 vagy −1. Meg akarjuk tudni a számok szorzatát. Legkevesebb hány kérdésre van szükség ehhez, ha egyszerre három szomszédos szám szorzatát tudakolhatjuk meg? 1744. [3] Fel lehet-e írni egy kör kerületére 10 különböző számot úgy, hogy mindegyik szám két szomszédjának számtani közepe legyen? 1745. [3] Egy végtelen négyzetrács mezőibe írjunk egészeket úgy, hogy mindegyik szám négy oldalszomszédjának számtani közepe legyen. Mutassuk meg, hogy ez csak úgy lehet, ha ugyanazt a számot írtuk mindegyik mezőbe.

152

Feladatok

1746. [4] Fel lehet-e írni egy kör kerületére az 1, 2, 3, . . . , 10 számokat úgy, hogy bármely két szomszédos szám különbsége 3, 4 vagy 5 legyen? 1747. [4] Fel lehet-e írni egy kör kerületére az 1, 2, 3, . . . , 13 számokat úgy, hogy bármely két szomszédos szám különbsége 3, 4 vagy 5 legyen? 1748. [4] Lehet-e egymás után írni 25 számot úgy, hogy bármely három egymás után következő szám összege pozitív legyen, de a 25 szám összege negatív? Fel lehet-e írni 25 számot egy kör kerületére úgy, hogy bármely három egymás után következő szám összege pozitív legyen, de a 25 szám összege negatív? 1749. [4] Lehet-e egymás után írni 50 számot úgy, hogy bármely 17 egymás után következő szám összege pozitív legyen, és bármely 10 egymás után következő szám összege negatív? 1750. [6] Adjunk meg minél több egész számból álló sorozatot úgy, hogy a sorozat bármely hét, egymás mellett levő számának összege negatív legyen; és a sorozat bármely öt, egymás mellett levő számának összege pozitív legyen. Legfeljebb hány eleme lehet ennek a sorozatnak? 1751. [4] Mutassuk meg, hogy nem lehet pozitív az a11 a22 a33 , a12 a23 a31 , a13 a21 a32 , −a13 a22 a31 , −a12 a21 a33 , −a11 a23 a32 számok mindegyike. 1752. [4] Bizonyítsuk be, hogy egy 13 oldalú szabályos sokszög csúcsaiból nem tudunk kiválasztani ötöt úgy, hogy közülük bármely két csúcs távolsága különböző legyen. 1753. [4] Mutassuk meg, hogy az 1, 2, 3, . . . , 20 számok közül nem lehet kiválasztani tizet úgy, hogy a belőlük képzett kéttagú összegek értékei különbözzenek egymástól. 1754. [4] 100 fős katonai alakulatnál a napi ügyeletet mindig hárman látják el. Meg lehete szervezni az ügyeletet egymást követő napokon át úgy, hogy bármely két fő csak egyszer ügyeljen együtt? 1755. [4] Egy országban a repülőjáratok néhány város között úgy vannak szervezve, hogy bármely városból legfeljebb 3 másikba indul járat, és bárhonnan bármely másik városba el lehet jutni legfeljebb egy átszállással. Legfeljebb hány városban van repülőtér? 1756. [4] Rajzoljunk egy négyzetrácsos papírra olyan (konvex vagy konkáv) sokszöget, amelynek oldalai rácsegyenesek. Mutassuk meg, hogy egy ilyen sokszög oldalainak száma páros. 1757. [3] Egy kocka éle 3 cm. Hat síkkal a kockát fel lehet vágni 27 kis kockára, amelyek mindegyikének 1 cm az élhossza: két-két, a kocka lapjaival párhuzamos sík kell. Azonban megtehetjük, hogy minden egyes vágás után a kapott részeket elmozgatjuk. Ily módon feldarabolható-e hatnál kevesebb vágással a kocka 27 kis kockára?

29. Különféle kombinatorikai feladatok

153

1758. [3] Egy 4 egység élhosszúságú kockát szét akarunk vágni 64 db 1 élhosszúságú kockára. Ezt megtehetjük egyszerűen 9 vágással, ha a szétvágással keletkező darabokat nem mozgatjuk el egymástól. Hány vágásra csökkenthető ez le, ha az egyes vágások után kapott darabokat alkalmas módon átrendezhetjük? 1759. [4] Írjuk fel egy papírszalagra harmincszor egymás után a 123-at: 123 123 123 . . . 123. Legfeljebb hány részre vágható szét a szalag úgy, hogy minden darabkán más-más számot kapjunk? 1760. [3] Valaki úgy megy fel a lépcsőn, hogy egy-egy lépésével vagy 1, vagy 2 lépcsőfokot lép át. Hányféleképpen juthat fel a 10. lépcsőfokra? 1761. [3] Hányféleképpen lehet egy 2 × 10-es téglalapot 2 × 1-es dominókkal kirakni? 1762. [4] Hány olyan nyolc számból álló, csak 0-t vagy 1-et tartalmazó sorozat van, amelyben nem fordul elő két szomszédos 1-es? 1763. [4] Szétoszthatunk-e 10 000 kavicsot 100 halomba úgy, hogy mindegyik különböző számú kavicsot tartalmazzon, de bármelyik halmot osztjuk is két részre, a keletkezett 101 halomra ez már nem teljesül? 1764. [4] Egy 10 résztvevős pingpong-bajnokságon mindenki egyszer játszott mindenkivel. Az egyes versenyzők győzelmeinek és vereségeinek számát jelölje rendre x1 , x2 , . . . , x10 , illetve y1 , y2 , . . . , y10 . Bizonyítsuk be, hogy x12 + x22 + · · · + 2 2 + x10 = y12 + y22 + · · · + y10 . 1765. [5] Arthur király kerekasztala mellett k különböző család 13 lovagja ül, ahol 1 < k < 13. Valamennyien vagy egy arany-, vagy egy ezüstserleget tartanak a kezükben; az aranyserlegek száma éppen k. Arthur király parancsot adott a lovagoknak, hogy a következő percben valamennyien adják át a serlegüket a jobbra ülő szomszédjuknak; majd ismételjék meg ezt a műveletet még tizenkétszer. Bizonyítsuk be, hogy valamelyik percben volt két, azonos családból származó lovag, akik aranyserleget tartottak a kezükben. 1766. [5] Arthur király ötven lovagja a kerek asztal mellett ül. Előttük egy-egy serleg, tele vörös vagy fehér borral. Éjfélkor mindenki átadja valakinek a serlegét, mégpedig mindazok, akiknél vörös bor van, a jobb szomszédjuknak, a többiek pedig a bal oldali másodszomszédjuknak. Bizonyítsuk be, hogy így lesz olyan lovag, akinek nem jut serleg. (Az asztalon vörös bor is, fehér bor is volt.) 1767. [5] Egy n × n-es táblázat mezőibe nemnegatív egész számokat írtunk úgy, hogy ha valamely sor és oszlop kereszteződésében nulla áll, akkor ebben a sorban és oszlopban a számok összege legalább n. Mutassuk meg, hogy a táblázatban lévő számok összege legalább n2 /2.

154

Feladatok

1768. [5] 0, 01, 0110, 01101001, 0110100110010110, . . . A sorozat következő elemét úgy kapjuk, hogy az utolsó 0–1 sorozathoz hozzáillesztjük a sorozat komplementerét (melyet az eredetiből úgy nyerünk, hogy a 0-t és az 1-est felcseréljük). A sorozat 12. eleme egy 2048 számjegyből álló 0–1 sorozat. Ebben a sorozatban milyen számjegy áll az 1996. helyen? 1769. [5] Van 1000 kártyánk, ezeket 000-tól 999-ig megszámozták. Van továbbá 100 dobozunk, ezek száma 00-tól 99-ig tart. Egy kártyát csak olyan dobozba szabad betenni, amelynek számát úgy kapjuk, hogy a kártyáról egy jegyet letörlünk. Mutassuk meg, hogy a kártyákat már 50 dobozba is berakhatjuk, de kevesebbe nem! 1770. [5] Egy asztalon 100 kavicshalom van, bennük rendre 1, 2, . . . , 100 darab kavics. Egy lépésben akárhány kiszemelt halmot csökkenthetünk; feltéve, hogy mindegyikből ugyanannyi kavicsot veszünk el. Legkevesebb hány lépésben tudjuk valamennyi kavicsot elvenni az asztalról? 1771. [4] A kilenctagú (1, 2, 3, 4, 5, 6, 7, 8, 9) számsorozatot állítsuk elő minél kevesebb, olyan 9 tagú számsorozat „összegeként”, amelyek mindegyikében csak kétféle szám szerepel (például: (0, 2, 2, 0, 0, 2, 2, 0, 0) egy ilyen sorozat). A 9 tagú sorozatok „összegét” úgy értelmezzük, hogy az azonos helyen álló számokat adjuk össze (például: (1, 1, 0, 0, 1, 0, 0, 0, 1) + (0, 2, 2, 0, 0, 2, 2, 0, 0) = = (1, 3, 2, 0, 1, 2, 2, 0, 1)). 1772. [3] Az 1, 2, 3, . . . , 100 számok közül tetszőlegesen kiválasztottunk 25 számot. Ezekhez lehet-e mindig további 25 számot választani úgy, hogy a kiválasztott 50 szám összege ugyanannyi legyen, mint a megmaradó 50 szám összege? 1773. [3] Az asztalra egy sorban kiraktunk 100 darab mérősúlyt úgy, hogy bármely két szomszédos súly között 1 gramm az eltérés. Bizonyítsuk be, hogy a mérősúlyok szétoszthatók két, 50–50 darabból álló csoportra úgy, hogy a két csoport azonos tömegű. 1774. [5] 2k érme között nincs három különböző súlyú. Egy kétkarú mérlegen súlyok használata nélkül k méréssel meg kell mondanunk, hogy van-e köztük két különböző súlyú; és ha van, kell mutatni egy könnyebbet és egy nehezebbet. Bizonyítsuk be, hogy ez lehetséges és mutassuk meg, hogyan. 1775. [6] 100 darab érme között nincs három különböző súlyú. Egy kétkarú mérlegen súlyok használata nélkül 7 méréssel meg kell mondanunk, hogy van-e köztük két különböző súlyú, és ha van, kell mutatni egy könnyebbet és egy nehezebbet. Bizonyítsuk be, hogy ez lehetséges és mutassuk meg, hogyan. 1776. [5] Van 6 egyenlő átmérőjű golyónk, melyek közül 2 piros, 2 fehér, 2 pedig kék. Az egyszínű golyók egyike aranyból, a másika ezüstből készült. Legkevesebb hány mérés kell ahhoz, hogy egy kétkarú mérleg segítségével ki tudjuk választani a három aranygolyót? (Az ugyanolyan anyagból készült golyók egyenlő súlyúak.)

29. Különféle kombinatorikai feladatok

155

1777. [4] Tudjuk, hogy 11, külsőre egyforma golyó közül kettő radioaktív. Egy méréssel meg tudjuk állapítani, hogy a golyók egy csoportjában van-e radioaktív vagy nincs. Mutassuk meg, hogy hét méréssel ki lehet választani a két radioaktív golyót. 1778. [6] Van egy kétkarú mérlegünk, továbbá 101 darab mérősúly, amelyek együttes tömege 200 gramm. Valamennyi mérősúly tömegének grammokban kifejezett mérőszáma egész szám. A súlyokat csökkenő sorrendben rakjuk be a serpenyőkbe úgy, hogy ha a mérleg egyensúlyban van, akkor a bal oldaliba; ha pedig nem, akkor a könnyebbikbe tesszük a következőt. Bizonyítsuk be, hogy miután valamennyi súlyt föltettük, a mérleg egyensúlyban lesz. 1779. [5] 100 kavicsot 50 kupacba raktunk, mindegyikbe legalább egyet. Bizonyítsuk be, hogy ha egyetlen kupacban sincs 50-nél több kavics, akkor a kupacok két csoportba rendezhetők úgy, hogy a két kupacban ugyanannyi kavics van. 1780. [5] Adott 35 pozitív egész szám, amelyek összege 100, és egyikük sem nagyobb 50-nél. Bizonyítsuk be, hogy van köztük néhány olyan, amelyek összege 50. 1781. [5] Van néhány arany és néhány ezüst érménk. Az érmék közül bármely kettő súlya különböző. Valaki sorba rendezte őket súlyuk szerint, de tévedésből a legnehezebbet nem az első, hanem az utolsó helyre rakta. (Így az 1. helyre a 2. legnehezebb, a 2. helyre a 3. legnehezebb stb. került.) Át akarjuk rendezni őket a helyes nehézségi sorrendbe, de többlet helyünk nincs, ezért egy lépésben mindig csak két (nem feltétlenül szomszédos) érmét tudunk felcserélni. Igazoljuk, hogy ha érméinket ilyen feltétel mellett rendeztük át, valamelyik lépésben egy arany és egy ezüst érmét is fel kellett cserélnünk. 1782. [4] Az 1 − 2 − 3 − 4 − · · · − 1999 kifejezésben zárójelpárok alkalmas elhelyezésével jelölhetjük ki a műveletek sorrendjét. Bizonyítsuk be, hogy vannak olyan zárójelezések, amelyek ugyanazt az eredményt adják. 1783. [4] Mutassuk meg, hogy egy 2n oldalú konvex sokszögnek mindig van olyan átlója, amely egyik oldallal sem párhuzamos! 1784. [4] Egy 12 × 12-es sakktábla bizonyos mezői feketék, a többiek pedig fehérek. A fekete mezők fehérre, a fehérek pedig feketére festhetők, de csak úgy, ha egy lépésben egy teljes sor vagy egy teljes oszlop valamennyi mezőjének megváltoztatjuk a színét. Elérhető-e mindig, hogy a sakktábla valamennyi mezője fekete legyen? 1785. [5] Egy 8×8-as tábla mezőiben egész számok állnak. Egy lépésben valamely 3×3as vagy 4 × 4-es részen mindegyik számot 1-gyel növeljük. Elérhető-e mindig, hogy minden szám 10-zel osztható legyen?

156

Feladatok

1786. [5] Egy iskolában a tanulók 10 fős csapatokat szerveztek. Egy diák több csapatnak is tagja lehet, vagy akár egyiknek sem. A csapatok száma 500. Bizonyítsuk be, hogy a diákokat el lehet helyezni két teremben úgy, hogy minden csapatnak mindkét teremben legyen tagja. 1787. [6] Legyen n 2-nél nagyobb páros szám. Egy n × n-es sakktábla mezőit kiszíneztük n2 színnel úgy, hogy minden színű mezőből pontosan kettő van. Bizonyítsuk 2 be, hogy el lehet helyezni n bástyát csupa különböző színű mezőre úgy, hogy semelyik kettő se üsse egymást. 1788. [6] Egy 3n + 1 tagú társaság bármely két tagja vagy teniszezni, vagy sakkozni, vagy pingpongozni szokott egymással. Mindegyiküknek n tenisz-, n sakk- és n pingpongpartnere van. Bizonyítsuk be, hogy van a társaságban három olyan ember, akik egymás között mind a három játékot játsszák. 1789. [6] Bizonyítsuk be, hogy 1-től 1986-ig a természetes számok kiszínezhetők pirossal és kékkel úgy, hogy ne forduljon elő 18 tagú számtani sorozat, melynek minden tagja ugyanolyan színű. 1790. [6] Bizonyítsuk be, hogy egy 1000 oldalú szabályos sokszög átlóit és oldalait színezhetjük úgy piros és kék színnel (mindegyik szakaszt csak az egyik színnel), hogy a sokszögnek ne legyen 20 olyan csúcsa, amelyek közötti szakaszok mindegyike ugyanolyan színű. 1791. [6] Egy 500 tagú vállalat dolgozói 2n nyelv közül beszélnek néhányat, minden dolgozó legalább n nyelvet beszél. Bizonyítsuk be, hogy ekkor kiválasztható 14 nyelv úgy, hogy minden dolgozó a 14 nyelv közül legalább egyen beszél. 1792. [7] Legyen adva egy X alaphalmaz n-elemű részhalmazainak egy F rendszere. Mutassuk meg, hogy ha |F| < 2n−1 , akkor X-nek van olyan, két színnel való színezése, hogy egyik F-beli n-es sem lesz egyszínű.

29. Különféle kombinatorikai feladatok

157

30. Konstrukciók 1793. [5] Az ötágú csillagon megjelölt 10 köröcske közül minél többre helyezzünk korongot. Korongot a következő módon lehet felrakni: valamelyik üres köröcskébe teszünk egyet, majd valamelyik szomszédos köröcskét átugorva (mindegy, hogy ott van korong vagy nincs) és egy üres köröcskére érkezve, a korong ott marad. Ugrani csak valamelyik egyenes vonal mentén lehet. 1794. [3] Egy kereskedőnek volt egy 40 kg-os mérősúlya, ami egyszer leesett és négy darabra tört szét. Amikor a darabokat megmérte, kiderült, hogy mindegyik darab egész számú kg-ot nyom, és a négy darabbal minden egész kg súlyt le lehet mérni egy kétkarú mérlegen 1 kg-tól 40 kg-ig. Hány kg-os darabokra tört szét a mérősúly? 1795. [3] Van 100 db, páronként különböző súlyú golyónk és egy kétkarú mérlegünk. Válasszuk ki minél kevesebb mérlegeléssel a legkönnyebb golyót. 1796. [3] Van 100 db, páronként különböző súlyú golyónk és egy kétkarú mérlegünk. Válasszuk ki minél kevesebb mérlegeléssel a legkönnyebb és a legnehezebb golyót. 1797. [3] Van 100 db, páronként különböző súlyú golyónk és egy kétkarú mérlegünk. Válasszuk ki minél kevesebb mérlegeléssel a két legnehezebb golyót. 1798. [3] 9 érme közül egy hamis, s ez könnyebb, mint a többi (a többi egyenlő súlyú). Egy kétkarú mérlegen súlyok felhasználása nélkül két mérlegeléssel keressük ki közülük a hamis érmét. 1799. [5] 12 pénzérme közül egy hamis. Tudjuk, hogy a hamis pénz súlya különbözik a többitől, de nem tudjuk, hogy könnyebb-e náluk vagy nehezebb. Az igazi pénzdarabok mind egyenlő súlyúak. Súlyok nélkül kétserpenyős mérlegen három méréssel meg kell állapítanunk, hogy melyik a hamis pénz, és könnyebb-e vagy nehezebb, mint a többi. 1800. [3] Egy repülőtéren a várakozó 128 utas között van egy terrorista, aki ruházatában fegyvert rejteget, s persze nem tudjuk, hogy melyik ez az utas. Van egy készülék, amely jelzi, ha a vizsgálati teremben levők között valaki fegyvert rejteget.

158

Feladatok

Csoportokba osztva kívánják megvizsgálni az utasokat. A lehető leggyorsabban, legkevesebb vizsgálattal szeretnék megtalálni a terroristát. Hogyan csinálják? 1801. [3] 10 láda pénz között az egyik ládában csupa 11 grammos érme van, a többiben 10 grammosak az érmék. Egykarú mérlegen mérősúlyokkal egyetlen méréssel állapítsuk meg, hogy melyik a nehezebb érméket tartalmazó láda. 1802. [4] Két rabló úgy szokott megosztozni a zsákmányon, hogy az egyik kétfelé osztja azt, és a másik azt a részt veszi el, amelyiket akarja. Ez így igazságos, mert mindkettőnek megvan a lehetősége arra, hogy megszerezze a zsákmány felét. Hogyan osztozkodjon három rabló, ha azt szeretnék biztosítani, hogy bármelyikük megkapja a zsákmány harmadát, bármit is csinál a másik kettő? 1803. [4] A folyón átfektetett kábelt — amelynek 49, egymástól elszigetelt áramkört kell átvezetnie — a szokásostól eltérően egyszínű szigetelésű drótokból sodorták. Így aztán nem lehet tudni, hogy a két parton látható drótvégződések közül melyik melyikkel tartozik össze (azaz, melyik ugyanannak a vezetéknek a két vége). Az azonosítást úgy kell elvégezni, hogy mindegyik drótvégződésre cédulát kötünk, és az összetartozókra mindkét parton ugyanazt a számot írjuk. Ehhez rendelkezésre áll a part mentén futó villanyvezeték, egy fázisceruza, valamint egy csónak a folyón való átkelésre. Szervezzük meg a munkát úgy, hogy minél kevesebbszer kelljen átkelni a folyón. 1804. [4] Egy versenyen 18 csapat mérkőzik egymással. Szervezzük meg a fordulókat úgy, hogy egy-egy fordulóban minden csapatnak legyen ellenfele, s a verseny során minden csapat mindegyikkel pontosan egy mérkőzést játsszék. 1805. [5] Mutassuk meg, hogy megszervezhető a sportélet egy 3n + 1 tagú társaságban úgy, hogy bármely két tag vagy teniszezik, vagy sakkozik, vagy pingpongozik egymással; és mindegyiküknek n tenisz-, n sakk- és n pingpongpartnere legyen. 1806. [3] Egy 4 × 4-es táblázat mezői közül kiválasztottunk hatot, s mindegyikbe rajzoltunk egy-egy csillagot. Mutassuk meg, hogy van két olyan sor és két olyan oszlop, melyek együtt tartalmazzák mind a hat csillagot! Helyezzünk el a táblázatban 7 csillagot úgy, hogy ne lehessen kiválasztani két olyan sort és két olyan oszlopot, amelyek együtt tartalmaznák mind a 7 csillagot. 1807. [4] Adél, Bori, Csilla és Dóri egy sötét, szűk alagúton szeretne átjutni. Van egy 12 percig égő lámpásuk. Adél 1, Bori 2, Csilla 4 és Dóri 5 perc alatt képes megtenni a távot. A sötétben félnek, ezért az alagútban lámpás nélkül nem mehetnek és a szűk alagútban egyszerre legfeljebb ketten férnek el. Átjuthatnake mindannyian a szűk alagúton? 1808. [4] Egy szigeten felszállásra készen áll néhány repülőgép. Mindegyik tartályában annyi üzemanyag van, amennyi egy világ körüli út felére elegendő. Repülés közben az egyik repülőgép tartályából át lehet szivattyúzni az üzemanyagot a másik gép tartályába. A speciális üzemanyag egyetlen ellátó telepe a szigeten

31. Teljes indukció

159

van. Az üzemanyag felvételével kapcsolatban feltételezzük, hogy a gépek sem a földön, sem a levegőben nem vesztenek időt. Hány egyforma repülőgépre van szükség a szigeten ahhoz, hogy biztosítsák egy gép világ körüli útját leszállás nélkül? Valamennyi gép sebessége egyforma, üzemanyagfelhasználása azonos, mindnek vissza kell térnie a szigetre. 1809. [4] Egy kutató két teherhordó segítségével át szeretne vágni a sivatagon a legközelebbi, 380 km-re levő oázisba. Napi 60 km-t tudnak megtenni, és mind a kutató, mind a teherhordók legfeljebb négy napra elegendő élelmet és vizet tudnak magukkal vinni. A kutatónak biztosítania kell, hogy a teherhordók visszatérhessenek oda, ahonnan elindultak. A sivatagban élelmiszert nem tárolhatnak. Átjuthat-e a kutató ezekkel a feltételekkel az oázisba, ha nem számít arra, hogy onnan elébe jönnek? 1810. [4] Két várost egy autóút köt össze. A városokban olyan autók vannak, amelyek éppen annyi benzint képesek tankolni, amennyi a két város közti út felének megtételéhez elegendő. Segédautók felhasználásával az egyik városból egy A autót akarunk eljuttatni a másik városba. A segédautók az út bizonyos pontjain benzint adhatnak át egymásnak, és A-nak; de csak annyit, hogy kiindulási állomásukra visszatérhessenek. Mennyi a szükséges segédautók minimális száma? 1811. [4] 10 cowboy párbajt vív a következő szabályok szerint: — Mindenki egy lövést ad le, s ez a lövés halálos. — Mindenki a hozzá legközelebbit lövi le; ha több ilyen is van, akkor közülük valamelyiket. — Mindenki ugyanabban a pillanatban adja le lövését. Lehetséges-e, hogy ennek a vérengzésnek csak két áldozata van?

31. Teljes indukció 1812. [3] Hol a hiba a következő bizonyításban? Állítás: Bármely n pozitív egészre a n−1 = 1, ahol a > 0 tetszőleges szám. Bizonyítás: Ha n = 1, akkor a n−1 = a 1−1 = a 0 = 1. Ha feltesszük, hogy a tétel igaz az 1, 2, . . . , n esetre, akkor azt kapjuk, hogy a n−1 · a n−1 1 · 1 a (n+1)−1 = a n = = = 1; a n−2 1 tehát a tétel (n + 1) esetére is igaz. 1813. [3] Hol a hiba a következő bizonyításban? 1 1 1 3 1 Állítás: + + ··· + = − . 1·2 2·3 (n − 1) · n 2 n

Feladatok

160

Bizonyítás: Indukciót alkalmazunk n-re. n = 1 esetén 3/2 − 1/n = 1/(1 · 2); ha feltesszük, hogy a tétel igaz n-re, akkor 1 1 1 3 1 1 + ···+ + = − + = 1·2 (n − 1) · n n · (n + 1) 2 n n(n + 1)

1 1 3 1 3 1 − = − . = − + 2 n n n+1 2 n+1 1814. [3] Hol a hiba a következő bizonyításban? Állítás: Minden lánynak ugyanolyan színű a szeme. Bizonyítás. Teljes indukcióval bizonyítunk. Legyen a lányok száma: n. n = 1-re az állítás nyilvánvalóan (semmitmondóan) igaz. Marad az n-ről (n + + 1)-re való átmenet. Hogy konkrét esetről beszéljünk, 3-ról 4-re lépünk át; az általános esetet az olvasóra bízzuk. Legyen a négy lány Anna, Bea, Cecília és Dorottya, vagy röviden A, B, C és D. Állítólagosan (n = 3) A, B és C szeme ugyanolyan színű. A B, C és D szeme szintén ugyanolyan színű (n = 3). Következésképp mind a négy lány, A, B, C és D szeme ugyanolyan színű; hogy teljesen világos legyen, nézzük a diagramot:    A, B,  C, D Ez bizonyítja az állítást n + 1 = 4 esetén; a 4-ről 5-re való átlépés nyilván nem sokkal nehezebb. 1815. [3] Igazoljuk a következő oszthatóságokat. a) 4 | 7n + 3n+1 , n = 1, 2, . . . b) 9 | 7n + 3n − 1, n = 1, 2, . . . c) 7 | 5 · 9n−1 + 24n−3 , n = 1, 2, . . . d) 17 | 7 · 52n−1 + 23n+1 , n = 1, 2, . . . e) 19 | 5 · 23n−2 + 33n−1 , n = 1, 2, . . . f) 17 | 62n + 19n − 2n+1 , n = 1, 2, . . . g) 9 | n3 + (n + 1)3 + (n + 2)3 , n = 1, 2, . . . h) 7 | 32n+1 + 2n+2 , n = 1, 2, . . . i) 133 | 11n+2 + 122n+1 , n = 0, 1, . . . j) 16 | 32n+2 + 8n − 9, n = 1, 2, . . . k) 19 | 52n+1 · 2n+2 + 3n+2 · 22n+1 , n = 1, 2, . . . l) 5 | 174n+1 + 3 · 92n , n = 1, 2, . . . m) 4 | 3 · 172n + 25n , n = 1, 2, . . . n) 3 | 5 · 17n + 52n , n = 1, 2, . . . o) 23 | 27n+3 + 32n+1 · 54n+1 , n = 1, 2, . . . n p) 3n+1 | 23 + 1, n = 0, 1, . . .

31. Teljes indukció

161

1816. [3] Mutassuk meg, hogy (1 + 2 + 3 + · · · + n)2 = 13 + 23 + 33 + · · · + n3 . 1817. [3] Mutassuk meg, hogy n · (n + 1)! (n + 2)! 1 · 2! 2 · 3! = − 2. + 2 + ···+ 2 2 2n 2n 1 · 3! 2 · 4! n · (n + 2)! 1818. [4] + 2 + ··· + =? 3 3 3n 1819. [3] a) Tudjuk, hogy a1 = 4, an+1 = 3an − 2, n = 1, 2, 3, . . . Mutassuk meg, hogy an = 3n + 1. b) Tudjuk, hogy a1 = 2, a2 = 8, an+2 = 4an+1 − 3an , n = 1, 2, 3, . . . Mutassuk meg, hogy an = 3n − 1. c) Tudjuk, hogy a1 = 1, a2 = 5, an+2 = 5an+1 − 6an , n = 1, 2, 3, . . . Mutassuk meg, hogy an = 3n − 2n . d) Tudjuk, hogy a1 = 1, a2 = 9, an+2 = 9an+1 − 20an , n = 1, 2, 3, . . . Mutassuk meg, hogy an = 5n − 4n . e) Tudjuk, hogy a1 = 3, a2 = 15, an+2 = 5an+1 − 4an , n = 1, 2, 3, . . . Mutassuk meg, hogy an = 4n − 1. f) Tudjuk, hogy a1 = 29, a2 = 85, an+2 = 5an+1 −6an , n = 1, 2, 3, . . . Mutassuk meg, hogy an = 2n + 3n+2 . g) Tudjuk, hogy a1 = 3, a2 = 6, an+2 = 3an+1 − 2an − 1, n = 1, 2, 3, . . . Mutassuk meg, hogy an = 2n + n. 1 1 an · an+1 , n = 1, 2, 3, . . . Mutassuk , a2 = , an+2 = 2 3 3an − 2an+1 1 meg, hogy an = n−1 . 2 +1

1820. [4] Tudjuk, hogy a1 =

2 an−1 +2 , n = 2, 3, 4, . . . Mutassuk meg, hogy an−2 an értéke egész szám minden n-re.

1821. [5] Tudjuk, hogy a0 = a1 = 1, an =

Igazoljuk a Fibonacci-sorozat alábbi tulajdonságait. (f1 = f2 = 1, fn+2 = fn+1 + fn , n = 1, 2, 3, . . . ) 1822. [4] f1 + f2 + · · · + fn = fn+2 − 1, n = 1, 2, 3, . . . 1823. [4] f1 + f3 + · · · + f2n−1 = f2n , n = 1, 2, 3, . . . 1824. [4] f2 + f4 + · · · + f2n = f2n+1 − 1, n = 1, 2, 3, . . . 1825. [4] f1 + 2f2 + 3f3 + · · · + nfn = (n − 1)fn+2 − fn+1 + 2, n = 1, 2, 3, . . . 1826. [4] f1 − f2 + f3 − f4 + · · · + (−1)n+1 fn = (−1)n+1 fn−1 + 1, n = 2, 3, 4, . . . 1827. [4] f12 + f22 + · · · + fn2 = fn fn+1 , n = 1, 2, 3, . . .

Feladatok

162 2 1828. [4] f1 f2 + f2 f3 + · · · + f2n−1 f2n = f2n , n = 1, 2, 3, . . . 2 − 1, n = 1, 2, 3, . . . 1829. [4] f1 f2 + f2 f3 + · · · + f2n f2n+1 = f2n+1

1830. [4] a) b) c) d) e)

3 | f4k , k = 1, 2, 3, . . . 4 | f6k , k = 1, 2, 3, . . . 5 | f5k , k = 1, 2, 3, . . . 7 | f8k , k = 1, 2, 3, . . . 9 | f12k , k = 1, 2, 3, . . .

1831. [4] fn2 = fn+1 fn−1 + (−1)n+1 , n = 2, 3, 4, . . . 1832. [4] f2n−1 = fn fn+1 − fn−2 fn−1 , n = 3, 4, 5, . . . 1833. [4] (−1)n = fn+1 fn+2 − fn fn+3 , n = 1, 2, 3, . . . 2 1834. [4] fn2 + fn+1 = f2n+1 , n = 1, 2, 3, . . . 2 2 1835. [4] fn+1 − fn−1 = f2n , n = 2, 3, 4, . . . 2 1836. [4] (fn fn+3 )2 + (2fn+1 fn+2 )2 = f2n+3 , n = 1, 2, 3, . . .

1837. [5] fn+h fn+k − fn fn+k+h = (−1)n fk fh , h = 1, 2, 3, . . . , k = 1, 2, 3, . . . , n = = 1, 2, 3, . . . 3 3 1838. [5] f3n = fn+1 + fn3 − fn−1 , n = 2, 3, 4, . . .

1839. [5] fm+n = fm−1 fn + fm fn+1 , m = 2, 3, 4, . . . , n = 1, 2, 3, . . . 1840. [5] fk | fk+2n + fk−2n , n = 1, 2, 3, . . . , k > 2n 1841. [6] Igazoljuk, hogy ha m | n, akkor fm | fn . 1842. [4] Mutassuk meg, hogy

1 1 1 1 1 + + + ···+ + < 1. f1 f3 f2 f4 f3 f5 f1999 f2001 f2000 f2002

1843. [4] fn4 − fn−2 · fn−1 · fn+1 · fn+2 = 1, n = 3, 4, 5, . . . 2 2 1844. [4] fn+1 = 4fn · fn−1 + fn−2 , n = 3, 4, 5, . . . k 1845. [5] fn2 | fn−1 − fkn−1 , n = 2, 3, 4, . . . , k = 1, 2, 3, . . . k 1846. [5] fn2 | (−1)k+1 fn−2 − fkn−2 , n = 3, 4, 5, . . . , k = 1, 2, 3, . . .

1847. [4] Mutassuk meg, hogy minden természetes szám előállítható a Fibonacci-sorozat páronként különböző elemeinek összegeként. 1848. [5] Bizonyítsuk be, hogy ha a az n!-nál nem nagyobb pozitív egész szám, akkor a előállítható az n! szám legfeljebb n db, páronként különböző osztójának összegeként.

31. Teljes indukció

163

1849. [3] Mutassuk meg, hogy egy négyzet feldarabolható n db négyzetre, ahol n ≥ 6. 1850. [5] Mutassuk meg, hogy van olyan N, hogy minden n ≥ N esetén egy kockát fel lehet darabolni n darab kisebb kockára. (Bizonyítható, hogy már N = 48 esetén is igaz az állítás.) 1851. [3] Mutassuk meg, hogy egy háromszög feldarabolható n db, hozzá hasonló háromszögre, ahol n ≥ 6. 1852. [4] Igazoljuk, hogy bármely háromszög feldarabolható n darab (n ≥ 4) egyenlő szárú háromszögre! 1853. [3] Néhány egyenes a síkot tartományokra bontja. Mutassuk meg, hogy ezek a részek két színnel kiszínezhetők úgy, hogy az oldalszomszédos tartományok különböző színűek legyenek. 1854. [5] Igazoljuk, hogy p | np − n, ahol p prímszám, n ∈ N (kis Fermat-tétel). 1855. [4] Legyen n 3-mal osztható természetes szám! Hagyjunk el az n − 1, n − 2, n − 3, . . . , 3, 2, 1 sorozatból minden harmadik számot! A megmaradók közül az első kettőt lássuk el pozitív előjellel, a következő kettőt negatív előjellel, az utánuk következő kettőt ismét pozitív előjellel és így tovább. Bizonyítsuk be, hogy az így kapott számok összege n-nel egyenlő. 1856. [4] Sorban egymás után írtunk 1992 db számot, ezek mindegyike +1 vagy −1. A következő műveletet végezzük: két szomszédos szám alá szorzatukat írjuk. Ezt az eljárást a kapott 1991 db számon is elvégezzük, majd újra és újra ismételjük ezt. Bizonyítsuk be, hogy ha az első sorban volt −1, akkor a kapott számháromszögben legalább 1992 db −1 van. 1857. [4] Mutassuk meg, hogy 2n | (n + 1)(n + 2) . . . (2n), n ∈ N . 1858. [4] Mutassuk meg, hogy 2n+1 − 1 egész szám közül mindig kiválasztható 2n db szám úgy, hogy összegük osztható legyen 2n -nel (n ∈ N ). 1859. [4] Bizonyítsuk be, hogy minden n pozitív egészre van n-jegyű, 1 és 2 jegyekből álló szám, mely többszöröse 2n -nek. 1860. [5] Igazoljuk, hogy ha n 2-nél nagyobb egész szám, akkor van olyan x és y páratlan egész, melyekre 2n = 7x 2 + y 2 . 1861. [4] Az 1, 2, 3, . . . , 2n számokból kiválasztjuk az a1 , a2 , . . . , an+1 számokat. Mutassuk meg, hogy ezek között van két olyan szám, amelyek közül egyik osztója a másiknak. [5] 1862. Mutassuk meg, hogy p < 4n , ahol p az n-nél nem nagyobb prímszámok p≤n

szorzatát jelöli.

p≤n

Feladatok

164

1863. [5] Mutassuk meg, hogy minden k egész szám végtelen sokféle módon felírható k = ±12 ± 22 ± 32 ± · · · ± n2 alakban, ahol n valamely természetes szám; a ± jelek közül pedig a megfelelőt kell választani. 1864. [4] Az a0 , a1 , . . . , a100 pozitív egészekből álló sorozatra a1 > a0 , a2 = 3a1 − 2a0 , a3 = 3a2 − 2a1 , . . . , a100 = 3a99 − 2a98 . Mutassuk meg, hogy a100 ≥ 2100 . 1865. [5] a1 = 1, a2 = a1 + a100 > 14. 1866. [5] a1 = 1, a2 = a1 + a9000 > 30.

1 1 1 , a3 = a2 + , . . . , an+1 = an + . Bizonyítsuk be, hogy a1 a2 an 1 1 1 , a3 = a2 + 2 , . . . , an+1 = an + 2 . Mutassuk meg, hogy a12 a2 an+1

√ 1 2 n 1867. [5] a1 = 1, a2 = 1 + , a3 = 1 + , . . . , an+1 = 1 + . Igazoljuk, hogy n ≤ a a a 1 2 n √ ≤ an ≤ n + 1. 1868. [5] 1 < a1 < 2, an+1 = an 2 − 2an + 2, n = 1, 2, 3, . . . Mutassuk meg, hogy 1 < < an < 2, n = 2, 3, 4, . . . . 1869. [6] A pozitív a1 , a2 , a3 , . . . számokra teljesül az an2 ≤ an − an+1 , n = 1, 2, 3, . . . 1 egyenlőtlenség. Mutassuk meg, hogy an < . n

2n 22n 22n 22n [5] 1870. Mutassuk meg, hogy ≤ < √ (ill. < √ , s ez az élesebb 2n n 2n 2n + 1 egyenlőtlenség könnyebben igazolható).

2n 22n 1871. [5] Mutassuk meg, hogy < , n > 1. n+1 n

2n 22n 22n 1872. [5] Mutassuk meg, hogy √ < 1. n 4n 3n + 1 3n (3n)! , n > 1. 1873. [3] Mutassuk meg, hogy < 2(n + 2) (2n!)2 1 1 3 5 2n − 1 1 1874. [4] Mutassuk meg, hogy √ < · · · . . . · 1. 2 4 6 2n 2 n 3n + 1 √ 1 1 1 1875. [4] Igazoljuk, hogy 1 + √ + √ + · · · + √ > 2( n + 1 − 1). n 3 2 1876. [3] Melyik ! szám a nagyobb: 2 ·· · 2 n db 2-es 22 vagy



33

··

3

!

n − 1 db 3-as

?

32. Kombinatorika a geometriában

165

1877. [5] Melyik ! szám a nagyobb: 3 ·· 3· n db 3-as vagy 33 1878.

[6]



44

··

4

!

n − 1 db 4-es

?

Legyenek a1 , a2 , . . . , an tetszőleges pozitív számok, amelyek nem mind egyenlők. Bizonyítsuk be, hogy √ n

a1 a2 . . . an
4), közülük semelyik három nem esik egy egyenesbe. 1 n Bizonyítsuk be, hogy legalább olyan konvex négyszög van, amelyeknek 5 4 csúcspontjai az adott pontok közül valók. 1904. [6] Adott a síkon 9 pont, közülük semelyik három sem esik egy egyenesre. Bizonyítsuk be, hogy kiválasztható közülük öt olyan pont, amelyek konvex ötszög csúcsait alkotják. 1905. [6] Adott a síkon n pont, melyek közül semelyik három nem esik egy egyenesre. Háromszögeket jelölünk ki úgy, hogy csúcsaikat az adott pontok közül választjuk, és

semelyik kettőnek nincs közös oldala. Bizonyítsuk be, hogy legalább 1 n 1 n háromszöget kijelölhetünk így, de -nél többet nem jelölhetünk ki. 9 2 3 2 1906. [4] Könnyen látható, hogy n = 1, 2, 3 mellett n kör legfeljebb 2n számú részre bontja fel a síkot. Legfeljebb hány részre lehet felosztani a síkot 4 kör megrajzolásával? 1907. [4] Lehet-e szabályos ötszög és tízszög alakú lapokból parkettát készíteni? 1908. [4] Hány szabályos test létezik? (Most nevezzük szabályos testnek az olyan testet, amely egybevágó szabályos sokszöglapokból áll, és minden csúcsában ugyanannyi lap találkozik.) 1909. [4] Adott a síkon 1000 pont, melyek közül semelyik három nincs egy egyenesen. Egymást nem metsző szakaszokkal egymás után összekötjük ezeket a pontokat. Ezt a műveletet mindaddig folytatjuk, míg végül már nem találunk olyan két pontot, melyeket összeköthetnénk az eddigieket nem metsző szakasszal. Bizonyítsuk be, hogy az összekötő szakaszok száma nem függ a pontok összekötésének sorrendjétől. 1910. [5] Legfeljebb hány átlót lehet berajzolni egy konvex sokszögbe úgy, hogy közülük bármely kettőnek legyen közös pontja (belső vagy végpontja)?

168

Feladatok

1911. [5] Egy n oldalú sokszög szögei között legfeljebb hány hegyesszög lehet? 1912. [4] Felbontható-e a tér 2000 darab diszjunkt, egybevágó részre? 1913. [4] Mutassuk meg, hogy a síkon adott végtelen sok pont között végtelen sok különböző távolság lép fel. 1914. [5] A síkon adva van n pont úgy, hogy bármely három által meghatározott háromszög területe legfeljebb 1 egység. Mutassuk meg, hogy a pontok lefedhetők egy 4 egység területű háromszöggel. 1915. [5] Egy szabályos tízszög csúcsai közül ötöt pirosra festettünk, a többit kékre. Mutassuk meg, hogy van két olyan egybevágó háromszög, hogy az egyik csúcsai kékek, a másiké pirosak. 1916. [4] Egy 13 oldalú konvex sokszög átlói is meghatároznak konvex sokszögeket. Legfeljebb hány oldala van egy ilyen sokszögnek? 1917. [4] Adott a síkon n (n > 3) pont, feketék és pirosak; és bármely 4 pont közül a pirosak és a feketék egy egyenessel szétválaszthatók. Mutassuk meg, hogy van olyan egyenes, mely az összes piros pontot elválasztja az összes feketétől. 1918. [4] Miért nem lehet 10 egység sugarú körbe 450 pontot elhelyezni úgy, hogy bármelyik két pont távolsága legalább 1 egység legyen? 1919. [5] Egy 20 × 25-ös téglalapban elhelyeztünk 120 db egységnégyzetet. Mutassuk meg, hogy a téglalapban elhelyezhető egy egységnyi átmérőjű kör, melynek egyik négyzettel sincs közös pontja. 1920. [5] Egy R sugarú kerek asztalon n db r sugarú pénzérme fekszik úgy, hogy egyik sem fedi — még részben sem — valamely másikat; és az asztalra újabb érme már el, mert nem találunk számára helyet. Mutassuk meg, hogy nem helyezhető

√ 1 R R −1 < n < . 2 r r 1921. [4] Egy test valamennyi oldallapja háromszög, és minden csúcs megjelölhető az 1, 2, 3 szám valamelyikével úgy, hogy minden él különböző számmal jelölt két csúcsot köt össze. Bizonyítsa be, hogy ekkor a test minden oldallapja befesthető két adott szín egyikével úgy, hogy bármely két, közös él mentén csatlakozó lap különböző színű legyen. 1922. [4] Rajzoljunk egy papírra tetszés szerint köröket, amelyek különbözőképpen metszhetik egymást. Ily módon a síkot tartományokra bontottuk. Mutassuk meg, hogy kiszínezhetjük ezt a térképet két színnel úgy, hogy bármely két, közös határvonallal rendelkező tartomány különböző színű.

32. Kombinatorika a geometriában

169

1923. [5] Rajzoljunk egy papírra tetszés szerint köröket, és minden körben húzzunk meg egy-egy húrt. Ily módon a síkot tartományokra bontottuk. Mutassuk meg, hogy ha bármely két húrnak legfeljebb egy közös pontja van; akkor kiszínezhetjük ezt a térképet három színnel úgy, hogy bármely két, közös határvonallal rendelkező tartomány különböző színű. 1924. [5] Mutassuk meg, hogy a négyzetrácson nem lehet úgy kiválasztani rácspontokat, hogy azok egy szabályos n-szög (n ≥ 5 és n = 6) csúcsai legyenek. 1925. [5] Mutassuk meg, hogy a négyzetrácson nem lehet úgy kiválasztani rácspontokat, hogy azok egy szabályos a) háromszög, b) hatszög csúcsai legyenek. 1926. [4] Mutassuk meg, hogy egy konvex, középpontosan szimmetrikus zárt görbén található 4 olyan pont, amelyek egy négyzet csúcsai. 1927. [6] Mutassuk meg, hogy bármely zárt térgörbének van négy olyan pontja, amelyek egy síkban vannak. Igaz-e ez az állítás öt pontra is? 1928. [5] Mutassuk meg, hogy ha egy sík pontjai 4 színnel vannak kiszínezve, akkor van olyan egyenes a síkban, melyen előfordul 3 különböző színű pont. 1929. [6] Mutassuk meg, hogy ha a tér pontjai 5 színnel vannak kiszínezve, akkor van olyan sík, melyen előfordul 4 különböző színű pont. 1930. [3] Kovácséknál egy este sok vendég fordult meg. Nem volt olyan, aki távozott és újra visszatért. Bármely kettő találkozott egymással a lakásban. Igaz-e, hogy volt olyan időpont, amikor mindnyájan egyszerre Kovácséknál voltak? 1931. [6] [Helly tétele.] Mutassuk meg, hogy ha a sík n darab konvex halmaza közül bármely háromnak van közös pontja, akkor az összesnek is van. 1932. [4] Mutassuk meg, hogy a sík bármely n pontja között a minimális távolság 3n-nél kevesebbszer fordul elő. 1933. [5] Mutassuk meg, hogy a sík bármely n pontja között a maximális távolság legfeljebb n-szer léphet fel. 1934. [6] Mutassuk meg, hogy ha egy ponthalmazban minden távolság egész, és a pontok nincsenek mind egy egyenesen; akkor a ponthalmazban csak véges számú pont lehet. 1935. [5] [A Sylvester feladat.] Meg lehet-e adni a síkon n pontot úgy, hogy azok ne legyenek mind egy egyenesen, és közülük bármely kettőn áthaladó egyenes tartalmazzon még egy további pontot is a megadottak közül?

170

Feladatok

1936. [7] Ha a sík n pontja között nincs három, mely egy egyenesre esne, és megrajzoljuk minden ponthármas köré a rájuk illeszkedő kört, akkor legfeljebb hány egységsugarú kört rajzoltunk? Jelölje az egységsugarú körök maximális számát m. Mutassuk meg, hogy

2 n 3/2 , ahol c egy pozitív konstanst jelöl. c·n ≤m≤ 3 2 1937. [5] [Izoperimetrikus probléma.] Adott kerületű síkidomok közül melyiknek legnagyobb a területe?

33. Gráfok 1938. [4] Néhány csapat körmérkőzést játszik (mindenki mindenkivel egy mérkőzést). Minden csapatnak van győzelme, s döntetlen nem volt. Mutassuk meg, hogy van három olyan csapat: A, B és C, hogy A legyőzte B-t, B legyőzte C-t és C legyőzte A-t. 1939. [4] 8 csapat körmérkőzést játszott, döntetlen nem volt. Bizonyítsuk be, hogy van köztük négy olyan A, B, C és D csapat, hogy A legyőzte B-t, C-t, D-t; B legyőzte C-t és D-t; végül C legyőzte D-t. 1940. [4] Két csapat teniszmérkőzést vívott úgy, hogy az egyik csapat minden játékosa egy mérkőzést játszott a másik csapat minden játékosával. A végén megállapították, hogy mindenkinek volt veresége is, győzelme is. Igazoljuk, hogy akkor van négy olyan játékos, akik „körbe verték” egymást. 1941. [4] Bizonyítsuk be, hogy ha egy társaságban mindenki mindenkivel egy pingpong mérkőzést játszott, akkor van a társaságnak olyan A tagja, hogy a társaság bármely további B tagjára a következő két állítás közül legalább az egyik igaz: (a) A legyőzte B-t. (b) Van a társaságnak olyan C tagja, hogy A legyőzte C-t és C legyőzte B-t. 1942. [4] Egy labdarúgó-bajnokságon 18 csapat vesz részt. Bizonyítsuk be, hogy a nyolcadik forduló után még biztosan van három olyan csapat, melyek közül semelyik kettő nem játszott egymással. 1943. [5] Bizonyítandó, hogy egy pingpong körmérkőzés résztvevői sorbaállíthatók úgy, hogy mindenki mögött olyasvalaki álljon, akit legyőzött. 1944. [5] Egy ismerkedési est résztvevői úgy álltak sorba, hogy az első és az utolsó ember kivételével mindenkiről tudjuk, hogy az előtte álló emberek közül ugyanannyit ismer, mint a mögötte állók közül. Bizonyítsuk be, hogy a két szélsőnek ugyanannyi ismerőse van.

33. Gráfok

171

1945. [5] Lehet-e úgy ismeretségeket szervezni n személy között, hogy ne legyen 3 fő, akinek ugyanannyi ismerőse van? 1946. [4] Egy üdülő bármely 3 lakója között van kettő, akik nem ismerik egymást, de bármely hét között van legalább kettő, akik ismerik egymást. Az üdülés befejeztével mindenki megajándékozza minden ismerősét egy-egy ajándéktárggyal. Bizonyítsuk be, hogy n nyaraló esetén legfeljebb 6n tárgy kerül ajándékozásra. 1947. [4] Egy iskolában 100 diák tanul. Közülük bárhogyan választva 4-et, ezek közül legalább egy ismeri a többi hármat. Bizonyítsuk be, hogy van olyan diák, aki ismeri a többi 99 diákot. Legalább hány olyan tanuló van, aki ismeri az összes többit? 1948. [4] Egy iskola 90 végzős diákja közül mindegyiknek legalább 10 barátja van ebben a társaságban. Bizonyítsuk be, hogy bármelyik meghívhat vendégségbe három másikat úgy, hogy a négy résztvevő mindegyikének legalább két barátja is jelen legyen. 1949. [4] Egy társaságban bárkinek van ismerőse; és tudjuk, hogy ha két embernek azonos számú ismerőse van, akkor nincs közös ismerősük. Mutassuk meg, hogy van, aki csak egy embert ismer a társaságból. 1950. [4] Egy n tagú (n > 3) társaságról tudjuk, hogy ha mindannyian együtt vannak, akkor bárki is megy ki a szobából, a többieknek ugyanannyi ismerőse marad a szobában. Mutassuk meg, hogy a társaságban vagy senki sem ismer senkit, vagy mindenki ismer mindenkit. 1951. [4] Egy baráti összejövetelen, ahol legalább 3 házaspár volt jelen; bármely 3 házaspárból vagy az asszonyok, vagy a férjek ismerik egymást. Elhelyezhetők-e a párok két teremben úgy, hogy az első teremben bármely két asszony, a másik teremben bármely két férj ismerje egymást? 1952. [4] Egy rendezvényen 2n személy vesz részt. Mindenki legalább n másikat ismer, s az ismeretségek kölcsönösek. Mutassuk meg, hogy kiválasztható közülük 4 személy, akik úgy körbeállíthatók, hogy mindenki ismerje a szomszédjait. 1953. [4] Egy 9 tagú társaságban bármely 3 ember között van két olyan, akik ismerik egymást. Bizonyítsuk be, hogy ekkor van közöttük 5 olyan, hogy mindegyiküknek legalább 4 ismerőse van. 1954. [5] Egy 9 tagú társaságban bármely 3 ember között van két olyan, akik ismerik egymást. Bizonyítsuk be, hogy ekkor van közöttük 4 olyan, akik páronként ismerik egymást. 1955. [5] Egy 11 fős társaságból bárhogyan is választunk ki kettőt, a többiek közül pontosan egy ismeri mindkettőjüket. Mutassuk meg, hogy van köztük olyan, aki mindenki mást ismer.

172

Feladatok

1956. [6] Egy társaságban mindenki legalább k másikat ismer. Mutassuk meg, hogy a társaság tagjai közül legalább (k +1)-et le tudunk ültetni egy kör alakú asztalhoz úgy, hogy mindenki ismerje a szomszédait. 1957. [4] Egy ország minden városát vagy hajó-, vagy repülőút köti össze. Bizonyítsuk be, hogy vagy hajóval, vagy repülővel bejárható az egész ország. 1958. [5] Legfeljebb hány kézfogásra kerülhetett sor egy 20 tagú társaságban, ha tudjuk, hogy akárhogyan is választunk ki a társaságból három embert, azok között biztosan akad kettő, akik nem fogtak kezet? 1959. [4] Barna úrhoz és feleségéhez három házaspár érkezett vendégségbe. Az örömteli találkozás alkalmából egyesek kezet is fogtak egymással. (Senki sem fogott kezet a saját feleségével.) Később Barna úr megkérdezte mindenkitől, hogy hány jelenlévővel fogott kezet. Kérdésére, amelyet hétszer tett fel, hiszen rajta kívül heten voltak a szobában, csupa különböző választ kapott. Hány vendéggel fogott kezet Barnáné? 1960. [4] A Kovács házaspárhoz négy házaspár érkezett vendégségbe. A bemutatkozások után Kovács házigazda megállapította, hogy rajta kívül a többi kilenc jelenlevő mindegyike más-más számú személlyel ismerkedett össze. Hány ember mutatkozott be Kovácsnénak?

34. Halmazrendszerek 1961. [3] Mutassuk meg, hogy egy n elemű halmaz részhalmazainak száma 2n . 1962. [4] A1 , A2 , . . . , Am ⊂ H, |H| = n, és Ai ∩ Aj = ∅. Mutassuk meg, hogy m ≤ 2n−1 . (|H| jelöli a H halmaz elemeinek számát.) 1963. [5] Legyenek A1 , A2 , . . . , Am egy n-elemű X halmaz részhalmazai. Tegyük fel, hogy közülük bármely kettőnek van közös eleme, és m < 2n−1 . Bizonyítsuk be, hogy ekkor X-nek létezik olyan részhalmaza, amely nem azonos egyik Ai -vel sem, de bármelyikkel van közös eleme. 1964. [7] S = {1, 2, . . . , n}; A1 , A2 , . . . , Am ⊂ S, és Ai ∩ Aj tartalmaz két szomszédos elemet minden i, j indexpárra. Mutassuk meg, hogy m ≤ 2n−2 . 1965. [7] A1 , A2 , . . . , Am ⊂ H, |H| = n, és Ai ∩ Aj = ∅, valamint Ai ∪ Aj = H. Mutassuk meg, hogy m ≤ 2n−2 . 1966. [4] A1 , A2 , . . . , Am ⊂ H, |H| = n, és Ai ∪ Aj = H, ha i = j . Mutassuk meg, hogy m ≤ n. 1967. [4] A1 , A2 , . . . , Am ⊂ H, |H| = n, és egyik Ai -t sem tartalmazza a többi részhalmaz uniója. Mutassuk meg, hogy m ≤ n.

34. Halmazrendszerek

173

1968. [5] Legfeljebb hány, nem üres részhalmaz választható ki egy 100 elemű halmazból úgy, hogy bármely két kiválasztott részhalmaz vagy diszjunkt legyen, vagy az egyik tartalmazza a másikat? 1969. [6] [Ray-Chaudhuri–Wilson tétel, 1975] A1 , A2 , . . . , Am ⊂ H, |H| = n, és k+1

n |Ai ∩ Aj | ≤ k, ha i = j . Mutassuk meg, hogy m ≤ . i i=0 1970. [2] A1 , A2 , . . . , Ak halmazok végesek. Mutassuk meg, hogy |A1 ∩ A2 ∩ . . . ∩ Ak | ≤

1 (|A1 | + |A2 | + · · · + |Ak |). k

1971. [4] A1 , A2 , . . . , Ak halmazok olyanok, hogy bármely kettő metszete egy elemű, s bármely három halmaz közös része üres. k(k − 1) Mutassuk meg, hogy |A1 ∪ A2 ∪ . . . ∪ Ak | ≥ . 2 1972. [5] A1 , A2 , . . . , Am ⊂ H, |H| = n és ezekből a halmazokból bármely kettő metszete nem üres, s bármely három halmaz metszete Jelölje#m lehetséges " üres. √ 1 + 8n + 1 legnagyobb értékét M. Mutassuk meg, hogy M = . 2 1973. [4] Mutassuk meg, hogy egy 9 elemű halmaz bármely négy 7 elemű részhalmazának közös része nem üres. 1974. [4] A1 , A2 , . . . , An+1 ⊂ H, |H| = n. Mutassuk meg, hogy kiválasztható közülük néhány úgy, hogy ezek uniója megegyezik a megmaradt halmazokból néhánynak az uniójával. 1975. [5] A1 , A2 , . . . , An+1 ⊂ H, |H| = n (n ≥ 5), |Ai | = 3, i = 1, 2, . . . , n. Mutassuk meg, hogy kiválasztható közülük két halmaz, melyeknek pontosan egy közös eleme van. 1976. [5] Adott 1978 halmaz, melyek mindegyike 40 elemű, és bármely két halmaznak van közös eleme. Mutassuk meg, hogy van olyan elem, amely mindegyik halmazban benne van. 1977. [5] Adott 1985 halmaz, melyek mindegyike 45 elemű, és bármely kettő uniója 89 elemű. Hány elemből állhat mind az 1985 halmaz uniója? 1978. [6] Legyenek H1 , H2 , . . . , H50 a |H| véges halmaz olyan részhalmazai, amelyekre 1 |Hi | > |H|, minden 1 ≤ i ≤ 50-re. Bizonyítsuk be, hogy létezik a H-nak 2 öt olyan h1 , h2 , . . . , h5 eleme, hogy valamennyi Hi részhalmaz tartalmaz ezek közül legalább egyet.

174

Feladatok

1979. [6] Legyenek H1 , H2 , . . . , H1066 a |H| véges halmaz olyan részhalmazai, amelyek1 re |Hi | > |H|, minden 1 ≤ i ≤ 1066-ra. Bizonyítsuk be, hogy létezik a H-nak 2 tíz olyan h1 , h2 , . . . , h10 eleme, hogy valamennyi Hi részhalmaz tartalmaz ezek közül legalább egyet. 1980. [5] Egy bizottság 40-szer ülésezett. Mindegyik ülésen 10 fő volt jelen. A bizottság bármely két tagja legfeljebb egy ülésen vett együtt részt. Bizonyítsuk be, hogy a bizottság legalább 64 tagból állt. 1981. [5] Egy osztályban bármely két gyerek jár közös szakkörbe, de mindenki legfeljebb két szakkörnek a tagja. Bizonyítsuk be, hogy van olyan szakkör, ahová az osztálynak legalább a kétharmad része jár. 1982. [5] Egy nemzetközi konferencián 9 tudós vesz részt. Egyikük sem beszél háromnál több nyelvet; és közülük bármely három között van kettő, akik beszélnek közös nyelvet. Bizonyítsuk be, hogy van olyan nyelv, amelyen legalább hárman beszélnek. 1983. [5] Egy bank páncélszekrényén több különböző zár található. Kulcsaikat úgy osztották szét a bank négy pénztárosa között, hogy a páncélszekrény kinyitásához legalább hármójuknak jelen kell lenni (de mind a négynek nem szükséges), hogy a náluk levő kulcsokkal ki lehessen nyitni az összes zárat. (Egy zárhoz többüknél is lehet kulcs, és egy embernél többféle kulcs is lehet.) Legkevesebb hány zár van a páncélszekrényen? 1984. [5] Egy nemzetközi bizottság 5 tagból, 5 állam 1–1 képviselőjéből áll. Azokat az iratokat, amelyeken a bizottság dolgozik, páncélszekrényben őrzik. Hány zárjának kell lennie a szekrénynek, és hány kulccsal kell ellátni a bizottság minden egyes tagját, hogy az iratokhoz csak akkor lehessen hozzáférni, ha a bizottságnak legalább három, tetszés szerinti tagja együtt van? 1985. [5] Tíz rabló egy többzáras ládában őrzi a kincsét. Minden rablónak bizonyos zárakhoz van kulcsa. A kulcsok úgy vannak elosztva, hogy semelyik három rabló se tudja a nála levő kulcsokkal kinyitni a ládát, de bármely négy közülük már hozzá tud férni a kincshez. Legalább hány zár szükséges a fenti feltételek teljesüléséhez? 1986. [5] A H véges halmaznak kijelöltük néhány valódi részhalmazát. Tudjuk, hogy a) a halmaz bármely két különböző eleme pontosan egy kijelölt részhalmazban van benne; b) bármely kijelölt R részhalmazhoz és abban nem lévő p elemhez pontosan m darab, p-t tartalmazó és R-t metsző kijelölt részhalmaz, és pontosan n darab, p-t tartalmazó és R-t nem metsző kijelölt részhalmaz van. Fejezzük ki m és n segítségével H elemeinek és kijelölt részhalmazainak számát.

34. Halmazrendszerek

175

1987. [5] Igazoljuk, hogy elég sok (egy bizonyos számúnál több) különböző halmaz közül mindig kiválasztható 2000 db halmaz úgy, hogy semelyikük se álljon elő másik kettő egyesítéseként. 1988. [5] Mutassuk meg, hogy egy n-elemű halmaz részhalmazaiból készíthető párok 3n + 1 közül olyan van, melyben a két részhalmaz metszi egymást. 2 1989. [5] Mutassuk meg, hogy ha az n-elemű H halmaz összes részhalmazát tekintjük, akkor |Ai ∩ Aj | = n · 4n−1 . (Az összegzésben i = j is megengedett.) i,j

1990.

[5]

Az A halmaz minden X részhalmazához hozzárendeljük az f (X) részhalmazt úgy, hogy ha X ⊆ Y , akkor f (X) ⊆ f (Y ). Mutassuk meg, hogy van A-nak olyan T részhalmaza, amelyre f (T ) = T .

1991. [5] Jelölje Tn egy n elemű halmaz különböző partícióinak (részhalmazokra bontásainak) számát. Bizonyítsuk be, hogy Tn ≤ n!. 1992. [5] Egy adott n természetes számra legyen Mn = {1, 2, . . . , n}. Az Mn valamely T részhalmaza „kövér”, ha a T egyetlen eleme sem kisebb, mint T elemeinek száma, és van olyan eleme, amely megegyezik a T halmaz elemeinek számával. Az Mn „kövér” részhalmazainak számát jelölje f (n). f (11) =? 1993. [7] [Sperner-tétel, 1928] A1 , A2 , . . . , Am ⊂ H, |H| = n és ezekből a halmazokból egyik sem

tartalmazza részként valamely másikat. Mutassuk meg, hogy m ≤ n ≤ n . [2] 1994. [7] [Erdős–Ko–Radó tétel, 1961] Ha A1 , A2 , A3 , . . . , Am egy n elemű H halmaz olyan k-elemű részhalmazai (k < n/2), hogy bármely kettőnek van közös ele

n−1 me, akkor m ≤ . k−1 1995. [7] Ha A1 , A2 , . . . , Am ⊂ H; B1 , B2 , . . . , Bm ⊂ H, |H| = n, |Ai | = k, |Bi | = l, i = 1, 2, . . . , m, k

+ l ≤ n, és Ai ∩ Bj = ∅ pontosan abban az esetben, ha i = j ; k+l akkor m ≤ . l 1996. [5] Bizonyítsuk be, hogy az első 10 000 természetes szám halmazának kijelölhető 16 részhalmaza úgy, hogy a 10 000 egész szám bármelyike előáll alkalmas kijelölt 8 részhalmaz metszeteként. 1997. [7] Nevezzük független metszőrendszernek az n-elemű H halmaz A1 , A2 , . . . , Am részhalmazait, ha (1) ∀x ∈ H előáll néhány Ai metszeteként; (2) Az A1 , A2 , . . . , Am rendszerből bármely Ai -t elhagyva (1) nem teljesül.

Feladatok

176

Mutassuk meg, hogy A1 , A2 , . . . , Am pontosan akkor független metszőrendszer, ha A1 , A2 , . . . , Am is független metszőrendszer. 1998. [7] Mutassuk meg, hogy ha A1 , A2 , . . . , Am az n-elemű H halmazon független metszőrendszer, akkor c1 · log2 n ≤ m ≤ c2 · n2 ; és ezek a korlátok nagyságrendjükben pontosak. (c1 és c2 pozitív konstansokat jelölnek.) 1999. [7] A1 , A2 , . . . , Am ⊂ H, |H| = n, |Ai | = 3; i = 1, 2, 3, . . . , m és |Ai ∩ Aj | ≤ 1, ha i = j . Mutassuk meg, hogy van olyan M ⊂ H, hogy minden i-re Ai ⊂ M √ és |M| ≥ [ 2n]. 2000. [7] Legyenek n és k adott természetes számok. Legyen A véges halmaz. Tegyük n(n + 1) fel, hogy |A| ≤ . Legyenek Ai (i = 1, 2, . . . , n + 1) olyan halmazok, k+1 n+1

melyekre |Ai | = n (1 ≤ i ≤ n + 1), |Ai ∩ Aj | ≤ k (i = j ), A = ∪ Ai . i=1

Határozzuk meg A elemeinek számát.

Megoldások, útmutatások

1. Fejtörők 1. A busz eleje balra néz, mert az ajtók a túlsó oldalon vannak. (Ez a hazai viszonyokra értendő, pl. Angliában ez másképp van.) 2.

3. A csúcsokat tükrözzük a háromszög középpontjára. 4.

5. A vadász az Északi–sarkon indult el a nyomok után (esetleg a Déli–sark közelében), mert máshonnan indulva a megadott útvonalon nem érkezhetne vissza a kiindulási helyre. Ha az Északi–sarkon lakik a medve, akkor az fehér színű. 6. A négy hajó mindegyike nem lehet egy síkban (például a tenger felszínén). A négy hajó egy szabályos tetraéder csúcsaiban helyezkedik el, így a negyedik hajó tengeralattjáró, esetleg léghajó.

Megoldások, útmutatások

178

7. 323 935 = 5 · 17 · 37 · 103. A kapitány 37 éves és a kisfia 5 éves (esetleg 1 éves). 8.

9. 101 − 102 = 1 10. 1975 + 146 = 210 + 1911. A feladat igazi, „szép” megoldása: 1995 + 146 = 210 + 1117. 11. 72 = 49. (A 6-ost megfordítva, 9-esként helyeztük el.) 12. A 9-es és a 8-as kártyákat cseréltük meg, és a 9-es kártyát megfordítva, 6-osként tettük le, így az összeg mindkét oszlopban 18. 13. (Deriválás.)

14. Mindegyik oszlopban az első két számból a nagyobbat írtuk a harmadik helyre.

12 15 30 17 26 22 30 24 20 25 20 13 28 18 24 20 30 20 26 22 30 15. Az ábrán az 1, 2, 3, 4, 5 számokat látjuk „visszatükrözött” alakjukkal együtt, így a sorozat következő eleme:

16. a) 23 (a prímszámok sorozata). b) 5040 (1, 1 · 2 = 2, 1 · 2 · 3 = 6, 1 · 2 · 3 · 4 = 24, . . . ) c) 89 (ez a Fibonacci–sorozat, a sorozat mindegyik eleme az őt megelőző két szám összege).

1. Fejtörők

179

d) 127 (a sorozat következő elemét úgy kapjuk, hogy az utolsó számhoz hozzáadjuk a szám jegyeinek összegét). e) 49 (a négyzetszámok sorozata). f) 8 (7 · 7 = 49, 4 · 9 = 36, 3 · 6 = 18, 1 · 8 = 8). 17. A 8-at írtuk fel 2-es, 3-as, 4-es, 5-ös és 6-os számrendszerben (1000, 22, 20, 13, 12), így a sorozat következő eleme a 8 hetes számrendszerben felírt alakja: 11. 18. A 111, 213, 141, 516, 171, 819, 202, 122, . . . sorozat valójában a 11, 12, 13, 14, 15, 16, 17, 18, 19, 20, 21, 22, . . . sorozat, csak a számjegyek tagolása, csoportosítása másképp történt. 19. A sorozat első eleme 1db 1-es, ezért a második elem: 11; itt 2 db 1-est írtunk le, így a következő elem: 21; és így tovább. A felírt sorozat soron következő eleme: 1113213211. 20. 42 = 16 → 4 16 → 461, 52 = 25 → 5 25 → 552, 62 = 36 → 6 36 → 663, 72 = 49 → 7 49 → 794, 82 = 64 → 8 64 → 846. A következő elem: 92 = 81 → 9 81 → 918. 21. A 0, 1, 2, 3, 4, 5, 6, . . . sorozat elemeinek első betűit felsorolva kapjuk a megadott sorozatot. 22. A sakktábla alapsorában álló bábuk (bástya, huszár, futó, vezér, király, . . . ) nevének első betűit adtuk meg. A hiányzó betűk: F , H , B. 23. A válasz: mennyország. (A szavakat egészítsük ki sorszámmal: elsősegély, második otthon, harmadik világ, negyedik birodalom, ötödik hadoszlop, hatodik érzék, hetedik mennyország.) c 24. A táblázatban bárhol választunk négy egymás melletti a

b számot, igaz d

rájuk az ab = cd + 1 összefüggés. 25. 79479479. 26. 41312432. 27. 12132003, 30023121, 23121300 vagy 31213200.

Megoldások, útmutatások

180

28.

29.

7

15

2

1

2

3

3

8

13

5

4

7

14

1

9

8

6

9

30.

1

5

4

6

2

9

7

8

3

31. 20

20 9 6 4

32.

3 2

8

11 5

8 1

7

4

20

3 1

9

12 5

9 2

7

2

11 7

4 3

1

6

1. Fejtörők

181

33.

1

2

2

1

1

{1

1

{1

{1 {2

{2

{1

3 74 + 985 1062 627 + 354 981

2 64 + 987 1053 219 + 438 657

2 46 + 987 1035 341 + 586 927

4 35 + 987 1026 154 + 782 936

3 45 + 978 1026 317 + 628 945

216 + 738 954

215 + 748 963

318 + 654 972

235 + 746 981

384 + 192 576

482 + 157 639

182 + 367 549

391 + 435 826

269 + 514 783

192 + 546 738

162 + 783 945

654 + 129 783

546 + 381 927

439 168 571 295 129 + 128 + 327 + 293 + 173 + 357 567 495 864 468 486 36. 495 + 459 954 4950 37. 6174 + 4590 + 1467 7641 9540 38. 123456789 + 864197532 = 987654321.

348 + 219 567

735 + 186 921

234 + 657 891

34.

35.

39. A törtkifejezésben 10 különböző betű van, tehát mind a 10 számjegyet használnunk kell, így a 0-t is. A 0 nem kerülhet a nevezőbe, csak a számlálóban állhat. Ezért a számlálóban levő szorzat értéke 0, emiatt a tört értéke is 0 lesz. 40. a)

9567 + 1085 10652

b)

9486 9376 + 1076 és + 1086 10562 10462

c)

6054 7894 1720 és 1038 + 9734 + 2054 17508 10986

Megoldások, útmutatások

182

d)

932 + 9338 10270

e) 26811 + 4708 31519

f)

29786 850 + 850 31486

h) 79 · 19 627 i) 103 · 273 627 79 206 746 711 721 + 746 1501 309 2726 28119 19 · 53 b) 99 · 91 c) 12 · 79 41. a) 97 · 11 és 97 95 891 84 97 57 99 108 1067 1007 9009 948 42. a) 775 · 33 b) 325 · 777 2325 2275 2325 2275 25575 2275 252525 b) 339 · 268 c) 285 · 39 43. a) 286 · 379 858 678 855 2002 2034 2565 11115 2574 2712 108394 90852 44. (2 : 3) : (4 : 5 : 6) = 5. g)

1 6 : = 7. 2 · 3 · 4 · 5 7 · 8 · 9 · 10 46. 1 + 2 + 3 − 4 + 5 + 6 + 78 + 9 = 100, 1 + 2 + 34 − 5 + 67 − 8 + 9 = 100, 1 + 23 − 4 + 56 + 7 + 8 + 9 = 100, 1 + 23 − 4 + 5 + 6 + 78 − 9 = 100, 12 + 3 + 4 + 5 − 6 − 7 + 89 = 100, 12 + 3 − 4 + 5 + 67 + 8 + 9 = 100, 123 + 4 − 5 + 67 − 89 = 100, 123 − 4 − 5 − 6 − 7 + 8 − 9 = 100, 123 + 45 − 67 + 8 − 9 = 100, 123 − 45 − 67 + 89 = 100.

45. 1 : 2 : 3 : 4 : 5 : (6 : 7 : 8 : 9 : 10) =

47. 98 − 76 + 54 + 3 + 21 = 100, 98 + 7 − 6 + 5 − 4 + 3 − 2 − 1 = 100, 98 − 7 + 6 − 5 + 4 + 3 + 2 − 1 = 100, 98 − 7 − 6 − 5 − 4 + 3 + 21 = 100, 9 + 8 + 76 + 5 + 4 − 3 + 2 − 1 = 100, 9 − 8 + 76 + 54 − 32 + 1 = 100. 48. 9 + 8 + 7 + 65 + 4 + 3 + 2 + 1 = 99, 9 + 8 + 7 + 6 + 5 + 43 + 21 = 99. 49. {1, 14, 15}, {2, 10, 12}, {3, 6, 9}, {4, 7, 11}, {5, 8, 13}. 50. {1, 4, 10, 13}, {2, 3, 11, 12}, {5, 6, 7, 8, 9}.      51. 10, 11, 12, . . . , 18, 19 40, 41, 42, . . . , 48, 49 és 20, 21, 22, . . . , 39 .

1. Fejtörők

183

52. 51, 52, 53, . . . , 100. 53. 51, 52, 53, . . . , 100 vagy 1, 3, 5, 7, . . . , 99. 54. 123121321. 55. A legnagyobb hányadost a 100, 200, . . . , 900 számok adják, a legkisebb hányadost pedig a 199 szám adja. 56. 599 . . . 999 (221 db 9-es). 57. 558. 58. 9831. 59. 720 = 6 · 5 · 4 · 3 · 2 · 1 = 10 · 9 · 8. 60. 1, 7, 9, 10, 8, 11, 2, 5, 3, 4, 6, 12. 61. 4163 vagy 6314. 62.

vagy

63. A négyszög egy oldalán legfeljebb két metszéspont lehet, ezért a metszéspontok száma legfeljebb 8.

vagy

Megoldások, útmutatások

184

64.

vagy

65.

66.

67.

68. Egy 200-jegyű szám számjegyösszege legfeljebb 200 · 9 = 1800 lehet. Vidor tehát egy 1800-nál nem nagyobb számot tudott meg Tudortól, s Vidor ennek a számnak a számjegyösszegét súgta meg Morgónak. Az 1800-nál nem nagyobb

1. Fejtörők

185

számok közül a 999-nek van a legnagyobb számjegyösszege. Tehát Morgó 27nél nem nagyobb számot tudott meg Vidortól, ám ezek közül csak a 19-nek a számjegyösszege kétjegyű. Így Vidor által Morgónak súgott szám a 19, Morgó 1+9 = 10-et súgott Hapcinak, Hapci pedig 1 + 0 = 1-et súgott Kukának. 69. Az öntögetés befejeztével mindegyik pohárban 1-1 dl folyadék lesz. Ezért amenynyi bor hiányzik a boros pohárban az 1 dl-ből, annak a helyén víz van, tehát a boros pohárból hiányzó bor mennyisége egyenlő a vizes pohárból hiányzó víz mennyiségével. Ez azt jelenti, hogy ugyanannyi víz van a boros pohárban, mint amennyi bor van a vizes pohárban. 70. Nem. Készítsük el a családfát. 71. Okoskodjunk „visszafelé”. A harmadik átmenetel előtt 12 krajcárja volt. Előzőleg fizetett 24 krajcárt, tehát a második átmenetel után 36, előtte 18 krajcárja volt. Ezért az első átmenetel után 18 + 24 = 42 krajcárja volt, s kezdetben 21 krajcár volt a zsebében. 72. Okoskodjunk „visszafelé”. A játékosoknak az ötödik játszma után (128, 128, 128, 128, 128) Ft-ja volt, a negyedik játszma után (64, 64, 64, 64, 384) Ft-ja, a harmadik után (32, 32, 32, 352, 192) Ft-ja, a második után (16, 16, 336, 176, 96) Ft-ja, az első után (8, 328, 168, 88, 48) Ft-ja volt, így eredetileg (324, 164, 84, 44, 24) Ft-juk volt. 73. Elérhető, hogy 9 szék legyen foglalt (10 nyilván nem lehet). Bemegy az első, leül az első székre. Bemegy a második, leül a harmadik székre. Bejön a következő, leül a második székre, és kimegy a harmadik széken ülő személy. Valaki ismét bemegy, leül a negyedik székre. Jön a következő, leül a harmadik székre, és kimegy az, aki a negyedik széken ül. Ekkor már az első három szék mindegyikén ülnek. Az eljárást így folytatjuk. 74. Válasszuk a P F feliratú ládát. Ha például pirosat húzunk, akkor az a láda P P (P F nem lehet, hisz hamis a felirat, és F F nyilván szóba sem jöhet). Ekkor a P P ládikóban P F nem lehet, mert akkor az F F ládában csak F F lehetne, de ekkor ez a felirat nem lenne hamis. Tehát a P P ládikó valójában F F , az F F láda pedig P F . Hasonlóan tárgyalható az az eset, amikor a P F feliratú ládából fehér színű golyót húzunk. 75. Az állítások mindegyike mást-mást állít, ezért nem lehet közöttük két igaz állítás: vagy egy állítás igaz, vagy egy sem. Ha nincs igaz állítás, akkor 5 állítás lenne hamis, de ekkor — ellentétben feltevésünkkel —, az ötödik állítás igaz lenne. Egy lehetőség marad: az állítások között 1 igaz van, azaz 4 állítás hamis, s valóban, a negyedik állítás igaz. 76. Az első két állítás hamis, a következő három igaz.

Megoldások, útmutatások

186

77. Az A-n és a C-n levő állítások ellentétesek, így egyikük igaz. Ezért a B-n levő állítás csak hamis lehet. B-ben van az arany. 78. Ha az arany a C ládikóban van, akkor mindhárom állítás igaz; ha a B ládikóban lenne az arany, akkor mindhárom állítás hamis lenne. Az A ládikóban van az arany, s ekkor az első két állítás igaz, a harmadik hamis. 79. Csak az A és az 1 feliratú kártyákat kell megfordítani. 80. a) A igaz, B hazug; b) A hazug, B hazug; c) A hazug, B igaz, C hazug; d) C igaz. 81. A kérdés: „Te melyik utat szoktad Mekkába vezető útként mutatni?”. Az igaz erre a kérdésre a helyes utat mutatja, és a hazug (mivel a rossz utat szokta mutatni, és ezt ebben a válaszban hamisan kell mondania) is a helyes utat mutatja. (Kérdezheti azt is, hogy: „A társad melyik utat mutatná Mekkába vezető útként?” Erre mindegyik a rossz utat mutatja.) 82. Egy jó kérdés: „Ebben a városban lakik Ön?”. Ha a hazugok városában van és igazmondót kérdez, az Nem-mel válaszol, és Nem választ kap akkor is, ha hazugot kérdez. Az igazmondók városában erre a kérdésre Igen választ kap. Tehát ha a kérdésre Igen-nel válaszolnak, akkor az igazmondók városában, ha Nem-mel válaszolnak, akkor a hazugok városában van a turista. 83. B igazat mond, A és C hazudik. 84. Tegyük fel az elején, hogy az (A) állítás igaz (később kiderül, hogy ez a feltételezés hamis). Következésképpen a (B) állítás is igaz volna, tehát legalább két állítás: (A) és (B) igaz volna. Ez ellenkezik a (B) állításával, hogy az (A), (B), (C), (D), (E) állítások közül legfeljebb egy igaz. Ellentmondásba ütköztünk, ezért (A) nem lehet igaz, csak hamis. Ha (A)-t hamisnak fogadjuk el, akkor (B)-t is annak kell elfogadnunk. Következésképpen pedig (C)-nek is hamisnak kell lennie. A (B) állítás hamis voltából következik, hogy az állítások közül legalább kettő igaz. Mivel (A), (B), (C) hamisak, ezért (D), (E) igaz.

2. Páros vagy páratlan? 85. Nem lehet. Öt páratlan szám összege páratlan, míg a 100 páros. 86. Nem lehet. Ha a szorzatuk páratlan, akkor mind a négy szám páratlan, s ekkor összegük páros. 87. Ha a három szám összege 1994, akkor van köztük páros szám, ezért szorzatuk páros lesz, így a szorzat nem végződhet 1-re.

3. Párbaállítás

187

88. Nem lehet, mert az 1, 2, 3, . . . , 1994 számok összege páratlan, és ez az összeg nem állítható elő két páratlan szám összegeként. 89. 3, 3, 1, 1, 1 vagy 9, 1, 1, −1, −1. 90. Lehet, pl. 9, 1, 1, 1, 1, −1, −1, −1, −1. 91. Nem. Ha a tíz szám szorzata 10, akkor közülük az egyik páros, s a többi páratlan. Azonban kilenc páratlan és egy páros szám összege páratlan. 92. Lehet, pl. 4, 2, 1, 1, −1, −1, 1, 1. 93. Nem, mert ha a szorzat 9, akkor mind a kilenc szám páratlan, így összegük páratlan. 94. Nem lehet, hasonló okok miatt, mint a 91. feladatban. 95. Lehet, pl. 4, 2, −1, −1, −1, −1, −1, −1. 96. Nem lehet. Tekintsük ugyanis az (a1 − 1), (a3 − 3), (a5 − 5), (a7 − 7) és (a9 − 9) számokat! E számok egyike páros szám. 98. Legyen a vízszintes irányú vágások száma a, a függőleges irányú vágásoké pedig b. a + b = 13, így a és b közül az egyik páratlan. Ha pl. a páratlan, akkor a vízszintes csíkok száma a +1, tehát páros, s a részek párosságát már a függőleges vágások nem változtatják meg. 99. Nem lehet, mert a hat páratlan szám összege mindig páros. 100. Nem lehet, mert az összeg mindig páros. 101. Ha a törteket közös nevezőre hozzuk, a számláló páros (100 db páratlan szám összege), a nevező páratlan (100 db páratlan szám szorzata), így a tört értéke nem lehet 1. 100 különböző egész szám reciprokának összege már lehet 1. Ez megmutatható pl. teljes indukcióval, vagy felhasználhatjuk az 1040. feladat eredményét. 103. Az egyik tényező páros.

3. Párbaállítás 104. a) 1 + 99 = 3 + 97 = · · · = 49 + 51, a végeredmény 2500. 5151 b) = 101. 51

Megoldások, útmutatások

188

105. a) 1 + (2 − 3 − 4 + 5) + (6 − 7 − 8 + 9) + · · · + (1990 − 1991 − 1992 + 1993) = = 1 + 0 + 0 + · · · + 0 = 1. b) Felhasználva az (n + 1)2 + (n + 4)2 + (n + 6)2 + (n + 7)2 = (n + 2)2 + (n + 3)2 + (n + 5)2 + (n + 8)2 azonosságot, az a) feladathoz hasonlóan megmutatható, hogy itt is 1 az összeg legkisebb pozitív értéke. 106. Az (a, 999 − a) párokban a két szám jegyeinek összege 27. A végeredmény: 499 · 27 + 27 + 1 = 13 501. 108. Ugyanannyi. A 4 és 5 lyukú lyukasztásokat párba lehet állítani. 109. A térrészekhez rendre hozzárendelhetjük a kocka egy-egy lapját, élét, csúcsát, s még ott maradt a kocka belseje. A térrészek száma: 6 + 12 + 8 + 1 = 27. 110. Hasonló az előzőhöz. 111. Számoljuk meg a lapokhoz illeszkedő részeket! Minden laphoz a tetraéder 6 darabkája csatlakozik, minden darabka csak egy lapra illeszkedik, ezért a darabolással kapott részek száma 4 · 6 = 24. 112. A mérkőzések száma 1991, ugyanis minden mérkőzéshez tartozik egy kieső játékos, s 1991 kieső játékos van. 113. A vágások száma mindig 59, ugyanis minden vágás 1-gyel növeli a részek számát, s a darabolás befejeztével 60 db kis négyzetünk van. 114. Állítsuk párba a szám osztóit: osztó–társosztó. Csak négyzetszámnál van, hogy pontosan egy osztónak önmaga a társa. (Az n szám a osztójához b a társosztó, ha a · b = n.) Megjegyzés. Bebizonyítható, hogy ha n prímtényezős alakja n = p1α1 p2α2 . . . pkαk , akkor az n szám pozitív osztóinak száma d(n) = (α1 + 1)(α2 + 1) . . . (αk + 1). 115. Használjuk az előző feladat állítását. 116. Az a sorszámú ajtó marad nyitva, mely számnak páratlan számú osztója van. A 114. feladat szerint ezek a négyzetszámok. 117. A végeredmény: n50 . Párosítsuk az osztókat a társosztókkal. 118. 24 · 32 · 53 = 18 000. 119. Figyelembe véve, hogy az n tökéletes szám pozitív osztóinak összege 2n, a keresett összeget n-nel szorozva, 2n-et kapunk. Tehát a kérdéses összeg értéke 2. 120. Ha a nevezőt n-nel szorozzuk, megkapjuk az osztók összegét (azaz a számlálót). 121–122. Indirekt úton bizonyítsunk, s használjuk az osztó–társosztó párosítást! 123. Párosítsuk az osztókat és a társosztókat!

3. Párbaállítás

189

124. Ha d páratlan osztója egy páros számnak, akkor 2d egy páros osztó. Tehát a páros osztók összege legalább kétszerese a páratlan osztók összegének. 125–126. Használjuk fel, hogy ha (n, k) = 1, akkor (n, n − k) = 1. 127. Vegyük figyelembe, hogy ha (m, n) = 1, akkor (n − m, n) = 1. Ennek alapján elég megmutatni, hogy azoknak az m számoknak a száma, amelyekre m < 2k és (m, 4k) = 1, páros. Ehhez pedig mutassuk meg, hogy (m, 4k) = 1 akkor és csak akkor, ha (m, 2k) = 1! Ám az olyan m < 2k számok száma, amelyekre (m, 2k) = 1 — könnyen látható — mindig páros. 128. A lehetőségek száma ugyanannyi, mint ahányféle módon választhatunk öt számjegyet ismétlődéssel az 1, 2, 3, . . . , 9 számjegyek közül.   n 129. , ugyanis a metszéspontok és 4–4 csúcs között kölcsönösen egyértelmű meg4 feleltetés létesíthető. (A metszéspontok egybe is eshetnek.)   n 130. , ugyanis a metszéspontok és 6–6 csúcs között kölcsönösen egyértelmű meg6 feleltetés létesíthető. (A háromszögek ponttá is zsugorodhatnak.) 132. Két vízszintes és két függőleges rácsegyenes kölcsönösen egyértelműen meghatároz egy téglalapot.     7 11 A rácsegyenesek ilyen kiválasztásainak száma: · . 2 2 133. 216. 134. a) Nem lehet, mert a 6. lépésben páros számon állunk. b) Lépéseink során 4-et kell jobbra, 3-at balra lépni,  ezek sorrendje tetszőleges. 7 A 7 lépésből azokat, melyeket jobbra lépünk, -féleképp tehetjük meg. 4 135. A bástyának, míg céljához nem ér, 14 lépést kell megtennie; ebből 7 előre,  7 14 fölfelé visz, ezek váltakozása tetszőleges lehet. A lehetőségek száma: . 7 136. a + b | a 3 + b3 , így pl. 1991 | 13 + 19903 . 137. A végeredmény: 1, ugyanis sin 0◦ = cos 90◦ , sin 1◦ = cos 89◦ , sin 2◦ = cos 88◦ stb. 138. sin2 1◦ + sin2 2◦ + sin2 3◦ + · · · + sin2 89◦ + sin2 90◦ = 45, 5. Használjuk fel, hogy sin α = cos(90◦ − α) és sin2 x + cos2 x = 1. Így sin2 1◦ + sin2 89◦ = sin2 1◦ + cos2 1◦ = 1, sin2 2◦ + sin2 88◦ = sin2 2◦ + cos2 2◦ = 1, . . .

190

Megoldások, útmutatások

139. lg tg 45◦ = 0, így a bal oldalon 0 áll. A jobb oldal másképp írva: lg( tg 1◦ · tg 2◦ · tg 3◦ ·. . .· tg 89◦ ), s a tangensek szorzata 1 (hiszen tg 1◦ · tg 89◦ = 1, tg 2◦ · tg 88◦ = 1 stb.), így ennek logaritmusa is 0. 140. Az első sorozatból tetszőleges számot választva, annak valamely számjegye után beszúrunk egy nullát. Ez a megfeleltetés (a számjegy és a mögé beszúrt nulla között) kölcsönösen egyértelmű az első sorozatbeli számok számjegyei és a második sorozatbeli számok 0 számjegyei között. 141. Legyen pl. n = 20, és a 20-at ábrázoljuk egy 20 egység hosszú mérőrúddal, rajta a beosztások 0, 1, 2, . . . , 19, 20. A 20 bármely felbontásához egyértelműen rendelhető a mérőrúd valamely feldarabolása (a rudat csak beosztás helyén lehet eltörni). Pl. a 20 = 2 + 5 + 11 + 2 felbontáshoz a rúdnak az a feldarabolása, amikor a rudat a 2, 7, 18 pontokban törjük el. Annyi felbontás van, ahányféleképpen feldarabolhatjuk a rudat. Törni az 1, 2, 3, . . . , 19 pontokban lehet, azaz 19 helyen. Egy-egy helyen vagy törünk, vagy nem; ezért a darabolás lehetőségeinek száma: 2 · 2 · 2 · . . . · 2 = 219 . Az általános esetben a válasz: az n természetes számot 2n−1 módon bonthatjuk fel pozitív egészek összegére. 142. Ahhoz, hogy a nagy téglatestet 330 db egységkockára vághassuk szét, 5 vízszintes, 10 függőleges és 4 elölről-hátra vágást kell végeznünk, azaz összesen 5 + 10 + 4 = 19 vágást. Amikor a testátló az egyik egységkockából a másikba halad, a 19 sík közül egyet mindig átdöf. Pontosan egyet és nem többet, mert 11, 6 és 5 relatív prímek. (Ellenkező esetben fejtegetésünk nem lenne érvényes.) Ezért a megkárosított kockák száma 20 lesz. 143. Egy-egy érdekes sorszámhoz párként választunk egy másik sorszámot úgy, hogy az első három számjegy helyébe azokat a számjegyeket írjuk, amelyek ezeket a jegyeket 9-re egészítik ki. 144. 0 ≤ n ≤ 999, ha n-hez 1000-et adunk, a szám jegyeinek összege 1-gyel nő. Az összegben n és n + 1000 közül pontosan az egyik szerepel. Továbbá n és 999 − − n közül pontosan az egyik szerepel az összegben. Így az összegben a 0, 1, 2, . . . , 999 számok fele szerepel, míg a megmaradó számokat 1000-rel növelve megkapjuk az összeg 999-nél nagyobb tagjait, tehát az összeg értéke 0 + 1 + 2 + + · · · + 999 + 500 · 1000 = 999 500. 145. A táblára írt számokból újabbakat készítünk. Mindegyik számból n db újabb számot nyerünk úgy, hogy annak egy-egy jegyét kiválasztjuk, és az „ellenkezőjére” változtatjuk (1 ↔ 2). Így nem kaphatunk olyan számot, mely már kezdetben is fel volt írva; sőt olyat sem kaphatunk, melyet egy másik, a táblán levő számból készítettünk. Ennek oka a feladatbeli feltétel: bármely két szám legalább három helyen különbözik.

4. Miért nem négyzetszám?

191

Ha a táblán kezdetben m db szám szerepelt, akkor most, az újabb számok felírása után a táblán levő számok száma: m(n + 1); és ez legfeljebb annyi, mint a csupa 1-esekből és 2-esekből álló n-jegyű számok száma: 2n . Azaz m(n + 1) ≤ 2n , 2n m≤ . n+1

4. Miért nem négyzetszám? Egy szám nem lehet négyzetszám, ha (i) utolsó számjegye 2, 3, 7, 8; (ii) osztható 3-mal, de 9-cel nem; (iii) 3-mal osztva 2 maradékot ad; (iv) 4-gyel osztva 2 vagy 3 a maradék; (v) két szomszédos négyzetszám között van. 146. (i) miatt nem lehet. 147. (i) miatt nem lehet. Más indoklás: a szám osztható 7-tel, de nem osztható 49-cel. 148. (i) miatt nem lehet. 149. (ii) miatt nem lehet. 150. (i), ill. (v) miatt sem lehet. 151. (ii), ill. (v) miatt sem lehet. Más indoklás: a szám osztható 5-tel, de nem osztható 25-tel. 152. (ii), ill. (v) miatt sem lehet. 153. abab = 1010a +101b = 101·(10a +b), azonban 101 prím és 10a +b nem osztható 101-gyel. 155. abcabc = 1001 · abc = 7 · 11 · 13 · abc. Ezért, ha abcabc négyzetszám, akkor abc osztható lenne 7 · 11 · 13 = 1001-gyel, de ez nem lehet. 156. abc + bca + cab = 111 · (a + b + c) = 3 · 37 · (a + b + c), s ez nem lehet négyzetszám, mert a + b + c nem osztható 37-tel. 157. (ii) miatt nem lehet. 158. (iii) miatt nem lehet. 160. (iii) miatt nem lehet. 161. (ii) miatt nem lehet.

Megoldások, útmutatások

192

162. Ha ez a szám négyzetszám lenne, nem végződhetne 06-ra vagy 66-ra mert akkor 4-gyel osztva 2 maradékot adna; ezenkívül 60-ra sem végződhet. Tehát 00-ra végződik. Ezt a számot 100-zal osztva ismét négyzetszámot kell kapnunk. A kapott számra az előző okoskodást ismételhetjük. (A 600 db-os megkötésnek a bizonyítás szempontjából nem volt szerepe.) Eszerint a feladatban megjelölt számok nem lehetnek négyzetszámok. 163. (ii) miatt nem lehet. 164. 1111, 5555, 9999 jegyekre nem végződhet, mert ekkor 4-gyel osztva 3 maradékot kapnánk. 6666-ra sem végződhet, mert ekkor 4-gyel osztva 2 a maradék. 4444-re pedig azért nem, mert ha ezt 4-gyel osztjuk, nem kapunk négyzetszámot, hiszen a hányados 11-re végződik, ami 4-gyel osztva 3 maradékot ad. Megjegyzés. 3 egyforma jegyre végződhet négyzetszám, pl.: 382 = 1444 vagy 44622 = 19909444. 166. (iv) miatt nem lehet. 167. 444 . . . 444 = 4 · 111 . . . 111, s 111 . . . 111 nem lehet négyzetszám, mert 4-gyel osztva 3 a maradék. Vizsgálható a 16-os osztási maradék is. Lehet indokolni a következőképpen is: a szám szorzattá alakítva 4 · 11 · 1010101 . . . 01 alakú, s ebből látható, hogy a szám osztható 11-gyel, de 112 -nel nem. 168. 144 . . . 444 = 4 · 36111 . . . 111, s 36111 . . . 111 nem lehet négyzetszám, mert 4-gyel osztva 3 a maradék. Vizsgálható itt is a 16-os osztási maradék. 169. (i), (iii), (iv) ill. (v) miatt sem lehet. 170. n2 < n(n + 1) < (n + 1)2 , vagy indokolhatunk úgy is, hogy n és n + 1 relatív prímek, így mindkettőnek négyzetszámnak kell lennie, de ez nem lehet. 171. (n − 1)n(n + 1) = n · (n2 − 1); n és n2 − 1 relatív prímek, ezért mindegyiknek négyzetszámnak kell lennie, de n2 − 1 sohasem négyzetszám. 172. n(n + 1)(n + 2)(n + 3) = (n2 + 3n + 1)2 − 1. 173. (iv) miatt nem lehet. 174. (iii) miatt nem lehet. 175. (iv) miatt nem lehet. 176. (n − 2)2 + (n − 1)2 + n2 + (n + 1)2 + (n + 2)2 = 5(n2 + 2), s ez nem négyzetszám, mert n2 + 2 nem osztható 5-tel, hiszen négyzetszám 3-ra, 8-ra nem végződhet. 177. (iii) miatt nem lehet. 178. (iv) miatt nem lehet. 179. Vizsgáljuk az összeg 4-es osztási maradékát.

5. Négyzetszámok

193

180. ab = n2 és ha ab + a = m2 lenne, akkor ab(ab + a) = a 2 b(b + 1) is négyzetszám, de b(b + 1) nem négyzetszám (170. feladat), így ab(ab + a) sem, végül ab + a sem az. 182. Ha x ≤ y, akkor y 2 < y 2 + x ≤ y 2 + y < (y + 1)2 . 184. (n2 + n)2 < n4 + 2n3 + 2n2 + 2n + 1 < (n2 + n + 1)2 . Vagy: n4 + 2n3 + 2n2 + 2n + 1 = (n + 1)2 (n2 + 1), s ez a szorzat csak akkor lehetne négyzetszám, ha n2 + 1 is négyzetszám lenne.  2  2 186. 1157995 < 11571990 +341990 < 1157995 + 2 . A megadott intervallumban egy négyzetszám van, de az páros. 187. Páros n esetén az összeg utolsó jegye vagy 3, vagy 7. Páratlan n esetén pedig 3-mal osztva 2 maradékot ad.

5. Négyzetszámok 188. Lehet, pl. 4 és 16. 189. Nincs, ugyanis a szomszédos négyzetszámok különbsége egyre nagyobb lesz. 190. (2n − 1)2 + (2n + 1)2 − 2 = 2 · (2n)2 . 191. n ≥ 1 esetén n2 + n + 41 < (n + 6)2 , tehát, ha n2 + n + 41 négyzetszám, akkor csak (n + k)2 alakú lehet, ahol 0 ≤ k < 6. Az egyetlen megoldás: n = 40. 192. A keresett számban a prímtényezők csak páros hatvánnyal szerepelhetnek. A páros számok közül így b) és e) kiesik, mivel 4-gyel nem oszthatók; a) osztható 8-cal, de 16-tal nem; d) osztható 3-mal, de 9-cel nem; c) 3-mal osztva 2 maradékot ad. Tehát az utolsó szám a négyzetszám. 2. megoldás. Az a), c), d) számok 9-cel osztva 8, 5, ill. 3 maradékot adnak. b) és e) utolsó jegye miatt nem lehet négyzetszám. Az utolsó szám 50 123 246-nak a négyzete. 193. Ha a szám jegyeinek összege 1991, akkor ez a szám 3-mal osztva 2-t ad maradékul, míg négyzetszámnál ez a maradék nem léphet fel. √ 194. 1991 = 44,6206230 . . . , 4462 = 198 916, 44622 = 19 909 444, 44632 = 19 918 369. 198. Legyen a sorozat differenciája d, egyik eleme m2 . Ekkor (m + d)2 = m2 + 2md + d 2 = m2 + (2m + d) · d szintén eleme a sorozatnak. 199. Sok lehetőség van, pl.: 10k + 3; 4k + 2; 3k + 2; 9k + 3 stb.

Megoldások, útmutatások

194

200. 2, 23 , 25 , 27 , 29 , . . . 201. a) Négyzetszám 4-gyel osztva 0 vagy 1 maradékot ad. Ezt felhasználva az egyik lehetséges válasz: a 4k + 3 alakú számok. b) Köbszám 9-cel osztva 0, 1 vagy 8 maradékot ad. Ennek ismeretében az egyik lehetséges válasz: a 9k + 3 alakú számok. 202. 10 = 32 + 12 , 100 = 82 + 62 , 1000 = 302 + 102 , 10000 = 802 + 602 , 100000 = = 3002 + 1002 , . . . 203. Ha a · b = m2 , b · c = n2 , akkor (ab) · (bc) = (n · m)2 , (ac) · b2 = (n · m)2 , mivel n · m 2 b2 = 0, így ac = . Ha az ac egész szám egy törtszám négyzete, akkor b az a törtszám egész szám. (Miért?) 204. a) Nem, mert 1987 4-gyel osztva 3 maradékot ad. b) Tegyük fel, hogy az egyenlet megoldható, és egy megoldása x = a és y = b. Mivel 1986 osztható 3-mal, így a 2 + b2 is. Tekintettel arra, hogy egy 3-mal nem osztható négyzetszám 3-mal osztva mindig 1 maradékot ad, ez csak úgy lehet, ha a és b is osztható 3-mal. Ekkor 9 | a 2 + b2 , de 9  | 1986, ezzel ellentmondásra jutottunk. c) Az előző gondolatmenetet követve: x = 3a, y = 3b, az egyenletből így az a 2 + b2 = 221 egyenletet nyerjük. Ennek megoldásai a = 10, b = 11, és a = 5, b = 14. Innen x = 30, y = 33 és x = 15, y = 42 (ill. x = 33, y = 30 és x = 42, y = 15). d) (x −y)(x +y) = 1989 = 32 ·13·17, így pl. egy megoldás x −y = 9, x +y = 221, azaz x = 115, y = 106. e) Négyzetszám 4-gyel osztva 0 vagy 1 maradékot adhat, ezért két négyzetszám különbsége 4-gyel osztva nem ad 2 maradékot. 205. a) (a 2 + b2 )(c2 + d 2 ) = (ac + bd)2 + (ad − bc)2 . b) (a 2 + 2b2 )(c2 + 2d 2 ) = (ac + 2bd)2 + 2(bc − ad)2 . 206. a) Ha x = a 2 + b2 , akkor 4x = (2a)2 + (2b)2 . Ha 4x = a 2 + b2 , akkor vagy a és b is páros, vagy a és b is páratlan. Ez utóbbi nem lehet, mert két páratlan szám négyzetének összege nem osztható 4-gyel. Tehát a = 2c, b = 2d, így az egyenletből helyettesítés után nyerjük az x = c2 + d 2 alakot. Megjegyzés. Az állítás következik a feladat erősebb, b) állításából. b) Ha x = a 2 + b2 , akkor 2x = (a + b)2 + (a − b)2 . Fordítva, ha 2x = a 2 + b2 ,   a+b 2 akkor vagy a és b is páros, vagy a és b is páratlan. Ekkor az x = + 2  2 a−b + felbontás x-et is két négyzetszám összegeként állítja elő. (A 2 második tag lehet 0.)

5. Négyzetszámok

195

207. Az állítás egyik része következik a 205. b) állításból. Ha 3x = a 2 + 2b2 , akkor 3 | a 2 + 2b2 , így 3 | a 2 + 2b2 − 3b2 = a 2 − b2 = (a − b)(a + b), tehát vagy 3 | a − b (ekkor 3 | a + 2b) vagy 3 | a + b (ekkor 3 | a − 2b). 3 | a − b esetén az     a + 2b 2 a−b 2 x= +2 , 3 3 3 | a + b esetén pedig az     a+b 2 a − 2b 2 +2 , x= 3 3 előállítást nyerjük, ahol a tagok egész számok. 208. a) (a 2 + 5b2 )(c2 + 5d 2 ) = (ac − 5bd)2 + 5(ad + bc)2 . b) Van. 84 = 4 · 21 = 6 · 14. E számok mindegyike a 2 + 5b2 alakú: 84 = 22 + 5 · 42 , 4 = 22 + 5 · 02 , 6 = 12 + 5 · 12 , 14 = 32 + 5 · 12 és 21 = 42 + 5 · 12 . Ezek mindegyike alapszám, kivéve természetesen a 84-et. c) (Hasonló a prímszámok végtelenségének Euklidesz-féle bizonyításához.) Tegyük fel, hogy csak véges sok alapszám van: x1 , x2 , . . . , xn . Az 1 + 5(x1 x2 . . . xn )2 szintén M-hez tartozik, nem osztható egyik xi -vel sem, tehát újabb alapszámot találtunk. 209. Előbb megmutatjuk, hogy két a 2 + 3b2 alakú szám szorzata is ilyen alakú: (x 2 + 3y 2 )(z2 + 3t 2 ) = (xz + 3yt)2 + 3(xt − yz)2 . Ha az a két szám, melyek összege a 2 + 3b2 alakú, A és B, akkor a feladat állításának igazolásához elegendő belátni, hogy A2 − AB + B 2 is a 2 + 3b2 alakú, hiszen A3 + B 3 = (A + B)(A2 − AB + B 2 ).     A+B 2 A−B 2 2 2 Ha A és B azonos párosságú, akkor A − AB + B = +3 ; 2 2 ha A és B különböző párosságú, és pl. A páros, akkor   2 2 A A 2 2 A − AB + B = −B +3 2 2 egy-egy megfelelő felírás. 210. Legyen n = 3x 2 + 32y 2 , akkor 96n = 96 · 3x 2 + 96 · 32y 2 = 3(32y)2 + 32(3x)2 ,

Megoldások, útmutatások

196

tehát 96n is érdekes. 97n = n + 96n = [3x 2 + 32y 2 ] + [3(32y)2 + 32(3x)2 ] = = 3[x 2 + (32y)2 ] + 32[y 2 + (3x)2 ] = = 3[x 2 + 64xy + (32y)2 ] + 32[y 2 − 6xy + (3x)2 ] = = 3(x + 32y)2 + 32(y − 3x)2 , tehát 97n is érdekes szám. Általánosabban megmutatjuk, hogy ha n = ax 2 + by 2 , akkor (ab + 1)n is előáll ilyen alakban (a, b, x, y egész számok). Ha n = ax 2 + by 2 , akkor (ab + 1)n = (ab + 1)(ax 2 + by 2 ) = a 2 bx 2 + ax 2 + ab2 y 2 + by 2 = = a 2 bx 2 + 2abxy + ax 2 + ab2 y 2 − 2abxy + by 2 = b(a 2x 2 + 2axy + y 2 ) + a(b2 y 2 − 2bxy + x 2 ) = = a(by − x)2 + b(ax + y)2 . (Speciálisan, ha a = 3 és b = 32, akkor ab + 1 = 97.) 211. Az egyenletet rendezve kapjuk, hogy z2 = (x − z)(y − z). Ha a d egész szám osztója (x − z)-nek és (y − z)-nek is, akkor z2 osztható d 2 -tel, tehát z osztható d-vel. De ekkor x = (x − z) + z és y = (y − z) + z alapján d az x, y és z számok mindegyikének osztója, így a feladat feltétele szerint csak 1 lehet. Tehát (x − z)nek és (y − z)-nek nincs 1-nél nagyobb közös osztója, azaz relatív prímek. Ezért és mert a szorzatuk négyzetszám, mindkettő külön-külön négyzetszám; vagyis alkalmas k és l egészekre x − z = k 2 , y − z = l 2 , és az átrendezett egyenlet alapján z = kl. Innen x + y = (x − z) + (y − z) + 2z = k 2 + l 2 + 2kl = (k + l)2 . 212. A feltételben szereplő egyenletből kapjuk: (3x + 3y + 1)(x − y) = x 2 és (2x + 2y + 1)(x − y) = y 2 . Ezek szorzata (3x + 3y + 1)(2x + 2y + 1)(x − y)2 = (xy)2 . Ha xy = 0, akkor az utóbbi egyenlőségből kapjuk, hogy x = 0 és y = 0; s ekkor igaz a feladat állítása. Ha xy = 0, akkor az egyenlőségből kapjuk, hogy (3x + 3y + 1)(2x + 2y + 1) is négyzetszám. Elegendő belátni, hogy (3x + 3y + 1) és (2x + 2y + 1) relatív prímek, hiszen ekkor mindkettő négyzetszám. Nos, ha d | 3x + 3y + 1 és d | 2x + 2y + 1, akkor d | (3x + 3y + 1) − (2x + 2y + 1) = x + y, de ekkor d | (2x + 2y + 1) − 2(x + y) = 1. 213. Legyen N = a1 a2 . . . a16 , az ai -k között tehát csak a 2, 3, 5, 6, 7, 8 jegyek fordulhatnak elő. Képezzük a következő 16-tagú sorozatot: a1 , a1 · a2 , a1 · a2 · a3 , . . . , a1 · a2 · a3 · . . . · a16 . Ezek prímtényezős felbontása: 22α1 +β1 · 32α2 +β2 · 52α3 +β3 · 72α4 +β4 alakú, ahol αi nemnegatív egész, βi pedig 0 vagy 1.

6. Két szomszédos egész szám szorzata

197

A 16-tagú sorozat elemeihez hozzárendelhető egy 0-kból és 1-esekből álló (β1 , β2 , β3 , β4 ) négyes. A különböző ilyen számnégyesek száma éppen 16. Ezért vagy szerepel közöttük a (0, 0, 0, 0), s akkor ezt olyan a1 · a2 · a3 · . . . · ak szorzathoz rendeltük, amely négyzetszám; ellenkező esetben van két azonos számnégyes. Ekkor annak a két számnak (szorzatnak) a hányadosa lesz négyzetszám, amelyekhez ez a két megegyező számnégyes tartozik. A feladat állítása 15-jegyű számra nem igaz; erre példa 232523272325232. 214. 12 egymást követő szám összege többszöröse 3-nak, míg 12 egymást követő négyzetszám összege 3-mal osztva 2 maradékot ad. 215. a) Ha n > 2, akkor n2 > n2 − n + 2 > (n − 1)2 . Válasz: n = 2. b) n3 − n + 2 3-mal osztva 2 maradékot ad, így nem lehet négyzetszám. c) Ha n > 2, akkor (n2 )2 > n4 − n + 2 > (n2 − 1)2 . Válasz: n = 2. d) n5 − n + 2 5-tel osztva 2 maradékot ad, így nem lehet négyzetszám. n

n

 1  1 216–218. Használjuk az 111 . . . 11 = · 999 . . . 99 = (10n − 1) összefüggést. Példá9 9 nak lássuk a 215. d) feladat megoldását. 44 . . . 488 . . . 89 = 4 8 4 · 102n − 4 · 10n + 8 · 10n − 80 + 81 = (10n − 1)10n + (10n−1 − 1)10 + 9 = = 9 9 9   2 · 10n + 1 2 4 · 102n + 4 · 10n + 1 = = . 9 3

6. Két szomszédos egész szám szorzata Két szomszédos egész szám szorzata (i) mindig páros; (ii) 0, 2 vagy 6 számjegyre végződik; (iii) 3-mal osztva 0 vagy 2 maradékot ad. 220. (i) miatt nem lehet. 221. (iii) miatt nem lehet. 222. (ii) vagy (iii) miatt sem lehet. 223. (ii) miatt nem lehet. 224. (i) miatt nem lehet.

Megoldások, útmutatások

198

1 2 (k − 1) két szomszédos egész szorzata. 4  2 1  2 1 4 (n + 2n3 + 3n2 + 2n) = n +n+1 −1 4 4

225. Ha k páratlan, akkor

227. Vegyük figyelembe, hogy a k(k+1) és a (k+1)(k+2) (k egész) számok között nincs olyan egész szám, amely előállítható két szomszédos egész szám szorzataként. Ezt felhasználva, az állítást először bizonyítsuk be n ≥ 0 egészekre, ezután n < < −6 számokra. Végül tekintsük a többi megvizsgálandó esetet. 228. n(n + 1) < n2 + 2n + 12 < (n + 1)(n + 2), ha n > 10; és (n + 1)(n + 2) < n2 + 2n + 12 < n(n + 1), ha n < −12. Így megoldásokat csak olyan páros n-ekre kell keresnünk, amelyekre −12 ≤ n ≤ 10. A megoldás: −12, −4, −2, 0, 2, 10. 229. (ii), ill. (iii) miatt semmilyen n sem lehet. 230. Mivel n2 + n + 1 páratlan szám ((i) miatt), ezért nem lehet páros osztója. 231. (ii) miatt az egyenletnek nincs megoldása. 232. Jelöljük a felírt számot a-val és feltételezzük, hogy a = k(k + 1) alakban írható. Mivel a 3-mal osztva 2 maradékot ad, k = 3m + 1, ahol m egész. k(k + 1) = = (3m + 1)(3m + 2) = 9(m2 + m) + 2, vagyis k(k +1) 9-cel osztva 2 maradékot ad. Az adott számjegyekkel felírt szám pedig 9-cel osztva nem adhat 2-t maradékul. 233. Ha n két szomszédos egész szám szorzata, vagyis n = k(k + 1), akkor 9n + 2 = = (3k + 1)(3k + 2) is ilyen. Fordítva, legyen 9n + 2 két szomszédos egész szám szorzata: 9n + 2 = k(k + 1). Mivel 9n + 2 nem osztható 3-mal, így k = 3m + 1, azaz 9n + 2 = (3m + 1)(3m + 2), s ilyenkor 9n + 2 = 9[m(m + 1)] + 2, tehát n = m(m + 1). 234. (iii) miatt nem lehet: vizsgáljuk meg az n = 3m, n = 3m − 1 és n = 3m + 1 eseteket! 235. Hasonló a 216–218. feladatok megoldásához.

7. Diofantoszi egyenletek 236–242. feladatokban az egyenlőségjel egyik oldalán páros, másik oldalán páratlan szám áll. 243. A bal oldalon levő kifejezés vagy páratlan, vagy 4-gyel osztható szám. 244. A bal oldal osztható 3-mal, a jobb oldal nem. 246–248. Az egyenlőségjel bal és jobb oldalán lévő számoknak különböző a 3-mal való osztási maradéka.

7. Diofantoszi egyenletek

199

249. Az egyenlőségjel bal és jobb oldalán lévő számok 4-gyel való osztási maradéka különböző. 250–253. Az egyenlőségjel bal és jobb oldalán lévő számok 8-cal való osztási maradéka különböző. 254–255. Az egyenlőségjel bal és jobb oldalán lévő számok 9-cel való osztási maradéka különböző. 257. Az egyenlőségjel bal és jobb oldalán lévő számok 7-tel való osztási maradéka különböző. 259. Az egyenlőségjel bal és jobb oldalán lévő számok 9-cel való osztási maradéka különböző. 260. Az egyenlőségjel bal és jobb oldalán lévő számok 16-tal való osztási maradéka különböző. 261. Az egyenlőségjel bal és jobb oldalán lévő számok 7-tel való osztási maradéka különböző. 263. Tekintsük az egyenlet mindkét oldalán szereplő számok 9-cel való osztási maradékát! 264. 7 | 12n − 5n , 7  | 11n . 265. Átalakítás után az (x − 1)2 + (y − 1)2 = −1 egyenletet nyerjük. 266. Átalakítás után az (x 2 − y)2 + 2y 2 = −2 egyenletet kapjuk. 267. Átalakítás után az (a − b)2 = b2 (4b − 1) egyenletet kapjuk. Az egyenlet jobb oldalán azért nem állhat négyzetszám, mert 4b − 1 4-gyel osztva 3 maradékot ad, így ez a tényező nem négyzetszám. 1 1 1 1 1 1 3 + ≤ + + = < 1, hiszen a és b egyike sem lehet 1. + a 2 ab b2 4 4 4 4 270–283. feladatoknál olyan alakra hozzuk az egyenletet, amelynek egyik oldalán egy szorzat, a másikon pedig egy szám áll. Ezt a számot bontsuk fel prímtényezőkre. 268.

270–275. feladatoknál az (a + 1)(b + 1) = ab + a + b + 1 azonossággal rokon szorzattá alakítást célszerű elvégezni. 276. a(2a 2 + b) = 7; innen vagy a = 1, 2a 2 + b = 7 és így a = 1, b = 5, vagy a = 7, 2a 2 + b = 1 és így b = −97. A 7 = (−1) · (−7) = (−7) · (−1) szorzatok vizsgálata adja az a = −1, b = −9, ill. az a = −7, b = −99 megoldást. 277. (a − 5)(b + 3) = −18. 278. a 2 − b2 = (a − b)(a + b) = 100, a − b = 2, a + b = 50 vagy a − b = 50, a + b = 2. 279. (a − 2b)(a + 2b) = 116.

200

Megoldások, útmutatások

280. (2a − 3b)(a + 4b) = 28. 281. 243 = 35 . 282. (a + b + 1)(a − b − 1) = 12. 2 2 283. a 2 = b3 +1 = (b + 1)(b  − b + 1) = (b + 1)(b + b − 2b − 2 + 3) = 3 . = (b + 1)2 · b − 2 + b+1

284. Átalakítva az egyenletet az (a − b)2 + (b − c)2 + (c − a)2 = 0 egyenletet kapjuk. 285. Átalakítva az egyenletet az (a − 1)2 + (b − 1)2 + (a − b)2 = 2 egyenletet kapjuk. 286. Átalakítva az egyenletet az (a −1)3 +(b+2)3 = 1 egyenletet kapjuk. A bal oldalon szereplő köbszámok közül az egyik 1, a másik 0. 287. Vizsgáljuk a kifejezések párosságát! 288. n! 0-ra végződik, ha n ≥ 5. Ha az egyenletben szereplő n értéke legalább 5, akkor a bal oldalon álló összeg utolsó számjegye 3, így ez a szám nem lehet négyzetszám. Ezért elegendő az n < 5 eseteket vizsgálni. 289. 145. 290. Bontsuk −16-ot három egész szám szorzatára minden lehetséges módon, s válasszuk ki azt a számhármast, mely kielégíti a másik két egyenletet. 291–293. Ha egy — nullára redukált és rendezett — algebrai egyenletnek van egész gyöke, akkor ez a gyök osztója a polinom konstans tagjának. 294. Tegyük fel, hogy a ≤ b ≤ c. Ekkor abc = a + b + c ≤ c + c + c = 3c, tehát ab ≤ 3. További rövid elemzés adja a megoldást, az 1, 2, 3 számhármast. 295–296. Osszuk az egyenlet mindkét oldalát abc-vel, és tegyük fel, hogy a ≥ b ≥ c. 297. Itt is tekintsük az a ≥ b ≥ c feltevést. 298–300. A 270–275. feladatok megoldásához hasonló. 301. Ha b2 − 4ac = 1994, akkor b páros; ám ekkor b2 és 4ac is osztható 4-gyel, így b2 − 4ac is, míg 1994 nem osztható 4-gyel, ezért a b2 − 4ac = 1994 egyenlet nem oldható meg az egész számok körében. Ha n = 2k + 1, akkor n2 = 4k 2 + 4k + 1, tehát páratlan szám négyzete 4-gyel osztva 1-et ad maradékul. Ha b2 − 4ac = 1995, akkor b páratlan, b2 − 4ac 4gyel osztva 1-et ad maradékul, míg 1995 4-gyel osztva 3 maradékot ad. Ezért a b2 − 4ac = 1995 egyenlet nem oldható meg az egész számok körében. Az x 2 + 46x + 30 = 0 egyenlet diszkriminánsa 1996.

7. Diofantoszi egyenletek

201

302. Ha a + b + c ≤ 11, akkor 28a + 30b + 31c ≤ 11 · 31 = 341. Ha a + b + c ≥ 13, akkor 28a + 30b + 31c ≥ 13 · 28 = 364, egyenlőség csak akkor, ha a = 13, b = c = 0; minden más esetben 28a + 30b + 31c ≥ 366. Az egyenletnek megoldása (gondoljunk arra, hogy egy év 365 napból áll): x = 1 (a február 28 napos), y = 4 (április, június, szeptember, november), z = 7 (a többi hét hónap). Más megoldás: x = 2, y = 1, z = 9. 303. Használjuk a következő becslést: k[x] ≤ [kx] ≤ k[x] + k − 1. Ezzel a feladatbeli összegre (jelöljük ezt S-sel) a 63m ≤ S ≤ 63m + 57 egyenlőtlenségeket nyerjük, ahol m = [x]. Ugyanakkor 12345 = 63k + 60. 304. Szorozzuk meg az egyenlet mindkét oldalát 4-gyel, ekkor az egyenlet a következő alakba írható: (2x +3y)2 −17y 2 = 488. Most már könnyű megmutatni, hogy az egyenlet nem oldható meg az egész számok körében. Vizsgáljuk a 17-es osztási maradékokat! Az egyenlet jobb oldala 17-tel osztva 12 maradékot ad, ám a bal oldalon ilyen maradék nem léphet fel. 305. Az (a + b + c)3 − (a 3 + b3 + c3 ) = 3(a + b)(b + c)(c + a) azonosságot használva (a + b)(b + c)(c + a) = 114, és a feladat első egyenlete másképp (a + b) + (b + c) + + (c + a) = 14. Mivel 114 = 1 · 2 · 3 · 19, így csak a + b = 19, b + c = −3, c + a = −2 lehet megoldás. Az egyenletrendszer megoldása: a = 10, b = 9, c = −12 (ill. ezeknek a számoknak más sorrendjei). 306. A második egyenlet 3-szorosából kivonva az első négyzetét: (a − b)2 + (b − c)2 + (c − a)2 = 14. 307. Dolgozzunk komplex számokkal! (m2 + n2 )2 = |m + in|4 = |(m + in)2 |2 = |m2 + + 2mni + i 2 n2 |2 = |(m2 − n2 ) + i(2mn)|2 = (m2 − n2 )2 + (2mn)2 . Ha (m, n) = 1, m > n, valamint az m és n számok különböző párosságúak, akkor (m2 − n2 , 2mn, m2 + n2 ) = 1, s ezek a számhármasok megoldásai az egyenletnek. 308. Dolgozzunk komplex számokkal! (m2 + n2 )3 = |m + in|6 = |(m + in)3 |2 = |m3 + + 3m2 in + 3mi 2 n2 + i 3 n3 |2 = |(m3 − 3mn2 ) + i(3m2 n − n3 )|2 = (m3 − 3mn2 )2 + + (3m2 n − n3 )2 . Ha (m, n) = 1, valamint az m és n számok különböző párosságúak, akkor (m3 − − 3mn2 , 3m2 n − n3 , m2 + n2 ) = 1, s ezek a számhármasok megoldásai az egyenletnek. 309. Dolgozzunk komplex számokkal! (m2 + n2 )4 = |m + in|8 = |(m + in)4 |2 = = |m4 + 4m3 in + 6m2 i 2 n2 + 4mi 3 n3 + i 4 n4 |2 = = |(m4 − 6m2 n2 + n4 ) + i(4m3 n − 4mn3 )|2 = = (m4 − 6m2 n2 + n4 )2 + (4m3 n − 4mn3 )2 . Ha (m, n) = 1, valamint az m és n számok különböző párosságúak, akkor (m4 − 6m2 n2 + n4 , 4m3 n − 4mn3 , m2 + n2 ) = 1, s ezek a számhármasok megoldásai az egyenletnek.

202

Megoldások, útmutatások

310. A végtelen leszállás (descente infinie) módszerével bizonyítunk. Tegyük fel, hogy a1 , b1 megoldás. Ekkor a1 nyilván páros, de b1 is páros lesz: a1 = 2a2 , b1 = 2b2 . Ha a12 = 2b12 , azaz (2a2 )2 = 2(2b2 )2 , akkor a22 = 2b22 , tehát a2 , b2 is megoldása az egyenletnek. Így az egyenletnek végtelen sok ai , bi megoldásához jutunk, melyekre |a1 | > |a2 | > · · · > |ak | > . . . (ugyanígy bi -kre is). Ez azonban nem lehetséges, hiszen a pozitív egészeknek nincs végtelen sok tagból álló, szigorúan csökkenő sorozata. Ellentmondásra jutottunk, az ellentmondás oka a kiinduló feltevés (az, hogy az egyenletnek létezik megoldása), tehát a kiinduló feltevés hamis. (Reductio ad absurdum.) 311. Alkalmazzuk a végtelen leszállás módszerét. 312. Alkalmazzuk a végtelen leszállás módszerét. Használjuk fel, hogy ha 3 | a 2 + b2 , akkor 3 | a és 3 | b. 313. Alkalmazzuk a végtelen leszállás módszerét. Használjuk fel, hogy ha 7 | a 2 + b2 , akkor 7 | a és 7 | b. 314. Alkalmazzuk a végtelen leszállás módszerét. Használjuk fel, hogy ha 3 | a 2 + b2 , akkor 3 | a és 3 | b. 315. Alkalmazzuk a végtelen leszállás módszerét. Lássuk be, hogy a is, b is, c is osztható 3-mal. 316. Alkalmazzuk a végtelen leszállás módszerét. Lássuk be, hogy a is, b is, c is osztható 2-vel. 317. Alkalmazzuk itt is a végtelen leszállás módszerét. Tegyük fel, hogy az egyenletnek vannak nullától különböző megoldásai. Tudjuk, hogy négyzetszám 4-gyel osztva 0 vagy 1 maradékot ad. Innen következtethetünk arra, hogy ha a1 , b1 , c1 megoldás, akkor mindegyik szám páros: a1 = 2a2 , b1 = 2b2 , c1 = 2c2 . Ekkor a22 + b22 + c22 = 4a22 b22 . Ebből következik, hogy a2 , b2 , c2 mindegyike páros: a2 = = 2a3 , b2 = 2b3 , c2 = 2c3 . Egyszerűsítés után az egyenlet: a32 + b32 + c32 = 16a32 b32 . Láthatjuk, hogy a3 , b3 , c3 mindegyike páros. Ezt a gondolatmenetet újra és újra megismételve kapjuk, hogy az |a1 | > |a2 | > · · · > |ak | > . . . sorozat mindegyik eleme páros, ami nem lehet. Ellentmondásra jutottunk, az ellentmondás oka a hibás feltevés. Tehát az egyenlet nem oldható meg a nullától különböző egészek körében. 318. Alkalmazzuk a végtelen leszállás módszerét. A gondolatmenet hasonló, mint az előző feladat megoldásában. 319. Alkalmazzuk a végtelen leszállás módszerét. A gondolatmenet hasonló, mint az előző két feladat megoldásában.

8. Prímszámok

203

320. Alkalmazzuk most is a végtelen leszállás módszerét. Azt látjuk be, hogy az a 4 + +b4 = c2 egyenlet nem oldható meg a pozitív egészek körében. Legyen (a, b, c) = = 1. Ismert, hogy ekkor a 2 = 2uv, b2 = u2 − v2 , u > v > 1, (u, v) = 1 és v páros; hiszen u nem lehet páros, mert ekkor u2 = 4A, v2 = 4B + 1, így b2 = u2 − v2 = = 4A − (4B + 1) = 4C + 3 lenne. Így aztán a 2 = 2uv miatt és mert v páros; u = x 2 , v = 2y 2 . A b2 = u2 − v2 összefüggésből b2 = x 4 − 4y 4 , azaz b2 + 4y 4 = x 4 . Innen y 2 = u1 v1 és x 2 = u21 + v12 , ahol u1 = a12 , v1 = b12 ; tehát x 2 = a14 + b14 , a1 < a és b1 < b, mert a 2 = 2uv = 4x 2 y 2 = 4u1 v1 (u21 + v12 ) = 4a12 b12 (a14 + b14 ) > a12 , így a1 < a; b2 = u2 − v2 = x 4 − 4y 4 = (x 2 + 2y 2 )(x 2 − 2y 2 ) ≥ x 2 + 2y 2 > > x 2 = a14 + b14 > b14 ≥ b12 , innen b1 < b. Tehát, ha az eredeti egyenletnek volt a, b, c megoldása a pozitív egészek körében, akkor van ezeknél kisebb pozitív egészekből álló a1 , b1 , c1 megoldása is. 321. Az egyenlet átalakítás után a (2a + 1)2 + (2b + 1)2 + (2c + 1)2 = 7 alakot ölti. Ha ennek az egyenletnek van megoldása a racionális számok körében, akkor az x 2 + y 2 + z2 = 7t 2 egyenlet megoldható az egész számok körében. Páratlan szám négyzete 8-cal osztva 1 maradékot ad; emiatt t nem lehet páratlan, hiszen ekkor az egyenlőség két oldalán a 8-as maradékok különbözők lesznek. Mivel t páros, a 4-es maradékot figyelve x, y, z mindegyike páros lesz: x = 2x1 , y = 2y1 , z = 2z1 és t = 2t1 . Az egyenletnek x1 , y1 , z1 , t1 is megoldása. Alkalmazhatjuk a végtelen leszállás gondolatát. 322. Ha létezik racionális számokból álló megoldás, akkor az x 2 + xy + y 2 = 2z2 egyenletnek van megoldása az egész számok körében. Ha x, y valamelyike vagy mindkettő páratlan, akkor a bal oldalon páratlan szám áll. Tehát x = 2x1 , y = 2y1 . Ekkor a bal oldalon álló szám osztható 4-gyel, így z is páros, z = 2z1 . Az egyenletnek x1 , y1 , z1 is megoldása. Alkalmazhatjuk a végtelen leszállás gondolatát.

8. Prímszámok 323. Páratlan. 324. 1 és 2. 325. −3, −1, 1. 326. 5 és 2. 327. A két összeadandó egyike páros, ez csak 2 lehet. A másik összeadandó 1989, ez pedig összetett, osztható 9-cel. 328. Mind a három prím nem lehet páratlan, így a legkisebb összeadandó a 2.

Megoldások, útmutatások

204

329. Három páratlan prímszám összege páratlan, emiatt az összeg 1234 csak úgy lehet, ha a három prím egyike a 2. Ekkor a három prímszám szorzata páros, így az nem lehet 87654321. Tehát a kívánt módon nem lehet három prímszámot megadni. 330. Legyen a két prím p és q, és q = p + 100; az egymás után írásukkal keletkező prím r. Ha p 3-mal osztva 1 maradékot ad, akkor q 3-mal osztva 2 maradékot ad, és ekkor az r szám osztható 3-mal, nem lehet prím (> 100). Ha p 3-mal osztva 2 maradékot ad, akkor q osztható 3-mal. Ha p 3-mal osztva 0 maradékot ad, akkor csak p = 3 lehetséges. Ekkor q = 103 (ami prím). Az egymás után írásukkal keletkező szám a 3103 (3103 = 29 · 107) és 1033 (ez prímszám). 331. Ha mind a hat prím páratlan, akkor az összeg páros szám, nem prím. A hat szám: 2, 3, 5, 7, 11, 13. 332. Két páratlan szám között levő egészek száma páratlan. 333. n egymást követő egész szám összege osztható n-nel, ha n páratlan. Megjegyzés. Ha n páratlan prím, akkor n egymást követő egész szám összege lehet prímszám. Pl. n = 7 esetén: −2, −1, 0, 1, 2, 3, 4. 334. Az összeg páros szám. 335. Nem. Ha lehetne, akkor mind a 3 sorban ugyanannyi lenne a számok összege, tehát a kilenc szám összege osztható lenne 3-mal, s ez nem teljesül. Más indoklás: Abban a sorban, amelyikben a 2 áll, a számok összege páros, míg a többi sorban páratlan. 336. y osztható 2-vel, y = 2; x osztható 3-mal, x = 3. Így z = 11. 337. p értéke csak 2 lehet. Ha q páratlan, akkor 2q +1 osztható 3-mal. Tehát p = q = 2, r = 5. 338. 2q 2 = p2 − 1 = (p − 1)(p + 1). p = 2 nem megoldás; ha p páratlan, akkor (p − 1)(p + 1) osztható 4-gyel is, tehát q = 2. Ezért p = 3. 340. 4, 8, 32 évesek. 341. Mivel a keresett szám jegyeinek szorzata kisebb, mint 93 és egy prím köbe, a szorzat lehetséges értékei: 23 , 33 , 53 , 73 . 342. 2n − 1, 2n , 2n + 1 három egymást követő egész közül az egyik osztható 3-mal. 343. p − 1, p, p + 1 három egymást követő egész közül az egyik osztható 3-mal, s ez a szám nem a p, tehát 3 | (p − 1)(p + 1) = p2 − 1. Továbbá, p − 1 és p + 1 két szomszédos páros szám, az egyik osztható 4-gyel, tehát 2 · 4 = 8 | (p − 1)(p + 1) = p2 − 1.

8. Prímszámok

205

344. A számok egyike osztható 3-mal. 345. a) Ha p = 3, akkor vagy p + 5, vagy p + 10 osztható 3-mal. b) Hasonló az a) ponthoz. c) 8p − 1, 8p és 8p + 1 közül az egyik osztható 3-mal. Ha p = 3, akkor 3  | 8p, tehát vagy 8p − 1, vagy 8p + 1 osztható 3-mal. d) Hasonló az c) ponthoz. e) Ha p = 3, akkor p2 + 2 mindig osztható 3-mal. f) Hasonló az e) ponthoz. 346–347. p = 3-ra igaz. Ha p = 3, akkor p2 −1 osztható 3-mal (lásd a 343. feladatot!), s ekkor p2 + 8 és 8p2 + 1 is osztható 3-mal. 348. p és q nem lehet egyszerre páros, ill. páratlan. Ha p = 2 és q páratlan, akkor p2 + q 2 − q osztható 2-vel, így q = 2; tehát p2 + q 2 − q = p2 + 2 = (p2 − 1) + 3. Ha p = 3, akkor ez a szám osztható 3-mal. A megoldás tehát q = 2, p = 3. 350. p és q közül az egyik egyenlő 2-vel. Ha p = q = 2, akkor pq − 1 és pq + 1 prímszámok. Tegyük fel, hogy p = 2 és q páratlan! Akkor ha q = 3, 2q − 1 és 2q + 1 közül az egyik osztható 3-mal. 351. 4p + 1 = (2k + 1)2 -ből p = k(k + 1) következik. Ez csak akkor prímszám, ha k = 1. p+q p+q 355. p + q = 2 · , és összetett szám, hiszen két szomszédos (páratlan) prím 2 2 között van. 356. a) b) c) d) e) f) g)

n4 + 2n3 + 2n2 + 2n + 1 = (n + 1)2 (n2 + 1), 6n + 3n + 2n+1 + 2 = (3n + 2)(2n + 1), 32n+1 − 22n+1 − 6n = (3n − 2n )(3n+1 + 2n+1 ), 8n2 + 10n + 3 = (2n + 1)(4n + 3), 25n4 + 9n2 + 1 = (5n2 + 1)2 − n2 , 4n3 + 6n2 + 4n + 1 = (n + 1)4 − n4 , |n4 + n3 − 2n2 − 3n − 3| = |(n2 − 3)|(n2 + n + 1).

357. Adjunk e prímek szorzatához 1-et. 359.

101

29

47

103

5

59 113

7

71

89 113

109

71 167

29

79

37

271 379

73 139 67

73

43 241 439

43

409 103 211

571 1051

181

823 1093

47

89 131

211

601

991

673

149

11 107

1021

151

631

1063

643

853 1033 613

883

Megoldások, útmutatások

206

307 607

97

569

59 449

127 337 547

239 359 479

577

269 659 149

67 367

829 1879

409

619 1039 1459 1669

199 1249

360. Ha f (x) = p, akkor f (x + p) osztható p-vel. 361. Következik az előzőből; vagy mutassuk meg, hogy ha a sorozat d differenciája nem osztható p-vel, akkor a sorozat p egymást követő eleme közt van olyan, amely osztható p-vel. 362. 7, 37, 67, 97, 127, 157. 363. 41, 47, 53, 59, 65, 71, 77, 83, 89. 364. Például: 11, 17, 23, 29, 35, 41, 47, . . . . Hiszen ha e számok valamelyike előállna két prím összegeként, akkor az egyik összeadandónak 2-nek kell lennie, s a másik így 6k + 3 alakú összetett szám lesz. 365. Nem, pl. 320 páros, így utolsó jegyét kell változtatni, viszont a 320 és 330 közötti számok mind összetettek. 366. c | ab, ezért c = xz, a = xy, b = zt, s ezek miatt d = yt. Ekkor a 2000 + b2000 + c2000 + d 2000 = (xy)2000 + (zt)2000 + (xz)2000 + (yt)2000 = = (x 2000 + t 2000 )(y 2000 + z2000 ). 367. Ha p összetett, p = mn, m > 1, n > 1, akkor p = (m − 1)(n − 1) + 1 + (m − 1) + + (n − 1) = a + b + c + d, s ekkor valóban ab = cd. Ha p = a + b + c + d, és ab = cd, akkor pb = ab + b2 + cb + db = cd + b2 + + cb + db = (c + b)(d + b). A p szám összetett, hiszen ha p prím volna, akkor az előző egyenlőségből p | c + b vagy p | d + b lenne, ami nem teljesülhet, hiszen p = a + b + c + d, ezért p > c + b, p > d + b. 368. Az, hogy a prímszámok száma végtelen, már 2000 éve ismert. Euklidesz bizonyításának eleganciája megragadja a figyelmes olvasót, s hasonló élményeket nyújt a többi bizonyítás is. (A bizonyítások többnyire vázlatosan vannak leírva.) 1. bizonyítás. (Euklidesz) Tegyük fel, hogy az állítás hamis: a prímszámok száma véges; a p1 , p2 , . . . , pn prímeken kívül nincs más prímszám. Vizsgáljuk az N = = p1 · p2 · . . . · pn + 1 számot. Ez nyilván nem osztható az eddig felsorolt prímek egyikével sem, tehát vagy prímszám, vagy olyan összetett szám, melynek csak az előzőktől különböző prímosztói vannak. Ezzel újabb prímszám létezését mutattuk meg, ami ellentmond a feltevésnek.

8. Prímszámok

207

2. bizonyítás. (Euler) Tegyük fel ismét, hogy hamis az állítás; a 2, 3, . . . , p prímeken kívül nincs több prím. Ekkor az 1 1 1 1 + + + + ··· = 2 3 4       1 1 1 1 1 1 1 1 1 = 1+ + 2 + 3 +. . . · 1+ + 2 + 3 +. . . · . . . · 1+ + 2 + 3 +. . . = 2 2 2 3 3 3 p p p 1 1 1 = · · ...· 1 − 12 1 − 13 1 − p1 azonosságban a bal oldalon álló kifejezés értéke a végtelenhez tart, míg a jobb oldal véges. Ellentmondásra jutottunk, tehát a feltevés hibás. 3. bizonyítás. Tegyük fel, hogy a 2, 3, 5, . . . , p prímeken kívül nincs több prím. Ekkor az 1 1 1 1 + 2 + 2 + 2 + ··· = 3 4 2    1 1 1 1 1 1 = 1 + 2 + 4 + 6 + ... · 1 + 2 + 4 + 6 + ... · ... 2 2 2 3 3 3   1 1 1 1 1 1 ...· 1 + 2 + 4 + 6 + ... = · ·...· p p p 1 − 212 1 − 312 1 − p12 π2 azonosságban a bal oldalon irracionális szám áll (= , lásd pl.: Skljarszkij– 6 Csencov–Jaglom: Aritmetika és algebra (Tankönyvkiadó, Budapest, 1967) c. könyvben a 233. feladatot!), míg a jobb oldalon racionális szám. Az ellentmondás oka a hibás feltevés. 4. bizonyítás. Ismét indirekt úton bizonyítunk. Ha k db prímszám van, akkor a belőlük képezhető p1α1 · p2α2 · . . . · pkαk alakú természetes számok száma, ahol a kitevők nem nagyobbak n-nél: (n + 1)k . E számok között persze szerepelnie kell az 1, 2, 3, . . . , 2n számok mindegyikének, tehát (n + 1)k > 2n lenne, de ez az egyenlőtlenség nem teljesül elegendően nagy n-re. 5. bizonyítás. Tegyük fel, hogy csak k db különböző prímszám van: p1 , p2 , . . . , pk . Legyen n = p1 · p2 · . . . · pk . Ekkor az egyedüli, n-hez relatív prím szám az 1, tehát φ(n) = 1, azaz φ(n) = (p1 − 1) · (p2 − 1) · . . . · (pk − 1) = 1, de ez az egyenlőség nem teljesülhet. (Az 1, 2, . . . , n−1számok közül az n-hez relatív prím számok számát az Euler 1 féle φ(n) = n · 1− függvény adja meg.) p p|n 6. bizonyítás. Ha tudunk olyan végtelen sorozatot konstruálni, melynek elemei páronként relatív prímek; abból már következik, hogy végtelen sok prím van. 1. konstrukció: Legyenek a és b egymáshoz relatív prím természetes számok. A sorozat a következő: a0 = a, an+1 = a0 · a1 · . . . · an + b, n = 0, 1, 2, . . . Lássuk be, hogy a sorozat elemei páronként relatív prímek. Legyen d egy közös

Megoldások, útmutatások

208

prímosztója ai -nek és aj -nek, ahol i < j . Az aj = a0 · a1 · . . . · ai · . . . · aj −1 + b előállítás miatt d | b. Mivel ai = a0 · a1 · · . . . · ai−1 + b és d | ai , d | b, így d | a0 · a1 · . . . · ai−1 , tehát d osztója a sorozat valamely, ai -t megelőző elemének is. Ezt folytatva kapjuk, hogy d osztója a-nak is, b-nek is; ami ellentmond a kezdeti feltevésnek. n 2. konstrukció: Nézzük a Fermat-féle számok sorozatát: Fn = 22 + 1; n = 0, n 1, 2, . . . A sorozat elemei páronként relatív prímek; ugyanis, ha d | 22 + 1 és n−m   m m m 2 n d | 22 + 1, m < n, akkor a 22 + 1 | 22 − 1 = 22 − 1 oszthatóság és az  2n   2n  2n előbbi oszthatóság miatt d | 2 − 1, így d | 2 + 1 − 2 − 1 = 2, s mert d páratlan, ezért d = 1. 7. bizonyítás. Felhasználjuk a Fibonacci-sorozatra vonatkozó Lucas tételt (D. E. Knuth: A számítógép-programozás művészete 1. (Műszaki Könyvkiadó, Budapest, 1987) c. könyv 101–102. oldalak). Jelölje fn a Fibonacci-sorozat n-edik elemét, akkor (fm , fn ) = f(m,n) . Ha csak véges sok prím van: p1 , p2 , . . . , pk , akkor Lucas tétele szerint az fp1 , fp2 , . . . , fpk elemek páronként relatív prímek, s mivel a felsoroltakon kívül más prím nincs, így mindegyik fpi -nek csak egy prímosztója lehet. Ennek mond ellent, hogy f19 = 4181 = 113 · 37. Megjegyzés. A prímek számának végtelenségére egyéb bizonyítások is ismertek.

9. Oszthatósági feladatok 369–370. Használjuk az oszthatósági szabályokat! 371. a) 119944.

b) 519948.

372. Mindkettő hamis, pl. 24, illetve 8 · 9. 373. a) A szám osztható 5-tel. b) A szám osztható 3-mal. c) d–e) f) g) h) i) j)

A szám osztható 3-mal. A szám a 3 + b3 alakú. A szám osztható 9-cel. A szám osztható 11-gyel. 111 . . . 121 . . . 111 = 111 . . . 110 . . . 000 + 11 . . . 111 = 11 . . . 111 · 100 . . . 001. Vizsgáljuk a szám tízes számrendszerben felírt alakját. A szám osztható 3-mal.

374. 347 777 743 = 333 333 300 + 11 111 110 + 3 333 333, azaz a szám osztható 1 111 111-gyel.

9. Oszthatósági feladatok

209

375. 49 + 610 + 320 = (29 + 310 )2 . 376. 210 + 512 = (25 + 56 )2 − 26 · 56 = (25 + 56 − 23 · 53 ) · (25 + 56 + 23 · 53 ) = = 14 657 · 16 657. Megjegyzés. 14 657 és 16 657 prímszámok. 377. Vegyük észre, hogy 53 + 96 = 83 + 66 = 109 + 40 = 149. Vezessük be a következő jelöléseket: A = 53, B = 83, C = 109, x = 149. Ekkor 53 · 83 · 109 + 40 · 66 · 96 = ABC + (x − A)(x − B)(x − C) = = x(x 2 − [A + B + C]x + [AB + BC + CA]). Azaz a megadott szám osztható 149-cel (s ez a szám nagyobb 149-nél), tehát összetett szám. 378. Legyen x = 991. Ekkor 989 · 1001 · 1007 + 320 = (x − 2)(x + 10)(x + 16) + 320 = x 3 + 24x 2 + 108x = = x(x + 6)(x + 18) = 991 · 997 · 1009. 379. Legyen x = 512, y = 675, z = 720. Mivel 2z2 = 3xy, így x 3 + y 3 + z3 = x 3 + y 3 − z3 + 3xyz = (x + y − z)(x 2 + y 2 + z2 − xy + xz + yz), ezért x 3 + y 3 + z3 = 5123 + 6753 + 7203 osztható x + y − z = 467-tel. Pontosabban: 5123 + 6753 + 7203 = 229 · 467 · 7621. 380. 77a = 34a + 43a = 43b + 43a = 43(a + b). Ezért 43(a + b) osztható 77-tel. Mivel (43, 77) = 1, emiatt a + b osztható 77-tel, és 77 összetett szám, így az a + b szám összetett szám. 381. Az m = n + 2, m = n − 2, m = n2 értékek bármelyikét választhatjuk. 382. Ha n páros, akkor a szám 3-mal osztható; ha n = 4k + 1 alakú, akkor 13-mal; ha n = 4k + 3, akkor 5-tel osztható a szám. 383. A nemnegatív egészek körében 0, a pozitív egészek körében 2520 a válasz. 384. Jelölje a középső számot n, ekkor a kilenc szám összege 9n. 385. A számok összege páratlan. 386. A szorzat osztható 10-zel. 387. A szorzat osztható 5-tel. 388. A négy szám között nincs 5-tel osztható. Így a lehetséges megoldások: 1, 2, 3, 4 vagy 6, 7, 8, 9 vagy 11, 12, 13, 14 stb. Az első négy szám szorzata kicsi, a harmadik számnégyes szorzata nagy, csak a 6, 7, 8, 9 számok lehetnek megfelelők, és csakugyan meg is felelnek. 389. 195, 285, 375.

210

Megoldások, útmutatások

390. A keresett szám osztható 5-tel és 9-cel. A válasz: 8 888 888 880. 391. Határozzuk meg, hogy mivel kell megszorozni ehhez az 1991-es számot. A szorzó utolsó jegye nyilván 2. A szorzást felírva, visszafelé haladva meg tudjuk állapítani a szorzó számjegyeit. A megoldás: 18 141 992. 392. 111 = 3 · 37. 393. Ha a törtek mindegyike egész, akkor n + 1 és n + 8 is osztható 3-mal, de ez nem lehet egyszerre (n + 8) − (n + 1) = 7 miatt. 394. a) 1 + 2 + · · · + 10 = 55, s ez páratlan. b) Az egyik szorzat osztható 7-tel, a másik nem. 395. Az egyik szorzat osztható 5-tel, a másik nem. 396. Ha a számok között van 7-tel osztható, akkor az egyik szorzat osztható lesz 7-tel, a másik nem. Ha a számok között nincs 7-tel osztható, akkor a hat szám szorzata nem lehet négyzetszám (mivel 7-tel osztva 6 maradékot ad), noha ez szükséges lenne ahhoz, hogy a két csoportban a számok szorzata egyenlő legyen. 397. 654 312. 398. 102 000 564. 399. 123654 vagy 321654. 400. 381 654 729. 401. 123 475 869. 402. Azt a legkisebb pozitív egész A számot keressük, amelyre a B = 999A = = 1000A−A számban nincs 9-es számjegy. A B szám mindkét alakjából látható, hogy A utolsó jegye nem lehet 1, hiszen ekkor B utolsó jegye volna 9. Nem lehet 0 sem A utolsó jegye, hiszen ekkor B tizedrésze is megfelelő volna. A B szám második alakjából látható, hogy emiatt A utolsó előtti jegye sem lehet 0, hiszen ekkor az 1000A − A kivonást szokás szerint számjegyenként végezve A többi jegyétől függetlenül B utolsó előtti jegyére 9-est kapunk. Ámde ekkor A százas helyiértékű jegye sem lehet 0, mert akkor B-ben a százas helyiértékű számjegy volna 9-es. Ha tehát A-ban minden számjegyet a lehető legkisebbnek választunk, akkor a 112-es számot kapjuk, mellette B = 111888 nem tartalmaz 9-est, tehát ez a keresett szám. 403. 7 639 128. (Ilyen tulajdonságú 8-jegyű szám nem létezik.) 404. Mivel a szám 9-re végződik, a páros számjegyeket és az 5-öt elhagyhatjuk. A keresett számban csak az 1, 3, 7, 9 számjegyek szerepelhetnek. A szám abc9 alakú, és 9 | a + b + c. Ezért a + b + c értéke 9, 18 vagy 27. Az ilyen tulajdonságú számhármasok a 333, 117, 171, 711, 999. Az elvárt feltételeknek csak a 3339, 7119 és 9999 felelnek meg.

9. Oszthatósági feladatok

211

405. A keresett szám: 8 757 193 191. 406. m = abcde = 45abcde. Ha m-nek van páros számjegye, akkor a szorzat osztható 10-zel, ezért e = 0, ám ebből m = 0 adódik. Tehát minden számjegy páratlan. e = 5, m osztható 25-tel, d = 7. m osztható 9-cel, így a + b + c + 12 osztható 9-cel, a + b + c = 15. Továbbá 45 · 5 · 7 · abc < 100 000, ezért abc ≤ 63. Ezeket kielégítő megoldások a páratlan számjegyek körében: 1, 7, 7, ill. 1, 5, 9. A feladat megoldását közöttük találjuk: 77 175. 407. Ha a 7-esek száma a, a 3-asok száma b, akkor a számjegyek összege 7a + 3b. Ez akkor osztható 7-tel és 3-mal, ha 3 | a és 7 | b. A legkisebb számot keresve először az a = 3, b = 7 esetet kell vizsgálnunk. A szóba jöhető legkisebb számok: 3 333 333 777, 3 333 337 773, 3 333 377 733, . . . Közülük az első kettő nem osztható 7-tel. A keresett szám: 3 333 377 733. 408. 7 | 1001, 7 | 222 222 és 7 | 119, ezért 7 | 11 900 000 + 222 222 = 12 122 222. (Egy másik szám: 22 122 212.) 409. Legyen a keresett szám 100B + 28. Nyilván 100B is osztható 28-cal, azaz B osztható 7-tel; továbbá B számjegyeinek összege 28 − (2 + 8) = 18 lévén, B osztható 9-cel is, tehát 63 többszöröse: 63, 126, 189, . . . Látható, hogy a legkisebb olyan szám, amelyben a számjegyek összege 18, a 189. A keresett szám: 189 · 100 + 28 = 18 928. 410. A számjegyek összege 56, így az 56 előtt álló számjegyek összege 45. A 9 999 956 szám nem osztható 56-tal, így a számban 56 előtt legalább 6 számjegy áll. Egy 56-tal osztható szám 8-cal is osztható, ezért az 56 előtt álló számjegy páros. Az utóbbiak alapján felírható legkisebb szám 19 999 856 nem osztható 56tal. Ezért az első jegy legalább 2. Így felírhatjuk a 28 999 856 számot, ám ez sem osztható 56-tal. A következőként felírható 29 899 856 szám lesz a keresett szám. 411. Ilyen számok: 166667 és 333334. 412. Vizsgáljunk egyszerűbb eseteket: 5-jegyű számok helyett 1-, 2-, 3-, 4-jegyű számokat. Két 1-jegyű számot könnyű megadni: 1 · 1 = 1, 1 · 3 = 3, . . . Keressünk két 2-jegyű számot! 11 · 11 = 121, 11 · 13 = 143, 13 · 13 = 169, . . . , 13 · 15 = 195. Az utolsóként felírt eset megoldást jelent. Ezt az eredményt használjuk fel a 3-jegyű számok kereséséhez: 133 · 135 = 17 955. Biztatóan alakul, látszik milyen alakú számokat keressünk: 1333 · 1335 = 1 779 555, 13333 · 13335 = 177 795 555. Az utolsóként felírt szorzás megoldást jelent a feladat kérdésére. 413. 1944 = 23 · 35 .

Megoldások, útmutatások

212

414. Legfeljebb 99 ilyen szám van. Például 10001, 10002, . . . , 10099. Ezek mindegyike 100 · 100 és 100 · 101 között van. 100 egymást követő felbonthatatlan szám nincsen, mert 100 egymást követő szám között van 100-zal osztható szám. 415. Legyen a keresett közös osztó d. Ez a szám mind a 49 számnak osztója, így osztója összegüknek, 999 = 33 · 37 = 27 · 37-nek. A számok mindegyike ≥ d, így 999 49d ≤ 999, azaz d ≤ < 21. A 999 legnagyobb osztója, mely kisebb 21-nél: 49 9, azaz a közös osztó legfeljebb 9 lehet. Van olyan eset, amikor 9 a legnagyobb közös osztó; ha a 49 szám közül 48 szám mindegyike 9 és a 49. szám az 567. A számok közös osztójának lehetséges legnagyobb értéke: 9. 416. Legyen a keresett közös osztó d. Ez a szám mind a 10 számnak osztója, így osztója összegüknek, 1001 = 7 · 11 · 13-nak. A számok mindegyike ≥ d, így 1001 < 101. Az 1001 legnagyobb osztója, mely kisebb 10d ≤ 1001, azaz d ≤ 10 101-nél: 91, azaz a közös osztó legfeljebb 91 lehet. Van olyan eset, amikor 91 a legnagyobb közös osztó; ha a 10 szám közül 9 szám mindegyike 91 és a tizedik szám a 182. A számok közös osztójának lehetséges legnagyobb értéke: 91. 417. Az összeg 0-ra végződik. 418. 1110 − 1 = (11 − 1)(119 + 118 + 117 + · · · + 11 + 1), a szorzat mindkét tényezője osztható 10-zel. 419. 1993 − 199 = 199(1992 − 1) = 199(199 − 1)(199 + 1) = 199 · 198 · 200. 420–430. feladatokban használjuk az a − b | a n − bn , a + b | a 2n − b2n , a + b | a 2n+1 + b2n+1 oszthatósági szabályokat. Például:  35  35 13 = 22 + 32 | 22 + 32 = 270 + 370 ; 18 | 1719 + 119 és 18 | 1917 − 117 , tehát 18 | (1719 + 1) + (1917 − 1), azaz 18 | 1719 + 1917 ; 31974 + 51974 = (31974 + 21974 ) + (51974 − 21974 ), és 32 + 22 | 31974 + 21974 , 53 − 23 | 51974 − 21974 , tehát mindegyik osztható 13-mal (32 + 22 = 13, 53 − 23 = 9 · 13), így összegük is. 431–435. feladatokat teljes indukcióval igazoljuk. 436.

2002 · 2003 · 2004 · . . . · 4002 = = (4003 − 2001) · (4003 − 2000) · (4003 − 1999) · . . . · (4003 − 1) = = 4003 · A − 1 · 2 · 3 · . . . · 2000 · 2001, ahol A valamely egész szám.

440. Legyen x = 1992, ekkor 19921993 −1992 = x(x 1992 −1), 19922 +1993 = x 2 +x +1. Mivel x 1992 − 1 osztható x 3 − 1 = (x − 1)(x 2 + x + 1)-gyel, ezért x(x 1992 − 1) osztható (x 2 + x + 1)-gyel.

9. Oszthatósági feladatok

213

441–442. feladatok megoldásánál használjuk a 420–430. feladatoknál alkalmazott öszszefüggéseket! 443. Legyen d = (2m − 1, 2n + 1). d | 2m − 1 | (2m )n − 1 = 2mn − 1 és d | 2n + 1 | (2n )m + 1 = 2mn + 1. Ezekből d | (2mn + 1) − (2mn − 1) = 2, azonban d páratlan, d = 1. 444. Lásd a prímszámok számának végtelenségére adott 6. bizonyítást (368. feladat)! 2n − 2 , akkor 2n − 1 = kn + 1. n n 22 −1 − 2 2kn+1 − 2 2(2kn − 1) (2n )k − 1 = = = 2 · . 2n − 1 2n − 1 2n − 1 2n − 1

446. Legyen k =

4(2p−1 + 1)(2p−1 − 1) , akkor 4 | n − 1 és a kis-Fermat tétel miatt 3 p | n − 1, azaz 2p | n − 1. De akkor 22p − 1 | 2n−1 − 1, így 22p − 1 | 2n − 2. A feladat feltételéből n | 22p − 1, ezért n | 2n − 2.

447. n − 1 =

449. a) (n + 3)(n + 4) < n2 + 8n + 15 < (n + 4)(n + 4). b) n2 + 3n + 5 = (n + 7)(n − 4) + 33. Ahhoz, hogy 121 | n2 + 3n + 5 legyen, (n + 7)(n − 4)-nek 121A + 88 alakúnak kell lennie. Tehát (n + 7) vagy (n − 4) osztható 11-gyel. De az első tag két tényezője egyszerre osztható vagy nem osztható 11-gyel, így (n + 7)(n − 4) osztható 121-gyel. 450. x 5 + 3x 4 y − 5x 3 y 2 − 15x 2 y 3 + 4xy 4 + 12y 5 = (x − 2y)(x − y)(x + y)(x + 2y)(x + 3y), s a 33 nem lehet öt különböző egész szorzata. 451. 19 + 81 | 1989 + 8189 , tehát a szám utolsó két jegye: 00. 452. Nem, mert pl. 1899 = 9 · 211 nem osztható 27-tel. 453. Figyeljük az a, b, c számok 3-as osztási maradékát. 454. A szám (1072 + 1063 + · · · + 109 + 1) · 111 111 111 alakban írható és mind a két tényező osztható 9-cel. 455. 1980 = 99·20. A szám 20-szal és 99-cel is osztható, mert 100 = 99+1 és 19+20+ +21+· · ·+80 = 99·31. Ugyanis, ha a felírt számot 100-as alapú számrendszerben felírt számnak tekintjük, akkor osztható 99-cel, ha a szám jegyeinek összege osztható 99-cel. (Természetesen bizonyíthatunk úgy is, hogy megmutatjuk a 9cel és 11-gyel való oszthatóságot.) 457. Közülük végtelen sok osztható 3-mal. Igaz az is, hogy közöttük végtelen sok 37-tel osztható van, mert 555 = 5 · 111 = 5 · 3 · 37, tehát ha a felírt számok valamelyikében az 5-ösök száma 3-mal osztható, akkor a szám osztható 37-tel. 458. Hasonló az előző feladathoz. Ezek között is végtelen sok 3-mal és végtelen sok 13-mal osztható van, mert 149 149 = 1001 · 149 = 13 · 11 · 7 · 149.

Megoldások, útmutatások

214

459. Mivel 333 333 = 333·1001 = (3·3·37)·(7·11·13) osztható 7-tel, azért 3 333 337 = = 10 · 333 333 + 7 is osztható 7-tel. Általában a sorozatnak azok a tagjai, melyek alakja 333 333 333

 333 . . . 333

 333 7, szintén oszthatók 7-tel. 6 db

6 db

6 db

460. Tekintsük a következő számokat: a1 = 3, a2 = 33, . . . , a32 = 33 . . . 3 (32 db 3-as). Mivel 31-gyel való osztáskor összesen 31 különböző maradék lehetséges, és nekünk 32 db számunk van; akad közöttük kettő, ai és aj (i < j ), melyek különbsége osztható 31-gyel. Ámde, ha 31 osztója az aj − ai = aj −i · 10j −i számnak, akkor 31 osztója aj −i -nek is, hiszen 31 prímszám, és a szorzat második tényezője, 10j −i nem osztható 31-gyel. Ez pedig azt jelenti, hogy a j −i = k darab hármasból álló szám oszthatók 31-gyel. Általában a sorozatnak azok a tagjai, melyek alakja 33 . . . 33 33 . . 33 . . . 33 . . 33 31, szintén oszthatók 31-gyel.

.

. k db

k db

k db

462. abcabc = abc000 + abc = 1000 · abc + abc = 1001 · abc = 7 · 11 · 13 · abc. 463. bca = 10 · abc − 999 · a = 10 · abc − 27 · 37 · a. 464. Jelöljön egy 1989-jegyű számot A, a szám utolsó jegyét a. Az „elcsúsztatott” szám: 10 · A − (101989 − 1) · a, s tudjuk, hogy 27 | A és  663 − 1 = 101989 − 1. 27 | 103 − 1 | 103 465. bcdea = 10abcde + a − 100000a = 10abcde − 99999a. Mivel abcde is, 99999 is osztható 41-gyel, ezért bcdea is osztható 41-gyel. 466. A számot jelölje A, utolsó jegyét a. A számjegy áthelyezésével kapott szám legyen B. Ekkor 10 · B − 999 999 · a = A, azaz 10 · B = 999 999 · a + A = 999 · 1001 · a + A = 999 · 7 · 11 · 13 · a + A. 467. Hasonló az előző négy feladathoz. 468. a − 6b = (5a − 4b) − 2(2a + b). 469. 4(100a + b) = 399a + (a + 4b) és 7 | 399. 470. 5(10a + b) = 49a + 7b + (a − 2b). 471. 4(3a + 4b) = 11(a + b) + (a + 5b). n + 11 n − 9 + 20 20 = =1+ , n−9 n−9 n−9 3n + 5 3(n + 3) − 4 4 b) = =3− , n+3 n+3 n+3 n2 + 1 n2 − 1 2 2 c) = + = (n − 1) + , n+1 n+1 n+1 n+1 d) A c) feladat megoldásához hasonló.

473. a)

10. Különféle számelméleti feladatok

215

474. Ha (n + 11)-nek és (n − 9)-nek van közös osztója, akkor az osztója (n + 11) − − (n − 9) = 20-nak is. Tehát, ha egyszerűsíthető a tört egy egész számmal, akkor az a szám csak a 20 osztói közül kerülhet ki. 475. b) Ha (12n + 1)-nek és (30n + 2)-nek van közös osztója, akkor az osztója 5(12n + + 1) − 2(30n + 2) = 1-nek is. c) 3(14n + 3) − 2(21n + 4) = 1. 476. b) Mivel 7(8n + 3) − 8(7n + 1) = 13, a közös osztó csak 13 lehet. 478. A legkisebb ilyen szám: 153 846. 479. A legkisebb ilyen szám: 105 263 157 894 736 842. 480. 1 034 482 758 620 689 655 172 413 793. A szám számjegyei az utolsó jegytől kezdve egymás után meghatározhatók. Írjuk fel a megadott szorzást. A 3-ra végződő számot 3-mal szorozva a kapott szorzat utolsó jegye 9, ezért a feladatban leírt feltétel miatt a szorzandó utolsó előtti jegye is 9 . . . A felírt 28-jegyű szám a legkisebb a megadott tulajdonságúak között. Az összes többit úgy kaphatjuk meg, hogy ezt az „alapszámot” tetszőlegesen sokszor egymás után írjuk. 481. Hasonló az előző feladatokhoz. 482. Ha az n szám elé az a számjegyet írjuk, akkor a feladat követelménye: a · 10k + + A = 58 · n, azaz a · 10k = 57 · n = 3 · 19 · n, s ez nem lehet, mert a bal oldal nem osztható 19-cel, tehát nincs ilyen szám. A második kérdésre már igen a válasz: 57 · 125 = 7125.

10. Különféle számelméleti feladatok 483. Három egymást követő egész szám összege és szorzata is osztható 3-mal. 484. (n − 1)3 < (n − 1)n(n + 1) = n(n2 − 1) = n3 − n < n3 . 485. Ha egy páros szám hatványszám, akkor a törzstényezős felbontásában 2-nek legalább 2 a kitevője, így osztható 4-gyel. Azonban négy egymást követő egész szám közül az egyik páros szám nem osztható 4-gyel. 486. a) Van közöttük három páros szám és egy 3-mal osztható páratlan szám. b) A számok között van olyan, mely nem osztható a 2, 3, 5 számok egyikével sem. 487. Tekintsük azt a számot, amelyik osztható 12-vel.

Megoldások, útmutatások

216

488. 18, 19, 20, 21, 22 és 9, 10, 11, 12, 13, 14, 15, 16 megoldások. Ezekből kapjuk a −17, −16, . . . , 17, 18, 19, 20, 21, 22 és a −8, −7, . . . , 8, 9, . . . , 15, 16 megoldásokat. 490. A szám mindig osztható 11 111-gyel, hiszen 105 = 9·11 111+1, ezért a felírt szám 11 111-gyel osztva annyi maradékot ad, mint az egymás után helyezett ötjegyű számok összege. 491. Nem, ugyanis az (a + 1) + (a + 2) + · · · + (a + k) = 2100 , azaz a k(2a + k + + 1) = 2101 egyenlőség azt jelentené, hogy 2101 -nek van 1-nél nagyobb páratlan osztója. (Negatív számokat is megengedve van megoldás.) 492. Nem. A végződés csak 2222, 4444, 6666, 8888 lehetne, azonban az ilyen számok egyike sem osztható 16-tal. Megjegyzés. 239 = 549 755 813 888. 493. 210 = 1024 > 1000 = 103 , ezért 2100 = (210 )10 > (103 )10 = 1030 . 213 = 8192 < 10 000 = 104 , ezért 2100 = 291 · 29 = (213 )7 · 29 < (104 )7 · 512 < < 1028 · 600 = 6 · 1030 . Ezek alapján 1030 < 2100 < 6 · 1030 . Ez azt jelenti, hogy a 2100 szám tízes számrendszerben felírt alakja 31 jegyű. 494. Ha a 21996 szám m-jegyű, akkor 10m−1 < 21996 < 10m . Ha az 51996 szám njegyű, akkor 10n−1 < 51996 < 10n . A két számban összesen m + n számjegy van. Szorozzuk össze a két egyenlőtlenséget! 10m−1 · 10n−1 < 21996 · 51996 < 10m · 10n . 10m+n−2 < 101996 < 10m+n , azaz m + n − 2 < 1996 < m + n, m + n = 1997. A két számban összesen 1997 számjegy van. 495. 25, 76. 496. 376, 625. 497. Mivel 1 + 2 + · · · + 13 = 91 és az 1, . . . , 13 számok közül bármelyiket helyettesítenénk egy nála több, mint 1-gyel nagyobb számmal, az összeg 92-nél nagyobb lenne. Így a keresett számok: 1, . . . , 12, 14. 498. 76 543 210. 499. 16-tal, ill. 17-tel osztható. 500. Ha egy szám nem osztható a-val, akkor persze 2a-val sem osztható. Tehát a hamis állítás nem vonatkozhatott a 2, 3, 4, 5, 6 számokkal való oszthatóságra. Ha hamis a 10-zel való oszthatóság, akkor a 2-vel és az 5-tel való oszthatóság állítása közül legalább az egyik hamis. Hasonlóan, ha hamis a 12-vel való oszthatóság, akkor a 4-gyel és a 3-mal való oszthatóság egyike is biztosan hamis. A két egymás után elhangzó hamis állítás egyike a 8-ra vonatkozott, ugyanis a többi, 2 és 13 közötti páros számot már kizártuk. A másik hamis állítás vagy a 9-re, vagy a 7-re vonatkozik. Ha a téves állítás 8-ra és 9-re vonatkozott, akkor a

10. Különféle számelméleti feladatok

217

felírt szám osztható a 2, 3, 4, 5, 6, 7, 10, 11, 12, 13 számokkal. Ezek legkisebb közös többszöröse: 3 · 4 · 5 · 7 · 11 · 13 = 60 060, ami nagyobb 50 000-nél, ezért nem ez a megoldás. Egy lehetőség marad: a felírt szám 50 000-nél kisebb, 7-tel és 8-cal nem osztható, de osztható a 2, 3, 4, 5, 6, 9, 10, 11, 12, 13 számokkal. E számok legkisebb közös többszöröse: 4 · 5 · 9 · 11 · 13 = 25 740. Mivel 2 · 25 740 > 50 000, ezért a felírt szám csak a 25 740 lehet. 501. A sorozat elemeinek szorzata 8, így a 0 nem eleme a sorozatnak. Ha a sorozat elb 1 1 a ső két eleme a és b, akkor felírhatók a sorozat elemei: a, b, , , , , a, b, . . . a a b b Tehát a sorozat első hat eleme periodikusan ismétlődik. Egy-egy periódusban a hat elem szorzata 1. b 1 b2 Az első 40 elem szorzata: a ·b · · = = 8, az első 80 elem szorzata: a ·b = 8. a a a 3 Ezt a két egyenletet összeszorozva: b = 64, azaz b = 4, a = 2. 502. Vegyük észre, hogy a sorozat 7-tel osztható tagjainak indexe osztható 8-cal; a sorozat 3-mal osztható tagjainak indexe pedig osztható 4-gyel. Ezért a sorozat 7-tel osztható tagjai 3-mal is oszthatók. 503. Az állítással ellentétben tegyük fel, hogy mindkét sorozat periodikus; akkor van közös periódushossz. Mivel an = bn (1 + cn ), így azt kapjuk, hogy az an sorozat is periodikus. 504. an+1 − 1 = an (an − 1) = an an−1 (an−1 − 1) = · · · = an an−1 . . . a1 (a1 − 1) = = an an−1 . . . a1 . 505. abcd − dcba = 999(a − d) + 90(b − c) = 1008, azaz 111(a − d) + 10(b − c) = 112, tehát a − d = 1, 10(b − c) = 1, s ez utóbbinak nincs megoldása. 506. ab = 10a + b > 9a ≥ a · b. 507. A szép számok: 6, 8, 10, 14, 15, 21, 22, 26, 27, 33, . . .  3  3 508. 1050 < 10150 + 5 · 1050 + 1 < 1050 + 1 . Indokolható azzal is, hogy köbszám 9-cel osztva nem adhat 7 maradékot. 511. A számok összege egy számmal osztva annyi maradékot ad, mint az összeadandók osztási maradékainak összege, 3-mmal való osztás esetén ez pedig megegyezik a számjegyek összegével. Azt kapjuk, hogy ez az összeg 3-mal osztva 0 maradékot ad, ezért az összeg nem lehet 100. 512. Ha egy számból levonjuk jegyei összegét, 9-cel osztható számot kapunk. Ezért az első elvétel után 9-cel osztható számok követik egymást. Legutoljára 9 aranyból vettek. Visszafelé követve, az aranyak száma ebben a sorrendben rendre: 9, 18, 27, 36, 45, 54, 63, 72, 81, (vigyázat! a 90 nem szerepelhet) 99, 108, 117, 126, 135. Innen leolvasható, hogy a főnök ötödik a névsorban, s neki 27 arany jutott a zsákmányból.

218

Megoldások, útmutatások

513. Az 1, 2, 3, . . . , 16 számokból ki kell hagyni négyet. Figyeljünk arra, hogy mindkét szorzatban minden prímnek ugyanolyan kitevővel kell szerepelnie! 11 és 13 elhagyása egyértelmű, továbbá a 2 és az 5 prímekkel osztható számokból is el kell hagyni. Mivel a megmaradó 12 szám összege páros, ezért a 11 és 13 számok mellé vagy két páratlan, vagy két páros számot kell választani. Páros számot mindenképp kell választani, ezért a másik két szám csak a 10 és a 4 lehet (a feladat kikötése miatt a 4 helyett a 16 nem hagyható el). A megmaradó 12 szám összege 98, tehát egy-egy padon az életkorok összege 49 év. A 16 mellé a 2, 6, 14 számokból kell kettőt választani (így teljesülhet az, hogy a 2-es tényezők száma mindkét szorzatban ugyanannyi, és a hat szám összege páratlan). Ezeket a lehetőségeket megvizsgálva két megoldást találunk: (16, 15, 7, 6, 3, 2), (16, 14, 9, 5, 3, 2). 514. A legkisebb szám az, amellyel bármely két szám összege osztható, és ennek a számnak többszöröse a többi szám. Ezen szám ismeretében akkor található ki mind az öt szám, ha 90-ig csak öt olyan szám van, mely osztható ezzel a számmal. Három ilyen szám van: 16, 17, 18. Ezek közül akkor egyértelmű a választás, ha a szám páratlan, azaz 17. A nyerő számok: 17, 34, 51, 68, 85. 515. Haladjunk sorban a kérdésekkel és a lehetséges válaszokkal! 1. „Nagyobb, mint 500?” Ha igennel válaszol: 501–1300 közötti szám (valójában — hiszen a válasz hamis — 13–500 közti szám). 2. „Négyzetszám?” Ha igennel válaszol: 16, 25, 36, 49, 64, 81, 100, 121, 144, 169, 196, 225, 256, 289, 324, 361, 400, 441, 484, 529, 576, 625, 676, 729, 784, 841, 900, 961, 1024, 1089, 1156, 1225, 1296 (valójában az összes többi szám). 3. „Köbszám?” Ha igen a válasz: 27, 64, 125, 216, 343, 512, 729, 1000. 4. „Az utolsó előtti számjegye 1-es?” Az erre adott válasz után kitalálható a házszám. Tekintsük át, hogy az eddigi kérdésekre kapható válaszok után milyen információink vannak! (A kérdésekre kapható igen és nem válaszokat I és N jelöli.) I I I : 729. NI I : 64. I NI : 512, 1000. NNI : 27, 125, 216, 343. I I N: 529, 576, 625, 676, . . . (túl sok megoldás). NI N: 16, 25, 36, 49, . . . (túl sok megoldás). I NN: túl sok megoldás. NNN: túl sok megoldás. Ezek alapján biztos, hogy Horváth az I NI válaszsort kapta kérdéseire, mert csak ez ad kétesélyes lehetőséget (amelyben az egyik szám utolsó előtti jegye 1-es). Mivel azonban Kovács hazudott, ennek ellentéte: NI I adja a helyes válaszokat, eszerint a házszám 64.

10. Különféle számelméleti feladatok

219

516. 1664 = 128 · 13 = 27 · 13. A gyerekek között van 13 vagy 26 éves. Ha az egyik gyerek 26 éves, akkor lehetne még 16, ill. 32 éves korú gyermeke az anyának (apának), de ekkor a szorzat nem lehet megfelelő. Az egyik gyerek 13 éves, két testvére van; egyik 8, másik 16 éves. 3 gyerek van a családban. 517. Anna nem tudta meghatározni a gyerekek életkorát. Ez azt jelenti, hogy szorzatukat többféleképpen is fel lehet bontani két 10-nél kisebb természetes szám szorzatára. Ez a következő lehetőségeket jelenti. (Egymás alá írtuk az egyforma szorzatokat adó párokat.) 1, 4 1, 6 1, 8 1, 9 2, 6 2, 8 2, 9 3, 8 4, 9 2, 2 2, 3 2, 4 3, 3 3, 4 4, 4 3, 6 4, 6 6, 6 Bori tudhatja, hogy az életkorokat a felsorolt számpárok valamelyike adja meg. Tudjuk, hogy a korkülönbség ismerete egyértelművé teszi a választást. A 18 számpár között csak a (6, 2), (8, 2), (9, 1) párok azok, melyek különbsége egyszer fordul elő. Ezek közül kell választani. Mivel az életkorok hányadosának ismeretében nem egyértelmű a választás, ezért 6 és 2, ill. 9 és 1 évesek nem lehetnek a gyerekek, mert itt a hányados ismeretében ki lehet választani a számpárt. A gyerekek csak 8 és 2 évesek lehetnek. 518. A 2450-et szorzatokra bontjuk. A három személy életkorára több lehetőség kínálkozik, mindegyikhez más összeg tartozik. Mivel a sekrestyés ismeri a saját életkorát, meg kellene találnia a helyes választ. Ez mégsem sikerül, amiből arra következtetünk, hogy több lehetőség áll előttünk. Megvizsgáljuk, hogy a három életkor összege milyen kombinációkban fordul elő; és azt tapasztaljuk, hogy csak a 64-es szám jelenik meg kétszer: 49 + 10 + 5 = 64, 50 + 7 + 7 = 64. Tehát a sekrestyés 32 éves. Amikor a pap megjegyzi, hogy az egyik személy idősebb nála, akkor e kijelentés alapján a sekrestyés választani tud a két lehetőség közül. Tehát a pap 49 éves. 519. Három olyan számot keresünk, melyek szorzata 225, összege egy ismeretlen szám; melyet az összeíró tudott, s még mindig kérdeznie kellett. A 225-nek csak két olyan felbontása van, melynél a 3–3 szám összege ugyanaz: 1, 15, 15, és 3, 3, 25. Ez utóbbi adja a megoldást. 520. Soroljuk fel azokból a pozitív egészekből álló számhármasokat, melyeknél a három szám szorzata 36. Ismerjük még a három szám összegét is, ám ebből nem állapítható meg egyértelműen, hogy melyik számhármasról van szó. Azaz, a felírt számhármasok közül válasszuk ki azokat, melyek összege másik számhármasnál is ugyanaz. Szerencsére csak az 1, 6, 6 és a 2, 2, 9 számhármas ilyen. Tehát a három gyerek életkorát ezek egyike adja meg. A választást a két idősebb gyerekre történő utalás segíti. Az életkorok 1, 6, 6 év. A beszélgetés a hónap 13. napján (1+6+6=13) történt.

220

Megoldások, útmutatások

521. András a különbség ismeretében akkor tudná megmondani Tamás számát, ha gondolt száma a és 0 < a ≤ 1995. Tehát a > 1995. Ha Tamás gondolt száma t, és t ≤ 3990, akkor tudná, hogy a = t + 1995. Ezért t > 3990. Ezt követően András már tudja Tamás számát, ami csak akkor lehet, ha 1995 < a ≤ 5985, s ekkor t = a + 1995. Ha 1-gyel nagyobb számra gondoltak volna, akkor András még nem tudná Tamás számát; ami úgy lehet, ha András száma 5985, Tamás száma 7980. 522. András első kijelentése után tudjuk, hogy a nála levő szám nem 1. Tamás első válasza után tudható, hogy a nála levő szám nem 1 vagy 2. András második válasza után tudjuk, hogy a nála levő szám nem 1, 2 vagy 3. Tamás második válasza után tudható, hogy a nála levő szám nem 1, 2, 3 vagy 4. . . . Tamás tizedik válasza után tudjuk, hogy a nála levő szám nem 1, 2, . . . , 19 vagy 20. Andrásnál vagy a 20 van és Tamásnál a 21, vagy a 21 van nála és Tamásnál a 22. 523. A szorzat ismeretében A nem tudta meghatározni a gondolt számokat, tehát a p = a · b szorzat nem lehet két prím szorzata. B az s = a + b összeg ismeretében már tudta, hogy p nem lehet két prím szorzata, tehát s nem állítható elő két prím összegeként. Ezért s nem lehet páros (hiszen a Goldbach-sejtés szerint a páros számok előállnak két prím összegeként), s a páratlanok közül sem lehet olyan, amely egy prímszámnál kettővel nagyobb. Így s lehetséges értékei: M = {11,17,23,27,29,35,37,41,47,51,53,57,59,65,67,71,77,79,83,87,89,93,95,97}. Ha s = 11, akkor 11 = 2 + 9 = 3 + 8 = 4 + 7 = 5 + 6, így p-nek a 18, 24, 28 és 30 számok egyikének kell lennie. Azonban a 18, 24 és 28 egyértelműen bontható két olyan tényező szorzatára, melyek összege M-ben van: 18 = 2 · 9 = 3 · 6, 2 + 9 = 11 ∈ M, és 3 + 6 = 9 ∈ / M; 24 = 3 · 8 = 4 · 6, 3 + 8 = 11 ∈ M, és 4 + 6 = 10 ∈ / M; 28 = 4 · 7 = 2 · 14, 4 + 7 = 11 ∈ M, és 2 + 14 = 16 ∈ / M. Ezért a legvégén B nem tudna választani. Ha pedig s = 11 és p = 30 lenne, akkor p = 2 · 15 = 5 · 6 alapján A nem tudna dönteni, mert 2 + 15 = 17 és 5 + 6 = 11 is a lehetséges s értékek között van. (A azért tudja a két szám összegét kitalálni, mert a p szorzat csak egyféleképp írható úgy a · b alakba, hogy a + b nem áll elő két prím összegeként.) Tehát s = 11 esetén nem tudnák kitalálni a gondolt számokat. Vegyük a következő lehetséges értéket: s = 17. Ekkor 17 = 2 + 15 = 3 + 14 = = · · · = 8 + 9 felbontások alapján p értéke 30, 42, 52, 60, 66, 70, ill. 72 lehet. Ha p = 30, akkor — mint előbb láttuk — A nem tudna dönteni, hiszen s-re két lehetséges érték is szóba jön. Ugyanígy kétesélyes a p = 42 lehetőség is, mert a 42 = 3 · 14 = 2 · 21 felbontásokból kapható 3 + 14 = 17, 2 + 21 = 23 számok mindegyike eleme M-nek. Ha s = 17 és p = 52, akkor az 52 = 2 · 26 = 4 · 13 felbontásokból kapható 2 + 26 = 28, 4 + 13 = 17 számokból csak a 17 eleme M-nek; A kitalálhatja s értékét, ill. a két gondolt számot: a 4 és 13 számokat.

10. Különféle számelméleti feladatok

221

A megmaradó lehetőségeket elemezve sem találunk további megoldásokat; így a gondolt számok: 4 és 13. Megjegyzés. A megoldásban kihasználtuk az s < 100 megkötést. Számítógéppel az s < 2 000 000 feltétel mellett is vizsgálták a feladatot, s akkor is csak a 4 és 13 adódott egyedüli megoldásnak. Valószínűnek látszik, hogy ez az egyetlen megoldás akkor is, ha s nagyságára nem teszünk semmilyen korlátozást. 524. x = 4,75. 525. a) 4, b) 6 és 25. 526. Csak a p4 (p prím) alakú számoknak van öt osztója. 527. a) 48, b) 60. 528. 60, 72, 84, 90, 96. 529. Ha az N szám prímtényezős alakja N = p1α1 · p2α2 · . . . · pkαk , akkor N osztóinak száma d(N) = (α1 + 1) · (α2 + 1) · . . . · (αk + 1). Egy szám osztóinak száma pontosan akkor páratlan, ha az a szám négyzetszám. Az N szám nem négyzetszám, mert prímtényezős felbontásában az 5 kitevője páratlan szám (= 3). Ezért az osztók száma nem lehet 25 323. Mivel a prímtényezős felbontásban 53 szerepel, ezért az osztók száma d(N) = . . . · (3 + 1) · . . . osztható 4-gyel, míg 25 322 nem osztható 4-gyel, ezért az osztók száma nem lehet 25 322. 530. 23 · 34 . 531. Nincs. 532. 215 · 320 · 524 . 533–538. feladatokhoz:  Legendre   tétele  szerint n! prímtényezős felbontásában a p n n n prím kitevője: + + +... p p2 p3 539. 333. (1000−{2-vel oszthatók száma}−{3-mal oszthatók száma}+{6-tal oszthatók száma}). 540. 1000 − n2 − n3 − n5 + n6 + n10 + n15 − n30 , ahol nk az 1000-et meg nem haladó, k-val osztható pozitív egészek száma („szita-formula”). 541. Az előző két feladat megoldásához hasonlóan járjunk el. 542. Amivel osztottunk, az osztója 539 − 327 = 212-nek. 543. 17.

222

Megoldások, útmutatások

544. A keresett számból vonjunk ki 1-et. Ez a szám már maradék nélkül osztható a 2, 3, 4, 5, 6 számokkal. 545. A keresett számhoz adjunk 1-et. 546. A keresett szám legyen x. 2x 5-tel osztva 1, 7-tel osztva 1, 9-cel osztva is 1 maradékot ad, tehát 2x − 1 osztható 5-tel, 7-tel, 9-cel, azaz 315-tel. Ezért 2x − − 1 = 315, 2x = 316, x = 158. 549. n = 8, n = 10, n ≥ 12. (Ha 2 | n, akkor n = 4 + (n − 4), ha 2  | n, akkor n = 9 + (n − 9).) 550. n ≥ 5, n = 6. Ha 2  | n, akkor n = 2 + (n − 2), ha n = 4k, akkor n = (2k − 1) + (2k + 1), ha n = 4k + 2, akkor n = (2k − 1) + (2k + 3). 551. n = 10, n ≥ 12. n ≥ 12 esetben a következőképpen láthatjuk be: ha n = 4k + 2, akkor n = (2k − 1) + 2 + (2k + 1), ha n = 4k + 3 és 3 | k, akkor n = (2k − 1) + 3 + (2k + 1), ha n = 4k + 3 és 3  | k, akkor n = (2k − 3) + 3 + (2k + 3), ha n = 4k + 4, akkor n = (2k − 1) + 4 + (2k + 1), ha n = 4k + 5 és 5 | k, akkor n = (2k − 1) + 5 + (2k + 1), ha n = 4k + 5 és 5  | k, akkor n = (2k − 5) + 5 + (2k + 5). Lehetne másképp is, pl. a 6k, 6k + 2, 6k + 4, 12k + 1, 12k + 3, 12k + 5, 12k + 7, 12k + 9, 12k + 11 alakú számokat állítanánk elő a kívánt alakban. 552. Hívjuk a racionális számok egy 13 elemű halmazát jó-nak, ha teljesül rá a feladat feltétele, tehát közülük bármely 12 szám két hatos csoportba osztható úgy, hogy az egyes csoportokban levő számok összege megegyezik. Szorozzuk meg a 13 racionális számot a nevezők egy közös többszörösével, ekkor olyan egész számokat kapunk, melyek jó halmazt alkotnak. Válasszuk ki a számok közül valamelyiket, és vonjuk ki ezt a számot mindegyikből. Az így kapott 13 szám jó halmazt alkot, és közte van a 0 szám. Ha a számok között van páros, akkor mindegyik szám páros. (Miért? Gondoljuk végig!) Vegyük mindegyik számnak a felét. Az így kapott 13 szám mindegyike egész szám és jó halmazt alkotnak. A számok között van a nulla, tehát mindegyik páros, így az előző eljárást megismételhetjük. Mivel ezt az eljárást a számokon végtelen sokszor megismételhetjük, ez csak úgy lehet, ha a számok mindegyike 0. Ez azt jelenti, hogy a feladatban szereplő 13 szám egyenlő. 553. Hasonló a következő feladathoz. 554. A számokból képezhető szorzatok száma 211 − 1. Vizsgáljuk a szorzatokat aszerint, hogy prímtényezős alakjukban a prímek kitevője páros vagy sem. A prímek között csak az első tíz prím szerepel, és ezért 210 a különböző lehetőségek száma arra, hogy egy-egy számban e tíz prím kitevője páros vagy páratlan. A

11. Számok reciprokainak összege

223

lehetőségek száma kevesebb a szorzatok számánál, ezért van két azonos viselkedésű szorzat. A két szorzat mindegyikéből hagyjuk el a közös tényezőket (ez nem változtat a két szorzatban a prímek kitevőjének azonos paritásán), és az így megmaradt két szorzatot összeszorozva négyzetszámot kapunk.     48 48 556. A 48 számból kéttényezős szorzatot lehet készíteni. Mivel > 210 , 2 2 így van két olyan szorzat, melynek prímtényezős alakja azonos kitevő-paritású. Legyen ez a két szorzat ab és cd. Ha a, b, c, d különböző számok, akkor ez a négy szám jó, azaz szorzatuk négyzetszám. Ha nem különbözők, például b = d, akkor ac négyzetszám. A maradék 46 számra az eddigi okoskodás megismétel  46 hető, mert > 210 , ezért ezek között is lesz vagy négy különböző szám, 2 vagy csak kettő, x és y, melyek szorzata négyzetszám. Ez utóbbi esetben a, b, x, y a megfelelő négy szám. 557. A feladat állítása már 1537 számra is igaz. Vegyünk közülük 513 db számot. Ezek között van kettő – (a1 , b1 ) –, melyek szorzata négyzetszám. (Az 513 szám között van kettő, melyek prímtényezős felbontásában a prímkitevők párossága azonos, ezért szorzatuk négyzetszám. 26-ig 9 prímszám van, ezért a pozitív egész kitevőjű hatványaik szorzatából álló számokban a prímkitevők párossága szerint 29 = 512 különböző lehetőség van.) Az 513 számból az (a1 , b1 ) számpárt kivesszük, helyettük a többiek közül hozzáveszünk kettőt. E között az 513 szám között is lesz kettő – (a2 , b2 ) –, melyek szorzata négyzetszám. Ily módon az 1537 szám közül kiválaszthatunk 513 számpárt, melyekben a két szám szorzata négyzetszám, xi2 = ai · bi . Az xi2 számokat aszerint tekintsük, hogy benne egy-egy prím (páros) kitevője 4-gyel osztva 0 vagy 2 maradékot ad! Így 29 = 512 eset lehetséges, tehát van e között az 513 négyzetszám között kettő, mely azonosan viselkedik ebből a szempontból, ezek szorzata negyedik hatvány lesz. Megtaláltuk azt a négy számot, melyek szorzata negyedik hatvány.

11. Számok reciprokainak összege 560. Legyen 2t ≤ n < 2t+1 . Vegyük a nevezők legkisebb közös többszörösét, s hozzuk a törteket erre a legkisebb közös nevezőre! Mindegyik tört számlálója páros lesz, 1 kivéve azt a törtet, melyet az t törtből kapunk. Így az összeg számlálója páratlan 2 lesz (hiszen páros számokat és egy páratlan számot adunk össze), nevezője páros, tehát a hányados nem lehet egész szám. n Második megoldás. Tekintsünk egy olyan p prímet, melyre < p ≤ n (ilyen 2 prím Csebisev tétele miatt létezik)! A törteket a legkisebb közös nevezőre hozva a nevező p többszöröse lesz, míg a számláló nem.

Megoldások, útmutatások

224

561.

1 1 1 1 1 = = − , ahol n > 1. < 2 n2 n − n n(n − 1) n − 1 n      n n   1 1 1 1 1 1 Ennek alapján − = 1+ 1− + − + < 1+ k2 k−1 k 2 2 3 k=1    k=2 1 1 1 1 1 + − + ···+ − = 2− . 3 4 n−1 n n

562. Hasonló az 560. feladat első megoldásához. 563. Hasonló az 560. feladat első megoldásához, csak itt a 2-hatványok helyett 3hatványokat figyelünk. 564. Hasonló az 560. feladat első megoldásához. Két lehetőség is kínálkozik: csak az egyik tört nevezője osztható 27 -nel, ill. csak az egyik tört nevezője osztható 34 -nel. 565. Hasonló az előző feladat megoldásához. A nevezők közül csak az egyik osztható 53 -nel. 566. A hatványszámok reciprokainak összege kisebb2-nél, hiszen:    1 1 1 1 1 1 1 1 1+ 2 + 3 + 4 + . . . + 2 + 3 + 4 + . . . +· · · = 1+ + +· · · = 2. 2 2 2 3 3 3 1·2 2·3 567. Levezethető az előző feladat állításából.    ∞  ∞ ∞ ∞    1 1 1 3 = 568. = 2 · = 3. α β α β 2 ·3 2 3 2 α=0 β=0 α=0 β=0 n  1 1 =2− . a a1 a2 . . . an i=1 i      1 1 1 n 570. Az összeg értéke: 1 + 1+ · ...· 1 + −1= . 2 3 n+1 2

569. Lássuk be teljes indukcióval, hogy

572. Az összeg értéke: 88. 573. a) Nem. Az összeg értéke mindig páratlan. b) Nem. Lásd az 560. feladatot! 574. Állítsuk párba az összeadandókat, elsőt az utolsóval, másodikat az utolsó előttivel stb. Egy-egy kéttagú összegben a közös nevezőre hozás után a számláló mindig p, a nevező pedig p-vel nem osztható szám. Ez a tulajdonsága meglesz a törtek összeadása után kapott törtnek is. 576. Előbb lássuk be az 1 1 1 1 1 1 1 1 1 + = + + ···+ 1 − + − + − ··· − 2 3 4 5 2n 2n + 1 n + 1 n + 2 2n + 1

12. Számok és számjegyek

225

összefüggést. Ez igazolható teljes indukcióval is, de most lássunk egy másik bizonyítást. 1 1 1 1 1 1 1 + + + ···+ = 1 + + + ··· + 2 3 2n 2 3 2n 1 1 1 1 1 1 1 1 + + + ···+ = 2 · + 2 · + 2 · + ···+ 2 · 2 3 n 2 4 6 2n A két egyenlőség különbsége adja a kívánt összefüggést. Ezeknek az összefüggéseknek a segítségével a feladatbeli összeg másképp írható:       1 1 1 1 1 1 1 1 + ···+ = + + + + ···+ + . 660 1319 660 1319 661 1318 989 990 Mivel 1979 prímszám és a zárójelekben a közös nevezőre hozás után mindenhol 1979 a számláló, a nevező pedig nem osztható 1979-cel, így igaz az állítás.   1 1 1 1 (k + 1) − (k − 1) 1 1 1 579. 3 < 3 = = · = − , k k − k (k − 1)k(k + 1)  2 (k − 1)k(k + 1) 2 (k − 1)k k(k + 1) 1 1 1 1 1 1 1 így a feladatbeli összeg < · − + − + − +···+ 2 2 · 3 3 · 4 3 · 4 4 · 5 4 · 5 5 ·6  1 1 1 1 1 + − < · = . (n − 1) · n n · (n + 1) 2 2 · 3 12

12. Számok és számjegyek 580–585. Használjuk fel, hogy n ≡ S(n) (mod 9). 586. Nem igaz. Legyen n = 99, m = 100, ekkor f (99) = f (18) + 1 = f (9) + 2 = 2, f (100) = f (1) + 1 = 1, és így f (99) > f (100). 587. Nem lehet. Lásd az 580. feladat állítását. 588. Tegyük fel, hogy létezik két ilyen szám, a és b. Legyen a < b. Ekkor a | b és a | b − a, továbbá 9 | b − a. Mivel a és 9 relatív prímek, így 9a | b − a, de ez nem lehet, mert 9a > b − a; 10a > b, hiszen 10a már nyolcjegyű szám. 589. Nyilván A, B és C is osztható 9-cel. A < 1994 · 9 ≤ 18 000, B < 37, C < 18, tehát C = 9. 590. Hasonló az előző feladathoz. 593. n = 69 999, n + 1 = 70 000. (n = 159 999, n + 1 = 160 000.) 594. n = 48 999, n + 1 = 49 000. (n = 98899 . . . 99 (a 8-as után 16 db 9-es), n + 1 = = 989000 . . . 00.) 595. n + 1 = 8999 . . . 9900 . . . 00 (13 db 9-es, 14 db 0).

Megoldások, útmutatások

226

596. a) Nincs. b) Nincs. 597. Legyen 10a < 21994 < 10a+1 , valamint 10b < 51994 < 10b+1 . Ekkor 10a+b < 21994 · 51994 = 101994 < 10a+b+2 , tehát a + b = 1993. Mivel 21994 (a + 1)-jegyű, 51994 (b + 1)-jegyű, így a keresett érték 1995. Megjegyzés. Lásd még a 494. feladat megoldását. 598. Az előző feladat megoldását követve kapjuk, hogy: k(21090701 ) + k(51090701 ) = 1090702. Mivel a két szám összege páros, különbségük is páros. 599. Nem. Legyen a két hatvány 2n és 2k , n > k. 9 | 2n − 2k = 2k (2n−k − 1), azaz 2n 9 | 2n−k − 1; de ez nem lehet, hiszen 2n−k = k < 10, ezért 2n−k − 1 < 9. 2 600. A mind a tíz számjegyet tartalmazó tízjegyű szám osztható 9-cel. A feladatban leírt átalakítások során kapott számok 9-es maradéka mindig ugyanaz, és így nem kaphatunk 9-cel osztható számot. 601. Az n számnak a p + 1, p + 2, …. , p + n számok valamelyike többszöröse. Legyen p = 1234567890 · 10k , ahol 10k > n, s ekkor az előbbi többszörös megfelelő tulajdonságú. 603. A sorozat első eleme a1 , differenciája d. Ekkor a sorozatnak két megfelelő eleme lesz a1 + 10n d és a1 + 10n+1 d, ha n elegendően nagy. 604. A polinom értékének pozitívnak kell lennie, ezért n > 11. P (n) ≤ n, ezért n < 13. Így n értéke csak 12 lehet, s ez ki is elégíti a megadott egyenlőséget. 605. n = 222 . . . 22111 . . . 11 (k db 2, 2k − 2k db 1). 606. Ilyenek például az olyan, 9-cel osztható számok, melyek nem tartalmaznak 3, 6, 9 számjegyeket. (Megfelelő számok azok is, melyekben van 0 számjegy.) 608. Ilyen szám pl. n = 111 . . . 11995125 (a szám elején 94 db 1-es). S(n) = 125. 609. Válasszuk a 18 szám közül azt az n számot, mely osztható 18-cal. Ekkor S(n) értéke csak 9 vagy 18 lehet, így S(n) | n. 614. Lássuk be, hogy a sorozat korlátos. 615. Ha k n + 1-jegyű, úgy bármely m > n esetén t = 10m − 1 jó szám. 617. n = 111 . . . 11000 . . . 00 (10 db 1-es, 10 db 0). 618. Van, pl.: 10 111 111 111. (Erre az is igaz, hogy S(n) = 10.) 619. Ha S(n) = k és S(n2 ) = k 2 , k(n) = t, akkor az n1 = 10t+1 · n + 1 számra S(n1 ) = = k + 1, S(n21 ) = (k + 1)2 .

13. Racionális és irracionális számok

227

629. n = 1995 · S(n). Tehát n osztható 3-mal, ekkor S(n) is osztható 3-mal, azaz n osztható 9-cel, így S(n) osztható 9-cel. Ezért n = 1995 · 9k, azaz n az 1995 · 9, 1995 · 18, . . . számok közül kerül ki. n = 1995 · 18 = 35 910 lesz a keresett szám. 630. Legyenek a számok: a = 5 555 554 445, b = 5 554 445 555, c = 4 445 555 555. Ekkor S(a + b) = S(11 110 000 000) = 4, S(b + c) = S(10 000 001 110) = 4, S(c + a) = S(10 001 110 000) = 4, S(a + b + c) = S(15 555 555 555) = 51 > 50. Hogyan lehet ezekre a számokra gondolni? S(2(a + b + c)) = S((a + b) + (b + c) + (c + a)) ≤ S(a + b) + S(b + c) + S(c + + a) ≤ 12. Ez azt jelenti, hogy az n = 2(a + b + c) szám felében a számjegyek összege lényegesen nagyobb, mint az n szám esetén. Ez lehetséges akkor, ha az n szám sok egyesből áll, míg a szám felében sok 5-ös számjegy van. Legyen n = = 31 111 111 110, ekkor S(n) = 12, S(n/2) = 51. Az n számot bontsuk fel három n/2-nél kisebb szám összegére úgy, hogy ezekben a számokban a számjegyek összege 4 legyen: n = 11 110 000 000 + 10 000 001 110 + 10 001 110 000, továbbá a + b = 11 110 000 000, b + c = 10 000 001 110, c + a = 10 001 110 000. Innen megkapjuk a fent megadott példákat. 631. Vannak ilyen számok, pl.: x = 999 999 999 999 891, y = 999 999 999 999 900, z = 1 000 000 000 000 008. Ekkor x + S(x) = y + S(y) = z + S(z) = 1 000 000 000 000 017. Teljes indukcióval igazolható, hogy tetszőleges k pozitív egészhez léteznek olyan x1 , x2 , . . . , xk pozitív egészek, amelyekre x1 + S(x1 ) = x2 + S(x2 ) = · · · = xk + + S(xk ). Ez a k = 3 esetben megválaszolja a feladat kérdését.

13. Racionális és irracionális számok 632. A bal oldalon irracionális, a jobb oldalon racionális szám áll. √ √ 633. Lehet. Pl.: 2 − 2, 2 + 2. 634. Nem lehet. Ha x, y irracionális számok, s x +y és x −y is racionális lenne, akkor (x + y) + (x − y) = 2x is racionális lenne, noha 2x irracionális. 635. Nincs. Legyenek x, y, z irracionális számok, s tegyük fel, hogy p = x + y, q = = y + z, r = z + x racionális számok. Ekkor 2y = p + q − r racionális számot jelöl, ami nem lehet, hiszen y irracionális. √ 636. Van. a = 2, b = log√2 3. p 637. Ha log2 5 = , ahol p és q egész számok, akkor 2p = 5q lenne. q 638. A tg 2x-re ismert összefüggés alapján az indirekt feltevés azt adja, hogy tg 10◦ , tg 20◦ racionális, majd az addíciós tételből tg (10◦ + 20◦ ) = tg 30◦ racionális lesz, holott tg 30◦ irracionális.

228

Megoldások, útmutatások

639. sin 3x = 3 sin x − 4 sin3 x miatt az indirekt szerint sin 15◦ értéke racio√ feltevés

2 √ 3 − 1 , amely irracionális. nális, de sin 15◦ = sin(45◦ − 30◦ ) = 4 p 640. logn (n + 1) = (p, q egész) feltevésből np = (n + 1)q , de ez nem lehet, mert az q egyenlőségjel egyik oldalán páros, a másikon páratlan szám áll. 641. Mutassuk meg, hogy a tizedes tört nem válik periodikussá. 642. Mutassuk meg, hogy a tizedes tört nem válik periodikussá. Dirichlet tétele szerint, ha a és b relatív prímek, akkor az ak +b, k = 0, 1, 2, . . . sorozatban végtelen sok prím van. Ha a tizedes tört periodikussá válik és a periódus hossza t, akkor a 102t k + 1 alakú prímek miatt a periódus csak nullákból állna. (Igazolható Csebisev tételének felhasználásával is, mely tétel azt állítja, hogy n és 2n (n ≥ 2) között mindig van prím.) p 643. Tegyük fel, hogy az összeg racionális, értéke . Ekkor az összeget q!-sal szoq rozva egész számot kell kapnunk. Azonban az összegben az első q összeadandó egész szám lesz a szorzás után, míg a többi összege 0 és 1 közötti szám. 644. Az előbbi megoldásához hasonló, itt (q!)2 -nel szorzunk. 645. A szám bináris számrendszerbeli felírásában a „tizedesvessző” után a számjegyek felírásában nincs periodikusság. p 646. Tegyük fel, hogy az összeg értéke racionális szám, = . A feltételből következik, q ai hogy van olyan k, melyre i ≥ k esetén > 3, és ai > 3ai−1 . a1 · a2 · . . . · ai−1 · q Ekkor p · a1 · a2 · . . . · ak−1 =   a1 · a2 · . . . · ak−1 · q a1 · a2 · . . . · ak−1 · q a1 · a2 · . . . · ak−1 · q = + + ···+ + a1 a2 ak−1   a1 · a2 · . . . · ak−1 · q a1 · a2 · . . . · ak−1 · q + + + ... . ak ak+1 A bal oldal értéke egész szám, a jobb oldalon az első zárójelben levő összeg értéke is egész, így a második zárójelben álló összegnek is egésznek kell lennie. 1 1 1 Azonban annak tagjai rendre kisebbek az , , , . . . számoknál, melyek 3 9 27 1 összege . 2 q p r s 647. Ha lg n = , lg(n + 1) = , azaz 10 = n p , 10 = (n + 1) r , akkor nqr = (n + 1)ps , q s de ez nem lehet, hiszen az egyik oldalon páros, a másikon páratlan szám áll.

13. Racionális és irracionális számok

229

648. Ha a sorozat két eleme racionális, akkor a sorozat differenciája is racionális, s ekkor a sorozat minden eleme racionális. 649. Az előző feladat következménye. √ √ √  √ √  √ x+ y x − y azonosság miatt x − y is racionális, s mivel 650. x − y = √ √ √ x + y racionális, így ezek összege 2 x is racionális. √ √ √ √ 651. Mutassuk meg, hogy z racionális. x + y + z = w, w racionális. Egysze 2 2 w + z − x − y + 4w2 z − 4xy √ , s ha a nevező nem rű számítások után z = 4w(w2 + z − x − y) nulla, akkor a tört értéke racionális. 652. Mindig irracionális szám lesz a kifejezés értéke. √ 654. Legyen ai = pi , i = 1, 2, 3, . . . , ahol pi az i-edik prímet jelöli. Tekintsük a következő táblázatot: a1 x1 , a1 x2 , a1 x3 , . . . a1 xn a2 x1 , a2 x2 , a2 x3 , . . . a2 xn a3 x1 , a3 x2 , a3 x3 , . . . a3 xn ... ... Ha itt mindegyik sorban van racionális szám, akkor van két sor, melyben valamely k-ra ai xk és aj xk racionális. Ebből következik, hogy ezek hányadosa is racionális, ami könnyen látható, hogy nem teljesül. 655. Ha mindegyik xi racionális szám, akkor bármely a irracionális szám megfelelő. Ha pl. x1 irracionális, akkor a1 = x1 , a2 = 2x1 , . . . , an+1 = (n + 1)x1 számok valamelyike jó, hiszen különben lenne olyan i, j , melyekre valamely k-ra ai + xk és aj + xk racionális, de ekkor különbségük is racionális lenne. 656. Indirekt feltevéssel élve, vegyük a három pontból két-két pont szakaszfelező merőlegesét! Ezek egyenletében az együtthatók racionális számok, így a két egyenlet közös megoldása is racionális. Azonban ez a megoldás a kör középpontjának két koordinátája, s ezek irracionális számok. 657. Legyenek P (a, √ √ b) és Q(c, d) két pont a körön, melyek koordinátái racionálisak. u = 2, v = 3 a kör középpontjának két koordinátája. Felírhatjuk, hogy (a − −u)2 +(b −v)2 = (c −u)2 +(d −v)2 , ebből 2u(c −a)+2v(d −b) = c2 +d 2 −a 2 −b2 . Ezt az egyenletet a fentiekben megadott számok nem elégítik ki. 2x ny (n + 1)z + + , s ez racionális, ha x, y, z racionális. 2 2 2 1 Ha a területet a Heron-képlettel számoljuk, akkor a t = 12n2 + 12n − 9 irra4 cionális értéket kapjuk. (A gyökjel alatt 4a + 3 alakú szám áll, amely nem lehet négyzetszám.)

658. A háromszög területe

Megoldások, útmutatások

230

659. Legyenek a négyzet csúcsai A, B, C, D és a kör egy pontja P , melyet A és D között veszünk fel. Az AP ívhez tartozó kerületi szög α, a kör sugara r. Ekkor AP = 2r sin α, CP = 2r cos α, √ √ 2 2 ◦ = (AP + CP ). Ha AP és CP BP = 2r sin(α + 45 ) = 2r(sin α + cos α) 2 2 racionális, akkor BP irracionális. 660. A négyszöget a koordináta tengelyekkel párhuzamos oldalú téglalapba foglalva, majd a téglalap területéből elhagyva a „felesleges” háromszögek területét, megkapjuk a négyszög területét, s ez racionális szám lesz,√ha a csúcsok koordinátái e · 2e · sin 45◦ 2 racionális számok. Másrészt: t = = e2 (e a négyszög kisebb 2 2 2 átlója), s ez irracionális, mert e racionális. 661. Igen, van. Legyen D az AB oldal felezőpontja, s válasszuk P -nek az AB szakasz egy pontját. Ha a P D = x távolság  racionális, akkor AP és P B is racionálisak, √ 3 1 (8x)2 + 48 alapján CP is raciotovábbá CP = CD2 + DP 2 = x 2 + = 4 8 1 nális lesz, ha 8x = 1, azaz x = értéket választjuk. 8 Megjegyzés. A hasonló kérdés négyzetre, vagyis hogy van-e olyan P pont az egységnyi oldalú ABCD négyzet síkjában, amelyre P A, P B, P C, P D mindegyike racionális, mindmáig megoldatlan probléma. A sejtés az, hogy nincs. 662. Legyen (m, n) a kiválaszott rácspont. Tekintsük a négyzetnek azt az (x, y) csúcsát, amelyre m−x, n−y mindegyike páratlan. E két pont távolságának négyzete (m − x)2 + (n − y)2 , amely 4-gyel osztva 2 maradékot ad, így nem lehet négyzetszám. Emiatt a két pont távolsága irracionális. 663. Van ilyen négyszög.√Válasszunk tetszőlegesen egy egységnyi területű trapézt, 3 melynek alapjai 1 és 2. Tegyük fel, hogy van olyan pont, amelyre mind a négy 2 háromszög területe racionális. A trapéz magassága √ . A trapéz alapjaira 3 1+ 2 β illeszkedő háromszögek magasságai α és √ , ahol α és β racionálisak. Mivel 3 2 √ β 2 3 α+ √ = √ ; innen adódik, hogy 2 gyöke lenne az αx 2 +(β+α−2)x+β = 0 3 3 2 1+ 2 egyenletnek, ami nem lehetséges. 1 1 + = 1 feltétel szükségességét. Legyen N > 1 egész szám. Az α β N + λ1 , ahol |λ1 | ≤ 1; ugyanígy [α · k] ≤ N feltételnek eleget tevő k-k száma α N N β-ra, s ezek együttes száma + λ1 + + λ2 = N. Az egyenlőséget N-nel osztva α β

664. Lássuk be az

14. Egész együtthatós polinomok

és az N → ∞ határátmenetet elvégezve

231

1 1 + = 1. α β

a 1 1 a , akkor + = 1 miatt β = , s ekkor fennáll az b α β a−b [α · b] = [β · (a − b)], tehát α nem lehet racionális. 1 1 Lássuk még be, hogy ha α > 0 és β > 0 irracionális számokra + = 1 α β teljesül, akkor bármely N pozitív egész szám szerepel az [α · k], k = 1, 2, . . . , vagy a [β · k], k = 1, 2, . . . sorozat valamelyikében, de csak az egyikben. Ha α racionális, α =

14. Egész együtthatós polinomok 665. Ha x 4 + x 3 + ax 2 + bx + c = (x − 1)f (x) + 1, úgy az x = 1 helyettesítés az 1 + 1 + a + b + c = 1 összefüggést adja. 666–667. Hasonló az előzőhöz. 668. x 2 − 3x + 2 = (x − 1)(x − 2). f (1) = 0 és f (2) = 0, tehát az f (x) polinomból az (x − 1) és az (x − 2) gyöktényező kiemelhető. 669. Az x = 1 helyettesítés után kapott érték adja a választ. 671. Mivel r gyöke az egyenletnek, így (r − a)(r − b)(r − c)(r − d) = 4. A bal oldalon négy különböző egész szám szorzata áll; azonban négy különböző egész szám szorzata csak úgy lehet 4, ha azok valamilyen sorrendben 1, 2, −1, −2. Ezért (r − a) + (r − b) + (r − c) + (r − d) = 1 + 2 − 1 − 2 = 0, tehát 4r = a + b + c + d. 672–678. feladatok megoldásánál az a − b | f (a) − f (b) összefüggést célszerű felhasználni, ahol f (x) egész együtthatós polinom, a és b egész számok; ill. azt a tulajdonságot, hogy ha f (x) egy páratlan számra páros értéket vesz fel, akkor minden páratlan számra páros értéket vesz fel, s ez hasonlóan így van egyéb paritások esetén is. 679. Megmutatjuk, hogy bármely m-re 3 | f (m). Az előző feladatok megoldásából tudjuk, hogy m − k | f (m) − f (k), m − (k + 1) | f (m) − f (k + 1), m − (k + 2) | f (m) − f (k + 2). Az m − k, m − (k + 1), m − (k + 2) három egymást követő egész szám valamelyike osztható 3-mal, ezért ott az f (m) − f (k + i) is osztható 3-mal, s mivel f (k + i) osztható 3-mal (0 ≤ i ≤ 2), így f (m) is osztható 3-mal. 680. Indirekt úton bizonyítsunk.

232

Megoldások, útmutatások

681. Tekintsük azt az esetet, amikor f (x) és g(x) fokszáma azonos: f (x) = b0 x k + b1 x k−1 + · · · + bk , g(x) = c0 x k + c1 x k−1 + · · · + ck , és b0 · c0 = 0. A feladat szerint minden n természetes számhoz van olyan αn egész, amelyre b0 f (x) b0 , így lim αn = . Az αn számok f (n) = αn · g(n). Mivel lim = x→∞ g(x) x→∞ c0 c0 egészek, ezért az egész számok αn sorozata csak úgy konvergálhat, ha bizonyos b0 n ≥ K-tól kezdve αn állandó, vagyis αn = α = , n ≥ K. Ez utóbbi egyenlőség c0 végtelen sok esetben teljesül, ezért minden x valós számra f (x) = αg(x). Ha f (x) = h(x) · g(x) + r(x), ahol az r(x) polinom fokszáma kisebb, mint g(x) fokszáma, és h(x), r(x) racionális együtthatós polinomok, akkor van olyan m egész szám, melyre h1 (x) = m·h(x), r1 (x) = m·r(x) egész együtthatós polinomok. Az m · f (x) = h1 (x) · g(x) + r1 (x) összefüggés és a feladat állítása miatt minden n egész számra g(n) osztója r1 (n)-nek. Ahogyan a bizonyítás első részében tettük, olyan okoskodással megmutatható, hogy r1 (x) ≡ 0. 682. Indirekt úton bizonyítjuk. Legyen r egész gyök, f (x) = (x − r) · g(x), r = p + kq, 1 ≤ p ≤ k, de f (p) = (p − r)g(p) = −kqg(p), s így k | f (p) lenne. 683. Legyenek a, b, c, d gyökei az f (x) polinomnak! Ekkor f (x) = (x − a)(x − b)(x − c)(x − d) · g(x). Ha f (x) = 7, akkor f (x) = 7 = = (x − a)(x − b)(x − c)(x − d) · g(x), de ez nem lehet, hiszen a 7 nem állítható elő négy különböző egész szorzataként. 684. Legyen g(x) a feladatban szereplő polinom, s f (x) = g(x) − 7. Ezzel a feladatot visszavezettük az előzőre. 685. b − c = f (a) − f (b) = (a − b)C, c − a = f (b) − f (c) = (b − c)A, a − b = f (c) − f (a) = (c − a)B. Ezekből |a − b| ≤ |b − c| ≤ |c − a| ≤ |a − b|, de ez páronként különböző a, b, c számokra nem teljesül. 686. Tegyük fel, hogy f (x) végtelen sok helyen prím értéket vesz fel. Ezeken a helyeken a g(x) és a h(x) egyike prímet, a másik +1-et vagy −1-et vesz fel értékül. Tehát a g(x) és h(x) polinomok egyike — mondjuk g(x) — végtelen sok helyen +1-et (vagy −1-et) vesz fel értékül. Ez azt jelenti, hogy a g(x) − 1 (vagy a g(x) + 1) polinomnak végtelen sok zérushelye van, ami nem lehetséges. 687. Tegyük fel, hogy f (x) = g(x) · h(x). Mivel f (x) hetedfokú, ezért g(x) és h(x) egyike legfeljebb harmadfokú. Legyen ez a polinom g(x). A hét különböző helyből legalább négy helyen a g(x) egyformán +1 vagy −1. Ha g(x) négy különböző helyen +1, akkor a g(x) − 1 polinomnak négy különböző gyöke lenne. 688. Tegyük fel, hogy a polinom előállítható két másik polinom szorzataként, azaz g(x) · h(x) alakban. Ekkor g(ai ) = −h(ai ) = +1 vagy −1. A g(x) + h(x) polinom-

15. Kombinatorika a számelméletben

233

nak van legalább n db zérushelye (a1 , a2 , . . . , an ), ezért a g(x) és h(x) polinomok közül legalább az egyik n-ed fokú. 689. Tegyük fel, hogy a polinom előáll két másik polinom szorzataként, azaz p(x) = = g(x) · h(x), ahol p(x) = (x − a1 )2 · (x − a2 )2 · . . . · (x − an )2 + 1. Ekkor g(ai ) = h(ai ) = +1 vagy −1. Lássuk be, hogy az előbbi „vagy” választás azt jelenti, hogy vagy minden ai -re +1, vagy minden ai -re −1. Ugyanis ellenkező esetben g(ar ) = +1 és g(as ) = = −1 azt jelenti, hogy a g(x) függvénynek van valamely α zérushelye ar és as között. Ezért g(α) · h(α) = 0, s ez azért nem lehet, mert p(x) ≥ 1 minden x-re. Tegyük fel például, hogy g(ai ) = h(ai ) = +1, i = 1, 2, 3, . . . , n. A g(x) − 1 és h(x) − 1 polinomok mindegyikének legalább n zérushelye van, így g(x) és h(x) mindegyike legalább n-ed fokú, magasabb fokú azonban egyik sem lehet, mert akkor szorzatuk nem lenne 2n-ed fokú. Arra jutottunk, hogy g(x) és h(x) is n-ed fokú, s g(x) − 1 = A(x − a1 ) · . . . · (x − an ), h(x) − 1 = B(x − a1 ) · . . . · (x − an ). (A = 0, B = 0.) Könnyen belátható, hogy p(x) nem áll elő két ilyen szerkezetű g(x) és h(x) polinom szorzataként.

15. Kombinatorika a számelméletben 690. A kiválasztott számok között lesz két szomszédos szám. Megjegyzés. Több is igaz: van két olyan szám; ai és aj , melyekre (ai , aj ) = 1 és ai ≤ n. 691. Legyenek a kiválasztott számok: 1 ≤ a1 < a2 < · · · < an+1 ≤ 2n. Tekintsük a 2n > an+1 − a1 > an+1 − a2 > · · · > an+1 − an ≥ 1 számokat. Ez az n db különböző szám és a kiválasztott n + 1 db szám mindegyike az 1, 2, 3, . . . , 2n számok közül való, ezért a 2n+1 db szám között vannak egyenlők: an+1 −ai = ak , azaz ak + ai = an+1 . 692. A számok közül legalább kettő páratlan. A páratlanok közül legnagyobb b, a többi páratlant jelölje b1 , . . . , br . Képezzük a b − bi különbségeket; ezek páros számok. E különbségek és a halmaz páros számainak száma együttesen 51, ezért valamelyik különbség megegyezik az 52 szám közüli egyik páros számmal. 693. A kiválasztott számok a1 < a2 < · · · < a69 ≤ 100. Nyilván a1 < 33, különben a69 > 100 lenne. Tekintsük az {a1 + a3 , a1 + a4 , . . . , a1 + a69 } és {a3 − a2 , a4 − a2 , . . . , a69 − a2 } halmazokat. Mindkét halmaznak 67 eleme van, és az elemek legnagyobbika, a1 + + a69 legfeljebb 132. A két halmaznak tehát van közös eleme, azaz létezik olyan n és m, amelyekre a1 + an = am − a2 , azaz a1 + a2 + an = am . Az állítás 69 helyett 68-ra nem igaz, lásd: 33, 34, . . . , 100.

Megoldások, útmutatások

234

694. a1 = 2α1 · b1 , a2 = 2α2 · b2 , . . . , an+1 = 2αn+1 · bn+1 , ahol a bi számok páratlanok. Mivel a bi páratlan számok 2n-nél kisebbek, ezért csak n-féle értékük lehet. Így az n + 1 darab bi szám között van két egyenlő: bk = bl . Ekkor az ak és al számok közül egyik a másiknak osztója, hiszen prímtényez˝os felbontásukban csak a 2 hatványkitevője különböző. Más bizonyítás. A 2n db számot beosztjuk n db kupacba: {1, 2, 4, 8, . . . }, {3, 6, 12, 24, . . . }, {5, 10, 20, 40, . . . }, . . . , {2n − 5}, {2n − 3}, {2n − 1}. A kupacoknak megvan az a tulajdonsága, hogy ha egy kupacból kiveszek két számot, akkor az egyik szám osztója a másiknak. Mivel az 1, 2, 3, . . . , 2n számok közül n + 1 számot választunk ki, ezért valamelyik kupacból két számot vettünk; így van két olyan szám, hogy egyik osztója a másiknak. Megjegyzés. A bizonyítás elvégezhető teljes indukcióval is, az indukciós lépésben indirekt feltevést alkalmazva. Ezt leírtuk az 1861. feladat megoldásánál. 696. Azt kell belátni, hogy az ai -k között szerepel 2-nek pozitív egész kitevős hatványa, hiszen akkor ennek a kétszerese is 2-hatvány; vagy pedig előfordul egy 2s − r, 2s + r alakú számpár.      m   N N N N 697. ai -nek többszöröse van N-ig. ≤ N. −1 < , így ai ai ai ai i=1  m  m m    N N 1 − 1 < N, és mivel m ≤ N, ezért < 2N, azaz < 2. a a a i i=1 i=1 i i=1 i Megjegyzés. A pontos korlát

31 . 30

698. Jelölésmódosítással kezdjük: N ≥ a1 > a2 > · · · > am ≥ 1. Állítjuk, hogy N ak ≤ . Ez k = 1-re igaz. Indukciós lépés (k + 1)-re: k ak · ak+1 ak · ak+1 ak − ak+1 ≥ (ak , ak+1 ) = ≥ . [ak , ak+1 ] N 2 N N2 ak · N N Ebből ak+1 ≤ =N− ≤N− N = . ak + N ak + N k+1 k +N Ezzel igazoltuk az indukciós lépést. √ N a[√N +1] ≤ √ < N . Tehát az N ≥ a1 > a2 > · · · > am ≥ 1 sorozatnak √ [ N + 1] √ √ N és N között legfeljebb N eleme van, ezért m ≤ 2 N . 699. Felhasználjuk az egyik korábbi feladat állítását: Ha az 1, 2, 3, . . . , 2n számok közül kiválasztunk n + 1-et, akkor azok között mindig lesz olyan kettő, hogy az egyik osztója a másiknak. 2n , akkor 2a1 < 2n, 3a1 ≤ 2n. Hagyjuk el (Indirekt bizonyítás.) Ha a1 ≤ 3 ekkor a kiválasztott számok közül a1 -et, helyette vegyük be a sorozatba a 2a1 , 3a1 számokat. Az így kapott sorozat is teljesíti az [ai , aj ] > 2n feltételt, és a

15. Kombinatorika a számelméletben

235

sorozatnak n + 1 eleme van. Azonban az előbb említett állítás miatt van az n + 1 szám között olyan kettő, hogy egyik a másiknak osztója. Ekkor mégsem teljesül 2n az [ai , aj ] > 2n feltétel, tehát hibás a kiinduló a1 ≤ feltevés. 3 700. Legyen a számok legkisebb közös többszöröse A. A feltétel azt jelenti, hogy A A A > > ··· > . Tudjuk, hogy A minden ai -nek többszöröse, így a most a1 a2 am felsorolt számok egészek. Mivel m darab különböző pozitív egész legnagyobbiA ka legalább m, így ≥ m, ahonnan a1 -gyel szorozva a bizonyítandó állítást a1 kapjuk. A Megjegyzés. Ha A = m! és ai = , akkor A az a1 , a2 , . . . , am számok m+1−i legkisebb közös többszöröse és éppen m · a1 -gyel egyenlő. Tehát az állítás nem javítható. 701. Legyen aj −1 és aj legkisebb közös többszöröse t ≤ N. Ekkor t/aj −1 és t/aj különböző egész számok, és közülük az első a kisebb, azaz t t N N − ≤ − . Ezeket az egyenlőtlenségeket j = 2, 3, . . . , i-re 1≤ aj aj −1 aj aj −1 N a nyilvánvaló 1 ≤ N/a1 egyenlőtlenséghez hozzáadva kapjuk, hogy i ≤ + a 1     N N N N N + − − + ···+ = . a2 a1 ai ai−1 ai N Megjegyzés. Ha N = m! és ai = , akkor a feltételben is és az állításban is épp i egyenlőség teljesül, tehát az állítás nem javítható. 702. Azt kell észrevenni, hogy minden ai -hez tudunk olyan pj prímet rendelni, melyre α α pj j | ai , de pj j  |a1 · a2 · . . . · ai−1 · ai+1 · . . . · ak . Különböző ai -khez különböző pj -k tartoznak. 703. A sorozat minden eleméhez egy számpárt rendelhetünk: ai → (li+ , li− ), ahol li+ jelenti az ai -vel kezdődő leghosszabb növekvő részsorozat elemeinek számát, és li− az ai -vel kezdődő leghosszabb csökkenő részsorozat elemeinek számát. Az így képzett számpárok különbözők. Hiszen, ha i < j és ai < aj , akkor li+ > lj+ ; ha pedig ai > aj , akkor li− > lj− . li+ és li− pozitív egész számot jelöl. Az olyan különböző számpárok száma, melyben li+ ≤ n, li− ≤ m, m · n. Mivel a sorozat elemeihez rendelt számpárok száma m · n + 1, ezért valamelyik i-re vagy li+ > n, vagy li− > m. Ez a tétel állítását jelenti. 704. ai + aj = ar + as , azaz ai − as = ar − aj . Tehát az ax−ay különbségeknek mind k különbözőknek kell lenniük. Egy k-tagú sorozatból különbség képezhető, e 2

236

Megoldások, útmutatások

  k különbségek abszolútértékei 1-től (n − 1)-ig terjedhetnek, ezért ≤ n − 1. 2   √ k (k − 1)2 < 2 · ≤ 2(n − 1) < 2n, tehát f (n) < 2n + 1. 2 √ Megmutatjuk, hogy ha az a1 , a2 , . . . , ak a kívánt tulajdonságú, és k < 3 n, akkor a sorozat bővíthető. Ugyanis találhatunk olyan a-t, amelyhez nincs olyan x, y, z, hogy a − ax = ay − az lenne; azaz a = ax + ay − az bármely x, y, z esetén. A k elemű sorozatból k 3 ilyen ax + ay − az szám képezhető (ez is csak akkor, ha mind különböz˝o számot eredményez). Mivel k 3 < n, így biztosan van olyan n-nél nem nagyobb a pozitív egész szám, mely nem áll elő ax + ay − az alakban. 708. 1. megoldás. Tekintsük a három legnagyobb számot: a1 ≥ a2 ≥ a3 . Ha a3 < 2, akkor a4 , . . . , a50 is kisebb 2-nél, tehát ezek összege kisebb 47 · 2 = = 94-nél, azaz az első három szám összege nagyobb 6-nál. 2. megoldás. Legyen az 50 szám a1 , a2 , . . . , a50 . Legyen S1 = a1 + a2 + a3 , S2 = a2 + a3 + a4 , . . . , S49 = a49 + a50 + a1 , S50 = a50 + a1 + a2 . Ekkor S1 + S2 + + · · · + S50 = 3(a1 + a2 + · · · + a50 ) = 300 = 50 · 6. Nos, ha az 50 szám összege 50 · 6, akkor nem lehet mind kisebb 6-nál. 3. megoldás. Vegyük most azösszes  3-tagú összeget, s adjuk össze ezeket.  Eb 49 49 ben az összegben egy szám -ször szerepel, vagyis az összeg 100 · . 2   2     50 49 50 Másfelől a 3-tagú összegek száma . Mivel 100 · = 6· , így nem 3 2 3 lehet minden 3-tagú összeg 6-nál kisebb. 709. Legyen a hét szám nagyság szerint: A > B > C > D > E > F > G. Megmutatjuk, hogy A + B + C ≥ 50. Ha ugyanis C > 15, akkor A + B + C ≥ ≥ (C + 2) + (C + 1) + C ≥ 51. Ha pedig C ≤ 15, akkor D + E + F + G ≤ ≤ (C − 1) + (C − 2) + (C − 3) + (C − 4) ≤ 50, tehát ebben az esetben is teljesül az A + B + C ≥ 50 egyenlőtlenség, hiszen a hét szám összege 100. 710. A 101 szám egyike a. A megmaradt 100 számot osszuk két 50-es csoportba, melyekben a számok összege S1 , ill. S2 . A feltétel szerint S1 < a + S2 , és S2 < < a + S1 . Ezeket összeadva kapjuk, hogy S1 + S2 < 2a + S1 + S2 , azaz a > 0. Másik megoldás. Legyenek a számok nagyság szerint rendezve: a1 ≤ a2 ≤ . . . ≤ ≤ a101 . Ekkor nyilván a2 + a3 + · · · + a51 ≤ a52 + a53 + · · · + a101 . Itt a bal oldalhoz a1 -et adva a feltétel szerint megfordul az egyenlőtlenség iránya, tehát a1 pozitív, és így a többi szám is pozitív. 711. Olyan különböző ai > ak , al > aj számokat kell találni, amelyekre ai − ak = = al − aj . Képezzük a 16 szám közötti összes pozitív különbséget; ezek száma 120. Mivel mindegyik különbség kisebb 100-nál, így vannak egyenlő különbségek. Ha van három egyenlő különbség, akkor kiválasztható közülük kettő, amelyekben különböző számok különbségét képeztük. Legyen ugyanis ai −aj = ak −al =

16. Számkonstrukciók

237

= am − an , ai < ak < am . Az első és második pár csak akkor nem jó, ha ai = al , az első és a harmadik pedig akkor, ha ai = an . Ez a két eset nem lehetséges, így a kívánt kiválasztás mindig lehetséges. Ha nincs három, egymással egyenlő különbség, akkor van legalább 21, amely kétszer lép fel. Tegyük fel indirekte, hogy a különbségpárok között nincs olyan, melyben négy különböző szám szerepelne. Ekkor mind a 21 pár ai − am , am − aj alakú volna. Mivel am legfeljebb 16 különböző értéket vehet fel (valójában 14-et, mert am a legkisebb és a legnagyobb nem lehet), volna olyan am , amely két különbségpárban is szerepel: aj > ak -ra ai − am = am − aj és ak − am = am − aj . Ezekből ai − ak = al − aj a különböző ai , aj , al számokra, ami ellentmond az indirekt feltevésnek. Így a 16 szám közül mindig kiválasztható négy olyan különböző, amelyre ai + aj = ak + al . 712. Az a1 , a2 , . . . ,  an  és az ai +aj összegeknek ki kell adniuk minden számot an+1 -ig; n < (n + 1)2 . így an+1 ≤ n + 2

16. Számkonstrukciók 713. 95 210. 714. 10 112 358. 715. 900 000. 716. 999 990. 717. 105. 718. 18, 36, 54, 72, 90. 719. 269, 807, 1345. 720. 129, 387, 645. 721. Több megoldás van: (192, 384, 576); (219, 438, 657); (273, 546, 819); (327, 654, 981). 722. 652, 978, 1304. 723. 183 = 5832, 184 = 104 976. Könnyen belátható, hogy csak egy ilyen szám van. 724. 567 és 5672 = 321 489; vagy 854 és 8542 = 729 316.

Megoldások, útmutatások

238

725. 532 · 14 = 98 · 76 = 7 448, 174 · 32 = 96 · 58 = 5 568, 186 · 27 = 93 · 54 = 5 022, 146 · 29 = 73 · 58 = 4 234, 134 · 29 = 67 · 58 = 3 886, 158 · 23 = 79 · 46 = 3 634.

584 · 12 = 96 · 73 = 7 008, 158 · 32 = 79 · 64 = 5 056, 259 · 18 = 74 · 63 = 4 662, 174 · 23 = 69 · 58 = 4 002, 138 · 27 = 69 · 54 = 3 726,

726. 39 · 402 = 15 678, 27 · 594 = 16 038, 54 · 297 = 16 038, 36 · 495 = 17 820, 45 · 396 = 17 820, 52 · 367 = 19 084, 78 · 345 = 26 910, 46 · 715 = 32 890, 63 · 927 = 58 401. 727. 42·138 = 5796, 18·297 = 5346, 27·198 = 5346, 39·186 = 7254, 48·159 = 7632, 28 · 157 = 4396, 4 · 1738 = 6952, 4 · 1963 = 7852. 728. 21 · 87 = 1 827, 15 · 93 = 1 395, 27 · 81 = 2 187, 35 · 41 = 1 435. 729. 153 = 13 + 53 + 33 , 370 = 33 + 73 + 03 , 371 = 33 + 73 + 13 , 730. 1634 = 14 + 64 + 34 + 44 , 9474 = 94 + 44 + 74 + 44 .

407 = 43 + 03 + 73 .

8208 = 84 + 24 + 04 + 84 ,

731. 54 748 = 55 + 45 + 75 + 45 + 85 , 92 727 = 95 + 25 + 75 + 25 + 75 . 732. 96 433 469. 733. 142 857. 734. 549, 1449. 735. 7, 29, 1, 1, 1, . . . , 1 (167 db 1-es). Könnyű belátni, hogy hasonló megoldás minden összetett számra megadható. 736. −2, 2, 25, 1, . . . , 1 (36 db 1-es), −1, . . . , −1 (61 db −1-es). −4, −2, 25, 1, . . . , 1 (89 db 1-es), −1, . . . , −1 (108 db −1-es). 737. 1, 1, 1, . . . , 1 (998 db), 2, 1000. vagy

739. 2

3

20

a

b

cd e

4

5

6

c

d

ab e

15

8

1

bd e

ac e

e

16. Számkonstrukciók

239

740. 27

10

1

5

3

18

2

9

15

741. 12 9 2 1 6 36 18 4 3 742. Ilyen táblázatot úgy is készíthetünk, hogy egy 3×3-as bűvös négyzet elemei helyére olyan 2-hatványokat írunk, ahol a kitevők az eredeti bűvös négyzet megfelelő elemei. 2

7

6

22

27

26

4

128

64

9

5

1

29

25

21

512

32

2

4

3

8

24

23

28

16

8

256

27

7

2

16

7

18

20

2

8

18

14

3

10

4

21

6

28

4

9

6

3

5

12

28

1

12

24

21

24

14

1

15

3

4

9

6

6

6

2

4

8

0

0

4

743.

744.

Megoldások, útmutatások

240

745.

1 −1 −1 1 −1 1 1 −1 1 −1 −1 1 −1 1 1 −1

746. 52 39 26 13 0

82 74 66 58 50

112 109 106 103 100

142 144 146 148 150

172 179 186 193 200

749. 1, 19, 2, 18, 3, 17, . . . , 8, 12, 9, 11, 10, 20. 750. 1, 2, 4, 8, 3, 6, 12, 11, 9, 5, 10, 7 vagy 1, 6, 10, 8, 9, 2, 12, 7, 3, 5, 4, 11. 751. (1, 2n, 2n + 1), (2, 2n − 1, 2n + 2), (3, 2n − 2, 2n + 3), . . . , (n − 1, n + 2, 3n − 1), (n, n + 1, 3n). 752. 1, 2, 7, 11, 24, 27, 35, 42, 54, 56 vagy 1, 3, 9, 14, 23, 24, 41, 53, 57, 60. 753. Hamis. Ellenpélda: 2 · 3, 2 · 5, 3 · 5, 7 · 11, 7 · 13, 11 · 13. Kis módosítással azonban már igaz az állítás: Bármely hat természetes számból vagy három, páronként relatív prím választható ki, vagy három olyan, amelyek közül bármely kettőnek van 1-nél nagyobb közös osztója. (Lásd a 1625. feladatot!) 754. Hamis. Ellenpélda: 2184, 2185, 2186, . . . , 2200. 755. 1939 db szám kiválasztható, több nem. A kiválasztott számok: 45, 46, 47, . . . , 1982, 1983. 756. Hamis. Ellenpélda: 2, 3, 5, 7, 8, 11, 12, 26, 27, 28, 29, . . . , 50 (összesen 32 db szám). 757. Az egyik számhalmaz álljon azokból az n természetes számokból, amelyekhez található olyan k, melyre (2k)! + 1 ≤ n ≤ (2k + 1)!. 758. Az egyik halmaz {1, 2, 3} ∪ {9, 10, 11, 12, 13, 14, 15} ∪ {25, 26, . . . , 34, 35} ∪ ∪{49, 50, 51, . . . , 63} ∪ . . . 760. A 2k, 3k, 4k + 1, 6k + 5, 12k + 7 alakú, a 2k, 3k, 4k + 1, 6k + 1, 12k + 11 alakú vagy a 2k, 3k, 4k + 1, 8k + 7, 12k + 11, 24k + 19 alakú számok. 761. (09, 18, 27, 36, 45), (18, 36, 54, 72, 90), (50, 61, 72, 83, 94).

(05, 16, 27, 38, 49), (10, 32, 54, 76, 98),

(01, 23, 45, 67, 89), (54, 63, 72, 81, 90),

16. Számkonstrukciók

241

762. a) 6, 12, 18. b) 120, 240, 360, 480, 600. Általában 2n + 1 db számra: A, 2A, 3A, . . . , (2n + 1)A, ahol A = (2n + 1)! 763. Például a 10k + 24, 10k + 25 alakú számpárok. (k ≥ 2) 764. 242, 243, 244, 245. 765. Van. Ha n osztható 2 · 3 · 5 · 7 = 210-zel, akkor az n − 10, n − 9, . . . , n + 10 számok mindegyike — kivéve az n − 1 és az n + 1 számokat — osztható a 2, 3, 5, 7 prímek közül legalább az egyikkel. Ha n = 9450, akkor n − 1 11-gyel, n + 1 pedig 13-mal osztható. Ily módon a 9440, 9441, . . . , 9460 számok teljesítik a kívánt elvárásokat. Megjegyzés. Vizsgáljuk meg a következő kérdést: „Van-e 14 olyan, egymás után következő pozitív egész szám, hogy a számok mindegyike osztható a 2, 3, 5, 7, 11 prímek közül legalább eggyel?” Erre a válasz: Nincsenek ilyen számok. 766. m = 2k − 2, n = 2k (2k − 2), m + 1 = 2k − 1, n + 1 = (2k − 1)2 . 767. 2, 3, 7. 768. 1, 2, 3, 7 vagy 2, 3, 7, 43. 769. −1, 1, −2, 2; 1, 2, 3, 6; 1, 2, 6, 9; 1, 3, 8, 12; 1, 4, 5, 10; 1, 6, 14, 21; 2, 3, 10, 15. 770. Helyezzük el a 16 számot egy 4 × 4-es táblázatba az ábra szerint. A vastaggal kiemelt számok alkotják az egyik csoportot, a többiek a másikat. (Ellenőrizzük, hogy megfelelő-e a számok szétosztása.) 1 2 3 4 5 6 7 8 9 10 11 12 13 14 15 16 771. 1, 3, 4, 5 vagy 1, 2, 3, 5. 772. 1, 2, 5, 7. 773. ak = k · n! + 1, k = 1, 2, 3, . . . , n. 774. 2, 22 , 23 , . . . , 2n . (n ≥ 3) 775. Legyenek p1 , p2 , . . . , p8 prímszámok. A keresett nyolc szám: a1 = p12 · p2 · . . . · p8 , a2 = p1 · p22 · . . . · p8 , . . . , a8 = p1 · p2 · . . . · p82 . 776. Válasszuk ki közülük a 3k + 1 vagy a 3k + 2 alakú számokat!

Megoldások, útmutatások

242

777. Ha már találtunk n db megfelelő számot, az a1 , a2 , . . . , an számokat, akkor a következő módon tudunk megadni n + 1 db, a kívánt tulajdonsággal rendelkező számot. Legyen N ennek az n db számnak és az n db számból képezhető páronkénti különbségeknek a legkisebb közös többszöröse! A keresett n + 1 db szám: N, N + a1 , N + a2 , . . . , N + an . 778. Tekintsük ezeknek a számoknak a hármas számrendszerben felírt alakját, s azokat a számokat válasszuk ki közülük, melyek felírásában a 2-es nem szerepel! 779. {1, 2}, {1, 3}, {1, 4}, {1, 5}, {1, 6}, {2, 3, 4, 5, 6} vagy {1, 2}, {1, 3}, {1, 4}, {1, 5}, {1, 6}, {1}. 780. A három halmaz lehet pl. a következő: A = {1, 2, 4, 5, 7, 8, . . . }, B = {2, 3, 5, 6, 8, 9, . . . }, C = {3, 4, 6, 7, 9, 10, . . . }. 781. Álljon A azokból a természetes számokból, amelyeknek minden, 0-tól különböző számjegye a szám végétől visszafelé számítva páratlan helyen áll, B pedig azokból a természetes számokból áll, melyeknek minden, 0-tól különböző jegye páros helyen helyezkedik el. 782. Hasonló az előzőhöz. 783. Tekintsük a 781. feladat megoldásában leírt konstrukciót. Tetszőlegesen választva egy b elemet B-ből, az A halmaz minden eleméhez hozzáadjuk ezt a választott számot, így kapjuk az Ab halmazt. Ezt az Ab halmazt B minden elemére elkészítve megkapjuk a keresett halmazokat. 784. A = {1, 2, 5, 6}, B = {3, 4, 7, 8}. Megjegyzés. Az 1, 2, 3, 4, 5, 6, 7, 8, 9 számokat a kívánt módon már nem tudnánk két csoportba osztani. 785. Használjuk fel a 105. b) feladat megoldásánál leírt azonosságot! 786. Az n2 + (n + 3)2 + (n + 5)2 + (n + 6)2 = (n + 1)2 + (n + 2)2 + (n + 4)2 + (n + 7)2 azonosság következménye, hogy bármely 8 — és így bármely 16 — szomszédos egész szám négyzete beosztható két egyenlő összegű csoportba. Elegendő tehát az első 11 négyzetszámot szétosztanunk. Ebben segít az n2 + (n + 2)2 + (n + 6)2 + +(n+7)2 +(n+8)2 +(n+10)2 = (n+1)2 +(n+3)2 +(n+4)2 +(n+5)2 +(n+9)2 +(n+11)2 azonosság. 788. 1210, 2020, 21 200, 3 211 000, 42 101 000, 521 001 000, 6 210 001 000. 789. 1, 7, 3, 2, 1, 1, 1, 2, 1, 1 vagy 1, 11, 2, 1, 1, 1, 1, 1, 1, 1. 790. x1 =0,95, x2 =0,05, x3 =0,34, x4 =0,74, x5 =0,58, x6 =0,17, x7 =0,45, x8 =0,87, x9 =0,26, x10 =0,66.

17. Melyik szám a nagyobb?

243

17. Melyik szám a nagyobb? 791. 23 < 32 . 792. 210 = 1024 > 103 . 793. 23 < 32 miatt 230 < 320 , így 2 · 230 < 3 · 320 vagy 231 < 321 794. 2023 = 23 · 1013 > 32 · 1012 = 3032 . 795. 992 < 99 · 101 = 9999. 796. 920 = 340 > 339 = 2713 . 797. 35 = 243 < 343 = 73 . 798. 199010 + 19909 = 19909 · (1990 + 1) < 19919 · 1991. 801. 2

100

+3

100

100

100

< 4 , mert 2 < 3  3 4 64 ugyanis már = > 2. 3 27

100

, és 2 · 3

100

< 4

100

 100 4 ; hiszen > 2, 3

999 1 1 1000 =1− 1− . 22222223 33333334 22222223 33333334 222222 222222 444443 222221 444442 806. = =1− 5 7 .

810. 3111 < 3211 = 255 < 256 = 1614 < 1714 . 811. 12723 < 12823 = 2161 < 2162 = 51218 < 51318 . 812. 113 = 1331 < 1369 = 372 .

244

Megoldások, útmutatások

 398  2148 813. 29 és 34 számoknál az alapokat és a kitevőket hasonlítsuk össze! √ √ 814. 2 · log12 145 > 2 · log12 144 = 4 = 16 > 15. 3 = log9 27 > log9 25. 2 6 6 7 816. log5 > log6 > log6 , tehát log5 6 − log5 5 > log6 7 − log6 6, azaz log5 6 > 5 5 6 > log6 7.

815. log4 9 > log4 8 =

817. lg2 11 = (1 + lg 1,1)2 > 1 + 2 · lg 1,1 = 1 + lg 1,21 = lg 12,1 > lg 12. √ √ π 3 7 < < log3 7, a legutóbbi egyenlőtlenség azért helyes, 818. sin 1 < sin = 3 2 8 mert 37 < 74 . 819. 1, 011000 > 1000. Ennek igazolásához felhasználjuk a Bernoulli-egyenlőtlenséget: (1 + x)n ≥ 1 + nx, ha x ≥ −1 valós szám, n ≥ 1 egész szám. (Az egyenlőtlenség igazolható teljes indukcióval.) 1, 018 ≥ 1, 08; 1, 011000 = (1, 018 )125 ≥ 1, 08125 . 1, 085 > 1, 4. Ezeket is felhasználva: 1, 011000 > 1, 425 > 1, 424 > 2, 78 > 74 = 2401 > 1000.  4 20 4 820. 2 · 3100 > 2100 + 3100 miatt elegendő belátni, hogy 480 > 2 · 3100 , vagyis = 35  20 256 = > 2. Ennek igazolásához felhasználjuk a Bernoulli-egyenlőtlenséget: 243 n (1 + x) ≥ 1 + nx, ha x ≥ −1 valós szám, n ≥ 1 egész szám. (Az egyenlőtlenség igazolható teljes indukcióval.)     1 256 20 256 1 20 > 1 + , ezért > 1+ ≥ 2. 243 20 243 20 Most belátjuk, hogy 2100 + 3100 > 479 . Megmutatjuk, hogy 3100 > 479 , azaz    20    10 480 480 256 20 19 361 10 9 < 4. Bizonyítás: = < = < = 100 100 3 3 243 18 324 8  5  5 81 9 59049 = < = < 4. 64 7 16807

18. Egyenletek és egyenletrendszerek 821. a) A bal oldalon 2 ≤ x ≤ 3 teljesül, de ilyen x-re a jobb oldal negatív. √ b) A bal oldalon álló összeg értéke legalább 1 + 2, s ez > 2. 2 1 c) x 2 + 2 > x 2 + 2 ≥ 2. x x

18. Egyenletek és egyenletrendszerek

245

2

d) 2sin x ≥ 20 = 1 ≥ sin x, így a sin x = 0, sin x = 1 egyenletrendszert kapjuk. Ennek az egyenletrendszernek nincs gyöke. √ e) 2 sin x cos x = 2 sin 60◦ , azaz sin 2x = 3. f) Vagy mindkét tényező +1, vagy mindkettő −1, hiszen −1 ≤ sin x ≤ 1. g) sin x · sin(x + π) ≤ 0. h) Az egyenlet bal oldalán álló kifejezés mindig pozitív. Vizsgáljuk az x < 0, 0 ≤ x ≤ 1, 1 < x intervallumokon a kifejezést! 822. x = 1. 823. Nincs megoldás. 824. x = −1. 825. Az egyenlőség csak akkor teljesülhet, ha mindkét összeadandó nulla: x = 1. 826. 2x + 3x szigorúan monoton növekvő, ezért a 35 értéket legfeljebb egyszer veszi fel: x = 3.  x  x  x  x 2 3 2 3 827. + = 1. A + függvény szigorúan monoton csökken, ezért 5 5 5 5 legfeljebb egy megoldás van: x = 1. 828. A bal oldalon álló kifejezés szigorúan monoton növekvő, ha x ≥ 5: x = ±5, más megoldás nem lehet. 829. Nyilván x ≥ 5, s ekkor a bal oldalon álló kifejezés szigorúan monoton növekvő, míg a jobb oldalon álló szigorúan monoton csökken. Ezért legfeljebb egy gyök lehet, s van is: x = 5. 830–831. Grafikus ábrázolás mutatja, hogy legfeljebb két gyök van: x = 2 és x = 3, 1 ill. x = 0 és x = . 2 832. A bal oldalon álló kifejezés szigorúan monoton növekvő, ezért az egyenletnek 1 legfeljebb egy megoldása van: x = . 3 833. Szorozzuk az egyenlet mindkét oldalát 2x -nel. Ekkor a bal oldalon álló kifejezés szigorúan monoton növekvő lesz; a jobb oldalon konstans áll, tehát legfeljebb egy megoldás van: x = −1. 834. Adjunk az egyenlet mindkét oldalához lg(x 5 − 24)-et. Ekkor a bal oldalon álló kifejezés szigorúan monoton növekvő lesz; a jobb oldalon konstans áll, tehát legfeljebb egy megoldás van. x = 4 gyök. 835. Az egyenletnek [0, 2π] intervallumon csak 0 és π/10 között lehet megoldása. π/12 megoldás. Több megoldás nincs, mert a [0, π/10] intervallumon az egyenlőség két oldalán álló függvény eltérő monotonitású.

246

Megoldások, útmutatások

836. cos x ≤ 1 ≤ (y − 1)2 + 1, tehát a megoldás: y = 1, x = 2kπ, k ∈ Z. 837. A bal oldali kifejezés növekvő, a jobb oldali csökkenő függvénye x-nek, így legfeljebb egy megoldás lehet. A megoldás: x = 1. 838. x = 105 megoldás, és ennek az elsőfokú egyenletnek több megoldása nem lehet. 839–841. Mindegyik egyenlet egyismeretlenes elsőfokú egyenlet, tehát legfeljebb egy gyöke lehet (ugyanis egyik egyenlet sem azonosság), s ez a gyök megsejthető. Az egyenletek gyökei: x = a + b + c, x = a + b + c, x = ab + bc + ca. 842. Az egyenletnek – ha nem azonosság – legfeljebb n számú különböző valós gyöke lehet. x = 1, 2, 3, . . . , n mindegyike megoldás. √ √ 843. Nyilván 2 √ ≤ x ≤ 3. Ekkor 0 ≤ x − 2 ≤ 1, ezért x − 2 ≤ x − 2. Ugyanígy 3 − x ≤ 3 − x. Utóbbi két egyenlőtlenséget összeadva azt kapjuk, hogy az egyenlet bal oldalán álló kifejezés legalább 1. Vizsgáljuk a jobb oldalt! x 2 − 5x + + 7 = (x − 2)(x − 3) + 1 ≤ 1, ha 2 ≤ x ≤ 3. Ezért az egyenletnek csak akkor lehet megoldása, ha mindkét oldalon 1 áll.  (x 2 + x − 1) + 1 x 2 + x 844. x 2 + x − 1 ≤ = , 2 2 2 2  (x − x + 1) + 1 x − x + 2 x − x2 + 1 ≤ = , tehát 2 2 2   x + x x − x2 + 2 x2 − x + 2 = x2 + x − 1 + x − x2 + 1 ≤ + = x + 1; 2 2 2 az innen adódó x − x + 2 ≤ x + 1 egyenlőtlenséget átrendezve az (x − 1)2 ≤ 0 egyenlőtlenséghez jutunk, melynek csak x = 1 a megoldása, és ez megoldása az egyenletnek is. 845. Átalakítás után az (x − 1)2 + (y − 1)2 = 0 egyenletet kapjuk. 846. Az egyenlet mindkét oldalát szorozzuk meg 2-vel, s egyszerű átalakítások után a (4x 2 − 1)2 + (4y 2 − 1)2 = 0 egyenletet kapjuk. 847. Átalakítás után a következő egyenlethez jutunk:  2  2  √ 6 2 4 4 √ − 2 x − 2 + √ − y − 1 = 0. 4 4 y−1 x−2 848. Átalakítás után az (x + y)2 + (x + 1)2 + (y − 1)2 = 0 egyenletet kapjuk.   849. Átalakítás után a 3(x + 1)2 + 4+ 5(x + 1)2 + 9 = 5−(x +1)2 egyenletet kapjuk. A bal oldal legalább 5, a jobb oldal legfeljebb 5. A megoldás x = −1. 850. Vonjuk ki a második egyenlet megfelelő oldalait az elsőéből, s a kapott egyenletet átalakítva az (x − y)2 + (y − z)2 = 0 egyenlethez jutunk. 851. (x 2 − a 2 )2 = 4ax + 1, (x 2 + a 2 )2 − 4a 2 x 2 = 4ax + 1, (x 2 + a 2 )2 = 4a 2 x 2 + 4ax + 1, (x 2 + a 2 )2 = (2ax + 1)2 .

18. Egyenletek és egyenletrendszerek

247

 2  2  2 1 2 1 1 1 x + x−y + x−z + x−t = 0 4 2 2 2 egyenletet kapjuk, tehát x = y = z = t = 0.

852. Az egyenletet átalakítva az

853. Az egyenletek megfelelő oldalait összeadva kapjuk az (x −3)2 +(y +5)2 +(z −1)2 = 0 egyenletet. Ennek x = 3, y = −5, z = 1 megoldása megoldása az egyenletrendszernek is. 854. Az egyenletek megfelelő oldalait összeadva és 2-vel szorozva: (x − y)2 + (y − z)2 + (z − x)2 = 0. 855. Az első két egyenlet megfelelő oldalainak összegéből a harmadikét kivonva az (x − y)2 + (y − z)2 + (z − x)2 = 0 összefüggéshez jutunk. 856. Az egyenletrendszer egy speciális megoldása az x = 0, y = 0, z = 0. Ha (x0 , y0 , z0 ) az egyenletrendszer egy megoldása és x0 , y0 , z0 közül pl. x0 = 0, akkor y0 = z0 = 0. Tehát feltehetjük, hogy x = 0, y = 0, z = 0. Szorozzuk 2-vel az egyenletek megfelelő oldalainak reciprokait és adjuk össze őket! A következő egyenletet kapjuk:       1 2 1 2 1 2 1− + 1− + 1− = 0. x y z 857. x, y, z, t egyszerre pozitív, ill. egyszerre negatív. Nézzük azt az esetet, amikor 1 mindegyik pozitív. Ha a pozitív, akkor a + ≥ 2, ezért most x, y, z, t mindegyia kének értéke legalább 1. Szorozzuk meg az egyenletek megfelelő oldalait 2-vel, 1 1 1 1 és adjuk össze őket; ekkor az x + y + z + t = + + + egyenlethez jutunk. Az x y z t előbbi észrevétel miatt a bal oldal értéke legalább 4, a jobb oldalon pedig 4-nél nagyobb szám nem állhat. Megoldások: x = y = z = t = 1, x = y = z = t = −1. 858. Tekintsük az egyenletek megfelelő oldalainak reciprokait. x y 1 y 859. x, y, z egyike sem 0. Legyen xyz = a. Ekkor az egyenletrendszer: + = , + a a 2 a x 1 y 1 z 1 z 5 z x 2 + = , + = . Ennek megoldása: = , = , = . Figyelembe véve a 6 a a 3 a 6 a 3 a 2 a jelentését azt kapjuk, hogy yz = 6, zx = 3, xy = 2. Ez utóbbi egyenletrendszer megoldása: x1 = 1, y1 = 2, z1 = 3 és x2 = −1, y2 = −2, z2 = −3. 3 860. Legyen y = tx. Ekkor az egyenletek: x 2 (1 + t) = 10, x 2 (t 2 + t) = 15. Tehát t = . 2 Megoldások: (2; 3), (−2; −3). √ 3 861. 3x 3 − 3x 2 − 3x = 1, 4x 3 = x 3 + 3x 2 + 3x + 1, 4x 3 = (x + 1)3 , 4x = x + 1, 1 . x=√ 3 4−1

Megoldások, útmutatások

248



1 t 2 + t + . Ekkor az egyenletrendszer x = f (y), y = f (z), z = 3 = f (x) alakú. Mivel f (t) minden t-re pozitív, ezért x, y, z pozitív. t > 0 esetén f (t) monoton növekvő, így ha x > y, akkor f (x) > f (y), azaz z > x, emiatt z > y. f (z) > f (y), vagyis y > x. Ellentmondásra jutottunk, tehát x = y = z. 1 Az előző feladatra támaszkodva kapjuk a megoldást: x = y = z = √ . 3 4−1

862. Legyen f (t) =

3

863. (x − y + z)2 − z2 = x 2 − y 2 , (x − y)(x − y + 2z) = (x − y)(x + y). Megoldás: x = y vagy y = z. 864. Legyen x = a 2 , y = b2 , z = c2 . Ekkor az egyenlet: (a − b + c)2 = a 2 − b2 + c2 , és ez megegyezik az előző feladatban szereplő egyenlettel. 865. (x − 1)3 + 8 = 0. 866. Szorozzuk 2-vel az egyenlet mindkét oldalát, s a kapott kifejezést másképp írva jutunk a (2x + 3)3 − 7 = 0 egyenlethez. 867. Átalakítás után az (x + 1)4 − 3 = 0 egyenletet kapjuk. 868. Átalakítás után az (x + 1)3 + 8x 3 = 0 egyenletet kapjuk. 869. Átalakítás után a 4x 4 − (x − 1)4 = 0 egyenletet kapjuk. 870. Átalakítás után az x 4 − (x − 1)2 = 0 egyenletet kapjuk. 871. Átalakítás után az (x 2 + 2x + 3)2 = 25 egyenletet kapjuk. 872–874. Jelöljük a gyökös kifejezéseket új változókkal, s ezekre írjunk fel egyenleteket. A kapott egyenletrendszert már könnyű megoldani. 875. Legyen y = x 5 . Ekkor az egyenlet az y y = 55 alakot ölti. 876. A bal oldalon egész szám áll, ezért a jobb oldalon levő kifejezés értéke −1, azaz x egész szám. Az egyenlet megoldását az (x + 1)(x 2 + 1) = 0 egyenlet megoldása adja. √ √ 877. A megoldások: 1, 2, 3. √ 3 878. A megoldás: 4. 879. A jobb oldalon álló szám csak 0 vagy −1 lehet.    √ √ √ 880. Legyen y = x x x x . . .. Ekkor az egyenlet a xy = 1994 alakot ölti, és √ √ y = 1994 miatt x = 1994.

18. Egyenletek és egyenletrendszerek

249

3 881. Az egyenletnek csak x ≥ esetén lehet megoldása. 4 √ 3 Tekintsük az f (x) = −3 + 4x függvényt. Ez a függvény x ≥ esetén értelmes 4 és szigorúan monoton nő. A feladat egyenlete x = f (f (f (x))) alakban írható. Ha x > f (x), akkor f szigorú monotonitása miatt x > f (x) > f (f (x)) > > f (f (f (x))). Ilyen x tehát nem lehet megoldása az egyenletnek. Hasonlóan látható, hogy x akkor sem megoldása az egyenletnek, ha x < f (x). √ Marad tehát az az eset, ha x = f (x), azaz x = −3 + 4x. Ennek gyökei: x1 = 1, x2 = 3, melyek kielégítik az eredeti egyenletet is. 882. Átalakítás után az (a − b)2 + (b − c)2 + (c − a)2 = 0 egyenletet kapjuk. Ezt használjuk fel az egyenlet megoldásánál is. 883. sin2n x ≤ sin2 x, cos2n x ≤ cos2 x. Adjuk össze a két egyenlőtlenséget. 884. Használjuk az előbbi eredményeket is. 885. A második egyenletből kapjuk, hogy −1 ≤ x, y ≤ 1. Ezután az első egyenlet miatt 0 ≤ x, y ≤ 1. Így már igazak az x 3 ≥ x 4 és y 3 ≥ y 4 , azaz 1 = x 3 + y 3 ≥ ≥ x 4 + y 4 = 1 egyenlőtlenségek. Tehát x 3 = x 4 , y 3 = y 4 . 886. Indirekt úton bizonyítunk. Tegyük fel, hogy az egyenletnek van három különböző valós gyöke: x1 , x2 , x3 . A gyökök és együtthatók közti összefüggés alapján: x1 + +x2 +x3 = −4; x1 x2 +x2 x3 +x3 x1 = 6. Ezekből x12 +x22 +x32 −x1 x2 −x2 x3 −x3 x1 = −2, azaz (x1 − x2 )2 + (x2 − x3 )2 + (x3 − x1 )2 = −4. Ez utóbbi egyenlőség nem lehet igaz, emiatt hamis volt a kiinduló feltevés. Tehát igaz a feladat állítása. √ 3 887. Legyen f (x) = x 3 − 6, ekkor f −1 (x) = x + 6. Az egyenlet f (f (x)) = f −1 (x) alakba írható, azaz f (f (f (x))) = x. Belátjuk, hogy ez csak úgy teljesül, ha f (x) = x. Ha f (x) = x, akkor f (f (x)) = f (x) = x, ezért f (f (f (x))) = f (x) = x. Ha pl. f (x) < x, akkor — mivel az f (x) függvény szigorúan monoton növekvő — f (f (x)) < f (x) < x, ezért f (f (f (x))) < f (x) < x. (Hasonlóképpen látható be, hogy az f (x) > x feltevés is ellentmondásra vezet.) Oldjuk meg az x 3 − 6 = x (f (x) = x) egyenletet! x 3 − x − 6 = 0, (x − 2)(x 2 + 2x + 3) = 0. Az egyenlet egyetlen valós gyöke: x = 2. 888. Az egyenlet (x 2 + x − 2)3 + x 2 + x − 2 = x 3 + x alakban is írható. Legyen f (x) = = x 3 + x, g(x) = x 2 + x − 2, h(x) = x. Az egyenlet f (g(x)) = f (h(x)) alakú. Mivel az f (x) függvény szigorúan monoton növekvő, ezért √ √ g(x) = h(x), azaz x 2 + x − 2 = x. Ennek gyökei: x1 = − 2, x2 = 2. 889. A p és q gyökök kielégítik az egyenletet, p2 + p2 + q = 0, q 2 + pq + q = 0.  tehát  1 1 A (p; q) megoldások: (0; 0), (1; −2), − ; − . 2 2

Megoldások, útmutatások

250

890. Ha x páratlan szám, akkor az x 10 +px 7 +q = 0 egyenlet bal oldalán három páratlan számot adunk össze; azok összege páratlan, így nem lehet egyenlő 0-val. Ha x páros szám, akkor az x 10 + px 7 + q = 0 egyenlet bal oldalán két páros és egy páratlan számot adunk össze; azok összege páratlan, így nem lehet egyenlő 0-val. Így beláttuk, hogy az x 10 + px 7 + q = 0 egyenletnek nincs egész gyöke, ha p és q páratlan egészek.

19. Egyenlőtlenségek 1 1 1 1 , , , ... , (1000 db szám). 1000 1000 1000 1000 892. Lehet. Pl.: 8 és −7. 891. Lehet. Pl.:

893. 1 = (a1 + a2 + · · · + a100 )50 . A hatványozást elvégezve megkapjuk a feladatban említett 50 tényezős szorzatokat, s az összegnek lesznek ezektől különböző pozitív tagjai is. √ 894. Ha 0 < a < 1, akkor a < a < 1. Ezért az első 60 tizedesjegy mindegyike 9. 895. Az összeadandók mindegyike legalább 1. 896. Szorozzuk az egyenlőtlenség mindkét oldalát gyike legalább 1.



n-nel. Az összeadandók minde-

1 . 2n 1 Az összeadandók mindegyike legalább 2 . n Hasonló az előzőkhöz. 1 1 Az összeg első ötven tagja legalább , a következő száz tag legalább . 100 200 Hasonló az előzőhöz, csak most a bontást 2-hatványok szerint végezzük.

897. Az összeadandók mindegyike legalább 898. 899. 900. 901.

1 1 , a következő nyolc kisebb, mint , ill. az első hat 5 8 1 1 tag legalább , a következő hét összeadandó legalább . 10 17

902. Az első öt tag legfeljebb

903. Használjuk fel, hogy n! > 2n−1 , ha n > 2. 904. Használjuk fel, hogy n2 > n(n − 1), továbbá támaszkodjunk az 1040. feladat eredményére.

19. Egyenlőtlenségek

251

906. Az összeg tagjai rendre kisebbek, mint 4, 5, 6.        √ 907. 100 + 99 + 98 + · · · + 2 + 1 <        √ < 110 + 110 + 110 + · · · + 110 + 121 = 11. Teljes indukcióval általánosabb állítás is igazolható:        √ √ Ha An = n + n − 1 + n − 2 + · · · + 2 + 1, akkor An < n+1. Sőt (teljes  1 1 indukcióval is) igazolható, hogy An < n + + . 2 2        √ 2 908. 1 + n + n + n3 + · · · + nn =             2  √  √ 1 n n n3 nn = n· + 2 + + + · · · + n, hiszen a törtek mindn < 2 4 8 2 n n n n n annyian egy természetes szám reciprokai, és így biztosan kisebbek 2-nél; ami azt jelenti, hogy minden gyökjel alatt 4-nél kisebb szám áll. (Ugyanis, ha ai < 2,     √ √ √ √ i = 1, 2, . . . , akkor a2 + a1 < 2 + 2 < 4 = 2, így a3 + a2 + a1 < √ √ < 2 + 2 = 2, és így tovább.) 2 n ≤ n, ha n > 3. Ezért az n = 2, n = 3 esetekre még ellenőrízni kell a feladat egyenlőtlenségét.   √ 909. Legyen an = 1 + 2 + · · · + n és    √ bn = 1 + 2 + · · · + n − 2 + 2n − 1 (n ≥ 2). Ha an kifejezésében a legbelső gyökjel alá n helyett n2 -et írunk, akkor éppen bn -et kapjuk; és mivel ezzel an értékét növeltük, fennáll az an < bn (n ≥ 2) egyenlőtlenség. Ha most bn kifejezésében a legbelső gyökjel alá 2n − 1 helyébe (n − 1)2 -et írunk, akkor éppen bn−1 -et kapjuk. Ezzel ismét növeltünk, vagy legalábbis nem csökkentettünk; feltéve, hogy 2n − 1 ≤ (n − 1)2 , ami n ≥ 4 esetén teljesül. Eszerint a b3 , b4 , . . . , bn sorozat monoton fogyó,  és így n ≥ 3 mellett b3 ≥ √ ≥ bn > an . Emiatt elég belátni, hogy b3 = 1 + 5 < 9/5 fennáll. Végül  √ a1 = 1 < a2 = 1 + 2 < b3 miatt igaz állításunk n = 1 és n = 2 mellett is.

Megoldások, útmutatások

252

910. 1. Általánosabb állítást igazolunk: ha n ≥ 2, akkor  megoldás.   √ m (m + 1) . . . n < m + 1, ahol 2 ≤ m ≤ n. A bizonyítást „fordított indukcióval” végezzük, azaz lefelé lépkedünk. m = n √ esetén a n < n + 1 állítás igaz. Feltehetjük, hogy lefelé, valamely m-ig igaz az egyenlőtlenség, az indukciós lépésben m − 1-re látjuk be az állítást.        √   √ (m − 1) m (m + 1) . . . n < (m − 1)(m + 1) = m2 − 1 < m2 = m = = (m − 1) + 1. 2. megoldás. A súlyozott számtani-mértani közép közötti egyenlőtlenség szerint, ha a1 , a2 , . . . , ar > 0, p1 , p2 , . . . , pr > 0 és p1 + p2 + · · · + pr = 1, akkor p p a1 1 a2 2 . . . arpr ≤ 1 a1 + p2 a2 + · · · + pr ar . p      √ n−1 Legyen xn = 2 3 4 . . . n = 21/2 · 31/4 · . . . · n1/2 . A kitevők összege: 1 1 1 1 + + · · · + n−1 = 1 − n−1 . Legyen a > 0. 2 4 2 2 1 1 1 1 n−1 Ekkor xn a 1/2 ≤ · 2 + · 3 + · · · + n−1 · n + n−1 · a. 2 4 2 2 1 1 1 n+2 Tudjuk, hogy · 2 + · 3 + · · · + n−1 · n = 3 − n−1 (ez igazolható pl. teljes 2 4 2 2 indukcióval).   n+2 a n−1 Így xn ≤ a −1/2 3 − n−1 + n−1 minden a > 0 esetén. Ha a = 1, akkor 2 2 n+1 megkapjuk a bizonyítandó egyenlőtlenséget: xn ≤ 3 − n−1 < 3. 2 a a b b 911. > és > . 1+a 1+a+b 1+b 1+a+b Adjuk össze az egyenlőtlenségeket. 912.

1 1 1 1 1 > , azaz + > 0, továbbá > 0. Utóbbi két b−c a−c b−c c−a a−b egyenlőtlenséget adjuk össze.

914. (a − b)(b − c)(a − c) > 0. 915. (a 2 − b2 )(a − b) ≥ 0. 916. (a 3 − b3 )(a − b) ≥ 0.   1 1 3 3 917. (a − b ) − ≥ 0. b a

19. Egyenlőtlenségek

253

919. Belátjuk, hogy a m+n + bm+n a m + bm a n + bn · ≤ teljesül, ha m és n azonos párosságú. 2 2 2 Átrendezve az egyenlőtlenséget, előbb a 2(a m+n + bm+n ) − (a m + bm )(a n + bn ) ≥ 0, majd az (a m − bm)(a n − bn ) ≥ 0 egyenlőtlenséghez jutunk. 920. Az előbb leírt egyenlőtlenség szerint: a + b a 3 + b3 a 4 + b4 a 2 + b2 a 4 + b4 a 6 + b6 · ≤ és · ≤ . 2 2 2 2 2 2 Ezekből megkapjuk a feladat állítását. 921. a) (x + 1)2 + (y − 2)2 + 1. b) (x 3 + x + 1)2 − 2. c) 2x 2 − 8xy + 17y 2 − 16x − 4y + 2062 = 2(x − 2y − 4)2 + 9(y − 2)2 + 1994, ill. = (x − 4y)2 + (x − 8)2 + (y − 2)2 + 1994. 922. Átrendezés, átalakítás után az (a − b)2 + (b − c)2 + (c − a)2 ≥ 0 egyenlőtlenséget kapjuk. 923. (2a − 1)2 + (2b − 1)2 + (2c − 1)2 ≥ 0. 924. (a 2 − b2 )2 + 2(ab − 1)2 ≥ 0.     1 2 1 2 1 925. a 4 − a + 1 = a 2 − + a− + > 0. 2 2 2 926. Átalakítások után: (a − b)2 + (a − 1)2 + (b − 1)2 ≥ 0. a

2 927. Átalakítások után: − b + c ≥ 0. 2   2 2 b b +3 928. a − − 1 + (c − 1)2 ≥ 0. 2 2 929. (a − b)2 + (a − c)2 + (a − d)2 + (b − c)2 + (b − d)2 + (c − d)2 + (b + c + d − a)2 ≥ 0.       2 b 2 3 2c 2 2 3d 2 930. a 2 +b2 +c2 +d 2 −ab−bc−cd −d + = a − + + + b− c− 5 2 4 3 3 4   5 4 2 + d− ≥0 8 5 a

2 a

2 a

2 931. a 2 + b2 + c2 + d 2 + e2 − a(b + c + d + e) = −b + −c + −d + 2 2 2 a

2 + − e ≥ 0. 2 Megjegyzés. Az x12 + x22 + · · · + xn2 ≥ xn (x1 + x2 + · · · + xn−1 ) egyenlőtlenség csak n = 2, 3, 4 és 5 esetén igaz.

254

Megoldások, útmutatások

Ugyanis a bal és jobb oldal különbsége: xn 2 xn 2 xn 2 x2 + x2 − + · · · + xn−1 − + (5 − n) n . x1 − 2 2 2 4  2  2

2 b c a b c a + + ≥ 0. 932. − − − b c c a a b  2 2  2 2     a b a a b b 2 933. −1 + −1 + − + + − 2 ≥ 0. b2 a2 b a b a 1 1 4 + ≥ egyenlőtlenséget. x y x+y √ √ √ 936. a + b ≥ 2 ab, b + c ≥ 2 bc, c + a ≥ 2 ca. Szorozzuk össze az egyenlőtlenségeket. a+b bc b+c ca c+a ab 938. ≤ , ≤ , ≤ . Ezeket adjuk össze. a+b 4 b+c 4 c+a 4 935. Használjuk fel az

939. a 2 + b2 ≥ 2ab, így (a 2 + b2 )c ≥ 2abc. Hasonlókat kapunk b, c-re és c, a-ra. Ezeket adjuk össze. √ 940. ab(a + b) ≥ 2ab ab. Hasonlót írjunk fel a másik két tagra is, s adjuk össze a három egyenlőtlenséget, s a kapott egyenlőtlenség jobb oldalára alkalmazzuk a közepek közti egyenlőtlenséget. 941. Hasonló az előzőhöz. 942. A bal oldalon álló szorzat mindkét tényezőjére írjuk fel a közepek közti egyenlőtlenséget, s ezeket szorozzuk össze.  ab bc ab bc 943. + ≥2 · = 2b. Írjunk fel hasonlókat és adjuk össze ezeket. c a c a 944. Legyen A = a + b − c, B = b + c − a, C = c + a − b. Ekkor az egyenlőtlenség (A+B)(B +C)(C +A) ≥ 8ABC alakban írható, s ez megegyezik a 936. feladattal. √ √ a 2 + (a 2 + c2 ) b2 + (b2 + c2 ) 945. a a 2 + c2 ≤ , b b2 + c2 ≤ , ezeket adjuk össze. 2 2 957. A pozitív számok súlyozott számtani és mértani közepei közt fennálló egyenlőtlenség felhasználásával 8 + 2 + 11 ab5 + bc5 + ca 5 = (ab5 + bc5 + ca 5 ) = 21 8ab5 + 2bc5 + 11ca 5 11ab5 + 8bc5 + 2ca 5 2ab5 + 11bc5 + 8ca 5 = + + ≥ 21 21 21 ≥ [(ab5)8 (bc5 )2 (ca 5 )11 ]1/21 +[(ab5 )11 (bc5 )8 (ca 5 )2 ]1/21 +[(ab5 )2 (bc5 )11 (ca 5 )8 ]1/21 = = a 3 b2 c + b3 c2 a + c3 a 2 b = abc(a 2b + b2 c + c2 a). Egyenlőség pontosan akkor áll, ha ab5 = bc5 = ca 5 , azaz a = b = c.

19. Egyenlőtlenségek

255

958. Bizonyítandó, hogy a 4 + b4 + c4 ≥ abc(a + b + c) = a 2 bc + b2 ac + c2 ab, azaz a 4 + b4 + c4 − a 2 bc − b2 ac − c2 ab ≥ 0. Ez igaz, mert 4a 4 + 4b4 + 4c4 − 4a 2 bc − 4b2 ac − 4c2 ab = (a 2 − b2 )2 + (b2 − c2 )2 + + (c2 − a 2 )2 + 2(a 2 − bc)2 + 2(b2 − ca)2 + 2(c2 − ab)2 ≥ 0. Második bizonyítás.         a 4 + b4 + c4 a+b+c a+b+c a+b+c 4 a+b+c 3 ≥ = · ≥ ·abc. 3 3 3 3 3 Harmadik bizonyítás. a 4 + b4 + c4 = (a 2 )2 + (b2 )2 + (c2 )2 ≥ a 2 b2 + b2 c2 + c2 a 2 = (ab)2 + (bc)2 + (ca)2 ≥ ≥ (ab)(bc) + (bc)(ca) + (ca)(ab) = abc(a + b + c). 963. A súlyozott számtani és mértani közép közötti egyenlőtlenséget fogjuk felhasz1 2 4 nálni az , , súlyokkal. 7 7 7 a 3 b + b3 c + c3 a =      1 3 2 3 4 3 2 3 4 3 1 3 4 3 1 3 2 3 = a b+ b c+ c a + a b+ b c+ c a + a b+ b c+ c a ≥ 7 7 7 7 7 7 7 7 7 ≥ (a 3 b)1/7 (b3 c)2/7 (c3 a)4/7 + (a 3 b)2/7 (b3 c)4/7 (c3 a)1/7 + (a 3 b)4/7 (b3 c)1/7 (c3 a)2/7 = = abc2 + ab2c + a 2 bc = abc(a + b + c). Más bizonyítás. A bizonyítandó egyenlőtlenség a triviális 0 ≤ ab(a − c)2 + bc(b − a)2 + ca(c − b)2 egyenlőtlenség átrendezése. 977. Használjuk fel, hogy a pozitív x, y és z valós számokra x 3 + y 3 + z3 x 2 + y 2 + z2 ≥ . x+y+z 3 a 3 + b3 + c3 b3 + c3 + d 3 c3 + d 3 + a 3 d 3 + a 3 + b3 + + + ≥ a+b+c b+c+d c+d+a d +a+b a 2 + b2 + c2 b2 + c2 + d 2 c2 + d 2 + a 2 d 2 + a 2 + b2 ≥ + + + = 3 3 3 3 = a 2 + b2 + c2 + d 2 . Igazolnunk kell még a felhasznált egyenlőtlenséget. Általánosabb egyenlőtlenséget igazolunk. Legyen xi > 0, i = 1, 2, . . . , n, és k természetes szám. Ekkor x1k + · · · + xnk x k+1 + · · · + xnk+1 ≤ 1 . n x1 + · · · + xn Ezt igazoljuk. xi Az ai = helyettesítéssel egyenlőtlenségünk az x1 + · · · + xn a1k + · · · + ank (1) ≤ a1k+1 + · · · + ank+1 n

Megoldások, útmutatások

256

alakot ölti. Elegendő belátni ezt az egyenlőtlenséget. Teljes indukcióval bizonyítunk. k = 0 esetén igaz az állítás. Feltehetjük, hogy az állítás teljesül (k − 1)-re. (1) bal oldalát átalakítva és a Schwarz–Cauchy– Bunyakovszkij egyenlőtlenséget alkalmazva: (k−1)/2

(∗)

a1k + · · · + ank (k+1)/2 a1 = a1 · n n  ≥

(a1k+1

+ ···+

ank+1 )1/2

(k−1)/2

+ · · · + an(k+1)/2 · a1k−1 ank−1 + · · · + n2 n2

an

n



1/2

.

a1k−1 + · · · + ank−1 ≤ a1k + · · · + ank . n  1/2 k−1 ank−1 k+1 k+1 1/2 a1 + ···+ 2 ≤ (a1 + · · · + an ) n2 n  k 1/2 ank k+1 k+1 1/2 a1 (a1 + · · · + an ) + ···+ . n n (∗) és az utóbbi összevetéséből:  k 1/2 ank a1k + · · · + ank k+1 k+1 1/2 a1 ≤ (a1 + · · · + an ) + ···+ n n n  k  1/2 a1 + · · · + ank ≤ (a1k+1 + · · · + ank+1 )1/2 n a1k + · · · + ank ≤ a1k+1 + · · · + ank+1 . n

Használjuk az indukciós feltevést:

Megjegyzés. Az igazolt általánosabb egyenlőtlenség mutatja, hogy a feladat általánosítható. 978. A számtani és mértani közép közti egyenlőtlenség alapján felírhatók az alábbi egyenlőtlenségek: a 3 + a 3 + b3 ≥ 3a 2 b a 3 + a 3 + c3 ≥ 3a 2 c b3 + b3 + c3 ≥ 3b2 c a 3 + b3 + b3 ≥ 3ab2 a 3 + c3 + c3 ≥ 3ac2 b3 + c3 + c3 ≥ 3bc2 2(a 3 + b3 + c3 ) ≥ 6abc Ezek összege a feladatbeli egyenlőtlenséget adja. (Vegyük figyelembe: 3(a + b)(b + c)(c + a) = 3a 2 b + 3ab2 + 3a 2 c + 3ac2 + 3b2 c + 3bc2 + 6abc.)  (4a + 1) + 1 980. (4a + 1) · 1 ≤ = 2a + 1, s hasonlóan a többi. 2 √ √ √ Így 4a + 1 + 4b + 1 + 4c + 1 < (2a + 1) + (2b + 1) + (2c + 1) = 5.

19. Egyenlőtlenségek

257

981. Hasonló az előzőhöz.

√ 995. (a 2 + b2 ) + (c√2 + d 2 ) ≥ 2(ab + cd) ≥ 4 abcd = 4. ab + cd ≥ 2√abcd = 2. bc + ad ≥ 2√bcad = 2. ac + bd ≥ 2 acbd = 2. √ 996. 1 + ai ≥ 2 ai , ezeket szorozzuk össze.

  1 1 1 1 ,q = , ahol 0 ≤ x < 1/2. + = 1. Ekkor 1/2 + x  1/2 − x  p   q  1 1 1 1 1 1 1 1 + = − + − = 1− + = p(p +  1) q(q + 1) p p + 1 q q+1 p+1 q+1 1 1 3/4 − x 2 =1− + = 1 − 2 · . 1 1 9/4 − x 2 1/2+x + 1 1/2−x + 1 3/4 − x 2 1 3/4 − x 2 1 1 Könnyen ellenőrizhető, hogy ≥ > , ezért ≤ 1 − 2 · < 3 9/4 − x 2 4 3 9/4 − x 2 1 1 1 1 1 < , azaz ≤ + < . 2 3 p(p + 1) q(q + 1) 2 1 1 Most igazoljuk az + ≥ 1 egyenlőtlenséget! p(p − 1)   1) q(q −   1 1 1 1 1 1 1 1 + = − + − = + −1= p(p − 1) q(q − 1) p−1 p q−1 q p−1 q −1 1 1 1 1 1/2 + x 1/2 − x + 1 − 1 = 1/2−x + 1/2+x − 1 = = 1 + −1 = 1/2 − x 1/2 + x − 1 1/2−x − 1 1/2+x 1/2+x 1/2−x

997. Legyen p =

=2·

1/4 + x 2 − 1 ≥ 2 − 1 = 1. 1/4 − x 2

998. Legyen az n db kis négyzet oldala rendre a1 , a2 , . . . , an hosszúságú. Ekkor a négyzetek területének összege, a12 + a22 + · · · + an2 legfeljebb a nagy négyzet területét adja, ezért a12 + a22 + · · · + an2 ≤ 1. A számtani és a négyzetes közép közötti egyenlőtlenségszerint a1 + a2 + · · · + an ≤ n

√ a12 + a22 + · · · + an2 n ≤ √ = n. n n

999. Legyen a kis kockák éle rendre b1 , b2 , . . . , bn . Ekkor a kockák térfogatának összege legfeljebb 1; b13 +b23 +· · ·+bn3 ≤ 1. A számtani és a harmadik hatványközép közötti egyenlőtlenségszerint 3 3 3 √ 3 b + b2 + · · · + bn 3 b1 + b2 + · · · + bn ≤ n 1 ≤ n2 = n2/3 . n  √ a 2 + b2 (a − b)2 + 2ab 2 2 = = (a − b) + ≥ 2 (a − b) = 2 2. 1000. a−b a−b a−b a−b

258

Megoldások, útmutatások

a 2 − b2 2ab a 2 + b2 =a+b >a+b− = . a−b a+b a+b 1 1 1 1002. a) + = + log2 π > 2 (lásd a 934. a) feladatot!). log2 π logπ 2 log2 π 1 1 b) + = logπ 2 + logπ 5 = logπ 10 > 2. log2 π log5 π 1001.

1003. Aközepek közti egyenlőtlenség szerint a bal oldali összeg nagyobb, mint 4 3 > 4 · 1,1. 4 2 1004. Ha 0 < sin x < 1, akkor sin2 x < sin x, ugyanígy ha 0 < cos x < 1, akkor cos2 x < cos x. Továbbá sin2 x + cos2 x = 1. Ezért 1 = sin2 40◦ + cos2 40◦ < sin 40◦ + cos 40◦ . √ 1005. Ha 0 < sin x < 1, akkor√sin2 x < sin x < sin x, ugyanígy ha 0 < cos x < 1, 2 2 akkor cos2 x < cos x < cos x.√Továbbá sin √ x + cos x = 1. 2 ◦ 2 ◦ ◦ ◦ Ezért 1 = sin 40 + cos 40 < sin 40 + cos 40 . 1006. Írjuk fel a közepek közti egyenlőtlenséget az 1, 2, 3, . . . , n számokra. 1007. Írjuk fel a közepek közti egyenlőtlenséget az a, b, b, . . . , b (n db b) számokra. 1 1 1 1008. Írjuk fel a közepek közti egyenlőtlenséget az n + 1 db 1 + , 1 + , . . . , 1 + , n n n 1 1 számra. (Az előző feladat speciális esete: a = 1, b = 1 + .) n 1 1 1 1009. Írjuk fel a közepek közti egyenlőtlenséget az n + 2 db 1 + , 1 + , 1 + , . . . , n n n 1 1 1 1 + , , számra. n 2 2 √ √ √ √ √ 3 3 3 1010. Írjuk fel a közepek közti egyenlőtlenséget a a, a, b, b, b számokra. 1011. Írjuk fel az  a közepek  köztiegyenlőtlenséget   a 1 b 1 b 1 3 c 1 3 c 1 3 c , , , , , számokra. b 2 c 2 c 3 a 3 a 3 a 1 1012. Mivel x(1 − x) ≤ (következik a közepek közti egyenlőtlenségből, vagy függ4 vényábrázolás is mutatja), így az egyenlőtlenségek bal oldalainak szorzatára  3 1 . a(1 − b)b(1 − c)c(1 − a) ≤ 4  √ a a 1014. a) x + ≥ 2 x = 2 a. x x  x3 + x + 2 1 1 1 1 4 2 b) = x + 1 + + ≥ 4 x 2 · 1 · · = 4. x x x x x

19. Egyenlőtlenségek

259

1015. (1 − x)3 (1 + 3x) = (1 − x)(1 − x)(1 − x)(1 + 3x) ≤   (1 − x) + (1 − x) + (1 − x) + (1 + 3x) 4 = 1. ≤ 4    4 (1 + x) + (1 + x) + (1 + x) + 3(1 − x) 4 3 3 1016. 3(1 + x) (1 − x) ≤ = . 4 2 1017. Legyen a négy gyök x1 , x2 , x3 , x4 . A gyökök és együtthatók közti összefüggéseket felhasználva írjuk fel a közepek közti egyenlőtlenséget.  x1 + x2 + x3 + x4 ≥ 4 x1 x2 x3 x4 = 1, 4 tehát x1 = x2 = x3 = x4 = 1. Ahonnan a következő azonosságot kapjuk: x 4 − 4x 3 + ax 2 − bx + 1 = (x − 1)4 . 1=

1019. Az egyenlőtlenség négyzetreemelés és rendezés után 0 ≤ (n − 3)(a 4 + b4 + c4 ) + (a 2 − b2 )2 + (b2 − c2 )2 + (c2 − a 2 )2 alakban írható. Innen látható, hogy a keresett érték n = 3. 1020. k =

2 . 3

1021. 1 +

(k + 1)2 2(n + 1) 1 = , a szorzat értéke . k(k + 2) k(k + 2) n+2

1024. Az első egyenlőtlenség másképp írva: x 2 < (y − z)2 , azaz (x + y − z)(x − y + z) < 0. Hasonló módon írjuk át a másik két egyenlőtlenséget is, majd az így nyert három egyenlőtlenséget szorozzuk össze. Így az ((x − y + z)(x + y − z)(y + z − x))2 < 0 egyenlőtlenséghez jutunk, melynek nincs megoldása. 1026. n! < (n + 1)n , (n!)n · n! < (n!)n · (n + 1)n , (n!)n+1 < [(n + 1)!]n . 1028–1030. Képezzük a sorozat egymás utáni elemeinek hányadosát.       ak 1 1 1 ak 1 ak 1 k+1 2 1031. 1 + 1+ 2 =1+ 2 + + ≥1+ 2 +2 · = . ak k k ak k 2 k ak k 2 k   2   n  n   1 ak k+1 Ezért 1+ 1+ 2 ≥ = (n + 1)2 . ak k k k=1 k=1 1033. Sp + Sq − Spq = (Sp + Sq − Sp ) − (S2p − Sp ) − (S3p − S2p ) − · · · − (Sqp − S(q−1)p ) ≤ 1 1 1 ≤ Sq − p · −p· − ··· − p · = 1. 2p 3p qp

260

Megoldások, útmutatások

20. Különféle algebrai feladatok 1034. a) Nulla, hiszen szorzunk nullával. b) Nulla. 1035. Használjuk az a 2 − b2 = (a − b)(a + b) azonosságot. 4 a) . 5 b) 96 · 104 = (100 − 4) · (100 + 4) = 10000 − 16 = 9984. c) 899 = 900 − 1 = 302 − 12 = 29 · 31. d) 77782 − 22232 = (7778 − 2223)(7778 + 2223) = 5555 · 10001. 1036. a) x 4 + x 3 + 2x 2 + x + 1 = (x 4 + x 3 + x 2 ) + (x 2 + x + 1) = (x 2 + x + 1)(x 2 + 1). b) x 3 + 2x 2 + 2x + 1 = (x 3 + x 2 ) + (x 2 + x) + (x + 1) = (x + 1)(x 2 + x + 1). c) x 4 + 2x 3 + 2x 2 + 2x + 1 = (x 4 + x 3 ) + (x 3 + x 2 ) + (x 2 + x) + (x + 1) = = (x + 1)(x 3 + x 2 + x + 1) = (x + 1)2 (x 2 + 1). d) x 4 + 2x 3 + 3x 2 + 2x + 1 = (x 4 + x 3 + x 2 ) + (x 3 + x 2 + x) + (x 2 + x + 1) = 2 = (x 2 + x + 1)(x 2 + x + 1) = (x 2 + x + 1) . e) x 4 + x 2 + 1 = x 4 + 2x 2 + 1 − x 2 = (x 2 + 1)2 − x 2 = (x 2 + x + 1)(x 2 − x + 1). f) x 10 + x 8 + x 6 + x 4 + x 2 + 1 = (x 4 + x 2 + 1)(x 6 + 1) = = (x 2 + x + 1)(x 2 − x + 1)(x 2 + 1)(x 4 − x 2 + 1). g) x 4 + 4 = x 4 + 4x 2 + 4 − 4x 2 = (x 2 + 2)2 − (2x)2 = (x 2 + 2x + 2)(x 2 − 2x + 2). h) x 4 −7x 2 +1 = x 4 +2x 2 +1 −9x 2 = (x 2 +1)2 −(3x)2 = (x 2 +3x +1)(x 2 −3x +1). i) x 5 + x 4 + 1 = (x 5 + x 4 + x 3 ) − (x 3 − 1) = x 3 (x 2 + x + 1) − (x − 1)(x 2 + x + 1) = = (x 3 − x + 1)(x 2 + x + 1). 1037. x 9 + y 9 = (x + y)(x 8 − x 7 y + x 6 y 2 − x 5 y 3 + x 4 y 4 − x 3 y 5 + x 2 y 6 − xy 7 + y 8 ) = = (x 3 + y 3 )(x 6 − x 3 y 3 + y 6 ) = (x + y)(x 2 − xy + y 2 )(x 6 − x 3 y 3 + y 6 ). Innen x 8 −x 7 y+x 6 y 2 −x 5 y 3 +x 4 y 4 −x 3 y 5 +x 2 y 6 −xy 7 +y 8 = (x 2 −xy+y 2 )(x 6 −x 3 y 3 +y 6 ). 1038. 2x 3 + 2y 3 − 3x 2 − 3y 2 + 1 = (x + y − 1)(2x 2 − 2xy + 2y 2 − x − y − 1). 1039. Az a 2 − b2 = (a − b)(a + b) azonosságot használva: 1002 − 992 + 982 − 972 + · · · + + 22 − 12 = 100 + 99 + 98 + 97 + · · · + 2 + 1 = 5050. 1 1 1 1 1 1 1 1 1 1040. Az = − azonosság alapján az összeg: − + − + − + k(k + 1) k k + 1 1 2 2 3 3 4 1 1 1 n + ···+ − =1− = , (n = 99). n n+1 n+1 n+1 2k + 1 1 1 1 1041. 2 = − . Végeredmény: 1 − . k (k + 1)2 k 2 (k + 1)2 (n + 1)2

20. Különféle algebrai feladatok

261

1042. k · k! = (k + 1)! − k!. Végeredmény: (n + 1)! − 1. k 1 1 1 = − . Végeredmény: 1 − . 1043. (k + 1)! k! (k + 1)! (n + 1)! k+1 k+2 = . Ezt alkalmazva, a feladat azonos az előzővel. 1044. k! + (k + 1)! + (k + 2)! (k + 2)! √ √ √ 1 1045. √ = k + 1 − k. Végeredmény: n + 1 − 1. √ k+ k+1 1 1 1 1 1046. √ =√ −√ . Végeredmény: 1 − √ . √ k (k + 1) k + k k + 1 k+1 n+1 1047. tg 2k α = ctg 2k α − 2 ctg 2k+1 α. Végeredmény: ctg α − 2n+1 ctg 2n+1 α.   1 1 sin(k + 1)α sin kα = − 1048. cos kα · cos(k + 1)α  sin α cos(k + 1)α  cos kα 1 sin(n + 1)α sin α Végeredmény: − . sin α cos(n + 1)α cos α 1 1049. = ctg 2k−1 α − ctg 2k α. Végeredmény: ctg α − ctg 2n α. sin 2k α      1 1 3 4 5 101 101 1 1050. 1 + 1+ ... 1 + = · · · ...· = . 2 3 100 2 3 4 100 2      1 1 1 1051. 1 − 2 1 − 2 ... 1 − = 1002    2  3     1 1 1 1 1 1 = 1− 1+ 1− 1+ ... 1 − 1+ = 2 2 3 3 100 100 1 3 2 4 3 5 4 100 99 101 = · · · · · · ·...· · · = 2  2 3  3 4 4 5  99  100 100  3 2 4 3 5 4 100 99 101 1 · · · · · ·...· · · = = · 2 2 3 3 4 4 5 99 100 100 1 101 101 = · = . 2 100 200   √ √ 1 1 1 1 k · k 1052. = = − = √ (k +1)k k  k+1 (k + 1) k    √ √ 1 1 1 1 1 1 = k √ +√ . Mivel k √ + √ < 2, ezért √ −√ k k  k+1 k k+1 k + 1 1 1 1 √ = 1 + . Ebből következik, hogy > xi+1 xi − xi2 xi (1 − xi ) xi xi x1001 1 = 2000, azaz x1001 < 0, 0005. > 1000 + x1 √ √ √ √ (4 + 15)3/2 + (4 − 15)3/2 (8 + 2 15)3/2 + (5 − 2 15 + 3)3/2 1056. √ √ = √ √ = (6 + 35)3/2 − (6 − 35)3/2 (12 + 2 35)3/2 − (7 − 2 35 + 5)3/2 √ √ √ √ √ √ ( 5 + 3)3 + ( 5 − 3)3 2(5 5 + 9 5) 7 = √ √ √ = √ √ = . √ 3 3 13 ( 7 + 5) − ( 7 − 5) 2(21 5 + 5 5)    4 1  4 1 2 + 4 4 + 4 · . . . · 204 + 14 (44 + 4)(84 + 4) . . . (404 + 4)      1057.  = = (24 + 4)(64 + 4) . . . (384 + 4) 14 + 14 34 + 14 · . . . · 194 + 14 =

(32 + 1)(52 + 1)(72 + 1)(92 + 1) · . . . · (392 + 1)(412 + 1) = 841, (12 + 1)(32 + 1)(52 + 1)(72 + 1) · . . . · (372 + 1)(392 + 1)

hiszen n4 + 4 = (n2 − 2n + 2)(n2 + 2n + 2) = [(n − 1)2 + 1][(n + 1)2 + 1].   1 · 2 · 4 + 2 · 4 · 8 + 3 · 6 · 12 + · · · + 100 · 200 · 400 1/3 1058. = 1 · 3 · 9 + 2 · 6 · 18 + 3 · 9 · 27 + · · · + 100 · 300 · 900     1 · 2 · 4(13 + 23 + 33 + · · · + 1003 ) 1/3 8 1/3 2 = = = . 1 · 3 · 9(13 + 23 + 33 + · · · + 1003 ) 27 3 1059. Hamis. Legyen x = y = 1. Erre a helyettesítésre az egyenlőségjel két oldalán más-más szám áll. 1062. f (t) = t + et szigorúan monoton növekvő, ezért f (x) = f (y) csak úgy lehet, ha x = y. 1063. 0 = (a + b + c)2 = a 2 + b2 + c2 + 2(ab + bc + ca). 1064. Átalakítás után: (a − b)2 + (b − c)2 + (c − a)2 = 0. a b c = = . Ekkor kb = a, kc = b, ka = c. Szorozzuk össze ezeket b c a az egyenlőségeket, s azt kapjuk, hogy k 3 abc = abc, tehát k = 1.

1065. Legyen k =

1066. Mivel abc = 1 és ab + bc + ca = a + b + c, ezért abc − ab − bc − ca + a + b + c − 1 = 0, tehát (a − 1)(b − 1)(c − 1) = 0. 1067. Hasonló az előzőhöz.

20. Különféle algebrai feladatok

263

1068. Legyen p = a + b + c, q = ab + bc + ca, r = abc, f (x) = x 3 − px 2 + qx − r. Az f (x) függvény x ≤ 0 esetén negatív értékeket vesz fel, tehát zérushelyei pozitív számok, s ezek a zérushelyek a gyökök és együtthatók közti összefüggések miatt az a, b, c számok. 1070. 0 = x + y + z − 2(xy + yz + zx) + 4xyz −

1 1 = (2x − 1)(2y − 1)(2z − 1). 2 2

1072. x 2 + y 2 + z2 = (x + y + z)2 − 2(xy + yz + zx) = 0. 1073. Támaszkodjunk az előző feladatra. 1074. Az egyenlőségjel bal oldalán álló kifejezést átalakítjuk: z+x x+y y+z+x z+x+y x+y+z y+z +1+ + 1 + +1 = + + = x y z x y z   1 1 1 = (x + y + z) + + = 0. x y z 1075. Szorozzuk a megadott egyenlőség mindkét oldalát rendre x, y, z-vel, s adjuk össze az így kapott három egyenlőséget. x2 y2 z2 + + + y+z z+x x+y       xy xz yx yz zx zy + + + + + + = y+z y+z z+x z+x x+y x+y =x+y+z Mivel a zárójelekben levő kifejezések értéke rendre x, y, z, így a bizonyítandó állításhoz jutottunk. 1077. a) b5 = b3 · b2 = (b + 1)b2 = b3 + b2 = (b + 1) + b2 = b2 + b + 1. Mivel b4 + 1 = = b3 · b + 1 = (b + 1)b + 1 = b2 + b + 1, így b5 = b4 + 1. b) Szorozzuk meg a b5 = b4 + 1 egyenlőséget b-vel: b6 = b5 + b = b4 + 1 + b. Innen b6 − 1 = b4 + b, azaz (b3 − 1)(b3 + 1) = b(b3 + 1). Mivel b3 + 1 = 0, így b3 − 1 = b, azaz b3 = b + 1.      1 1 1 2 3 1078. x + 3 = x + − 3 = 0. x+ x x x     1 2 1 1 2 1079. x + = x + 2 + 2 = 7 + 2 = 9, így x + = 3. x x x     1 1 1 1 3 27 = x + = x3 + 3 + 3 x + , ezért x 3 + 3 = 18. x  x  x  x      1 1 1 1 1 2 3 5 5 126 = 7 · 18 = x + 2 · x + 3 = x + 5 + x + = x + 5 + 3, x x x x x 1 5 emiatt x + 5 = 123. x

264

Megoldások, útmutatások

1080. 0 = (a + b + c)2 = a 2 + b2 + c2 + 2(ab + bc + ca). Mivel a 2 + b2 + c2 = 1, így 2(ab + bc + ca) = −1. Innen 1/4 = (−1/2)2 = (ab + bc + ca)2 = a 2 b2 + b2 c2 + c2 a 2 + +2(ab·bc+bc·ca+ca·ab) = a 2 b2 +b2 c2 +c2 a 2 +2abc(b+c+a) = a 2 b2 +b2 c2 +c2 a 2 , 1 hiszen b + c + a = 0. Tehát a 2 b2 + b2 c2 + c2 a 2 = . 4 1 2 2 2 2 4 4 4 2 2 2 2 1 = (a + b + c ) = a + b + c + 2(a b + b c + c2 a 2 ) = a 4 + b4 + c4 + 2 · , 4 1 4 4 4 azaz a + b + c = . 2 1081. a 4 + b4 + c4 = a 2 (b + c)2 + b2 (a + c)2 + c2 (a + b)2 = = 2(a 2 b2 + b2 c2 + c2 a 2 ) + 2abc(a + b + c) = (a 2 + b2 + c2 )2 − (a 4 + b4 + c4 ), azaz 2(a 4 + b4 + c4 ) = (a 2 + b2 + c2 )2 . 1082. 0 = 2(a 8 + b8 + c8 ) − (a 4 + b4 + c4 )2 = = −(a 2 + b2 + c2 )(a 2 + b2 − c2 )(a 2 − b2 + c2 )(b2 + c2 − a 2 ). 1083. Az x 2 + ax + b + 1 = 0 egyenlet gyökei x1 és x2 . Ekkor a = −[x1 + x2 ], b + 1 = x1 x2 , b = x1 x2 − 1. Innen a 2 + b2 = (−[x1 + x2 ])2 + (x1 x2 − 1)2 = x12 · x22 + x12 + x22 + 1. Mivel a 2 + b2 = x12 · x22 + x12 + x22 + 1 = (x12 + 1)(x22 + 1), ahol (x12 + 1) > 1 és (x22 + 1) > 1 egészek, ezért a 2 + b2 két, 1-nél nagyobb egész szám szorzata, tehát összetett szám. 1085. a 3 = 3ab − 2c + 6xyz. 1086. (x + y + z)2 − (x 2 + y 2 + z2 ) = 9 − 5 = 4, tehát xy + yz + zx = 2. (x 2 + y 2 + z2 )2 − (x 4 + y 4 + z4 ) = 25 − 17 = 8; innen kapjuk, hogy x 2 y 2 + y 2 z2 + + z2 x 2 = 4. Az (xy + yz + zx)2 = x 2 y 2 + y 2 z2 + z2 x 2 + 2xyz(x + y + z) azonosság miatt xyz(x + y + z) = 0, azaz xyz = 0. 1087. Az egyenlet (x 2 − 2)2 = 5x 3 + 7x alakban is felírható. 1088. Ha teljesülne √ a megadott egyenlőség, √ akkor az (a − b · 3)2 = 99 999 − 111 111 · 3 egyenlőség is fennállna; de ez nem lehet, mert itt a jobb oldalon negatív szám áll. √ √ 1089. Ha teljesülne a megadott egyenlőség, akkor az (5 − 3 · 2)m = (3 − 5 · 2)n egyenlőség is fennállna; de ez nem lehet, mert a bal oldalon 1-nél kisebb, a jobb oldalon 1-nél nagyobb abszolút értékű szám áll. 1090. Hasonló az előzőkhöz. √ √ 1091. ( 26 + 5)1993 − ( 26 − 5)1993 értéke egész szám. Ezért azt kell belátni, hogy  1993 √ √ 1 1 1 1 ( 26 − 5)1993 < . 26 − 5 = √ < = . 10 5 + 5 10 26 + 5

20. Különféle algebrai feladatok

1092. (45 + (45 −

√ √

1975)30

1975)30

265

√ + (45 − 1975)30 értéke páros szám (= 2n), továbbá  30 √ 50 = < 1, emiatt [(45 + 1975)30 ] páratlan, √ 45 + 1975

hiszen = 2n − 1. Az említett összeg páros, mert √ √ (45 + 1975)30 + (45 − 1975)30 =

  28 30 26 30 2 2 14 15 = 2 4530 + 30 45 · 1975 + 45 · 1975 + · · · + 45 · 1975 + 1975 . 2 4 28

√ √ √ √ 19 1093. [(15 + 220)19 + (15 + 220)82 ] + [(15 + (15√− 220)82 ] értéke 10-zel √− 220) osztható egész szám, továbbá (15 − 220)19 + (15 − 220)82 < 1. Így a tizedesvessző előtti számjegy 9-es. 1094. A tizedesvessző melletti jegyek: . . . 7, 9 . . . . 1095. A diszkrimináns az m minden értékére pozitív, hiszen 4 az értéke, azonban szükséges még az m + 1 = 0 feltétel is. 1096. A gyökök és együtthatók közti összefüggés szerint a két gyök szorzata c, tehát c mindig negatív. 1097. Helyettesítsünk az x helyére 0-t. 1098. Az y = ax 2 + bx + c függvénynek a feltétel szerint nincs zérushelye. Nézzük a függvény értékét az x = 1 helyen: y = a + b + c < 0, tehát a parabola az y-tengely alatt helyezkedik el, a függvény értékei negatív számok. Így az x = 0 helyen is negatív értéket vesz fel, ez az érték c. 1099. Az f (x) = x 2 + bx + ac polinomra a feladat feltétele miatt f (c) = c2 + bc + + ac = (c + b + a)c < 0 teljesül. Az f (x) = x 2 + bx + ac függvényt ábrázoló parabola álló helyzetű és f (c) < 0 miatt előjelet vált, tehát a függvénynek vannak zérushelyei. Ez azt jelenti, hogy az x 2 + bx + ac = 0 egyenletnek vannak valós gyökei, tehát D = b2 − 4ac > 0. 1100. Az ábra szerint mindegyik parabolának van két zérushelye; ezért a 2 > 4bc, b2 > > 4ca és c2 > 4ab. Mivel a, b, c pozitív számok (hiszen a parabolák szárai felfelé mutatnak), az egyenlőtlenségek összeszorozhatók: a 2 b2 c2 > 64a 2 b2 c2 . Nem találunk olyan a, b, c valós számokat, amelyekre ez az egyenlőtlenség teljesülne. 1101. Három különböző parabolának legfeljebb 6 metszéspontja van. Az x = 1 helyen mindegyik függvény értéke a + b + c. Tehát az x = 1 helyen lennie kell egy újabb közös pontnak, ami nem lehet: ugyanis ez már a 7. metszéspont lenne. Tehát nincsenek olyan a, b, c valós számok, amelyek kielégítenék a feladat követelményeit. Azért sem léteznek ilyen parabolák, mert az x = 1 helyhez tartozó pont mindhárom parabola közös pontja, s ilyen esetben a három parabolának összesen legfeljebb 4 közös pontja lehetne.

266

Megoldások, útmutatások

1102. x = 1 mindig megoldás. 1104. Az f (x) = ax 2 + bx + c polinom 0 és 1 közötti zérushelyei x1 és x2 . Ekkor f (x) = = a(x − x1 )(x − x2 ). f (0) és f (1) pozitív egészek, ezért f (0) · f (1) ≥ 1, azaz 1 1 a 2 x1 (1 − x1)x2 (1 − x2) ≥ 1. Mivel x1 (1 − x1) ≤ , x2 (1 − x2 ) ≤ , és nem állhat 4 4 fenn mindkét helyen egyenlőség, így a 2 > 16, a ≥ 5. a = 5 lehetséges, pl.: 5x 2 − 5x + 1 = 0. 1105. Tekintsük az x = 0, x = 1, x = −1 helyettesítéseket.   1 a b 2 1107. f (x) = ax +bx +c, f (0) = c, f = + +c, f (1) = a +b +c. Mivel 0 = 2a + 2 4 2     1 1 + f (1), tehát az f (0), f , f (1) számok között + 3b + 6c = f (0) + 4f 2 2 vannak ellentétes előjelűek, így a polinomnak van 0 és 1 közötti zérushelye. 1108. a) Vizsgáljuk a bal oldali kifejezést. Ez x = a-ra pozitív, x = b-re negatív, x = c-re pozitív értéket vesz fel, tehát van zérushelye a és b, ill. b és c között. b–c) Hasonló a)-hoz. d) A bal oldali kifejezést jelölje f (x), s vizsgáljuk az f (a)·f (b) és az f (b)·f (c) szorzatokat. e–f) Vizsgáljuk a bal oldali kifejezés értékét x = 0 és x = 1 esetén. 1109. Legyen az Ai pont az (a1 + a2 + · · · + ai ; b1 + b2 + · · · + bi ) (i = 1, 2, . . . , n) koordinátákkal megadva. Az egyenlőtlenség bal oldalán az OA1 A2 . . . An töröttvonal hossza szerepel, amely nem rövidebb az OAn szakasznál, melynek hossza a jobb oldalon áll. (O = (0; 0)) 1110. Az a + b + c oldalú ABCD négyzet AB oldalára mérjük egymás után az a, c, b szakaszokat, a BC oldalra a b, a, c szakaszokat, s az osztáspontokon át húzzunk párhuzamosakat az oldalakkal. Kijelölhető A és C között egy három szakaszból álló töröttvonal úgy, hogy ennek hossza az egyenlőtlenség bal oldalán álló kifejezés, s ez nyilván legalább akkora, mint az AC átló. 1111. Vegyünk fel három, O pontból induló szakaszt! Az OA, OB, OC hossza rendre a, b, c, továbbá OA és OB, valamint OB és OC között 60◦ -os szög van. Ekkor a feladatban szereplő egyenlőtlenség az |AB| + |BC| ≥ |AC| egyenlőtlenséggel azonos. ab . 1112. Itt is segít a geometriai megfogalmazás. A keresett érték: x = a+c 1113. Hasonló az előzőkhöz. Legyen P (0; −1), Q(0; 1), X(x; x). A |P X| + |QX| ≥ |P Q| egyenlőtlenséget a koordináták segítségével felírva kapjuk a minimumot, 2-t.

20. Különféle algebrai feladatok

267

 1114. Hasonló az előzőhöz. Itt P

 √  √  1 2 2 2 − ; ,Q ;− , X(x; 0). 3 3 3 3

1 -nál lesz. 6 1115. O(0; 0), A(2a; 2b), B(a + c; b). Az |OB| + |BA| ≥ |OA| egyenlőtlenséget írjuk fel a koordinátákkal. Egyenlőség x =

1116. Az ABC derékszögű háromszög BC, CA, AB oldalainak hossza 3, 4, 5. A háromszög belsejében levő P pont olyan, hogy a BP C szög derékszög, a CP A y szög 120◦ -os. A P A, P B, P C szakaszok hosszát x, √ , z jelöli. Ekkor a három 3 egyenlet a három kis háromszögben a keresett kifejezés √ √ felírt koszinusz-tétel, pedig a háromszög területének 4 3-szorosa, 24 3. 2x 2y 2z ,z= ,x= . A kifejezések 2 2 1−x 1−y 1 − z2 emlékeztetnek tg 2α képletére. Legyen x = tg α, akkor y = tg 2α, z = tg 4α, kπ x = tg 8α. Ez azt jelenti, hogy tg α = tg 8α, melynek megoldása α = . Az 7 egyenletrendszer megoldásai: 2kπ 4kπ kπ x = tg , y = tg , z = tg , ahol k egész szám. 7 7 7

1117. Az egyenleteket átalakítva: y =

1118. Legyen x = cos α, akkor 2x 2 −1 = cos 2α, 8x 4 −8x 2 +1 = cos 4α. Így az egyenlet 8·cos α·cos 2α·cos 4α = 1, ezt sin α-val szorozva a sin 8α = sin α egyenlethez 2kπ (2n + 1)π jutunk. Ennek gyökei: α = és α = . Az egyenlet gyökei: x = 7 9 2kπ (2n + 1)π = cos és x = cos , ahol k és n egész számok. 7 9 1120.  Legyen a = cos2 x, b= cos2 y, c = cos2 z(0◦ ≤ x, y, z ≤ 90◦√ ). Ekkor a a(1 − b)(1 − c) + b(1 − a)(1 − c) + c(1 − a)(1 − b) − abc ≤ 1 egyenlőtlenség cos x sin y sin z + cos y sin x sin z + cos z sin x sin y − cos x cos y cos z ≤ 1 alakban írható. Ez pedig igaz, mert a bal oldali kifejezés = − cos(x + y + z). ◦ ◦ 1121. Legyenek  α ésβ olyan szögek, melyekre 0 < α, β < 45 és tg α = x, tg β = y. x−y   = | tg (α − β)|, és ez valóban kisebb 1-nél. Ekkor  1 + xy 

1122–1124. Hasonló az 1121. feladat megoldásához. 1125. Egységoldalú négyzetben kijelölhetünk a, b területű téglalapokat, melyek közös részének területe ab. Ekkor a két téglalap együttes területe a közös rész nélkül a + b − 2ab lesz, s ez kisebb a négyzet területénél.

Megoldások, útmutatások

268

1126. Hasonló az előzőhöz. Most egy egységoldalú szabályos háromszög egy-egy oldalára felmérjük a c, b és az a hosszúságú szakaszt úgy, hogy a háromszög csúcsainál rendre c és 1 − a, b és 1 − c, a és 1 − b hosszúságú szakaszok találkozzanak. Így a háromszög sarkaiban kijelöltünk egy-egy kisebb háromszöget. A bizonyítandó egyenlőtlenség geometriai tartalma már látható lesz: a három kis háromszög együttes területe kisebb, mint a szabályos háromszög területe. 1127. A bal oldali összeg összeszámlálja, hogy n-ig hány négyzetszám (12 , 22 , . . . ), hány köbszám (13 , 23 , . . . ), hány negyedik hatvány (14 , 24 , . . . ) stb. van. A jobb oldalon megszámláljuk, hogy n-ig mennyi 2-hatvány (21 , 22 , . . . ), 3-hatvány (31 , 32 , . . . ), 4-hatvány (41 , 42 , . . . ) stb. van. Ha a listákon álló első elemeket elhagyjuk, akkor a bal oldali listákon ugyanazok a számok maradnak meg (és mindegyik ugyanannyiszor), mint a jobb oldali listákon. p(x) függq(x) vénynek véges sok: annyi, ahány gyöke van a p(x) = 0 egyenletnek; s ha a p(x) polinom n-edfokú, akkor legfeljebb n.

1128. Az y = sin x függvénynek végtelen sok zérushelye van, míg az y =

1 1 1 1129. Legyen Sn = 1 + + + · · · + . Mutassuk meg, hogy lim Sn = ∞ és n→∞ 2 3 n Sn lim = 0. Lássuk be, hogy nincsenek olyan p(x) és q(x) polinomok, amen→∞ n p(n) p(n) lyekre lim = ∞ és lim = 0. n→∞ q(n) n→∞ n · q(n) ∞ ∞   k 1 + . (Végtelen sok tagú összegen csak akkor k k k 3 3 3 k=1 k=1 k=1 végezhetünk ilyen átalakításokat, ill. egyéb műveleteket, ha belátjuk, hogy az összeg létezik, azaz az összeg értéke véges.) ∞  1 1 = (végtelen mértani sor) k 3 2 k=1 Legyen |x| < 1, P = x + 2x 2 + 3x 3 + 4x 4 + . . . , x · P = x 2 + 2x 3 + 3x 4 + 4x 5 + . . . , x P − xP = x + x 2 + x 3 + x 4 + · · · = , 1−x x azaz P = . (1 − x)2  k ∞ ∞   x 1 1/3 9 k Tehát kx = . Ezért 3 k· =3· = . 2 2 (1 − x) 3 (1 − 1/3) 4 k=1 k=1 ∞ ∞ ∞   3k + 1 k  1 9 1 11 Emiatt S = =3 + = + = . 3k 3k k=1 3k 4 2 4 k=1 k=1

1130. S =

∞  3k + 1

= 3

1131. Az összeg az abszolútértékek felbontása után az S = ±(a1 − a2 ) ± (a2 − a3 ) ± ±(a3 − a4 ) ± · · · ± (a1999 − a2000 ) ± (a2000 − a1999 ) alakot ölti, ahol a ± jelből a

21. Függvényegyenletek

269

megfelelőt választjuk. Ebben az összegben 2000 tag előtt + jel, 2000 tag előtt − jel fog állni. S ≤ 2(2000 + 1999 + · · · + 1001) − 2(1000 + 999 + · · · + 1) = 2(1000 + 1000 + · · · + + 1000) = 2 000 000. Ez az érték elérhető, ha pl. az a1 , a2 , a3 , . . . , a1998 , a1999 , a2000 sorozat a 2000, 1, 1999, 2, 1998, 3, . . . , 1002, 999, 1001, 1000 sorozatot jelöli. 1132. f (x) = f (−x).  2π 1133. f (x) dx = 0, ahol f (x) a feladatban megadott kifejezést jelöli. 0

1134. f1 (x) = −

1−x , f2 (x) = x, f3 (x) = f0 (x), fn+3 (x) = fn (x), f1994 (1994) = 1994. x

21. Függvényegyenletek 1135. Nincs megoldás. Ha x = 1, akkor f (1) = 1, ha x = −1, akkor f (1) = −1. 1136. Helyettesítsünk x helyébe (1−x)-et, a 2f (1−x)+3f (x) = 4(x −1)−1 egyenletet kapjuk. Ez és az eredeti egyenlet adja a megoldást: 11 f (x) = −4x + . 5 1137. Nincs ilyen függvény. Helyettesítsünk x helyébe (−x)-et. 1138. Helyettesítsünk x helyébe (−x)-et. 1139. Helyettesítsünk x helyébe (−x)-et. 1140. Helyettesítsünk x helyébe (−x)-et. 1141. Nincs ilyen függvény. Legyen t = x − 1, akkor f (t) − f (−t) = t + 1, f (−t) − f (t) = −t + 1 Ezek megfelelő oldalainak összege a 0 = 2 „egyenlőséget” adja. 1142. Helyettesítsünk x helyébe

1 -et. x

1143. f (x) = x − 1. 1144. Helyettesítsünk x helyébe (1 − x)-et. 1145. Helyettesítsük az egyenletbe az x = t és az x = 1 − t értékeket: 2f (t) + f (1 − t) = t 2 , 2f (1 − t) + f (t) = (1 − t)2 . 1 Ebből f (t) = (t 2 + 2t − 1). 3

Megoldások, útmutatások

270

x−1 , és vizsgáljuk a következő sorozatot: 0 < x1 < 1, xn = 1146. Legyen φ(x) = x = φ(xn−1 ), n = 2, 3, . . . Ekkor a sorozat elemei felírhatók x1 segítségével: x1 , x1 − 1 1 x2 = , x3 = , x4 = x1 , . . . Ennek alapján: x1 1 − x1 f (x1 ) + f (x2 ) = 1 + x1 , f (x2 ) + f (x3 ) = 1 + x2 , f (x3 ) + f (x1 ) = 1 + x3 , ahonnan 1 1 1 1 x+ + . f (x1 ) = (1 + x1 − x2 + x3 ), azaz f (x) = 2 2 x 1−x         1 1 1 1 x 1 1147. f (x) + f 1 − = ; f 1− +f = ;f + f (x) = x x x 1−x x−1 1−x 3 2 x −x +1 = 1 − x egyenletekből kapjuk, hogy f (x) = . 2x(1 − x) 1148. f (x) = 1 − x 2 . 1149. Helyettesítsünk x helyébe (1 − x)-et. A megoldás: f (x) = x + 1. 1150. Helyettesítsünk x helyébe

1 x−1 -et, ill. -et. A megoldás: f (x) = 1 − x. 1−x x

1151. Legyen y = x. Így az egyenlet: f (2x) − f (0) = 4x 2 . Jelölje a az f (0) értékét. A megoldás: f (x) = x 2 + a. 1152. Legyen y = 0. 1153. Legyen y = 1. Így az egyenlet: f (x) · f (1) = f (1), ezért f (x) ≡ 1.  1154. Legyen y = x. Így az egyenlet: f 2 (x) = f (0), vagyis f (x) = ± f (0) minden x-re. A folytonosság miatt f (x) = a, a konstans. Ezt helyettesítve az eredeti egyenletbe: a 2 = a. Tehát f (x) ≡ 0 vagy f (x) ≡ 1. 1155. f (x) ≥ x − 1 és f (x) ≤ x − 1, tehát f (x) = x − 1. 1156. A bal oldali egyenlőtlenségből f (x) ≤ x 2 + x. Míg a jobb oldali egyenlőtlenség szerint f (x − 1) ≥ x 2 − x. Írjunk x helyébe (x + 1)-et, ekkor az f (x) ≥ x 2 + +x egyenlőtlenséget kapjuk. A két egyenlőtlenség egybevetése adja a megoldást: f (x) = x 2 + x. 1157. x = y = 0 adja, hogy egyrészt f (0) ≤ 0, másrészt f (0) ≤ 2f (0), tehát f (0) ≥ 0, s ezekből f (0) = 0. f (x) + f (−x) ≥ f (0) = 0, azaz f (x) ≥ −f (−x) ≥ x. A megoldás f (x) = x. 1158. x = y = 0 adja, hogy f (0) értéke 0, −1 vagy 1. Az y = −x helyettesítés alkalmazásával eljutunk a megoldáshoz: ha f (0) = 0, akkor f (x) = 0; ha f (0) = = 1, akkor f (x) = 1; ha f (0) = −1, akkor f (x) = −1. 1159. Legyen y = 1. Ekkor az egyenlet az xf (x) = f (1) alakot ölti. x = 0 adja, hogy f (1) = 0, innen f ≡ 0.

21. Függvényegyenletek

271

1160. Legyen y = 0. A megoldás: f (x) = 1 vagy f (x) = 0. 1161. Legyen x = y = 1. Ekkor f (1) = 0 vagy f (1) = −2. Most az y = 1 helyettesítés adja, hogy az első esetben f (x) = x − 1, a második esetben pedig f (x) = −x − 1. 1162. Hasonló az előzőhöz. 1163. Legyen x = y = 1. Ekkor f (1) = 0 vagy f (1) = 2. Most az y = 1 helyettesítés adja, hogy az első esetben f (x) = 1 − x, a második esetben pedig f (x) = 1 + x. 1164. Legyen y = x. Ekkor: 2xf (x) = 2xf 2 (x), azaz xf (x)(f (x) − 1) = 0. Ennek folytonos megoldásai: f (x) ≡ 0, f (x) ≡ 1. Második megoldás. Legyen x = = y = 1. Innen kapjuk, hogy f (1) = f 2 (1), tehát vagy f (1) = 0, vagy f (1) = 1. Ha f (1) = 0, akkor legyen y = 1, s kapjuk a megoldást: f (x) ≡ 0. Ha f (1) = 1, akkor is legyen y = 1; ekkor az egyenlet x(f (x)−1) = 0 alakra hozható, melynek egyetlen folytonos megoldása van: f (x) ≡ 1. 1165. Legyen y = x. Ekkor az egyenlet [f (x) − x]2 − 1 = 0 alakra hozható. 1166. Végezzük el az x = 0, y = t, majd az x = t, y = 2t és végül az x = t, y = −2t helyettesítéseket! A kapott három egyenletből kifejezhető f (t). A megoldás: f (x) = x+a. 1167. Hasonló az előzőhöz.

π

: 2 f (t) + f (−t) = 2a cos t,

1168. Tekintsük az alábbi helyettesítéseket, ahol a = f (0), b = f

x = 0, y = t helyettesítéssel az π π x = + t, y = helyettesítéssel az f (π + t) + f (t) = 0, 2 2 π π x = , y = + t helyettesítéssel az f (π + t) + f (−t) = −2b sin t. 2 2 egyenleteket kapjuk. Az első két egyenlet megfelelő oldalainak összegéből kivonjuk a harmadik egyenletéit, s osztunk 2-vel. A megoldás: f (t) = a cos t + b sin t. ! 1169. Legyen a = f (0) és y = 0. Ekkor a következő egyenlethez jutunk: a f (x)+cos x = = 0. Tehát vagy minden x-re f (x)+cos x = 0, vagy a = 0. Utóbbi esetet vizsgálva, tekintsük az x = y helyettesítést. !2 Ekkor 0 = f (0) = f 2 (x) + 2f (x) cos x − sin2 x, tehát f (x) + cos x = 1, s itt csak f (x) = 1 − cos x a megoldás, hiszen az f (x) = −1 − cos x függvényre f (0) = 0. Tehát két megoldás van: f (x) = − cos x és f (x) = 1 − cos x. Megjegyzések. Az egyenlet más úton történő megoldásairól és egyéb vizsgálatokról olvashatunk A Matematika Tanítása 1984. augusztusi számában, ill. a Matematikai Lapok (Kolozsvár) 1985. októberi számában Vincze Endre: Egy függvényegyenlet folytonos és nemfolytonos megoldásai c. cikkében.

272

Megoldások, útmutatások

π

π π

π

π π

1170. f (x) = f x − + = f x− cos + f cos x − = 2 2

2 2 2 2 π

π

π cos x − =f sin x. A megoldás: f (x) = a sin x. =f 2 2 2 1171. f (2x) = f (x + x) = f (x) + f (x) = 2f (x), f (3x) = f (2x + x) = f (2x) + f (x) = = 2f (x) + f (x) = 3f (x) és általában: f (nx) = nf (x), n ∈ N . Lássuk be, hogy az utóbbi egyenlőség igaz minden n ∈ Z esetén, felhasználva x = f (x) egyenlőségből a könnyen látható f (0) = 0 tulajdonságot. A qf    q    x 1 p x p az f = f (x) azonosságot nyerjük. Így f x = pf = f (x), q q q q q p, q ∈ Z, tehát minden r racionális számra f (rx) = rf (x). A folytonosság miatt ez tetszőleges r valós számra is igaz. Tehát f (r) = f (r · 1) = rf (1), azaz a megoldás: f (x) = cx, c = f (1). 1172. Ha valamely x-re f (x) = 0, akkor f (x) ≡ 0. A többi megoldást keresve, ha x = y = 0, akkor f (0) = 1. Az előző feladat megoldásához hasonló lépésekben !r beláthatjuk, hogy f (rx) = f (x) és a megoldás: f (x) = a x , a = f (1). 1173. Legyen b > 0, b = 1, t = logb x, s = logb y, ekkor az egyenlet az f (bt bs ) = = f (bt ) + f (bs ) alakot ölti; s a g(z) = f (bz ) jelölést használva ez az egyenlet g(t + s) = g(t) + g(s) alakban írható, melynek az 1171. feladat szerint a megoldása g(x) = cx, tehát f (t) = f (blogb t ) = g(logb t) = c logb t, c = f (b). 1174. Hasonló az előzőhöz. 1175. Legyen g(x) = f (x) + 1, ekkor g(x + y) = g(x) + g(y). (Lásd az 1171. feladatot!)   a+b f (a + b) + f (0) 1176. Az x = a + b, y = 0 esetben f = . Innen és az eredeti 2 2 egyenletből az f (a + b) = f (a) + f (b) − A egyenletet kapjuk, ahol A = f (0). Az előző feladat megoldását követve f (x) = cx − A. 1177. Az 1171. és az 1172. feladat megoldásával rokon. Lássuk be előbb, hogy f (x r ) = rx r−1 f (x) igaz minden r természetes számra, minden r egészre (itt alkalmazzuk 1 az y = helyettesítést), továbbá minden racionális számra is. f (x) folytonossága x miatt az egyenlőség minden r valós számra igaz. Legyen r = logx y. Ekkor y lg y f (y) f (x) f (y) = f (x) , melyből = ≡ a, a állandó. A keresett megoldás: x lg x y lg y x lg x f (x) = cx lg x, ahol c állandó. 1178. y = 1 helyettesítés adja, hogy f (x + 1) = f (x) + 1. A megoldás további lépései: 1 f (0) = 1; f (x + n) = f (x) + n; f (n) = n + 1. Az x = n, y = helyettesítés adja, n     1 1 p p 1 hogy f = 1 + . Az x = p, y = helyettesítésből pedig az f = +1 n n q q q összefüggéshez jutunk. A folytonosság miatt f (x) = x + 1, x ∈ R.

21. Függvényegyenletek

273

    1 1 1 g − = 0 adódik. Ha 1. megoldás: Az x = y = − helyettesítésből f − 2 2 2   1 1 1 f − = 0, akkor tekintsük az x = − , y = helyettesítést, melyből 0 = 1 2 2 2 következne. 2. megoldás: f (0)g(y) = y + 1, f (x)g(0) = x + 1, f (0)g(0) = 1. Az első két egyenlőség megfelelő oldalainak szorzatát elosztva a harmadikkal az f (x)g(y) = = xy + x + y + 1 egyenletet kapjuk, s ezt vessük egybe az eredeti egyenlettel! 3. megoldás: f (5)g(3) = 9, f (3)g(5) = 9, f (5)g(5) = 11, f (3)g(3) = 7. Az első két egyenlőség bal oldalának szorzata egyenlő az utolsó két egyenlőség bal oldalának szorzatával, azonban 9 · 9 = 11 · 7.

1179. Nincs.

1180. Nincs. Hasonló az előző feladathoz. 1181. Helyettesítsük (x; y) helyére rendre a (0; 0), (0; 1), (1; 0), (1; 1) számokat. 1182. Nincs. x = 1-re h(4) + h(2) = 14, x = −1-re h(2) + h(4) = 4. 1183. Nincs. f (g(x)) > g(x) > g(f (x)). 1184. Nincs. A feltétel miatt nyilván f (x) > x minden x-re, vagy f (x) < x minden x-re. Vegyük az első esetet. Ekkor f (f (x)) > f (x) > x. 1185. Készítsük el az f (x) függvény gráfját. Az a valós számból vezessen nyíl az f (a) számhoz. Egy pontból kövessük a nyilak útját. Ha f (x) gráfját már sikerült megszerkeszteni, abból f (f (x)) gráfját könnyen megkaphatjuk, ha az eredeti gráf két, egymáshoz csatlakozó nyílból álló irányított útját egyetlen nyíllal helyettesítjük. Könnyen belátható, hogy f (x) gráfjának egy lánca az f (f (x)) gráfjában két láncra bomlik, ugyanis páratlan hosszúságú körnek ugyanolyan hosszúságú kör felel meg, viszont egy 2k hosszúságú kör két, k hosszúságú körre bomlik fel. Mindez azt jelenti, hogy f (f (x)) gráfjában minden páros hosszúságú körből páros soknak kell lennie. Ha tehát egy függvény gráfjában valamilyen k-ra páratlan sok 2k hosszúságú kör van, akkor ez a függvény biztosan nem áll elő f (f (x)) alakban. x 2 − 2 gráfjában egyetlen 2 hosszúságú kör van, hiszen az x 2 − 2 = y, y 2 − 2 = x egyenletrendszer, azaz az (x 2 − 2)2 − 2 = x, vagyis a 0 = x 4 − 4x 2 − − x + 2 = (x 2 − x − 2)(x 2 + x − 1) egyenlet megoldásaiból csak egy 2 hosszúságú kör állítható össze. 1186. A bizonyítást többféle módon is végezhetjük. Kiszámolhatjuk f (f (f (x)))−x-et, majd az így kapott nyolcadfokú egyenletet megoldjuk (könnyen felírhatjuk elsőés harmadfokú tényezők szorzataként). Az is elegendő, ha mutatunk csatlakozó intervallumokat, ahol a függvény előjelet vált. Kevésbé számolós megoldás a következő. Vizsgáljuk az f (x) függvény grafikonját a [−2; 2] intervallumon. Ezt követően ugyanezen az intervallumon fel tudjuk vázolni az f (f (x)) függvény, majd pedig az f (f (f (x))) függvény grafikonját.

Megoldások, útmutatások

274

x

=f 1187. f (x) = f 2 = f (0) állandó.

x

x

= · · · = f n . A függvény folytonossága miatt f (x) = 4 2

1188. f (x, y, z) = 2f (y, z, x) = 4f (z, x, y) = 8f (x, y, z), innen f (x, y, z) ≡ 0. 1189. f (x) páratlan, g(x) páros függvény, hiszen f (x) = f (2x − x) = f (2x)g(x) − g(2x)f (x), f (−x) = f (x − 2x) = f (x)g(2x) − g(x)f (2x), g(x) = g(2x − x) = g(2x)g(x) − f (2x)f (x), g(−x) = g(x − 2x) = g(x)g(2x) − f (x)f (2x). Innen f (0) = 0, g(0) = g(0 − 0) = g 2 (0), amiből g(0) = 0 vagy g(0) = 1. 1190. Az x = y = 0 helyettesítés adja az f (0) = 2f (0)g(0) és a g(0) = g 2 (0) − f 2 (0) 1 összefüggéseket. Ha f (0) = 0, akkor az első összefüggésből g(0) = következik, 2 ami ellentmond a második összefüggésnek, így f (0) = 0. Ezért g(0) = 1 vagy g(0) = 0. Az utóbbi eset az f (x + y) = f (x)g(y) + g(x)f (y) összefüggés miatt f (x) ≡ 0 következményt vonja maga után (y = 0). 1192. Teljes indukcióval mutassuk meg a könnyen megsejthető megoldást: f (n) = = logn (n!). 1194. Ha n = m = 1, akkor f (2) = 4. n = 2, m = 1 helyettesítésre f (3) = 8. n = 3, m = 1 helyettesítésre f (4) = 13. Esetleg még néhány próbálkozás, s megsejthető, n(n + 3) hogy f (n) = − 1, és ez teljes indukcióval igazolható. 2 1195. 1. megoldás: Előbb teljes indukcióval lássuk be, hogy f (2n ) = 2n . Majd a 2n = f (2n ) < f (2n + 1) < f (2n + 2) < · · · < f (2n+1 ) = 2n+1 egyenlőtlenséglánc miatt kapjuk, hogy f (2n + i) = 2n + i, i = 1, . . . , 2n − 1. Tehát: f (n) = n. 2. megoldás: Itt is teljes indukcióval bizonyítunk. f (n) = n nyilvánvaló n = 1 és n = 2 esetén. n = 2k-ra f (n) = f (2k) = f (2)f (k) = 2k = n. Ha n = 2k + 1, akkor n − 1 = 2k = f (2k) < f (2k + 1) < f (2k + 2) = n + 1 adja, hogy f (2k + 1) = 2k + 1. 1196. Ha m = n = 0, akkor f (0) = 0. Legyen m = n, ekkor f (2n) = f (3n), tehát f (4k) = f (6k) = f (9k). Másrészt, ha n = 3m, akkor f (4m) + f (2m) = f (9m), tehát f (2m) = 0. Ha m = 0, akkor 2f (n) = f (3n), 1 1 azaz f (n) = f (3n) = f (2n) = 0, így f (n) ≡ 0. 2 2 1197. f (n) = 1.

21. Függvényegyenletek

275

1198. Legyen n ≤ 100 és n + 11 > 100, azaz 90 ≤ n ≤ 100. f (n) = f (f (n + 11)) = f (n + 11 − 10) = f (n + 1), így f (90) = f (91) = · · · = f (100) = f (101) = 91. Legyen most n < 90; akkor van olyan m, hogy 90 ≤ n + 11m ≤ 100, és f (n) = = f 2 (n + 11) = · · · = f m+1 (n + 11m) = f m (f (n + 11m)) = f m (91) = 91, (ahol definíció szerint f k+1 (x) = f (f k (x))). Tehát f (18) = 91. 1199. f (32) = 63, f (63) = f (f (32)) = 125, f (125) = f (f (63)) = 249, f (249) = = f (f (125)) = 497, f (497) = f (f (249)) = 993, f (993) = f (f (497)) = 1985, f (1985) = f (f (993)) = 3969. Látható, hogy az f (x) = 2x − 1 függvény kielégíti a felírt egyenleteket. 1 1 1200. f (999) = f (f (1000)) = = . Az f (x) függvénynek két értékét már f (1000) 999 1 . A folytonosság miatt van olyan a, ismerjük: f (1000) = 999 és f (999) = 999 1 1 amelyre f (a) = 500. De akkor f (500) = f (f (a)) = = . f (a) 500 1201. x = 1, x = 0, x = −1 az f (x) = 0 egyenlet gyökei, ezért f (x) = (x 3 − x)g(x). Ezt az egyenletbe helyettesítve a g(x + 1) = g(x) összefüggést nyerjük. Akkor a g(x + 1) − g(x) polinomnak végtelen sok zérushelye van, ezért g(x) ≡ konstans, tehát f (x) = c(x 3 − x). 1202. x = 0, x = 1 az f (x) függvény zérushelyei, ezért f (x) = (x 2 − x) · g(x). Ezt a feltétel egyenletébe helyettesítve a g(x) ≡ g(x − 1) összefüggést nyerjük. Akkor a g(x)−g(x −1) polinomnak végtelen sok zérushelye van, ezért g(x) ≡ konstans, tehát f (x) = c(x 2 − x). 1203. Helyettesítések mutatják, hogy az f (x) polinomnak zérushelyei a 0, 1, 2, 3, 4, . . . számok; s ez azt jelenti, hogy f (x) ≡ 0, hiszen csak ennek a polinomnak van végtelen sok zérushelye. 1204. Ha x = 0, akkor 0 = (−12) · f (0), tehát a polinomnak gyöke a 0. A feltételből látható, hogy ha (α − 1) gyöke a keresett polinomnak — akkor a bal oldal nulla — és α = 12, akkor a jobb oldalon f (α) = 0, tehát α is gyök. A 0-val együtt tehát az 1, 2, . . . , 11 számok is gyökök, tehát f (x) = x(x − 1) · (x − 2) · . . . · (x − 11) · g(x). Ezt az alakot a feltételbe írva, majd alkalmasan egyszerűsítve azt kapjuk, hogy például minden 12-nél nagyobb x-re g(x) = g(x − 1). Ez azt jelenti, hogy g(x) = c (=konstans), így f (x) = = c · x(x − 1) · (x − 2) · . . . · (x − 11), ahonnan f (12) = c · 12!, vagyis c = 1. Emiatt f (x) = x(x − 1) · (x − 2) · . . . · (x − 11). 1205. Teljes indukcióval megmutatható, hogy f (n) = nf (1), n ∈ N . Ez azt jelenti, hogy az f (x) − xf (1) polinomnak végtelen sok zérushelye van, így a megoldás: f (x) = cx, ahol c = f (1).

Megoldások, útmutatások

276

1206. Legyen g(x) = (x + 1)f (x) − x, g(x) (n + 1)-edfokú polinom. Tudjuk, hogy g(x) értéke az x = 0, 1, . . . , n helyeken nulla, ezért g(x) = (x + 1)f (x) − x = = Ax(x − 1)(x − n). x = −1 esetén 1 = A(−1)n+1 (n + 1)!. Ezért  − 2) . . . (x n+1 1 (−1) x(x − 1)(x − 2) . . . (x − n) x+ . f (x) = x+1 (n + 1)! n , attól függően, hogy n páratlan vagy páros. Innen f (n + 1) = 1 vagy n+2 1207. Helyettesítsünk az f (x 2 −x +1) = g(x 2 +x +1) egyenlőségben x helyébe (−x)-et: f (x 2 + x + 1) = g(x 2 − x + 1). Emiatt f (x 2 − x + 1) = g(x 2 + x + 1) = g((x + 1)2 − (x + 1) + 1) = f ((x + 1)2 + + (x + 1) + 1) = f (x 2 + 3x + 3). Így f (1) = f (7) = f (21) = . . . , azaz az f (x) polinom végtelen sok helyen ugyanazt az értéket veszi fel, ezért a polinom azonosan konstans.     f (x) + f (−x) f (x) − f (−x) 1208. f (x) = + . 2 2 1209. Tegyük fel, hogy x 2 = g(x) + h(x), g(x + p) = g(x), h(x + q) = h(x), p, q > 0 minden x ∈ R esetén. Írjunk x helyébe rendre 0, p, q, p + q értékeket. 0 = g(0) + h(0) = g(p) + h(q), p2 = g(p) + h(p), q 2 = g(q) + h(q), (p + q)2 = g(p + q) + h(p + q) = g(q) + h(p). Az első és az utolsó egyenlőség megfelelő oldalainak összegéből kivonjuk a második és harmadik egyenlőséget, s ekkor a 2pq = · · · = 0 összefüggést nyerjük. Tehát az f (x) = x 2 függvény nem állítható elő két periodikus függvény összegeként. Ugyanezt állíthatjuk az f (x) = x 3 függvényről is. A bizonyítás hasonlóképpen végezhető. 1210. Nincs. Egy ilyen függvény értékkészlete megszámlálható, mert racionális helyen felvett értékei megszámlálhatók, a többi helyen racionális értékeket vesz fel, tehát értékei ott is megszámlálhatók. Azonban egy nem konstans folytonos függvény értékkészlete kontinuum számosságú halmaz. 1211. Vizsgáljuk a g(x) = f (x +1)−f (x) függvényt. Ez a függvény egyrészt folytonos, másrészt csak irracionális értékei lehetnek. Ilyen (nem konstans) függvény pedig nem létezik. 1 1 1212. f (x + 2a) = − , f (x + 4a) = − = −(−f (x)) = f (x). f (x) f (x + 2a)

21. Függvényegyenletek

277

√ √ 1213. f (x + 2) + f (x) = 2f (x + 1), f (x) + f (x√− 2) = 2f (x − 1), ! összegük: 2f (x) + f (x + 2) + f (x − 2) = 2 f (x + 1) + f (x − 1) = 2f (x), tehát f (x + 2) + f (x − 2) = 0, f (x + 4) = −f (x), f (x + 8) = −f (x + 4) = f (x). π Megjegyzés. Létezik az egyenletet kielégítő függvény: f (x) = sin x. 4 1214. f (x + a)f (x) = b és b = f (x + 2a)f (x + a) szorzata bf (x) = bf (x + 2a). 1215. Az f (x + ka)

=

f (x) cos kφ − sin kφ összefüggést igazoljuk, s így f (x) sin kφ + cos kφ

f (x + na) = f (x). 1216. a) 1. megoldás: Ha p > 0 a periódus, akkor cos(x + p)2 = cos x 2 minden x-re, ezért (x + p)2 = x 2 + 2kπ, vagy (x + p)2 = −x 2 + 2kπ. Rögzített k mellett a két egyenletet legfeljebb három x érték elégíti ki (hiszen az első egyenlet elsőfokú, a második másodfokú), ezért az összes k egészre adott p mellett megszámlálható sok x-re teljesülhet csak az elsőnek felírt egyenlőség. 2. megoldás: A cos x 2 függvény k-adik zérushelye:  π xk = (2k − 1) . Azonban az xk+1 − xk különbségek nullához tartó, csök2 kenő sorozatot alkotnak. b) Hasonló a)-hoz. √ √ 1217. a) Legyen p > 0 a periódus: cos x + p = cos x. Legyen ekkor  x = 0, √ √ √ cos p = 1, tehát p = 2kπ.√Legyen x = p, ekkor cos 2p = cos p =  √ 2p √ n = 1, tehát 2p = 2nπ. Innen √ = 2 = ; s ez azt jelentené, hogy 2 p k racionális. b) Hasonló a)-hoz. 1218. a) Ha f (x + p) = f (x), p > 0, akkor cos x + cos ax = cos(x + p) + cos a(x + p). Legyen x = 0, ekkor 2 = cos p + cos ap, ami csak úgy lehet, ha mindkét n összeadandó 1; azaz p = 2kπ, ap = 2nπ, tehát a = , így a racionális lenne. k  2 b) Ha f (x) periodikus, akkor f (x) = − sin x − a sin ax is periodikus, és ekkor periodikus f (x) + f  (x) = (1 − a 2 ) sin ax is, tehát sin ax periodikus, s az eddig felsorolt függvényeknek van közös periódusa. Azonban a sin x és sin ax függvényeknek nem lehet közös periódusa, mert a irracionális. 1219. Hasonló az 1218. a) feladat megoldásához.

278

Megoldások, útmutatások

22. Vektorok a geometriában 2π −−→ szöggel O körül. Az összegben az össze1220. a) Forgassuk el az OAi vektorokat n adandók csak felcserélődtek, de az összeg ugyanaz maradt. Az összegvektort ez az elforgatás nem változtatta meg, ezért az nullvektor. −−→ −−→ −−→ −→ −−→ −→ −−→ −→ −−→ b) XA1 + XA2 + · · · + XAn = (XO + OA1 ) + (XO + OA2 ) + · · · + (XO + OAn ) = −→ −−→ −−→ −−→ = n · XO + (OA1 + OA2 + · · · + OAn ), s a zárójelben levő összeg az a) feladat állítása szerint nullvektor. 1221. Az előző feladat b) állítását felhasználva a kérdésre a válasz: Ha A közelebb van O-hoz, mint B. 1222. Helyezzük a sokszöget koordináta-rendszerbe, legyen a sokszög középpontja az origóban és egyik csúcsa se legyen rajta az x-tengelyen. A sokszög csúcsaiba a csúcsok első koordinátáit írjuk. Ezek a számok megfelelő számok, gondoljunk csak az 1220. a) feladat állítására. −−→ −→ −−→ −→ 1223. |A1 X|2 = (A1 O + OX)2 = |A1 O|2 + |OX|2 + 2A1 O · OX. Ezért |A1 X|2 + · · · + −→ −−→ −−→ −−→ +|An X|2 = n(r 2 +d 2 )+2OX·(A1 O + A2 O +· · ·+ An O). A jobb oldali összeg utolsó tagja 0, hiszen a zárójelben levő összeg az 1220. a) feladat szerint nullvektor. 1224. Az előző feladat speciális esete. −−→ −→ −−→ 1226. Legyen a kör középpontja O, a kör sugara r. P Ai = P O + OAi . −−→ −−→ −→ −−→ −→ −−→ |P Ai |2 = P Ai · P Ai = (P O + OAi )2 = 2r 2 + 2P O · OAi . −→ −−→ −→ −−→ |P Ai |4 = (|P Ai |2 )2 = (2r 2 + 2P O · OAi )2 = 4r 4 + 8r 2 · P O · OAi + 4r 4 · cos2 φi , −→ −−→ (ahol φi a P O és OAi által bezárt szöget jelenti) ugyanis −→ −−→ −→ −−→ −→ −−→ (P O · OAi )2 = (|P O| · |OAi | · cos φi )2 = |P O|2 · |OAi |2 · cos2 φi = r 2 · r 2 · cos2 φi . Így a vizsgált összeg: −→ −−→ −−→ −−→ |P A1 |4 + |P A2 |4 + · · · + |P An |4 = 4n · r 4 + 8r 2 · P O · (OA1 + OA2 + · · · + OAn ) + 4 2 2 2 + 4r · (cos φ1 + cos φ2 + · · · + cos φn ). −−→ −−→ −−→ Ismert, hogy OA1 + OA2 + · · · + OAn = 0, −→ −−→ −−→ −−→ ezért 8r 2 · P O · (OA1 + OA2 + · · · + OAn ) = 0. Már csak azt kell megmutatni, hogy a cos2 φ1 + cos2 φ2 + · · · + cos2 φn összeg értéke állandó.     2π 2π 2 2 2 2 2 2 cos φ1 + cos φ2 + · · · + cos φn = cos α + cos α + + cos α + 2 · + n n   2π + · · · + cos2 α + (n − 1) · n Vizsgáljuk az utóbbi összeget. A 2 cos2 x = 1 + cos 2x összefüggés alapján

22. Vektorok a geometriában

279

      2π 2π 2π 2 2 2 2 cos α + cos α + + cos α + 2 · + · · · + cos α + (n − 1) · = n  n n     1 2π 2π n + cos 2α + cos 2 α + + · · · + cos 2 α + (n − 1) · . = 2 n n Nem nehéz belátni, hogy       2π 2π 2π +cos 2 α + 2 · +· · ·+cos 2 α + (n − 1) · = 0. cos 2α+cos 2 α + n n n Az eddigi megállapítások összegzésével eljutunk a kérdezett összeg értékéhez: n |P A1 |4 + |P A2 |4 + · · · + |P An |4 = 4n · r 4 + 4r 4 · = 6n · r 4 . 2 1228. x = 2b − a, y = 2c − b, z = 2a − c. x 2 = 4b2 + a 2 − 4a · b, y 2 = 4c2 + b2 − 4b · c, z2 = 4a 2 + c2 − 4c · a. Innen x x 2 + y 2 + z2 = 5(a 2 + b2 + c2 )− − 4(a · b + b · c + c · a). Mivel a + b + c = 0, így ennek négyzete b a 2 + b2 + c2 + 2(a · b + b · c + c · a) = 0, és ezt felhasználva a korábban kapott egyenlőségben, adódik a kívánt eredmény. −→ − → − → −→ − → − → −→ − → − → 1232. P A = P S + SA, P B = P S + SB, P C = P S + SC. Ezek négyzeteit összeadva:

a z b

a c

c y

− → − → − → − → P A2 + P B 2 + P C 2 = 3 · P S 2 + SA2 + SB 2 + SC 2 + 2P S · (SA + SB + SC). Az összeg utolsó tagja nulla, mert a zárójelben levő összeg értéke nullvektor.

1233. Használjuk fel az előző feladat eredményét. 1234. Hasonló az előző feladathoz. −→ −→ 1240. Legyen az adott paralelogramma ABCD. Akkor |AC| · |BD| > |AC · DB| = −→ −→ −→ −→ = |(AB + BC)(AB − BC)| ≥ ||AB|2 − |BC|2 |. −→ −→ −→ −→ 1241. Legyen a tér egy pontja O, jelölje az OA, OB, OC, OD vektorokat rendre a, b, c, d. Ekkor az AC 2 + BD2 + AD2 + BC 2 ≥ AB 2 + CD2 egyenlőtlenség átírható a következő alakba: (c − a)2 + (d − b)2 + (d − a)2 + (c − b)2 ≥ (b − a)2 + (d − c)2 . Négyzetreemelés és átrendezés után nyerjük a nyilvánvalóan igaz (a +b−c−d)2 ≥ 0 egyenlőtlenséget. Így igazoltuk a feladat állítását. Egyenlőség akkor és csak akkor áll fenn, ha a + b − c − d = 0, azaz ha az ACBD négyszög paralelogramma.

280

Megoldások, útmutatások

−−→ −→ −−→ −→ −−→ −→ 1242. |P A1 | + |P A2 | + · · · + |P An | = |OA1 − OP | + |OA2 − OP | + · · · + |OAn − OP | = −−→ −→ −−→ −−→ −→ −−→ −−→ −→ −−→ = |OA1 − OP | · |OA1 | + |OA2 − OP | · |OA2 | + · · · + |OAn − OP | · |OAn | ≥ −−→ −→ −−→ −−→ −→ −−→ −−→ −→ −−→ −−→ ≥ (OA1 − OP ) · OA1 + (OA2 − OP ) · OA2 + · · · + (OAn − OP ) · OAn = OA1 2 + −−→ −−→ −→ −−→ −−→ −−→ + OA2 2 + · · · + OAn 2 − OP · (OA1 + OA2 + · · · + OAn ) = n, hiszen a feladat −−→ −−→ −−→ −−→ feltételei szerint OA1 + OA2 + · · · + OAn = 0, és OAi 2 = 1. −→ −→ −→ −→ −→ −→ −→ −→ −→ −→ −→ 1243. a) Legyen OP = OA+ OB + OC. CP = OP − OC = OA+ OB ⊥ OA− OB = −→ −→ −→ = BA, így (CP ⊥ BA miatt) P rajta van a C-ből induló magasságegyenesen. Hasonlóan mutatható meg, hogy P rajta van a többi magasságegyenesen is, ezért valóban P ≡ M. −−→ −→ −→ −→ b) Az előbbi OM = OA + OB + OC összefüggés mindkét oldalát négyzetre −→ −→ −→ −→ −→ −→ −→ −→ emelve: d 2 = 3r 2 + 2(OA · OB + OB · OC + OC · OA). Mivel AC = OC − −→ −→ −→ − OA, így ennek mindkét oldalát négyzetre emelve: b2 = 2r 2 − 2OA · OC; hasonlókat felírva a másik két oldalra megkapható a kívánt eredmény. 1244. Következik az 1243. b) feladatból. 1245. Következik az 1243. a) feladatból.

   1 1 1 1 1249. s a = a+b, s b = b+a, s a ⊥ s b , emiatt s a ·s b = 0, azaz a+b b + a = 0, 2 2 2 2 4 a 2 + b2 4 melyből rövid számolással kapjuk a cos γ = · ≥ összefüggést. 5 2ab 5 4 1250. Az előző megoldást követve a cos γ = eredményt kapjuk. 5 1251. a) Tekintsük a háromszögbe írt kör középpontjából az érintési pontokba mutató vektorok összegének négyzetét, amely nemnegatív: (x + y + z)2 = 32 − − 22 (cos α+ cos β+ cos γ ) ≥ 0. Ezt rendezve a kívánt eredményhez jutunk. ( a háromszögbe írt kör sugara.) b) Tekintsük a háromszög köré írt kör középpontjából a csúcsokba mutató vektorok összegének négyzetét, amely nemnegatív: (x + y + z)2 = 3r 2 + 2r 2 (cos 2α+ cos 2β+ cos 2γ ) ≥ 0. Ezt rendezve a kívánt eredményhez jutunk. (r a háromszög köré írt kör sugara.) 1253. Tekintsük a p(a; b), q(c; d), r(e; f ), s(g; h) vektorokat. A négy vektor között van kettő, melyek által bezárt szög legfeljebb 90◦ , s ezek skaláris szorzata nemnegatív. (A feladatban szereplő kifejezések e négy vektorból képezhető párok skalárszorzatai.) 1256. Legyenek az A, B, C, P pontokba mutató helyvektorok rendre a, b, c, p. Fejezzük ki ezek segítségével a tükrözések után kapott pontokba mutató helyvektorokat.

22. Vektorok a geometriában

281

1261. A hatszög szomszédos oldalait tekintsük mint egymás után fűzött vektorokat. Ezek összege nullvektor. Továbbá e hat vektorból nullvektor annak a három vektornak az összege is, melyek az adott háromszög oldalvektorai. Ezért a megmaradó három vektor összege is nullvektor. −→ −→ −→ −→ −→ −→ 1268. AG = EF = LK = CB = DA = F H . 1270. Fejezzük ki az eredeti háromszög oldalvektoraival a keletkezett háromszögek oldalvektorait. 1274. Legyen az x vektor 60◦ -os elforgatottja (az óramutató járásával ellentétesen) x  . Az ábrán látható vektorok szakaszfelező pon−−→ −→ tokat kötnek össze, és DN = b + a, DM = = a  + b . Mivel (a + b) = a  + b , így állításunkat igazoltuk.

A

M

P

a

a

0

b

b

0

B

1278. Előbb mutassuk meg, hogy ha az ABCD (konvex) négyszög AD, ill. BC oldalá−→ C1 nak felezőpontja E, ill. F , akkor EF = 1 −→ −→ = (AB + DC). A feladat megoldásáC2 2 nál lássuk be azt, hogy EF ⊥ BC és b A 2 · |EF | = |BC|. A fenti állítás szerint b −→ 1  −→ B EF = (b + c ), valamint igaz a BC = 2 = b + c egyenlőség is. (Itt az x vektor 90◦ -os elforgatottja az óramutató járásával ellentétesen: x  .)

D

C

B1 B2

F c

c

0

1279. Legyen az x vektor 90◦ -os elforgatottja az óramutató járásával megegyezően x  . Nyilván igaz az (x + y) = x  + y  egyenlőség. −→ a + b = 2s, EF = a  + b . Tehát |EF | = 2s és EF ⊥ s.

R

N

C

E

F a a A

E 0

;b C s b

0

B

0

Megoldások, útmutatások

282

1281. A vektorok szakaszfelező pontokat kötnek össze, és x  az x vektor 60◦ -os elforgatottját jelöli az óramutató járásával ellentéte−→ − → sen. Látható, hogy P S  = P R, és ez állításunk igazolását jelenti.

B

b

C

0

a

b

A

a

P

c

S

c

F 1282. Legyen az x vektor 60◦ -os elforgatottja (az óramutató járásával ellentétesen) x  . Ennek a műveletnek a tulajdonságai: (x + y) = x  + y  , x  = −x, x + x  = x  . Most a + a  + b + b + c + +c = 0, azaz (a +b+c)+(a +b +c) = 0, innen a + b + c = 0. Ezeket felhasználva belátható a −(a  + b) = (b + c) összefüggés, melyből következik a feladat állítása.

0

D 0

R

E a

c0

a0

c b

b0

23. Területátalakítások 1284–1292. Néhány vonal berajzolása után páronként egyenlő területű vonalkázott és jelöletlen részeket kapunk. 1284. A belső ponton át húzzunk párhuzamosakat az oldalakkal. 1285. Rajzoljuk meg a négyszög vonalkázott részén áthaladó átlóját. 1286. A középvonalak metszéspontját kössük össze a csúcsokkal. 1288. A vonalkázott háromszög csúcsaiból indítsunk a hatszög oldalaival párhuzamos egyeneseket. 1292. A belső ponton át húzzunk párhuzamosakat az oldalakkal. 1293–1300. Ha egyenlő területű idomokból egyenlő területű részeket veszünk el, egyenlő területű részeket kapunk.

23. Területátalakítások

283

1296. Az ábrán két egyenlő területű négyszöget találhatunk. 1300. Használjuk fel az 1284. feladatot. 1301. Két szemközti pöttyözött háromszög területének összege a négyszög területének negyede. 1302. A vonalkázott téglalap átdarabolható a „nagy” téglalap területének felét kitevő háromszögbe. 1303. A jelöletlen részek átdarabolhatók a vonalkázott körcikkbe. 1304. A területképletekből kapjuk, hogy a két félkör együttes területe megegyezik a negyed körív területével. 1307. A két szemközti vonalkázott háromszög együtt a pöttyözött paralelogramma felét teszi ki. 1308. Használjuk fel az 1285. feladatot. 1309. Hosszabbítsuk meg a „kis” négyzet oldalait! 1311. Hasonló az 1288. feladathoz. 1312. 1/2. 1313. 1/2. 1314. 7/9. 1315. 5/9. 1316. 2/9. 1317. 2/8. 1318. 1/5. 1319. 1/2. 1320. 5/12. 1321. 1/3. 1322. 1/3. 1323. 1/2. 1324. 1/3. 1325. 1/3. 1326. 5/24. 1327. 1/7.

Megoldások, útmutatások

284

1328. 7/60. 1329. a) b) c) d)

7, 19, 5, 3.

1331. Nem. Használjuk az 1284. feladatot. 1332. Tükrözzük a félkört az átmérőre, s lássuk be, hogy a körbe írt téglalapok közül a négyzet területe a legnagyobb. Ezt úgy bizonyíthatjuk be a legkönnyebben, ha berajzoljuk a téglalap egyik átlóját. 1333. Használjuk az 1284. feladatot. 1 1 1 1334. P A2 + P B 2 + P C 2 + P D2 = (P A2 + P B 2 ) + (P B 2 + P C 2 ) + (P C 2 + P D2 ) + 2 2 2 1 2 2 + (P D + P A ) ≥ P A · P B + P B · P C + P C · P D + P D · P A ≥ 2TAP B + 2 + 2TBP C + 2TCP D + 2TDP A = 2T . Az egyenlőtlenségek helyén csak akkor áll egyenlőség, ha P A = P B = P C = P D és AP B ^ = BP C ^ = CP D^ = DP A^ = 90◦ . Ezekkel a tulajdonságokkal a P pontból indítva a P A, P B, P C, P D szakaszokat, a kapott ABCD négyszög négyzet lesz, a P pont pedig a négyzet középpontja.

C

1335. P az adott pont, F az AC oldal felezőpontja, e  f . P R egyenes felezi a háromszög területét.

A

P

f

F

R

e

B

D

1336. F a BD átló felezőpontja, P R AC. AR felezi a négyszög területét.

A

F

P

B

R

C

24. Geometriai konstrukciók

285

24. Geometriai konstrukciók 1337. Igen (konkáv négyszög). 1338.

1339.

1340.

1341. a)

b)

4

6

4

3 4

6

4

3 4

6 1342.

=

+

+ 1

2 1

vagy

+ 1

+ 1

p

2

= 2

2

3

Megoldások, útmutatások

286

1343.

1344.

vagy

vagy

vagy

1345.

1346.

1347.

1348.

24. Geometriai konstrukciók

287

1349. Legkevesebb 6 kockából állhat az építmény. Az elhelyezést felülnézetből mutatja az ábra. Pl. a 2 szám azt jelzi, hogy ott két kocka van egymáson. (Legfeljebb 20 kocka lehet.)

1 2

1350. Pl. a térbeli kereszt, azaz ragasszunk egy kocka minden lapjára egy vele egybevágó kockát! Egységkockákból egyéb testek is készíthetők, melyek teljesítik a kívánt feltételt.

2 1

1351.

1352. Az ábrán a szakaszok 1 egység hoszszúak.

1353. Igen. Az ábrán a szakaszok 1 egység hosszúak.

1354. Van.

2

2 2

1355. Hamis. Ellenpélda a 3 egység átmérőjű körön a szabályos 12-szög. 1356. Egy lehetséges konstrukció: egy félkörívnél rövidebb íven vegyünk fel n pontot. 1357. Szabályos ötszög csúcsai és a köré írt kör középpontja. 1358.

Megoldások, útmutatások

288

1359.

40

40

1361. E és F oldalfelező pontok.

1360.

E

F

1362.

1363. 30

30

1

1364.

6

1 1

2

4

2

24. Geometriai konstrukciók

289

1365.

1366.

1367.

1368.

1369. A feldarabolás során van olyan háromszög, melynek van csúcsa a négyzet belsejében (különben lenne derékszögű háromszög), sőt van két háromszögcsúcs is a négyzet belsejében. Mindegyik csúcs legalább öt háromszögnek csúcsa, ez összesen 2 · 5 = 10 háromszög, legfeljebb két háromszöget számoltunk kétszer, így a háromszögek száma legalább 8.

B

B

A 8

C A

9

10

11

1370.

4 3

9 5

4 8

1

2

3 7

6

9

D

1

7 2

6 8

5

C

Megoldások, útmutatások

290

1371.

1372. Néhány megoldás.

36◦ 36◦

360◦ /7

72◦

36◦

36◦

72◦

72◦

72◦

540◦ /7

180◦ /7

540◦ /7

1373. Elvégezhető a felosztás; az ábrák mutatnak egy-egy megoldást.

1374.

24. Geometriai konstrukciók

291

1375.

A B

C

1376.

1377.

A B

1378.

1379. Ötszög alapú gúla.

C

Megoldások, útmutatások

292

1380. Van.

1381. Írjunk 1/2 sugarú köröket az (n; n2 ) koordinátájú pontok köré, ahol n pozitív egész szám. 1382. Nem lehet. Vegyünk egy olyan egyenest, mely egyik parabola-tengellyel sem párhuzamos. A parabolatartományok ennek az egyenesnek véges hosszúságú részét fedik le. 1383. Vegyünk egy pontot, körülötte egy tetraédert, és vegyük fel a tetraéder oldallapjai köré írt köröket. A pont és egy-egy kör meghatároz egy kúppalástot. Ezekbe a kúpokba „ejtsünk bele” egy-egy gömböt; úgy választva a sugarukat, hogy ne érintsék egymást. 1384. a) Nem lehet. P -ből a körök mindegyike 180◦ -nál kisebb szögben látszik. Így e szögek összege kisebb 4 · 180◦ = 720◦ -nál. Míg, ha lenne megfelelő elrendezés, akkor e szögtartományok kétszeresen fednék le a síkot, tehát összegük legalább 720◦ lenne. b) Lehet. Vegyünk fel 5 db, P -ből induló félegyenest úgy, hogy bármely két szomszédos félegyenes 72◦ -os szöget zárjon be egymással! Két-két ilyen szomszédos, 72◦ -os szögtartományból álló 144◦ -os tartományba írjunk a szögszárakat érintő kört! 1386. a) Lehet.

b) Nem lehet. Ugyanis 1991 páratlan szám, ezért bárhogyan is veszünk fel egy egyenest, annak valamelyik oldalára a sokszögnek több csúcsa esik, mint a másikra.

25. Invariáns tulajdonságok

293

1387. a) Igen, lehet. Az első ábrán látható alakzatból 496 db-ot (4·496 = 1984), a másodikból egyet véve, ezeket egymás után helyezzük, úgy hogy a szélső érmék érintsék egymást, s alakítsunk ki egy záródó kör alakú láncot! b) Nem lehet. Számoljuk meg az érintési pontokat! Minden érme 3 másikat érint; ez összesen 1991·3 érintési pont, de minden pontot kétszer számoltunk, tehát ennek a számnak párosnak kell lennie.

25. Invariáns tulajdonságok 1388. Nem lehet. A papírlapok száma 9-esével nő. 1389. Nem lehet. Egy törlés-felírás alkalmával a páratlan számok száma nem változik vagy kettővel csökken. Kezdetben öt páratlan szám volt, így az utolsóként megmaradó szám páratlan lesz. 1390. A törlések során a 0 számjegyek száma nem változik, vagy kettővel csökken aszerint, hogy különböző vagy egyenlő két jegy helyébe írunk. Így a századik törlés előtt a még meglévő két számjegy páros sok (0 vagy 2) 0 jegyet ad, tehát a két számjegy egyenlő, vagyis a századik törlés után az 1 számjegy marad a táblán. 1391. Ha egy tárgyat 6 részre osztunk, akkor 1-ből 6 rész lesz, a részek száma tehát 5tel nő. Ha a tárgyat 11 részre daraboljuk, akkor a darabok számát 10-zel növeltük, minthogy 1 helyett 11 darabbal rendelkezünk. Ezek szerint Béla „osztogatásai” során minden alkalommal az 5 többszörösével (1-szeresével vagy 2-szeresével) növelte a részek számát, végül tehát a részek számának egy 5-többszörösnél 1gyel nagyobb számnak kell lennie. Az 1993 nem ilyen, Béla tehát hibázott a számlálás során. 1392. Minden törlés-felírás után a táblán levő számok összege 1-gyel csökken. 19 törlés-felírás után a táblán egy szám marad, mégpedig: (1 + 2 + · · · + 20) − 19 = 191. 1393. Ha a táblán levő számok a1 , a2 , . . . , ak , akkor az S = (a1 +1)·(a2 +1)·. . .·(ak +1) kifejezés értékét vizsgálva, az a műveletek során változatlan marad. Kezdetben S = 21!, ugyanez S értéke akkor is, amikor egyetlen szám marad a táblán; ezért ez a szám: 21! − 1.

294

Megoldások, útmutatások

1394. A fehér babszemek száma egy-egy kivételnél vagy nem változik, vagy kettővel csökken. Kezdetben 75 fehér babszem volt, ezért az utolsónak megmaradó babszem fehér lesz. 1396. Nem, hiszen a középső mezőben mindig olyan szám áll, amely a négy sarokban levő szám összege. 1397. Nem lehet. Legyen a és b a két kupacban levő gyufák száma és S = a − b. A változtatások során S értékének hárommal való osztásakor a maradéka nem változik. 1398. Nem lehet. Legyen a, b és c a csúcsokban levő gyufák száma és S = a + b + c. A változtatások során S értékének hárommal való osztásakor a maradéka nem változik. 1399. Nem lehet. Legyen a, b, c és d a csúcsokban levő gyufák száma és S = a − b + + c − d. A változtatások során S értékének hárommal való osztásakor a maradéka nem változik. 1400. Nem lehetséges. Figyeljük az asztalon levő kupacok számának és az asztalon maradt kavicsok számának S összegét. Ez az S érték a lépések során nem változik. Kezdetben S = 1001 + 1 = 1002. Ha a kívánt állapotban n kupac van, akkor az elvárások miatt S = n + 3n = 4n, de nincs olyan n egész szám, amelyre 4n = 1002. 1401. Nem lehet. Legyen a, b és c a szürke, a barna, ill. a zöld kaméleonok száma és S = a − b. A változtatások során S értékének hárommal való osztásakor a maradéka nem változik. 1402. A megmaradó szám az 1-es. Egy-egy alkalommal megváltozik a 0-k, 1-esek, 2-esek számának paritása. 1403. Nem változik a felírt számok összegének 13-as maradéka. 1404. A táblán levő három szám négyzetösszege a lépések során nem változik, hiszen     a+b 2 a−b 2 2 2 a +b = √ + √ . Mivel a megadott számhármasok négyzetösszege 2 2 nem egyenlő, ezért célunk nem érhető el. 1405. Nem kaphatjuk meg. Figyeljük a táblára felírt számok négyzetének összegét. Ez az összeg minden lépésben nő. 1406. Nem lehet. a) A csúcsokban levő számok összege mindig páratlan. b) A négyzet egyik átlójának két végén levő szám összege S1 , a másik két szám összege S2 . A változások során |S1 − S2 | értéke nem változik.

25. Invariáns tulajdonságok

295

1407. Nem lehet. a) A csúcsokban levő számok összege mindig páratlan. b) Osszuk két csoportba a kocka nyolc csúcsát úgy, hogy az ugyanabban a csoportban levő négy csúcs között ne legyen kettő, mely ugyanannak az élnek két végpontja (ez a felosztás egy-egy szabályos tetraéder csúcsait jelöli ki). Az azonos csoportban levő számok összege legyen S1 , ill. S2 . A változások során |S1 − S2 | értéke nem változik. 1408. Válasszunk ki 3, nem egymás mellett ülő személyt, s úgy figyeljük az előttük levő tányérok számának változását. 1409. Hasonló az előzőhöz. 1410. Válasszunk ki 3 nem szomszédos személyt, és figyeljük a következő S összeg változását. S értékét úgy kapjuk, hogy a 3 személy előtt levő tányérok összegéből levonjuk a másik 3 személy előtt levő tányérok összegét. S értéke a megengedett lépések során nem változik. Emiatt nem érhető el a kívánt állapot. 1411. Nem lehet. Legyen S a nők előtt levő poharak számának összege! Az átalakulások lépéseiben S vagy nem, vagy 2-vel változik. Mivel kezdetben S = 5, ezért nem érhető el, hogy S = 0 legyen. 1412. Nem lehet. A fákat számozzuk meg valamilyen körüljárás szerint, rendre az 1, 2, 3, . . . , 44 számokkal. Az i-edik fához hozzárendeljük az Si = iai számot, ahol ai az i-edik fán ülő majmok száma. Figyeljük az S összeg változását, ahol S = S1 + S2 + · · · + S44 . Az S értéke egy-egy alkalommal vagy nem, vagy 44-gyel változik az 1. és 44. fák közti átugráskor. Kezdetben S = 1 + 2 + · · · + 44 = 22 · 45, az elérni kívánt állapotban S = 44 · k (ha a majmok a k-adik fán vannak), de ez nem érhető el, mert 44  | (22 · 45 + 44n). 1413. Nem lehet. Lásd az előző két feladatot. 1414. A matematikusnak igaza van. Ugyanis megadható olyan, a klubokba sorolástól függő, természetes szám értékű függvény, melynek értéke minden egyes javító lépés során legalább 1-gyel csökken. Ezért az algoritmus véges sok lépés után megszakad, ami azt jelenti, hogy létrejöttek a kívánt tulajdonságú klubok. A = 7 × (a 11-esek klubján belüli ismeretségek száma) + 11× (a 7-esek klubján belüli ismeretségek száma). Egyszerű számolás mutatja, hogy a függvény értéke 11-es → 7-es csere esetén legalább 7-tel, 7-es → 11-es csere esetén legalább 11-gyel csökken. Ugyancsak megfelelő függvény: A = (a klubokon belüli ismeretségek összes száma) + 4 × (a 7-esek klubjának létszáma). 1415. Nem lehetséges. Tekintsük ugyanis a gyomos parcellákat körbevevő kerület hoszszát. Ez a kerület a további gyomosodás során nem nő. (Miért?) A 9 mező kerülete legfeljebb 9 · 4 = 36 egység, míg a 10 × 10-es tábla kerülete 40 egység.

Megoldások, útmutatások

296

Megjegyzés. Látható, hogy 10 gyomos parcella esetén már megtörténhet, hogy az egész tábla elgazosodosik. 1417. Legyen a sáska, szöcske, tücsök jele: A, B és C, és a kezdeti sorrendjük ABC. Az ABC, BCA, CAB, ACB, BAC, CBA sorrendek közül az első hármat nevezzük szabályosnak, az utolsó hármat szabálytalannak. Az ugrálások során szabályos és szabálytalan sorrendek váltakozva követik egymást. Emiatt 1999 (azaz páratlan számú) ugrás után nem állhat vissza az eredeti sorrend. 1418. Igen, elérhető. Ha egy poharat páratlan sokszor forgatunk meg, akkor meg lesz fordítva. Hatszor fordítunk meg öt-öt poharat, először az első, majd a második, . . . , végül a hatodik pohár kivételével a többi poharat. 1419. Igen, elérhető (hasonló az előző feladathoz). Ha egy-egy alkalommal négy elem előjelét változtatjuk meg, akkor a kívánt állapot nem érhető el, mert a változtatások száma páros (4 többszöröse), míg az elérni kívánt állapothoz páratlan számú változtatást kell végezni. 1420. 1. megoldás: Helyettesítsük a + jelet +1-gyel, a − jelet −1-gyel. A számok szorzata az átalakítások során nem változik. 2. megoldás: Helyettesítsük a + jelet 0-val, a − jelet 1-gyel! A számok összege minden átalakítás után páratlan marad. 3. megoldás: Egy-egy lépésben a − jelek száma vagy nem változik, vagy 2-vel csökken. 1421. Helyettesítsük a + jelet +1-gyel, a − jelet −1-gyel. Az átalakítások során nem változik a megjelölt mezőkben álló számok szorzata.

* *

*

*

* *

1422. Nem mindig. A megjelölt mezőkön álló számok összege minden lépésben 3k-val változik.

*

*

• • • • • • • • • • • • • • • • • • • • • • • • • • • • • • • • • • • • • • • • • • • • • • • •

1423. a) Nem lehet. Állítsuk párba a szemközti csúcsokat. Egy-egy átalakítás minden egyes pár egyik elemének előjelét változtatja meg. Az eljárást páratlan sokszor elvégezve, a párba állított csúcsokban az előjelek különbözők lesznek, páros sokszor elvégezve pedig azonosak — ezek alól kivétel az (A1 , A7 ) páros. Ezért nem lehet, hogy egyidejűleg az (A2 , A8 ) párban különböző, az (A3 , A9 ) párban azonos legyen az előjel.

25. Invariáns tulajdonságok

297

1424. Nem lehet. Vegyük fel a koordináta-rendszerben a négyzet három csúcsát: (0; 1), (1; 1), (1; 0). Egy tükrözés alkalmával a tükrözött pont egy-egy koordinátája vagy változatlan marad, vagy páros számmal változik. Mivel a megadott pontok mindegyikének van páratlan szám a koordinátái között, így nem kaphatunk olyan pontot, melynek mindkét koordinátája páros. 1425. Nem lehetséges. Figyeljük meg, hogy az adott négy pont mindegyikénél, és a keletkező új pontoknál is a pont két koordinátájának különbsége osztható 3-mal. 1426. Nem lehet. Könnyű belátni, hogy a sorozatban négy páratlan szám után egy páros, majd négy páratlan, egy páros, . . . követi egymást ismétlődve. 1427. Legyen a sorozat hét egymást követő eleme x1 , x2 , x3 , x4 , x5 , x6 , x7 . Tekintsük az S(a, b, c, d, e, f ) = 2a +4b+6c+8d +10e+12f függvényt. S(x1 , x2 , x3 , x4 , x5 , x6 ) és S(x2 , x3 , x4 , x5 , x6 , x7 ) utolsó jegye ugyanaz (lássuk be!). Emiatt a sorozatban bármely k esetén az S(xk+1 , xk+2 , xk+3 , xk+4 , xk+5 , xk+6 ) utolsó számjegye mindig ugyanaz. S(1, 0, 1, 0, 1, 0) = 18, míg S(0, 1, 0, 1, 0, 1) = 24. Ezek utolsó számjegye különböző, így a sorozatban nem szerepelhet a 0, 1, 0, 1, 0, 1 részsorozat. 1428. a) Nem lehet. A műveletek megőrzik az első számpár legnagyobb közös osztóját közös osztóként. b) Lehetséges. 1429. a) Lehetséges. b) Nem lehet. Az átalakítások során a számpár két elemének különbsége mindig osztható 7-tel. 1430–1433. feladatokban az inverziók számának párosságát vizsgáljuk. Az 1, 2, 3, 4, 5 számok 3, 4, 5, 2, 1 sorrendjében inverzióban áll pl. a 3 a 2-vel (mert ebben a számpárban a nagyobb szám áll elöl), míg pl. a 3 a 4-gyel nincs inverzióban. A számok felírt sorrendjében az inverziók száma 7. 1430. Egy-egy csere megváltoztatja az inverziók számának párosságát, így a 15. csere után páratlan sok inverzió lesz, tehát nem juthatunk el az eredeti sorrendhez. 1431. Az első kérdésre nem a válasz, mivel egy ilyen átrendezés nem változtatja meg az inverziók párosságát, s a kiinduló helyzetben az inverziók száma páros, míg az elérni kívánt állapotban páratlan. 1432. Nem lehet. Egy művelet az inverziók számát nem változtatja, ha vízszintes irányban toltunk el egy lapocskát, s páratlan számmal változtatja, ha függőlegesen toltuk el. Kezdetben az inverziók száma páros, az elérni kívánt állapotban páratlan, s ez azért nem érhető el, mert a függőleges irányú mozgatások száma — ha az üres mezőt ismét az alsó sorban akarjuk látni — páros. 1433. Hasonló az előzőhöz.

298

Megoldások, útmutatások

1434. Nem lehet. Adott irányt kijelölve haladjunk végig pozitív körüljárás szerint egy sokszög kerületén! Utunk során kizárólag a megadott iránnyal párhuzamos oldalak hosszát vegyük figyelembe, összegezzük pozitív előjellel, ha a megadott iránnyal egyezően; negatív előjellel, ha azzal ellentétesen jártuk be a szóban forgó oldalt! A sokszögre így számolt érték invariáns a feldarabolással és az eltolással szemben. Háromszögnél három (valójában hat) irány is van, mikor ez az érték nem nulla; míg téglalapnál minden irányra nullát kapunk. 1435. x = −1-re az egyik polinom 11-et, a másik −9-et vesz fel értékként. A megadott változtatás a polinom x = −1 helyen felvett értékét 1-gyel változtatja. Ezért lesz olyan polinom, melynek x = −1 gyöke. 1436. Az előírt művelet növeli a sokszög területét. Mivel csak véges sok különböző sokszöget kaphatunk, így az eljárás néhány lépés után véget ér. 1437. Legyen S a táblázatban levő számok összege! Ha valamely sorban vagy oszlopban a számok összege negatív, akkor ott változtassuk meg az előjeleket; ezzel S nő. S-nek véges sok értéke lehetséges. Az eljárások befejeztével S nem lehet negatív, hiszen (az előbbiek szerint) ekkor növelhető lenne. 1438. Legyen az S állapotjelző függvény értéke a szomszéd számpárok szorzatának összege! Ebben az összegben az a, b, c, d sorrendre az ab + bc + cd szerepel, míg a leírt cserét követően ac + cb + bd, mely (a − d)(b − c) < 0 miatt nagyobb a korábbi részösszegnél (ab + cd < ac + bd). Tehát egy ilyen lépésben S értéke nő, de S-nek csak véges sok értéke lehet (n db adott szám esetén), ezért a növekedés egyszer véget ér. 1439. Vegyük észre, hogy egy-egy lépés nem változtatja meg a csúcsokban levő számok összegét; míg S értéke minden lépésben csökken, ahol S a másodszomszédos csúcsokban levő számok különbségei négyzetének összege. Mivel S értéke pozitív egész, így az eljárás néhány lépés után véget ér. 1440. Az n-edik lépésben kapott számnégyest jelölje (an , bn , cn , dn ). Nyilván an + bn + cn + dn = 0, és így 2 2 2 2 an2 + bn2 + cn2 + dn2 ≥ 2(an−1 + bn−1 + cn−1 + dn−1 ). 1441. Tekintsük a fekete mezőket körbefogó, rácsszakaszokból álló, legrövidebb zárt töröttvonal S kerületét. Az átalakítás lépéseiben S értéke mindig csökken. 1443. Tekintsük az „igazi” barát-párok S számát, tehát azokét a párokét, melyekben a két törpe egymás barátja, s ugyanolyan színű házikóban laknak. Az átfestések során ez az S szám mindig nő. 1445. Helyezzük el tetszőlegesen a manókat és építsünk kerítést két parcella közé, ha azok tulajdonosai utálják egymást. Megmutatjuk, hogy minden kerítés felszámolható két manó parcellájának megcserélésével úgy, hogy közben nem keletkezik új kerítés. Innen már nyilván következik, hogy néhány csere után az összes kerítés megszüntethető.

25. Invariáns tulajdonságok

299

Legyen A és B két szomszédos manó, akik utálják egymást. Keresünk olyan C manót, akivel B felcserélhető anélkül, hogy új kerítést kellene építeni, s közben az A és B közötti kerítés lebontható. Azaz olyan, A-tól különböző C manót kell keresni, (1) aki B eddigi szomszédait nem utálja; (2) akinek egyetlen szomszédját sem utálja B. Az (1) feltétel legfeljebb 3 · 4 = 12 manót zár ki, hiszen B négy szomszédja legfeljebb ennyi különböző manót utálhat. A (2) feltétel ismét 12 manót zár ki a C-ként szóbajöhetők közül, ugyanis a B-t utálók szomszédjai legfeljebb ennyien vannak. Vegyük észre azonban, hogy a B manót mindkétszer kizártuk. (Ugyanis B utálja B valamely szomszédját — nevezetesen A-t; továbbá B valamely szomszédját — A-t — utálja B, ez utóbbi a (2) feltétel.) Így ez a két feltétel legfeljebb 23 manót zár ki; köztük van B manó is, tehát C = B teljesül. A C = A feltétel esetleg kizár még egy manót, de így is csak 24 manót zártunk ki; tehát van (1)-nek és (2)-nek eleget tevő C manó, s ekkor B ezzel a C-vel parcellát cserélhet. 1446. Valahogyan szétosztjuk őket. Ha valamelyik csoportban valakinek két ellenfele van, akkor őt áttesszük a másik csoportba. Ez a művelet fokozatosan csökkenti az azonos csoportban levő ellenfélpárok S számát. 1447. Ültessük le őket tetszőlegesen. Ha ez még nem megfelelő, akkor keressünk két szomszédot, At és B-t, akik ellenségek. Keressük meg A-nak olyan C barátját, hogy C-nek jobb oldali D szomszédja B-nek barátja legyen (ez a feladat feltétele miatt létezik)! A BC íven ülőket ültessük fordított sorrendben. Egy ilyen transzformáció során csökken az ellenfél-szomszédpárok S száma.

A

B

C

D

1448. Színezzünk 4 színnel tetszőlegesen. Ha az egyszínű pontokat összekötő szakaszok száma nagyobb n-nél, akkor van olyan pont, melyből legalább három, egyszínű pontokat összekötő szakasz indul. Ezt a pontot színezzük át, s ekkor csökken az egyszínű szakaszok száma. 1449. A piros és a kék pontokat kössük össze szakaszokkal. Ha valamely két szakasz metszi egymást, akkor a metszéspont megszüntethető, ha a között a négy pont között a két szakaszt másképp vesszük fel. Kínálkozik, hogy itt az állapotjelző függvény a metszéspontok számát jelentse, de az előbbi lépés nem biztos, hogy

Megoldások, útmutatások

300

csökkenti annak értékét. Figyeljük inkább a szakaszok összhosszának változását. Ez az átalakítások során mindig csökken, azonban a csökkenésnek egyszer véget kell érnie, s ekkor nem lehet, hogy két szakasz metssze egymást. 1450. Hasonló az előzőhöz. 1451. Írjunk a táblázat mezőibe betűket, majd ezekkel fejezzük ki az átalakítások során kapott táblázatokat! 1452. Nem. Ha az utolsó átalakítás után csupa 0-t kaptunk, akkor előzőleg mindegyik szám egyforma volt, mindegyik 1-es. Ez azt jelenti, hogy ezt megelőzően a számok 0, 1, 0, 1, . . . , ami nem lehet, mert 9 páratlan. 1453. Hasonló, mint az 1451. feladat. 1454. Ha a sorozat elemeinél páros szám helyett +1-et, páratlan helyett −1-et írunk, akkor az átalakítások megegyeznek az előző feladat átalakításaival. Ez azt jelenti, hogy néhány lépés után mindegyik szám páros lesz. Figyeljük meg, hogy a számok közt a legnagyobb nem nőhet.

26. Feladatok a sakktáblán 1455. A huszár fekete mezőről fehérre, fehérről feketére lép. Ha a bal alsó sarok fekete, akkor innen indulva a 63. lépésben nem léphet a jobb felső fekete mezőre, hiszen ekkor fehér színű mezőre lép. 1456–1459. Hasonló az előzőhöz. 1460.

1

20

5

14

9

6

15

10

19

4

11

2

17

8

13

16

7

12

3

18

1461. Tekintsük a négysoros táblát, két belső és két szélső sorral. Egy szélső mezőre a huszár csak belső mezőről léphet, szélső mezőről pedig belsőre lép. Mivel ugyanannyi szélső mező van, mint belső, ezért a huszár minden második lépésében szélső mezőre lép, s mivel a huszár felváltva lép fehérről feketére, így a szélső mezők mindegyikének fehérnek (vagy feketének) kellene lennie. 1462. Hasonló az előzőhöz: a 4 szélső sorból a huszár a három középső valamelyikére léphet. 1463. Nem. Legyen a jobbra, a felfelé, a balra lefelé történő lépések száma rendre x, y, z. A feltétel szerint: x + y + z = n2 − 1, y = z, x = z + 1. Ezekből 3z + 2 = n2 , de négyzetszám 3-mal osztva nem adhat 2 maradékot.

26. Feladatok a sakktáblán

301

1464. a) Nem lehet. Fekete (fehér) mezőn álló bábu ismét fekete (fehér) mezőre lép. b) Nem lehet. Színezzük a tábla sorait felváltva fehérre, feketére! Ekkor a 9 figurából pl. 6 áll fekete, 3 fehér mezőn. Ez a lépések során nem változik, pedig a célul kitűzött állapotban 6 figurának fehéren, 3-nak feketén kell állnia. Ez egyben újabb indoklása az a) feladat megoldásának. 1465. 32 huszár elhelyezhető, pl. ha mindegyiket fehér mezőre tesszük. Többet nem lehet elhelyezni, hiszen a tábla ábra szerinti 4 × 2-es részére legfeljebb 4 huszár helyezhető (ugyanis az azonos számmal jelölt mezőkből legfeljebb az egyiken állhat huszár), így az egész táblán legfeljebb 8 × 4 huszár állhat.

1

2

3

4

2

1

4

3

1466. Az egyszínű mezők száma 12 és 13, ezért 13 huszár elhelyezhető a kívánt módon. Többet nem lehet feltenni, mert — az ábrákon látható bejárási sorszámozás miatt — nem állhatnak huszárok szomszédos sorszámú mezőn.

1467.

• •••

1

20

9

14

3

1

10

15

22

3

10

15

2

19

24

16

21

2

9

14

21

8

25

4

13

11

8

17

4

23

16

11

6

23

18

20

25

6

13

18

7

22

17

12

5

7

12

19

24

5

•• ••

• •• •• • • •• •• •



• ••• •

•• • • ••

• •



•• • • ••

•• ••



••••• •••••

• •• •• •• ••

•• •• •• ••

1468. Osszuk a táblát 2 × 2-es részekre. Egy 2 × 2-es részben legfeljebb egy király állhat, így a táblán legfeljebb 16 király lehet. Ennyit el is lehet helyezni.

Megoldások, útmutatások

302

1469.







1470.



• •















1471. A bekeretezett területeken kell állnia királynak; különben lenne olyan mező, mely nem áll ütés alatt. Tehát legalább 9 király szükséges; s az ábra mutatja, hogy ennyivel a cél el is érhető.



















1472. 10-et el lehet helyezni (az ábrák mutatják). Többet nem, még bástyát sem. Ha 11 bástya van, akkor kell lennie 3 sornak, melyben 2–2 bástya áll. Ezeknek különböző oszlopokban kell elhelyezkedniük, azonban a maradék két oszlopban csak 2 × 2 bástya állhat.

• •

• •









••

• •

••

• •

• •

••

• • •



•• • •

• •

•• •



• ••



• •

26. Feladatok a sakktáblán

1473.

303

• • • • • • • •

• • • • • • • •

• • • • • • • •

• • • • • • • •

1474–1475. Figyeljük meg a „közlekedési szabályokat”.









8

1 4 7 6 2 3 8 5

1476. Mindig ugyanazt a számot kapjuk az öszszegzéskor. Ugyanis a számokat az ábra szerint felbontva két szám összegére, látható, hogy a nyolc szám összege: 0 + 8 + + 16 + · · · + 56 + 1 + 2 + 3 + · · · + 8.

1

2

7

3 6

4

5

...

.. .

0+1

0+2

0+3

8+1

8+2

8+3

... ...

0+8 8+8

16+1 16+2 16+3 16+8 .. .. .. .. .. . .. .. . . . . . . ... 56+1 56+2 56+3 56+8 ...

1477. Az előző feladat eredményét alkalmazzuk. Számozzuk meg a táblázat mezőit, soronként kitöltve az 1, 2, 3, . . . , 8, 10, 11, 12, . . . , 17, 19, 20, . . . számokkal (vigyázat!, pl. a második sort nem 9-cel kezdtük, hanem 10-zel). Így értük el, hogy fekete mezőkön mondjuk páratlan számok, a fehéreken páros számok állnak. Ha elhelyeztük a nyolc bástyát, adjuk össze az e mezőkön álló számokat, a végeredmény mindig ugyanaz a páros szám, tehát az összeadandók között páratlan számból páros sok van, ezért a fekete mezőkön álló bástyák száma páros. 1478. Figyeljük a tábla színezését; s gondoljunk arra, hogy a fehér és fekete mezők száma eltérő; s a bogarak fehér mezőről feketére, feketéről fehérre lépnek. 1479. Nem. Ha a táblázatban levő számokat soronként adjuk össze, akkor pozitív az összeg; ha oszloponként, akkor ugyanez az összeg negatív lenne.

Megoldások, útmutatások

304

1480. Lehet. 1 1 1 1 1 1 −4 1 −4 1 1 1 1 1 1 1 −4 1 −4 1 1 1 1 1 1

3 3 3 3 3 −4 −4 −4 −4 −4 3 3 3 3 3 −4 −4 −4 −4 −4 3 3 3 3 3

1481. Nem. Lásd az 1479. feladatot. 1482. Válasszuk ki valamelyik számot, pl. az 1-et; s úgy állítsuk párba azokat a mezőket, melyen az 1 áll, hogy a párban álló mezők egymás tükörképei legyenek, ha a főátlóra tükrözünk. Mivel öt ilyen mező van, valamelyik mezőnek önmaga a képe; ezért a főátlóban ott áll az 1-es szám. A hasonló állítás általában is igaz n × n-es táblázatra, ha az n páratlan szám, s ha n páros, akkor nem igaz. 1483. A táblázat bármelyik mezőjéről bármelyik másikra eljuthatunk legfeljebb 2n − 2 lépésben úgy, hogy minden lépésben egy mezőről valamelyik szomszédjára lépünk. Mivel a szomszédos mezőkbe írt számok különbsége legfeljebb 1, így a táblázatba írt számok maximumának és minimumának különbsége legfeljebb 2n − 2. Ez azt jelenti, hogy a táblázatban legfeljebb 2n − 1 különböző szám fordul elő. Ebből következik, hogy a táblázatba írt számok között van lega  olyan,  amelyik n n2 n2 n n2 n2 helyen szerepel. > = , így ≥ . lább 2n − 1 2n − 1  2n 2 2n − 1 2 n Van tehát olyan szám, amelyből legalább darab van a táblázatban. 2 Megjegyzés. Több is igaz. Van olyan szám, amelyik minden sorban szerepel; vagy pedig olyan szám, amelyik minden oszlopban előfordul. Van tehát olyan szám, amelyik legalább n-szer szerepel. 1484. Tekintsük a sakktábla egy tetszőleges számozását; és válasszuk ki azt a két mezőt, amelyen az 1, ill. a 64 áll. Az 1-est tartalmazó mezőről oldalszomszédos mezőkön lépkedve legfeljebb 14 lépésben eljutunk a 64-es számhoz. Ha egy-egy lépésben legfeljebb 4-gyel nagyobb számra lépünk, akkor a 14. lépésben legfeljebb az 57 = 1 + 14 · 4 számra léphetünk. Emiatt legalább az egyik lépésben (legalább) 5-tel nagyobb számra léptünk. 1485. Tekintsük a sakktábla egy tetszőleges számozását, és helyezzünk el egy királyt — amelyik egy lépésben szomszédos mezőkre léphet — azon a mezőn, amelyik az 1-es számot kapta. Bárhol áll is a király, legfeljebb 7 lépésben a sakktábla akármelyik mezejére, így a 64-es számúra is eljuthat. Az útja során megtett távolságok összege legalább 63, így a király legalább egy alkalommal „9-et lépett”, hisz 7 · 8 csak 56. Ez azt jelenti, hogy bárhogyan is számozzuk meg a sakktáblát, van két olyan szomszédos mező, melyek távolsága legalább 9, így a keresett szám legalább 8.

26. Feladatok a sakktáblán

305

Azt viszont könnyű belátni, hogy a keresett szám éppen a 8. Ha ugyanis „folyamatosan” számozzuk meg a sakktáblát, azaz az n-edik sor k-adik mezőjére az (n−1)8+k számot írjuk, akkor a szomszédos mezők távolsága 1, 8, 7 vagy pedig 9 lehet. Van tehát olyan számozás, amikor nem lép fel 9-nél nagyobb távolság a szomszédos mezők között. 1486. Azt bizonyítjuk, hogy létezik olyan (a1 , b1 ), . . . , (an , bn ) diszjunkt, szomszédos mátrixelemekből álló pár, hogy max aj < min bj . Ekkor azon t indexre, 1≤j ≤n

1≤j ≤n

melyre bj maximális, fennáll bt = max bj ≥ n − 1 + min bj ≥ n + max aj ≥ n + at , 1≤j ≤n

1≤j ≤n

1≤j ≤n

és ezért (at , bt ) a kívánt tulajdonságú szomszédos elempár. Jelölje α a mátrix sormaximumainak minimumát. Tegyük fel, hogy ez a ν-edik sorban vétetik fel. 1. eset. Minden sorban van α-nál kisebb elem. Ekkor α definíciója szerint minden sorban van α-nál nem kisebb elem is, így minden sorból kiválasztható olyan szomszédos elempár, melynek egyik tagja α-nál kisebb, a másik nem kisebb. Az így kiválasztott n pár megfelel a követelményeknek. 2. eset. Van olyan sor, mondjuk a μ-edik, hogy minden eleme legalább α; α definíciója miatt μ = ν és ezért a mátrixelemek különböző volta miatt a μedik sor minden eleme nagyobb, mint α. Ekkor tehát minden oszlopban van α-nál nagyobb és α-nál nem nagyobb elem egyaránt. Innen az állítás ugyanúgy bizonyítható, mint az 1. esetben. 1487. a) Az ábra mutatja a kitöltést. 0 0 0 0 8 8 8 9 9 9 b) Jelölje qk és rk azoknak az oszlopoknak, ill. 0 0 0 1 1 1 1 9 9 9 soroknak a számát, amelyben előfordul a k szám0 0 0 1 1 1 2 2 2 2 jegy. 3 3 3 1 1 1 2 2 2 3 Minden oszlop fölé odaírjuk, hogy mely szám3 3 4 4 4 4 2 2 2 3 jegyekből található legalább egy darab az 3 3 4 4 4 5 5 5 5 3 adott oszlopban. Így az oszlopok fölé nyilván 6 6 4 4 4 5 5 5 6 6 q0 + q1 + · · · + q9 darab számot írtunk. Ugyan6 7 7 7 7 5 5 5 6 6 akkor, ha feltételezzük, hogy minden oszlop6 7 7 7 8 8 8 8 6 6 ban legfeljebb három különböző szám van, ak9 7 7 7 8 8 8 9 9 9 kor az összes oszlop fölé legfeljebb 30 darab szám kerülhet. Így q0 + q1 + · · · + q9 ≤ 30. A sorokra hasonlóan kapjuk, hogy r0 + r1 + · · · + r9 ≤ 30. Mivel minden számjegy tízszer fordul elő, így qk rk ≥ 10. Ezért qk + rk ≥ √ ≥ 2 qk rk > 6, azaz (q0 + r0 ) + (q1 + r1 ) + · · · + (q9 + r9 ) > 60, ami ellentmond az előző két egyenlőtlenségnek. Az ellentmondás azt igazolja, hogy van legalább négy különböző számot tartalmazó sor vagy oszlop.

Megoldások, útmutatások

306

1488. k ≥ 3, hiszen az 1-est tartalmazó mezőnek négy másik mezővel van közös oldala, ugyanakkor a papírlapon csak 2 darab 2-es szerepel. Így az egyik szomszédos mezőbe kettőnél nagyobb szám kerül, tehát különbségük legalább 2. A k = 3 értékhez tartozó kitöltés többféle módon is megadható. 1489. Az 1, 2, . . . , 120 számok mindegyikét ötször írhatjuk be, tehát e számok összesen 600 mezőt fednek le. Másrészt minden olyan mezőt e számokkal kell lefednünk, amelyekre soruk és oszlopuk sorszámát összeszorozva 120-nál nem na120 = gyobb számot kapunk. Az első sorban 120 ilyen mező van, a másodikban 2   120 120 = 60, a harmadikban , . . . , az i-edik sorban olyan mező van, amely 3 i megfelel. Ezeket összeadva 602 mezőt számolunk meg. Ezek szerint több mezőt kell 120-nál nem nagyobb számokkal lefednünk, mint ahány szám rendelkezésünkre áll; így a táblázat nem tölthető ki a feladat kívánalmainak megfelelően. 1491. Számozzuk meg a végtelen négyzetrács mezőit az ábra szerint az 1, 2, 3, 4 számokkal! A 100 mezőből legalább 25 mezőn ugyanaz a szám áll, s ezeknek a mezőknek megvan a kívánt tulajdonságuk.

1 3 1 3 1 3 1

2 4 2 4 2 4 2

1 3 1 3 1 3 1

2 4 2 4 2 4 2

1 3 1 3 1 3 1

2 4 2 4 2 4 2

1 3 1 3 1 3 1

2 4 2 4 2 4 2

1 3 1 3 1 3 1

2 4 2 4 2 4 2

1 3 1 3 1 3 1

2 4 2 4 2 4 2

1 3 1 3 1 3 1

2 4 2 4 2 4 2

1492. A 29 × 29-es négyzetrács mezőit jelöljük meg az ábra szerint pontokkal! Öszszesen 100 db 2 × 2-es részt jelöltünk meg. Vegyük észre, hogy ha bármiképpen vágtunk ki egy 2 × 2-es részt, az csak egy, a pontokkal megjelölt 2 × 2-es részt metsz. ... ... ... ... ... ... .. .. .. .. .. .. .. .. .. .. .. . .. .. .. .. .. .. . . . . . . . . . . . . . . . . ... ... ... ...

•• ••

•• ••

•• ••

•• ••

•• ••

•• ••

•• ••

•• ••

•• ••

•• ••

•• ••

•• ••

•• ••

•• ••

•• ••

26. Feladatok a sakktáblán

307

1493. Általánosabban vizsgáljuk a feladatot, a tábla mérete legyen m × m. Helyezzünk a kiválasztott mezőkbe csillagot. Ha az oszlopokban levő csillagok száma rendre   m    xi m ≤ . x1 , x2 , . . . xm , akkor 2 2 i=1 Ennek oka: ha egy oszlopban kiválasztunk két csillagot, akkor másik oszlopban ugyanannak a két pozíciónak mindegyikében nem lehet csillag (mert ekkor felbukkanna a tiltott téglalap); tehát oszloponként a csillagpárok más-más pozíciókat (sor-párokat) jelölnek ki, így ezeknek   a pároknak a száma legfeljebb annyi, m mint az összes sor-pár száma, azaz . 2     m m m   xi  m xi (xi − 1) m(m − 1) Legyen xi = k. ≤ , azaz ≤ . Innen: 2 2 2 2 i=1 i=1 i=1 m m   xi2 ≤ m(m − 1) + xi = m(m − 1) + k. i=1

m 

i=1

xi2 ≥ (x1 + x2 + · · · + xm )2 /m = k 2 /m, így k 2 /m ≤ m(m − 1) + k, i=1 √ m + m 4m − 3 ahonnan k ≤ . Ez m = 7 esetén a k ≤ 21 egyenlőtlenséget 2 jelenti. A mellékelt ábra mutatja, hogy 21 mező kijelölhető a kívánt módon.

Mivel

























1494. Játsszuk a játékot fekete-fehér sakktáblán. Látható, hogy 9 figura fekete, 6 pedig fehér mezőn áll, s ez is marad minden lépés után. Ezért a kívánt állás nem valósítható meg.

1495. Jelöljük a három színt az 1-es, 2-es, 3-as számokkal. Színezzük a tábla mezőit az ábra szerint 3 színnel úgy, hogy az egyik átlós irányban (a vízszintessel 45◦ -os szöget bezáró irányban) azonos színűek legyenek a mezők. Mindegyik színnel 11–11 mezőt színeztünk ki. Kezdetben az azonos színű mezőkön álló bábuk száma: a = 11 (az 1-es színű mezőkön álló bábuk száma), b = 11 (a 2-es színű mezőkön álló bábuk száma), c = 10 (a 3-as színű mezőkön álló bábuk száma).

Megoldások, útmutatások

308

Hogyan változnak ezek a számok egy lépés során? Ahonnan ugrunk és amelyik mezőt átugorjuk, e két szín esetén eggyel–eggyel csökken az ilyen színű mezőn álló bábuk száma; míg ahová érkezünk, az olyan színű mezőkből eggyel több mezőn lesz bábu. Tehát a, b, c mindegyike 1-gyel változik, ezért c párossága mindig különbözni fog a és b párosságától. Ha csak egy bábu marad, az úgy lehet, hogy a = 0, b = 0, c = 1. Azaz a megmaradó bábu csak olyan mezőn állhat, amelynek a színe ugyanolyan, mint a középső mező színe. Ez 11 mezőt jelent. Most színezzük újra a játéktáblát, hasonlóan az elő1 3 2 zőhöz, csak a másik átlós irányban. Ismét azt kapjuk, 2 1 3 hogy az egyedül maradó bábu 11 mező valamelyikén állhat. Azonban csak 5 olyan mező van, amely 2 1 3 2 1 3 2 az előző 11 mező között is szerepelt; csak ezeken áll3 2 1 3 2 1 3 hat a megmaradó bábu (vagy a tábla közepén, vagy 1 3 2 1 3 2 1 a négy oldal valamelyikének középső mezőjén). 3 2 1

1 3 2 1496. A hiányos sakktáblán nem egyenlő a fekete és a fehér mezők száma, s a dominók ugyanannyi fehér és fekete mezőt fednek le. Ha egy fekete és egy fehér mezőt „tiltunk le”, mint pl. az ábrán látható, akkor a megrajzolt „útvonal” mutatja a lefedés módját.

1497. A sakktáblát az ábra szerint számozva, bárhogyan is helyezünk a táblára 1 × 3-as dominót, az mind a három számból lefed egyet-egyet. Ha a táblát lefedtük volna, akkor ugyanannyi lenne a táblán az 1, 2, 3 számok mindegyikéből (mindegyikből 21), de pl. az ábrán az 1-esből 22 db van. Tehát a tábla nem fedhető le.

1 3 2 1 3 2 1

2 1 3 2 1 3 2 1

3 2 1 3 2 1 3 2

1 3 2 1 3 2 1 3

2 1 3 2 1 3 2 1

3 2 1 3 2 1 3 2

1 3 2 1 3 2 1 3

2 1 3 2 1 3 2 1

26. Feladatok a sakktáblán

309

Tekintsük a 8 × 8-as sakktáblának a következő ábrák szerinti két számozását. 1 3 2 1 3 2 1 3

2 1 3 2 1 3 2 1

3 2 1 3 2 1 3 2

1 3 2 1 3 2 1 3

2 1 3 2 1 3 2 1

3 2 1 3 2 1 3 2

1 3 2 1 3 2 1 3

2 1 3 2 1 3 2 1

1 2 3 1 2 3 1 2

2 3 1 2 3 1 2 3

3 1 2 3 1 2 3 1

1 2 3 1 2 3 1 2

2 3 1 2 3 1 2 3

3 1 2 3 1 2 3 1

1 2 3 1 2 3 1 2

2 3 1 2 3 1 2 3

Ezekre is bárhogyan helyezünk egy 1 × 3-as dominót, az mind a három számból lefed egyet-egyet. Az első táblán az 1-ből, a másodikon a 2-ből van 22 db. Tehát, ha a táblára feltettünk 21 db 1 × 3as dominót, akkor a lefedetlen mező az egyik táblán az 1-es, a másikon a 2-es számot viseli. Négy ilyen mező van; ezek bármelyikének „letiltása” esetén már lefedhető a tábla. 1498. Nem lehet. Hasonló az előzőhöz, csak most a mezőket az 1, 2, 3, 4 számokkal számozzuk. 1499. Nem lehet. Színezzük a sakktáblát a szokásos módon. Az adott síkidom 1 vagy 3 fekete mezőt fed le; a 10 × 10-es tábla lefedéséhez szükséges 25 db dominó páratlan számú fekete mezőt fed le, tehát nem fedheti le az 50 fekete mezőt. 1500. Nem lehet. A megadott dominók páratlan számú fekete mezőt fednek le. 1501. Nem lehet. Színezzük a sakktábla sorait felváltva fehérre, feketére. A dominók páratlan számú fekete mezőt fednek le. 1502. Nem lehet. Tekintsük a táblának az ábra szerinti színezését, s vizsgáljuk a dominók által fedett fekete mezők számának párosságát.

310

Megoldások, útmutatások

1504. Nem lehet. Tekintsük a téglalapnak az ábra szerinti színezését. Mivel 1 × 4-es dominókkal lefedhető, ezért a fekete mezők száma páros. Azonban, ha 1 db 2 × 2-es dominót elhelyeztünk, akkor páratlan számú fekete mező marad, s a téglalap többi része csupán 1 × 4-es dominókkal már nem fedhető le. (A feladatban levő egyik feltételre nem volt szükség. Melyikre?)

1505. Nem lehet. A keresett téglalapot sakktáblaszerűen színezve 10 fehér és 10 fekete mező lesz a táblán. A megadott alakzatokkal nem lehet lefedni 10 fekete mezőt. 1506. Színezzük a tábla sorait felváltva fehérre, feketére. Ekkor 5 sor fehér (= 70 fehér mező), 4 sor fekete (= 56 fekete mező). A 11 db 2×3-as dominó 33 fekete mezőt fed, a 10 db 3 × 2-es dominó pedig páros számú fekete mezőt. Tehát a dominók páratlan számú fekete mezőt fednek, miközben a fekete mezők száma 56, ami páros szám. 1507. A 101 × 101-es tábla mindegyik sorába írjuk be az 1, 2, 1, 2, 1, . . . , 1, 2, 1 számokat ebben a sorrendben. A táblára egy 2 × 2-es dominót helyezve, a dominó alatt levő számok összege 6, egy 3 × 3-as dominó alatt pedig 12 vagy 15 a számok összege — azaz a dominók alatt levő számok összege osztható 3-mal. Azonban a 101 × 101-es táblán levő számok összege nem osztható 3-mal, ezért a tábla nem fedhető le a kívánt módon. 1508. Nem lehet. Ugyanis a 39 = 5a + 11b egyenletnek nincs megoldása a nemnegatív egészek körében. 1509. Nem lehet. Tekintsük a táblának az ábra szerinti színezését. Egy-egy dominó legfeljebb egy fekete mezőt fed le, s mert 20 fekete mező van, így legalább 20 dominó szükséges a tábla lefedéséhez.

1510. Legfeljebb 14 dominó helyezhető el a táblán. Osszuk fel a táblát 2 vízszintes és 3 függőleges egyenessel részekre úgy, hogy a szomszédos párhuzamos egyenesek távolsága 3 egység legyen. Ezután színezzük ki a részeket sakktábla-szerűen

26. Feladatok a sakktáblán

311

feketére, fehérre. Így 48 fekete és 42 fehér mezőt kapunk (vagy fordítva). Minden 42 dominó három fehér és három fekete mezőt fed le. Ezért legfeljebb = 14 3 dominót tehetünk a táblára. (14 dominót el is tudunk helyezni a táblán.) 1511. A feladat „csak akkor” részét szükséges bizonyítani, a másik irány ugyanis nyilvánvaló. Töltsük ki a táblázat mezőit a 0, 1, 2, . . . , k − 1 számokkal úgy, hogy a táblán bárhogyan helyezünk el egy (k × 1)-es dominót, az csupa különböző számot fedjen le („átlós színezés”). Ennek vizsgálatával is eredményre juthatunk. Hasonló ötlettel, komplex számok segítségével gyorsabban érünk célba. Legyen ε = cos(2π/k) + i sin(2π/k) az első k-adik egységgyök, és tegyük fel, hogy az n × m-es táblát lefedtük (k × 1)-es dominókkal. Ha a tábla mezőit oly módon töltjük ki komplex számokkal, hogy az r. sor s. mezőjére az εr+s−2 egységgyököt írjuk, akkor minden (k×1)-es dominó a k. egységgyököknek egy teljes rendszerét fedi le, amelyeknek 0 az összege. Így a táblán levő összes egységgyök is 0-t ad összegül, azaz     n m  1 − εn 1 − εm εr+s−2 = εr−1 · εs−1 = 0= · . 1−ε 1−ε 1≤r≤n r=1 s=1 1≤s≤m

Az egyenlőség két oldalát összehasonlítva kapjuk, hogy εn = 1 vagy εm = 1, azaz k | n vagy k | m. 1512. Helyezzük a téglalapot olyan sakktáblára, melyben az alapnégyzetek oldalhosszai 1/2 hosszúságúak. Könnyű belátni, hogy a felosztásban szereplő kis téglalapok mindegyike a sakktáblából ugyanakkora területű fehér részt fed le, mint fekete részt. (Ez abból következik, hogy mindegyik kis téglalapnak valamelyik oldala egész.) Tehát a nagy téglalap is ugyanakkora területű fehér részt fed le, mint feketét. Így a feladat állítását bebizonyítjuk, ha megmutatjuk, hogy ez csak abban az esetben lehetséges, amikor a nagy téglalap valamelyik oldala egész. Tegyük fel, hogy ez nem így van. Vágjuk ki a téglalap jobb alsó sarkából azt a téglalapot, melynek oldalai olyan hosszúak, amennyivel a két oldal hosszúsága meghaladja a hozzá legközelebb eső egészt. A megmaradt konkáv hatszögben ugyanakkora területű fehér, mint fekete rész van (miért?), ezért ez a kivágott részben is így van. Azonban ha ezt megvizsgáljuk, meglátjuk ennek lehetetlenségét. 1513. a) Ha egy rácsegyenest keresztez egy dominó, akkor keresztez még legalább egy másik dominó is, hiszen az egyenes bármelyik oldalán páratlan számú fedetlen mező maradt. Ha mind a 10 rácsegyenest keresztezi dominó, akkor ez legalább 20 db dominót jelent, azonban a tábla lefedéséhez 18 db dominó szükséges. b) Hasonló az a)-hoz. 1514. Nem lehet, ugyanis egy-egy átfestés nem változtatja meg a fekete mezők számának párosságát.

312

Megoldások, útmutatások

1515. Igen, lehet. Bármely színezés megvalósítható; ugyanis néhány átfestéssel elérhető, hogy csak egy tetszőlegesen választott mező színe változzék meg. Ehhez színezzünk át minden olyan sor–oszlop párt, mely a kiválasztott mezőt tartalmazza. 1518. Nem, hiszen bármely 7 × 11 × 1-es rétegben a kis téglák 3n helyet foglalnak el, azonban 3  | 7 · 11. 1519. Nem. A kocka 2 × 2 × 2-es részeit felváltva színezzük sakktáblaszerűen fehérre, feketére! Egy 1 × 2 × 4-es téglát bárhogyan is helyezünk el, annak fele fekete, fele fehér lesz. Tehát, ha a kirakás megvalósítható, akkor a 6 × 6 × 6-os kocka fele fehér, fele fekete — s ez nem teljesül. 1520. Azt lássuk be, hogy a láda három, a, b, c élhossza közül legalább kettő páros, és ezek egyike osztható 4-gyel! Ha pl. a és b is páratlan lenne, akkor a láda nem lenne kitölthető a megadott téglákkal, hiszen bármely a × b × 1-es rétegből a téglák páros számú egységkockányi helyet foglalnak el. Az előző feladat megoldását követve belátható: az nem lehetséges, hogy a, b, c mindegyike páros, ám egyik sem osztható 4-gyel. A láda térfogata osztható 16-tal. Ezekből következik a várt állítás. 1521. Színezzük sakktáblaszerűen az egységkockákat felváltva fehérre, feketére! A 26 kockát úgy vesszük el, hogy felváltva veszünk fehér és fekete kockát, tehát a kockák felének feketének kell lennie, azonban ez nem teljesül. 1522. Nem. Helyezzük a Rubik-kockát egy Descartes-féle derékszögű koordináta-rendszerbe oly módon, hogy a merőleges oldalélek illeszkedjenek egy-egy tengelyre, és válasszuk a kocka egy kis négyzetének élhosszát 1-nek! Tekintsük a koordináták összegének változását, miközben egyik lapcsúcsból a másikba lépünk. Az összeg mindig páros számmal változik, tehát megőrzi paritását. A lapcsúcsok felénél a koordináták összege páros, másik felénél páratlan, ezért az útvonal bejárja az összes azonos paritású csúcsot. Tehát át kell haladnunk a kocka valamely csúcsán, de a csúcs három négyzetlapra illeszkedik, így a harmadik átló berajzolása után nem tudunk tovább haladni. 1523. A térfogatokat összehasonlítva legfeljebb 54 db 1 × 1 × 4-es téglát helyezhetünk a kockába. Azonban ez nem érhető el, legfeljebb 52 db 1 × 1 × 4-es téglát helyezhetünk el. Színezzük a kocka 27 db 2 × 2 × 2-es részét (sakktáblaszerűen) felváltva fehérre, feketére. Ekkor egy 1×1×4-es tégla két fehér és két fekete 1×1×1-es kockából áll. Mivel a színezés során 104 db 1 × 1 × 1-es kocka fehér és 112 fekete (vagy 104 fordítva), ezért legfeljebb = 52 1 × 1 × 4-es téglát helyezhetünk az adott 2 kockába. Nem nehéz megmutatni, hogy ennyit el lehet helyezni. 1524. Kirakható, de csak úgy, hogy az 1 × 1 × 1-es kockák a 3 × 3 × 3-as kocka valamelyik testátlóján vannak. Ha a kockát felszeleteljük 3 rétegre valamelyik

27. Skatulyaelv

313

oldallapjával párhuzamosan, akkor mindegyik réteg térfogata 9 egység, és egy ilyen réteg nem tölthető ki 2 × 2 × 1-es téglákkal. Ezért mindegyik rétegben kell lennie 1 × 1 × 1-es kockának. Ha a kockát más oldallapjával párhuzamosan szeleteljük fel, ugyanezt állapíthatjuk meg. Ugyanezt megtesszük még a harmadik irányból is. Csak úgy lehet minden rétegben 1×1×1-es kocka, ha ezek a kockák valamelyik testátló mentén helyezkednek el. 1525. Ha k páros, akkor a színezés elvégezhető. A kockát osszuk fel 2 × 2 × 1-es blokkokra, s ezeket váltakozva színezzük feketére, fehérre. Belátjuk, hogy ha a színezés elvégezhető, akkor k páros. Kössük össze a szomszédos fehér színű kockák középpontját; minden középpontból két szakasz indul. Ezek a szakaszok zárt töröttvonalat alkotnak; egy vagy több ilyen zárt vonal keletkezik. Minden zárt vonalon páros számú csúcspont van, hiszen a szakaszok három, páronként merőleges irányban futhatnak. Ha egy csúcspontból pl. 3 egységnyit távolodunk előre a szakaszokon, akkor ugyanennyit meg kell tenni visszafelé is, hogy záródjon az útvonal. Emiatt a fehér kockák száma páros, és páros a fekete kockák száma is. 1526. Nem lehet. A fekete átlók száma az egyik irányban 7, a másikban 8. Így, ha megszámláljuk a fekete mezőkön álló bábukat, egyik irányú átlókon haladva 7 páratlan szám összege adja a végeredményt, másik irányú átlókon haladva 8 páratlan szám összege a végeredmény. Ez a két összeg nem lehet egyenlő, ezért nem valósítható meg a kívánt elrendezés. 1527. Legfeljebb 48 bábu helyezhető el. Ugyanis van 16 átló, melyek mindegyike páratlan számú mezőből áll, és ezeknek az átlóknak nincs közös mezőjük. Ezért mindegyik átlónak legalább egyik mezőjén nem áll bábu, emiatt legfeljebb 48 bábut tehetünk fel. Ez meg is valósítható, ha a két, 8 mezőből álló átlóba nem teszünk bábut. 1528. Lehet. Az első sor: 1, 1, 0, −1, −1, 0,1, 1, 0, −1, −1, 0, 1, 1, 0, −1, −1. A második sorban minden szám nulla, a harmadik sorban az első sor számainak (−1)-szerese áll, a negyedik sorban megint csupa nulla, stb. 1529. Ha 50 mezőn átmegy az egyenes, akkor a tábla belsejében 49 rácsegyenest metsz, azonban csak 48 ilyen egyenes van.

27. Skatulyaelv 1530. 4 · 9 + 1 = 37. 1531. a) 4 · 3 + 1 = 13. b) 20 + 20 + 20 + 4 = 64.

Megoldások, útmutatások

314

1532. a) b) c) d) e) f)

12, 20, 12, 23, 24, 7.

1533. a) b) c) d) e)

3, 22, 5, 7, 22,

f) 24, g) 11, h) 26. 1534. a) b) c) d) e) f) g) h) i) j) k)

4, 52, 42, 32, 6, 12, 56, 46, 54, 54, 2k + 2.

1535. 10 + 10 + 1 = 21. 1536. A fogak számára 33 lehetőség van: 0, 1, 2, 3, . . . , 32. Ha 33 skatulyába 34 nevet helyezünk, akkor lesz olyan skatulya, melyben van két név. 1537. A fogazat azonosítását megtehetjük egy 32-jegyű, 0 és 1 jegyekből álló számmal: 0 áll a k-adik helyen, ha a k-adik fog hiányzik, különben 1-es. Az ilyen különböző sorozatok — így a lehetőségek — száma: 232 . A válasz: 232 + 1 = = 4 294 967 297 (nincs ilyen ország a Földön, melynek ennyi lakosa van). 1538. Igaz.

27. Skatulyaelv

315

1539. Válasszunk ki egy cédulát, és legyen az erre írt szám m. Ekkor a többi cédulán szereplő n számok mindegyikére: m−1 000 000 ≤ n ≤ m+1 000 000. Ha minden ilyen n szám véges sokszor szerepel, akkor csak véges sok cédula lehet. 1540. Két szelvény kevés, három elegendő. 1541. Osszuk fel a sakktáblát 16 db 2 × 2-es részre! Nem állhat mindegyiken 2 bábu. 1542. Van 17 azonos színű tehén, mivel 4 szín van és 4 · 16 < 65. Ezek között — mivel csak legfeljebb négyféle korú tehén van — találhatunk 5 azonos korút, hiszen 4 · 4 < 17. Az 5 tehén között van 3, melyek ugyanabba a faluba tartoznak. 1543. A körlapot osszuk fel 6 db egybevágó körcikkre! Lesz két pont, mely ugyanabba a körcikkbe esik. 1544. Az előbbi felosztást most úgy végezzük el, hogy az egyik pont rajta legyen valamelyik, a felosztásban résztvevő egyenesen. 1545. A körbe írjunk szabályos hatszöget, és az oldalak, mint átmérők fölé írjunk 1 egység átmérőjű köröket, továbbá még egy ilyen kört rajzoljunk a kör középpontja körül. Ezt a hét kört úgy forgatjuk el az adott kör középpontja körül, hogy az adott 8 pontból egyik se illeszkedjék e körök valamelyikére. Így elértük, hogy a 8 pontból legalább kettő ugyanannak az 1 egység átmérőjű körnek a belsejében van, ezért távolságuk 1-nél kisebb. 1546. Osszuk fel a háromszöget 9 db egybevágó háromszögre. 1554. Az ábra szerint öt részre bontjuk a téglalapot. Ezek valamelyikében a 6 pontból legalább kettő van, s egy ilyen részben √ lévő pontok között fellépő legnagyobb távolság: 5.

865 − 1 = 432 olyan 1555. Elegendő megmutatnunk, hogy a háromszög felosztható 2 részre, amelyek mindegyike lefedhető egy 1 átmérőjű zárt félkörlemezzel, mivel akkor a 432 rész közül legalább egyben legalább 3 pontnak kell lennie. Ehhez osszuk az eredeti háromszög mindegyik oldalát 12 egyenlő részre, és az osztáspontokon keresztül húzzunk párhuzamosakat a háromszög oldalaival. Így 144 darab egybevágó, az eredetihez hasonló kis háromszögre daraboltuk fel a háromszöget. Mindegyik kis háromszög feldarabolható 3 kisebb, az adott háromszöghöz hasonló, egybevágó háromszögre, s ezeknek a derékszögű háromszögeknek az átfogója 1 egység hosszú. Ily módon eljutunk a kívánt felosztáshoz.

316

Megoldások, útmutatások

1556. Osszuk fel a háromszöget 12 részre úgy, hogy a részek mindegyike lefedhető legyen egy 5/17 átmérőjű körrel. Ehhez előbb rajzoljuk be a háromszög középvonalait, majd a kapott négy háromszög mindegyikét osszuk fel három egybevágó háromszögre.

30◦

30◦ 30◦

1557. Tekintsük a háromszög ábra szerinti szabályos hatszöglemezekkel történő lefedését! Megmutatjuk, hogy az állítás 20 pont helyett√már 17re is igaz. A hatszögek átmérője: 1/ 3 . Ha valamelyik fél-hatszögben 3 pont van, akkor készen vagyunk. Ellenkező esetben valamelyik hatszögben 4 pont van. Vegyük a hatszög köré írt körnek a 4 pont egyikén átmenő átmérőjét! Az átmérő egyik oldalára legalább két pont fog esni. 1558. Fedjük le a háromszöget az ábrán látható módon 6 darab egybevágó szabályos hatszöggel. Rövid számolás mutatja, hogy a hatszögek lefedhetők egy-egy, 0,2 m-nél kisebb sugarú körrel. A céltáblát 7 találat érte, ezért a 6 hatszög közül van olyan, amelyik a találatok közül legalább kettőt tartalmaz. Ennek a két találatnak a távolsága kisebb 40 cm-nél. 1559. Osszuk fel a négyzetet 16 db 2 cm oldalú négyzetre! Ezek egyikében lesz 3 pont. Lássuk be, hogy e pontok mint csúcsok által meghatározott háromszög területe legfeljebb fele a négyzet területének. 1560. Osszuk fel a négyzetet 25 db egybevágó, 1/5 oldalú √ négyzetre. Ezek egyikében lesz 3 pont, s e négyzet köré írható kör sugara 1/ 50 < 1/7. 1561. Osszuk fel a táblát 16 db egybevágó négyzetre. 1563. Osszuk fel a kockát 125 db egybevágó kockára. 1564. Indirekt módon bizonyítjuk. Tegyük fel, hogy bármely két pont között a távolság legalább egy egység. Rajzoljunk mindegyik pont köré 1/2 sugarú köröket; ezek nem metszik egymást, s benne vannak az eredetivel koncentrikus, 10,5 sugarú körben. Akkor a területeket összehasonlítva 450π(1/2)2 ≤ (10,5)2 π, azaz 450 ≤ ≤ 441 lenne. 1565. Tegyünk mindegyik pontra mint középpontra ilyen gyűrűt. Ezek összterülete 650· · 5π = 3250π. E körgyűrűk az adott körlemezzel koncentrikus, 19 egység sugarú körön belül vannak. Ennek a körnek van olyan pontja, melyet a körgyűrűk közül legalább 10 lefed, hiszen 9 · 192 π = 3249π < 3250π. E pont mint középpont köré írt körgyűrű lefedi a (legalább) tíz körgyűrű középpontját.

27. Skatulyaelv

317

1566. A kör sugara legyen r. Tegyük fel, hogy az állítás nem igaz. Akkor a pontok köré írt r/3 sugarú körlemezek nem metszik egymást. E 17 körlemez terüle17π 2 tének összege 17π(r/3)2 = r . A 17 körlemez lefedhető az eredeti körrel 9 r 16π 2 koncentrikus r + sugarú körrel. Ennek a körnek a területe r , ami kisebb 3 9 17π 2 r -nél, tehát nem fedi le a 17 körlemezt. Ellentmondásra jutottunk, hamis 9 volt a feltevés. Ezzel igazoltuk az állítást. 1567. Tekintsük a körnek a szabályos n-szög csúcsai által létrehozott n ívét. Ezek

közül az ívek közül az egyikben, pl. A1 A2 -ben legalább két pont van, amelyek 2π a szabályos (n + 1)-szög csúcsai, pl. B1 és B2 . Mivel az A1 A2 ív hossza ,a n 2π 2π B1 B2 ív hossza , az A1 B1 és a B2 A2 ívek hosszának összege . n+1 n(n + 1) 1568. Válasszuk azt az Ai Aj átlót, melyhez O a legközelebb van. 1569. A 8 csúcsból legalább 3 azonos színű, s ezek közt van kettő, melyeket nem köti össze a kocka valamely éle. 1570. Tekintsük a „nagy” háromszög három csúcsát és a három oldalfelező pontot. Ezt a 6 pontot lefedi az 5 „kis” háromszög, tehát valamelyikük két pontot is lefed; ezért ez a háromszög legalább a negyedét lefedi a „nagy” háromszögnek. 1571. Lássuk be, hogy egy ilyen egyenes a négyzet egyik középvonalát 2:3 arányban osztja. A két középvonalon összesen 4 ilyen pont van, ezért ezek valamelyikén legalább 3 átmegy a 9 egyenesből. 1572. Osszuk a körívet négy negyedkörívre úgy, hogy az egyik osztópont a 4 pont egyike legyen! (A megoldás menete hasonló az 1544. feladatéhoz.) 1573. Az öt pontból egyet válasszunk Északi-sarknak, s vegyük fel azokat a főköröket, melyek e ponton és egy-egy további ponton mennek át. Tekintsük e négy főkör és az Egyenlítő metszéspontjait. Vagy az Északi-sark és egy másik pontból álló páros a megfelelő, vagy az Egyenlítőn levő metszéspontokra alkalmazzuk az előző feladat megoldását. 1574. A köröket merőlegesen vetítsük a négyzet valamelyik oldalára. A vetületek hoszszának összege 1,02, ezért a vetületek között van átfedés. Az átfedett szakasz valamely pontjában állítsunk merőlegest a négyzet oldalára, ez az egyenes legalább két kört fog metszeni. 1575. A körök átmérőinek összege előzőhöz.

10 > 3. Innen a feladat megoldása hasonló az π

Megoldások, útmutatások

318

1576. Vetítsük a gömböket egy kiválasztott egyenesre. A „nagy” gömb 6 egység hosszú vetületének szakaszán lesz a többi vetülete, melyek hosszának összege 50. Mivel 50 > 6 · 8, ezért van olyan pont, mely legalább 9 vetületnek is közös pontja. E pontban állítsunk merőleges síkot az egyenesre. 1577. Vetítsük a töröttvonal szakaszait merőlegesen a négyzet két, egymásra merőleges oldalára: az ai szakasz vetületei xi , yi . Nyilván xi + yi ≥ ai . Ezért n n n n     xi + yi = (xi + yi ) ≥ ai > 1000. i=1

i=1

i=1

i=1

Tehát a bal oldali két összeg közül legalább az egyik > 500, így azon az egységnyi hosszú négyzetoldalon a vetületek az oldal valamely pontját legalább 501szer lefedik. E pontban állítsunk merőlegest a négyzetoldalra, s ez az egyenes legalább 501 pontban metszi a töröttvonalat. 1578. Hasonló az előzőhöz. 1579. Csak a tábla peremét figyeljük. Itt körben 28 mező, ill. megjelölt pont van, s már ezek elválasztásához is kevés 13 egyenes. 1580. Válasszunk két pontot, A-t és B-t, melyek távolsága nem kisebb 1-nél (ha nincs ilyen pontpár, akkor tetszőlegesen választhatunk), s rajzoljunk mindegyik körül 1 egység sugarú kört! A 25 pont bármelyike benne van valamelyik körben, ezért az egyik körben mindenképp lesz 13 pont. 1581. Válasszuk ki a kör egy AB átmérőjét. Ekkor

1000 

|APi | +

i=1

1000 

|BPi | =

i=1

+|BPi |) ≥ 2000, (hiszen |APi |+|BPi | ≥ |AB| = 2), ezért vagy vagy

1000 

1000 

1000 

(|APi | +

i=1

|APi | ≥ 1000,

i=1

|BPi | ≥ 1000.

i=1

1582. Vegyünk fel egy 1 egység oldalú szabályos háromszöget, ennek lesz két azonos színű csúcsa. 1583. Vegyünk két különböző színű pontot, melyek távolsága kisebb, mint 1 egység. Ezek köré írjunk 1 egység sugarú kört, s figyeljük a körök valamelyik metszéspontját. 1584. Használjuk fel az 1353. feladat megoldásának ábráját. 1585. Figyeljük egy szabályos hatszög csúcsait és a hatszög köré írt kör középpontját. Megjegyzés. A sík két színnel kiszínezhető úgy, hogy ne legyen olyan egységnyi oldalú szabályos háromszög, amelynek csúcsai ugyanolyan színűek.

27. Skatulyaelv

319

1587. Tekintsük az öt piros pont által meghatározott ötszöget. A tízszög középpontja körül ezt az ötszöget forgassuk el valamelyik irányba 36◦ − 36◦ -kal, amíg önmagába vissza nem tér. Ha az így meghatározott ötszögek valamelyikének csúcsai között található három kék, akkor a feladatbeli állítás nyilván teljesül. Tegyük fel, hogy egyik ötszögben sincs három kék csúcs. Ekkor a teljes körülforgatás közben összesen legfeljebb 2 · 9 = 18 kék csúcspont található. A körülforgatás során egy-egy tízszög-csúcson áthaladt mind az öt ötszög-csúcs, ezért minden egyes kék pontot ötször számoltunk. Mivel öt kék pont van, ezért összesen huszonöt pontot kellett volna összeszámolnunk. Itt ellentmondásra jutottunk, tehát a feltételezett eset nem állhat fenn. 1588. Válasszunk két szomszédos, azonos színű csúcsot. Ha a két pont bal vagy jobb szomszédja ugyanolyan színű, már megtaláltuk a kívánt háromszöget. Ellenkező esetben az eredetileg kiválasztott két pont és az általuk meghatározott szakasz felezőmerőlegesén levő csúcs, vagy a két pont két szomszédja és az utóbbi csúcs alkotta háromszög valamelyike megfelelő. Másik Tekintsünk 7 egyszínű csúcsot a 13-szög csúcsai közül. Össze megoldás.  7 sen = 21 olyan szakasz van, amelyeknek ezek a csúcsok a végpontjai. Mi2 vel a szabályos 13-szög csúcsai között csak 6 különböző hosszúságú szakasz húzható, ezért a 21 szakasz között van legalább 4, amelyeknek hossza egyenlő. Mivel ezek 7 pontot kötnek össze, ezért nem lehet minden végpontjuk különböző. Egyenlő hosszúságú szakaszok csatlakozása esetén viszont egyenlő szárú háromszög keletkezik, melynek csúcsai az előzők értelmében egyező színűek. Megjegyzés. A feladat állítása igaz marad, ha 13 helyébe 5-öt, 7-et vagy tetszőleges, 8-nál nagyobb egész számot mondunk. 1589. Tekintsük a sokszög három szomszédos csúcsát. Összesen 13 ilyen hármas van. Ezek két színnel való színezése 23 = 8-féleképp történhetik. Így találunk a 13 háromszög között kettőt, melyek ugyanúgy vannak színezve. 1590. Tekintsük az (1; i), (2; i), (3; i), (4; i), (5; i), (6; i), (7; i) 7 pontból álló rendszereket. Ezeknek 67 -féle különböző színezése lehet. Vegyünk 67 + 1 db ilyen 7 pontból álló rendszert. Ezek közt két lesz azonosan színezett. A 7 pont között van két azonos színű pont. Ezt a két pontot válasszuk ki mind a két, 7 pontból álló rendszerből. 1591. Egy rácspont két koordinátája párosság szerint négyféleképp viselkedhet: párospáros, páros-páratlan, páratlan-páros, páratlan-páratlan. Az öt rácspont között van két azonos típusú. A két pontot összekötő szakasz felezőpontja szintén rácspont. 1592. Egy-egy rácspont három koordinátájának párossága szempontjából 8-féle osztályba sorolható. Mivel 9 rácspontunk van, lesz közöttük kettő, amelyeknek első koordinátája azonos párosságú; ugyanígy a második koordináták, és a harmadik

Megoldások, útmutatások

320

koordináták is azonos paritásúak. Ezért e két pontot összekötő szakasz felezőpontjának koordinátái egész számok lesznek. 1594. Indirekt úton bizonyítunk. Tegyük fel, hogy nincs háromszög a kívánt tulajdonsággal. Ekkor van olyan ABC háromszög, melynek csúcsai kékek. Ha nem lenne, akkor a síknak legfeljebb egyetlen egyenese tartalmazhatna kék pontokat, s ekkor lenne piros csúcsokkal és súlyponttal háromszög. Legyen ABC súlypontja S, amely az indirekt feltevés miatt piros. Nagyítsuk Sből négyszeresére az ABC háromszöget, a képháromszög A B  C  . Az S pont az A B  C  háromszögnek is súlypontja. Ekkor az A BC háromszögnek A súlypontja (miért?). Indirekt feltevésünk miatt A piros. Hasonlóan mutatható meg, hogy B  is, C  is piros. Így az A B  C  háromszög csúcsai is, súlypontja is pirosak, ellentmondásra jutottunk. 1595. E prímek utolsó jegye csak 1, 3, 7 vagy 9 lehet. Tehát az öt szám között lesz kettő, melyek ugyanarra a számjegyre végződnek. 1596. Négyzetszámok utolsó jegye csak 0, 1, 4, 5, 6 vagy 9 lehet. Tehát a hét szám között lesz kettő, melyek ugyanarra a számjegyre végződnek. 1597. Négyzetszám 3-mal osztva 0 vagy 1 maradékot adhat. 1598. Négyzetszám 4-gyel osztva 0 vagy 1 maradékot adhat. 1599. Általában is igaz: n + 1 db egész szám között van kettő, melyek különbsége osztható n-nel. Ugyanis a számok n-nel osztva n-féle maradékot adhatnak; így lesz köztük kettő, melyek ugyanazt a maradékot adják, s ezek különbsége osztható n-nel. 1600. Általában is igaz: n + 2 db egész szám között van kettő, melyek összege vagy különbsége osztható (2n + 1)-gyel. 1601. Egy szám n-nel osztva 0, 1, 2, . . . , n − 1 maradékot adhat. Ezekbe a skatulyákba megfelelően elhelyezve a 2-hatványokat, lesz olyan skatulya, melyben végtelen sok szám lesz, s ezek közül bármely kettő különbsége osztható n-nel. 1602. Az 1, 11, 111, 1111, 11111, . . . , 111 . . . 11 (1992 db 1-es) számok között van kettő, melyek 1991-gyel osztva ugyanazt a maradékot adják; ezek különbsége osztható 1991-gyel, s ez a különbség 111 . . . 11100 . . . 000 alakú. Ha 111 . . . 11· · 10k osztható 1991-gyel, akkor 111 . . . 11 is osztható 1991-gyel. 1603. Készítsük el a következő számokat: a1 , a1 + a2 , a1 + a2 + a3 , . . . , a1 + a2 + +a3 + · · · + an . Ha e között az n db szám között nincs n-nel osztható, akkor van kettő, melyek n-nel osztva ugyanazt a maradékot adják. Ezek különbsége: ak+1 + ak+2 + · · · + am osztható n-nel. 1604. Három egész szám között van kettő, melyek ugyanolyan párosságúak.

27. Skatulyaelv

321

1605. Egy szám 3-mal osztva 0, 1 vagy 2 maradékot adhat. Ha az öt szám között van három, melyek ugyanazt a maradékot adják, akkor ezek összege osztható 3-mal. Ellenkező esetben kiválasztható három szám, melyek rendre 0, 1 és 2 maradékot adnak 3-mal osztva. Ezek összege osztható 3-mal. 1606. Három egész szám között van kettő, melyek összege osztható 2-vel; így van az adott 7 szám között is, legyenek ezek a és b. A megmaradó öt szám között is van kettő, c és d, melyek összege páros; végül az utolsó három között is találunk a+b c+d e+f , , egész számok, tehát közöttük van kettőt, e-t és f -et. Most 2 2 2 a+b c+d és . Így tehát kettő, melyek összege osztható 2-vel. Legyenek ezek 2 2   a+b+c+d 1 a+b c+d + = is egész, vagyis a + b + c + d osztható 4-gyel. 2 2 2 4 1607. Az a1 , a2 , . . . , a37 számok között van 13 olyan, mely 3-mal osztva ugyanazt a maradékot adja (hiszen 37 > 3 · 12). Ha e 13 szám indexét vesszük, az ilyen indexű bi -k között van 5 olyan, melyek 3-mal osztva ugyanazt a maradékot adják (hiszen 13 > 3 · 4). Ha ennek az öt számnak az indexét vesszük, az ilyen indexű ci -k között (lásd 1605. feladat!) van három, melyek összege osztható 3-mal. Ez az ilyen indexű bi -kre és ai -kre is igaz. 1608. Az 1605. feladat mutatja, hogy legfeljebb 4 prímszámot lehet így megadni. Négy ilyen prím pl.: 7, 11, 13, 23 vagy 19, 23, 37, 41. 1609. A 2-hatványok utolsó két jegyéből álló szám legfeljebb 100-féle lehet (ez a korlát könnyen csökkenthető). Tehát, ha veszünk 101 különböző 2-hatványt, lesz köztük kettő, melyek utolsó két jegye azonos. Ez már a periodikusságot jelenti, hiszen, ha 2n és 2m utolsó két jegye megegyezik, akkor 2n+1 és 2m+1 utolsó két jegye is azonos. 1610. Figyeljük az (fk , fk+1 ) számpár utolsó három jegyéből álló számpárt! Ezek legfeljebb 106 -félék lehetnek. Tehát van két olyan számpár, melyek utolsó három jegye megegyezik. Ez a sorozat képzési szabálya miatt jelenti a periodikusságot. 1611. Lássuk be, hogy a sorozat elemei kisebbek egy a0 -tól függő korlátnál. 1612. Az 1608. feladat megoldásához hasonlóan van két 3-hatvány is, 3k és 3n , melyek utolsó négy számjegye megegyezik. Ez azt jelenti, hogy 104 | 3k − 3n = 3n (3k−n − 1), tehát 104 | 3k−n − 1, s ez pontosan a feladat állítása. 1613. Hasonló az előző feladathoz, ill. a következő feladatnak speciális esete. 1614. Az m, m2 , m3 , . . . , mn+1 számok között van kettő, melyek n-nel osztva ugyanazt a maradékot adják; tehát n | ma − mb , azaz n | mk − 1.

322

Megoldások, útmutatások

1615. 100-ig 25 prím van. Ezek adnak 25 skatulyát, s még vegyük hozzá az 1-esét. A 27 számot elhelyezzük ezekbe a skatulyákba. Egy számot annak a prímnek a skatulyájába teszünk, mely osztója neki; ha több lehetőség is van az elhelyezésre, akkor is csak egyet választunk, tetszés szerint. Az 1-es skatulyájába csak az 1 kerülhet. Lesz két szám, mely ugyanabban a skatulyában van. 1616. Tegyük fel, hogy nincs három egyforma számjegy a 20-jegyű számban. Ekkor a számban mind a 10 számjegy pontosan kétszer szerepel. Ebben a számban a számjegyek összege: 2(0+1+· · ·+9) = 90. A szám osztható 3-mal, ami ellentmond annak, hogy a szám egy 3-tól különböző prímszámnak a hatványa. 1617. A bizonyítást Laczkovich Miklós: Sejtés és bizonyítás (TypoTEX, 1998) c. kitűnő könyvéből vettük. Például 10001 = 1002 + 12 pozitív osztói 1, 73, 137 és 10001. Ezek mindegyike előáll két négyzetszám összegeként: 1 = 12 + 02 , 73 = 82 + 32 , 137 = 112 + 42 . Az általános tételt bizonyítandó tegyük fel, hogy n osztója a 2 + b2 -nek. Nyilván feltehetjük, hogy n maga nem négyzet. Azt fogjuk belátni, hogy vannak olyan x, y egészek, melyek nem mindegyike nulla, és amelyekre teljesül, hogy |x|, √ |y| ≤ 2[ n], továbbá n osztója x 2 + y 2 -nek. E feltételekből következik, hogy √ 2 2 2 x + y ≤ 2[ n] < 2n, valamint x 2 + y 2 nullától különböző többszöröse n-nek. Ez csak úgy lehetséges, ha x 2 + y 2 = n, ez tehát bizonyítani fogja állításunkat. Mivel (a, b) = 1, ezért (n, b) = 1 (miért?), tehát n akkor és csak akkor osztója x 2 + y 2 -nek, ha n osztója a b2 x 2 + b2 y 2 = b2 x 2 − a 2 y 2 + (a 2 + b2 )y 2 = (bx − ay)(bx + ay) + (a 2 + b2 )y 2 számnak. Így n | x 2 + y 2 biztosan teljesül, ha n | bx √ − ay. Tekintsük a bx − ay számokat, ahol 0 ≤ x, y ≤ [ n]. Ezek száma √ ([ n] + 1)2 > n. Mivel ezen számok n-nel osztva csak n-féle maradékot adnak, ezért a skatulya-elv szerint van két különböző (x1 , y1 ) és (x2 , y2 ) pár úgy, hogy bx1 − ay1 és bx2 − ay2 azonos maradékot adnak n-nel osztva. Legyen x = x1 − x2 és y = y1 −y2 . Ekkor bx 2 −ay2 ) osztható n-nel. Másrészt √ −ay = (bx1 −ay1 )−(bx√ |x| ≤ max(x1 , x2 ) ≤ [ n], és hasonlóan |y| ≤ [ n]. Végül x és y nem lehetnek mindketten nullák, mert az (x1 , y1 ) és (x2 , y2 ) párok különbözőek voltak. Ezzel a bizonyítást befejeztük. 1619. Ha nincs 8 páronként metsző egyenes, akkor legfeljebb 7 olyan irány van, melyek valamelyikével az 50 egyenes bármelyike párhuzamos. 50 = 7 · 7 + 1, ezért van 8 egyenes, mely egymással párhuzamos. 1620. Minden intervallum befoglalható nem-csökkenő intervallumok egy sorozatába a tartalmazás szerint, így „láncokat” alkothatunk. Egy intervallumot k-adrendűnek nevezünk, ha azok a láncok, melyeknek ő a végső intervalluma, maximálisan k-hosszúak. Ha van az intervallumok között (n + 1) olyan, hogy egyik sincs a másikban, akkor készen vagyunk. Ha ez nem teljesül, akkor van (n + 1) hosszúságú lánc, hiszen 1, 2, 3, . . . , n-edrendű intervallumokból — feltevésünk szerint

27. Skatulyaelv

323

— legfeljebb n–n darab van, s ezeknek az intervallumoknak a száma legfeljebb n2 , noha az intervallumok száma n2 + 1. 1621. Minden helyiértéken a megadott 11 számból legalább két számban ugyanaz a tizedesjegy szerepel. 1622. Három egymást követő számjegyből álló számhármas összesen 103 lehet, míg a tizedesjegyek végtelen sorozatában végtelen sok, egymás utáni három számjegy van. 1623. Válasszuk ki a poliéder egyik legtöbb oldalú lapját! Ha ez a lap egy n-oldalú sokszög, akkor ehhez n db másik lap illeszkedik. Azonban ezek oldalainak száma csak 3, 4, 5, . . . , n lehet. Ezért van közöttük két azonos oldalszámú. 1624. Egy n csúcsú egyszerű gráfban a fokszámok a 0, 1, 2, . . . , n−1 értékeket vehetik fel, azonban a 0 és az n − 1 nem léphet fel egyszerre a gráfban. Az n csúcson a fokszámok tehát n − 1 különböző értéket vehetnek fel, ezért van két azonos fokszám. 1625. A 6 személyből egyet választva, az a többi öt közül hármat ismer (vagy hármat nem ismer). Ha e 3 személy közül valamely kettő ismeri egymást, akkor ők ketten a kiszemelt személlyel egy 3 fős klikket alkotnak (a 3 fő kölcsönösen ismeri egymást). Ha a 3 személy közül semelyik kettő nem ismeri egymást, akkor ez egy 3 fős antiklikk. Az állítás 5 főre nem igaz. 1626. Használjuk az előző és a 635. feladat állítását! 1628. Valamelyik tudóst kiválasztva, az a többi 16-tal három nyelven levelezik, így van 6 olyan partnere, akivel ugyanazt a nyelvet (pl. angol) használja, hiszen 3·5 < 16. Ha e 6 személy között van kettő, akik egymás között angolul leveleznek, akkor megvan a keresett hármas. Ellenkező esetben a 6 személy egymás között a német vagy a francia nyelvet használja. Közülük az egyik az öt másikból legalább 3mal ugyanazt a nyelvet (pl. német) használja, hiszen 2 · 2 < 5. Ha e három közül valamelyik kettő egymás között a németet használja, akkor megtaláltuk a keresett három főt, különben pedig ez a 3 levelező partner egymás között a francia nyelvet használja. 1629. Ugyanolyan, mint az előző vagy az 1625. feladat. 1630. Jelöljük az i-edik napon lejátszott játszmák számát ai -vel; i = 1, 2, . . . , 77. Képezzük az s1 = a1 , s2 = a1 + a2 , . . . , sk = a1 + a2 + · · · + ak , . . . , s77 = a1 + + a2 + · · · + a77 összegeket. Mivel mindegyik nap játszik a sakkmester, ai ≥ 1 (i = 1, 2, . . . , 77). Emiatt s1 < s2 < · · · < s77 . Képezzük most az s1 + 21, s2 + 21, . . . , s77 + 21 számokat. Így az si és az si + 21 számokból összesen 2 · 77 = = 154 darab van. Mivel a sakkmester hetenként legfeljebb 12 mérkőzést játszik, s77 ≤ 11 · 12 = 132, s77 + 21 ≤ 153. Azaz (a1 ≥ 1 miatt) a 154 db si és si + 21 szám bármelyikének értéke 1 és 153 közé esik (a határokat is megengedve). Ezért

324

Megoldások, útmutatások

lesz közöttük kettő, amelyik megegyezik. Az si számok mind különbözők, így különbözők az si + 21 számok is, tehát az egyezés valamelyik sk és valamelyik si +21 szám között van. sk = si +21-ből sk −si = 21, azaz ak +ak−1 +· · ·+ai+1 = 21. Sakkmesterünk tehát az (i + 1)-edik nappal kezdve, a k-adikkal bezárólag éppen 21 partit játszik le. 1631. Ha a kiinduló sortól különböző sorok bármelyikében megjelenik egy n szám, az azt jelenti, hogy az előző sorban valamelyik számból n darab egyenlő található. Jelölje k azoknak a különböző számoknak a számát, amelyek mindegyikéből az előző sorban pontosan n darab található. Ha k = 1, akkor a tekintett sor azon helyei alá, ahol n áll; a következő sorban is n-et írunk. Ha k > 1, akkor a következő sorban n alá kn kerül. Tehát a kiinduló sor számai alatt lefelé induló sorozatok mindegyike nemcsökkenő. Ha találunk két olyan, egymást követő sort, amelyek megegyeznek; akkor az ismétlődés minden további sorban be fog következni. Az első ismétlődő sort véges sok lépés után megtaláljuk, hiszen addig minden sorban legalább egy szám alá írt érték nő, másrészt viszont a kiinduló sor után leírt számok értéke legfeljebb 1000. Megjegyzés. Érdekes megvizsgálni hasonló módon pl. a verseket. Leírjuk egy vers első sorát, majd a sor alá számokat írunk. Minden betű alá azt a számot, mely megmutatja, hogy az a betű hányszor szerepel ebben a sorban. A számsorozatból feladatunk szabályai szerint képezzük az újabb számsorozatokat. Legfeljebb hányadik sor az, ahonnan kezdve már biztosan ismétlődnek a sorok? 1632. Válasszuk ki a sorok közül azt, amelyiken legtöbb bábu van, és hagyjuk el. (Ha több ilyen sor van, ezek bármelyikét választhatjuk.) Ezt ismételjük, amíg n sor nem marad. Ha az utolsó lépésben legalább 2 bábut hagytunk el, akkor az előző lépésekben is legalább 2–2 bábut hagytunk el, összesen legalább 2n bábut, így legfeljebb n bábu maradt az n × 2n-es táblán. Ha az utolsó lépésben 1 bábut hagytunk el, akkor a maradék n sor mindegyikén legfeljebb 1–1 bábu van. Tehát az n sor elhagyása után az n × 2n-es táblán legfeljebb n bábu maradt, ezeket pedig n oszlop elhagyásával eltüntethetjük. 1633. Az összegek lehetséges értékei: 5, 3, 1, −1, −3, −5. Mivel tíz összeg van, ezek között van két azonos. 1634. Az összegek száma: 2n + 2. Az összegek lehetséges értékei: n, n + 1, . . . , 3n, azaz összesen 2n + 1 lehetőség. 1635. Legyen m a legkisebb szám a táblázatban! Abból a mezőből, ahol ez a szám áll, oldalszomszédos mezőkön lépkedve legfeljebb 198 lépésben bármely mező elérhető, ezért a legnagyobb szám legfeljebb m + 198 · 20 = m + 3960. A táblázatban tehát legfeljebb 3961 különböző szám lehet. Mivel 2 · 3961 < < 100 · 100, így valamelyik szám legalább háromszor szerepel.

28. Matematikai játékok

325

28. Matematikai játékok 1636. A kezdő az asztal szimmetria-középpontjába helyezi az első érmét, majd mindig az ellenfele által elfoglalt helynek a középpontra való tükrös képére tesz. 1637. Először egy főátlót rajzol meg, majd mindig azt az átlót húzza meg, amelyik középpontosan tükrözött képe az ellenfél által legutóbb húzott átlónak. Különben mindegy, hogy követ-e valamilyen stratégiát, mert mindig a kezdő nyer. Ha az oldalak száma 2n + 1, akkor a második nyer. Ugyanis egy N oldalú sokszögbe a feltételek szerint átlókat rajzolva, a játék végére a sokszöget háromszögekre bontottuk. A háromszögek száma N − 2, hiszen a háromszögek szögeinek összege egyenlő a sokszög belső szögeinek összegével. Mindez azt jelenti, hogy a sokszögbe N − 3 átlót rajzoltunk (lásd a 113. feladatot!). 1638. Ha n páros, akkor a kezdő nyerhet, ha úgy játszik, ahogy az előző feladat megoldásánál leírtuk. Ha n páratlan, akkor a második nyerhet, ha mindig ellenfele lépésének a sokszög középpontjára tükrözött képét rajzolja be. 1640. A kezdő első lépése: a középső, ill. a középső két bábut dönti föl. A továbbiakban mindig ellenfele lépésének a középső(k)re vett tükörképét dönti el. 1641. A másodiknak van nyerő stratégiája, hiszen a kezdő első lépése után a játék megegyezik az előző feladat játékával. 1642. A másodiknak van nyerő stratégiája. Mindig ellenfele lépésének a kocka középpontjára tükrözésével kapott csúcsba helyezi az ugyanolyan színű korongot. 1643. Mindegyik játékos megakadályozhatja ellenfelét a győzelemben, ha első lépéseként három kitérő élt színez. 1644. A kezdő nyerhet, ha a (60, 70) állapotból a (60, 60) állást alakítja ki első lépésben, majd mindig az (m, m) állapotra törekszik. 1645. A kezdő nyerhet, ha első lépésében az (1, 64, 65) állást alakítja ki; majd mindig az (1, 2m, 2m + 1) vagy a (0, m, m) állapot valamelyikére törekszik. 1646. Ha kezdetben mindkét kupacban páros számú kavics volt, akkor a második nyerhet; különben a kezdő játékos. A nyerő stratégia az, hogy az asztalon mindkét kupacban páros számú kavicsot kell hagyni. 1647. A második játékos mindig „szimmetrikusan lép”: az első játékos által elfoglalt mezőnek a tábla középpontjára vett tükörképét foglalja el. 1648. Az első játékos első lépésében elfoglalja a tábla középső mezőjét, majd mindig „szimmetrikusan lép”: a második játékos által elfoglalt mezőnek a tábla középpontjára vett tükörképét foglalja el. 1649. A második játékosnak van nyerő stratégiája: a tábla középpontjára szimmetrikusan játszik (tehát ellenfele utolsó lépésének tükörképét lépi).

Megoldások, útmutatások

326

1650. Az a) és b) esetben a második játékosnak van nyerő stratégiája. (Középpontos szimmetria.) A c) esetben az első játékosnak van nyerő stratégiája: első lépésében elfoglalja azt az oszlopot, amely tartalmazza a téglatest szimmetriaközéppontját. Mindegyik esetben a második játékosnak van nyerő stratégiája. 1651. A kezdő játékosnak van nyerő stratégiája: előbb elfoglalja a tábla középpontját, majd arra szimmetrikusan játszik (tehát az ellenfél utolsó lépésének tükörképét lépi). 1652. a) Nyerhet a kezdő. Lásd az 1636. feladatot. b) A második nyerhet. Az ábra középpontjára szimmetrikusan játszik. 1653. A kezdő nyerhet. Elsőként 1 cukorkát áttesz, majd a 10 cukorkát tartalmazó kupacból 2 cukorkát elvesz; így mindegyik kupacban 8–8 cukorka marad. Legközelebb a cukorkák számát 6–6 darabra állítja be, s később is a 2n–2n darabszámot éri el. 1654. A 10 × 10-es táblázaton ábrázolhatjuk a játék pozícióit. Legyen a bal alsó sarok a nulladik sor nulladik mezője. Az m-edik sor és az n-edik oszlop közös mezője a játéknak azt az állapotát tükrözi, amikor az első kupacban m kavics, a második kupacban pedig n kavics van (0 ≤ m, n ≤ 9). +

+ +

+ +

+ +

+ +

+ +

+ +

+ +

+

+ +

+ +

+

+ +

+

+

+ +

+ +

+ + +

+

+

Visszafelé okoskodva (a bal alsó sarokból indulva) megtaláljuk a nyerő pozíciókat, ezeket a + jelek jelzik. Innen láthatjuk, hogy a kezdő játékos nem tud nyerő pozícióra lépni; ám a második igen, ha szemmel tartja a nyerő pozíciókat. A második játékosnak van nyerő stratégiája. 1655. A második játékos nyer, ha mindig ellenfele lépésének a tábla középpontjára tükrözésével kapott mezőre teszi bábuját. 1656. A második játékos nyer, ha mindig ellenfele lépésének a tábla középpontjára tükrözött mezőit színezi ki. 1657. A második nyerhet, ha mindig az átlóra tolja fel a bábut.

28. Matematikai játékok

1658. A kezdő mindig a rajz szerinti állapotot alakítja ki. ... b) a) ...

327

... ... ...

.. .. . .

1659. Ha n = m, akkor a kezdő nyerhet; ő mindig k × k-s méretű táblát hagy meg. Különben pedig a második számára követhető ez a nyerő stratégia. 1660. A kezdő nyerhet. Először két darab 5×5-ös táblára töri a csokit; majd mindaddig, amíg nem tud 1×1-est letörni, megismétli ellenfele törését a másik csokidarabon. 1661. A kezdő nyerhet. Először két darab 5 × 5-ös táblára töri a csokit, majd mindig megismétli ellenfele lépését a másik csokidarabon. 1662. Ezzel kapcsolatos az 1625. feladat. 5 pont esetén ügyes játékkal mindegyik játékos megakadályozhatja a másik győzelmét, s ekkor döntetlennel ér véget a játék. 6 pont esetén a kezdőnek van nyerő stratégiája. 1663. Felvehetjük a hat pontot úgy is, hogy azok egy szabályos hatszög csúcsait alkossák. A hatszög egyik, átlótól különböző szimmetriatengelye legyen t. A t tengelyre a hatszögnek két oldala (legyen ez a és b) és egy átlója merőleges. A kezdő játékos első húzása legyen a t-re merőleges átló. A továbbiakban mindig az ellenfél szakaszának a t egyenesre vonatkozó tükörképét rajzolja, illetve ha az ellenfél a vagy b egyikét húzza, ő berajzolja a másikat. Az adott eljárásnak megfelelő szakasz nyilván mindig behúzható. Lássuk be, hogy a kezdő nem zárhat háromszöget elsőként! Ez azt jelenti, hogy a kező nyer. 1664. A második nyerhet. Kiválasztja valamelyik 1 × 1-es rácsnégyzet középpontját, s ellenfele lépésének e pontra tükrözésével kapott pontot választja. 1665. A kezdő nyerhet, ha az adott szám utolsó számjegye nem nulla; különben a második nyerhet. Mindig a táblán levő szám utolsó jegyét vonja le a számból (így ő mindig 0-ra végződő számot hagy a táblán). 1666. Okoskodjunk „visszafelé”. Aki 4 gyufát hagy az asztalon, az már nyert. Előzőleg 8, korábban rendre 12, 16, 20, 24 szálat kell meghagyni. Ezt a kezdő megteheti. 1667. A második játékos el tudja érni azt, hogy a kezdő ne nyerhessen: mindig 4-re egészíti ki a kezdő által elvett gyufák számát. 1668. Az előbbi megoldáshoz hasonló. A kezdő nyerhet: 25, 20, 15, 10, 5 szál gyufát kell az asztalon hagynia.

328

Megoldások, útmutatások

1669. Az előző feladatok megoldásához hasonló. A kezdő nyerhet: 35, 28, 21, 14, 7 szál gyufát kell az asztalon hagynia. 1670. Az előző feladatokhoz hasonló. A kezdő nyerhet: 37, 30, 23, 16, 9, 2 szál gyufát kell az asztalon hagynia. 1671. Mindig annyi szál gyufát hagy az asztalon, amely szám a 6 többszöröse (ugyanis vehet 1, 2, 3, 4, ill. 5 szál gyufát). 1672. Bárhogyan játszanak is a játékosok, mindig a kezdő játékos a győztes; hiszen ha (a, b) = d, akkor d | a + b = 101, de 101 prímszám, ezért d = 1. (Érdekes megvizsgálni a játékot 101 helyett összetett számot választva.) 1673. Ha n osztható 3-mal, akkor a második nyerhet. Az első játékos vételét mindig 3-mal osztható számra egészíti ki. Ha n nem osztható 3-mal, akkor az első nyerhet. Első alkalommal az el nem vett gyufák számát 3-mal osztható számra állítja be, s a további lépéseiben is erre törekszik. 1674. Ha n páratlan, akkor a kezdő nyerhet; hiszen ő mindig léphet páros számra, de innen a második csak páratlanra léphet. Ha n páros, akkor (az előzőek szerint) a második nyer. 1675. Két esettel foglalkozunk aszerint, hogy a golyók g száma néggyel osztva 1 vagy 3 maradékot ad. 1. eset. Ha g = 2k + 1 és k páratlan, akkor mindegyik játékos legfeljebb k golyót vehet el egyszerre. Legyen a kezdő húzó A, a második húzó B. Ha A elsőre k darab (tehát páratlan sok) golyót húz , akkor B válasza után legalább 1, de legfeljebb k darab golyó marad. Így másodszorra attól függően, hogy B páros vagy páratlan sok golyót húzott, A egy kivételével valamennyi golyót elveszi, illetve egyetlen golyót sem hagy B-nek. Ez esetben tehát A mindig nyer, hiszen B-nek páratlan sok golyója lesz. 2. eset. Ha g = 2k + 1 és k(> 2) páros, akkor A előszörre k − 2 golyót vegyen el! Ha most B a maradék k + 3 golyóból 3-nál többet és legfeljebb k darabot vesz el, akkor ha az így adódó maradék páros, A ezt mind felveszi; ha pedig páratlan, akkor 1 kivételével veszi fel A ezeket. Ha B a maradék k + 3 golyóból csak egyet vesz el, akkor másodjára A is csak egyet húzzon, s ezzel előállt az 1. esetben tárgyalt helyzet, hiszen a maradék k + + 1 golyóból most B kénytelen húzni és legalább egyet meghagyni. Ha tehát a maradék páros, akkor A egy kivételével valamennyit vegye el; ha pedig páratlan a maradék, akkor valamennyit fel kell vennie. Ezzel beláttuk, hogy ha g ≥ 3 és g = 5, akkor mindig A nyer. A g = 1 esetben A veszít. Ha g = 5, akkor is A veszít, mert bármennyit is húzzon elsőre, B is ugyanannyit fog húzni, s ezzel eléri, hogy A-nál 1 vagy 3 golyó legyen.

28. Matematikai játékok

329

1678. Összesen 1990 két részre osztás történik (lásd a 113. feladatot!), ezért mindenképp a második játékos győz. 1679. a) A második nyerhet. b) A kezdő nyerhet. Cél: a legnagyobb kupacban mindig 2k − 1 db kavics legyen. 1680. Okoskodjunk „visszafelé”. Akinél a szorzat 56 és 111 között van (a határokat is beleértve), az ellenfelének következő lépése után 1000 fölé jut. Ugyanígy látható, hogy az előző nyerő intervallum a 4, 5, 6 számokból áll. Tehát a kezdő nyerhet. 1681. Ismét „visszafelé” okoskodunk. A nyerő számok (fordított sorrendben): 100, 90, 80, 71, 62, 53, 44, 35, 30, 23, 20, 14, 11, 5, 2. Tehát a második játékosnak van nyerő stratégiája. 1682. „Visszafelé” okoskodva megkereshetők a nyerő mezők. Az ábrán ezek a megjelölt mezők. A kezdő játékos mindig ezek valamelyikére lép. Egyszerűbben is leírható, hogyan lépjen a kezdő játékos: első lépése egyet jobbra fel, majd mindig ellenfele lépését ismétli meg.

1683. Belátjuk, hogy ha n · m páros, akkor a kezdő játékosnak van nyerő stratégiája, ha n · m páratlan, akkor a másodiknak. Ha n · m páros, akkor n vagy m páros. Ezért a tábla felosztható (2 × 1)-es téglalapokra. A kezdő játékos első lépése az induló mezőt tartalmazó téglalap második mezője, ezután is mindig a királyt tartalmazó téglalap másik mezőjére lép az első játékos. Ha n · m páratlan, akkor n is, m is páratlan. A sakktáblát, a bal felső sarok kivételével, lefedjük (2 × 1)-es téglalapokkal. Bárhová is lép a kezdő, a második játékos a királyt tartalmazó dominó másik mezőjére lép. 1686. Készítsük el a játék fáját! A fagráf csúcsai a játék egy-egy állapotát tükrözik; s a gráf élei mutatják, hogy mely állapotból mely állapotba lehet eljutni. A gráf magasságára teljes indukcióval igazolható az állítás. 1687. A játékosok lépésétől függetlenül a táblán egyedül maradó szám 1-es, ha m páratlan; ha pedig m páros, akkor 2-es marad a végén. Ennek oka, hogy a táblán levő 1-esek száma lépésenként vagy nem változik, vagy 2-vel csökken.

330

Megoldások, útmutatások

1688. Az 1686. feladat szerint valamelyik játékosnak biztosan van nyerő stratégiája. Megmutatjuk, hogy a második játékosnak nem lehet. Ugyanis, ha lenne, akkor a kezdő bármilyen kezdése esetén meg tudná nyerni a játékot. Kezdjen most az első játékos az 1 számmal! Ezzel szerepcsere történt: az első ezek után úgy játszhat, ahogyan a második játszana, ha az első kezdene azzal a számmal, amit a második írt az 1 után a táblára. Ellentmondásra jutottunk, hiszen ebben a játékban most a kezdőnek és a másodiknak is nyerő stratégiája lenne. Ha n = 10, akkor a kezdő első lépése a 6. Ezután az ellenfele által választott számnak a következő párosításban kijelölt párját választja: (4, 5), (7, 9), (8, 10). 1689. Ahogyan az előző feladatban, itt is a „helycserés” bizonyítás mutatja, hogy a kezdőnek van nyerő stratégiája. Ha n = 72, akkor a kezdő stratégiája a következő: Első választása a 6. Ha a második első lépése a (36, 24) valamelyike, akkor ő a másik számot választja, s megnyerte a játékot. Ha a második első választása a (9, 8) valamelyike, akkor ő a megmaradt számot választja; majd a (12, 18), (24, 36) párokból mindig azt, melynek a társát ellenfele elvitte. Ha a második első választása a (18, 4) valamelyike, akkor ő a megmaradt számmal válaszol; aztán a (8, 12), (24, 36) párokból a megmaradt számot veszi el. Ha a második a 12-t választja, akkor ő 9-cel válaszol, majd a (8, 18), (24, 36) párokból mindig a megmaradt számot veszi el. 1690. Mindenképpen a második játékos nyer. A felírt számok csak a kiinduló két szám (25 és 36) legnagyobb közös osztójának többszörösei közül kerülhetnek ki (euklideszi algoritmus). A 36-nál kisebb összes szám felírható, így 34 számot írnak fel a játékosok (hiszen két szám már fel volt írva), az utolsó számot a második játékos írja. 1691. Ha a négy együttható a, b, c, d lesz, a második játékosnak elegendő úgy megválasztani az utoljára felírt együtthatót, hogy a + b + c + d = 0 legyen. Ennek az egyenletnek x = 1 gyöke. 1692. Elérhető a cél: x 3 + kx 2 − x − k = (x 2 − 1)(x + k) = 0. 1693. Elérhető a cél: x 3 + ax 2 + x + a = (x 2 + 1)(x + a) = 0. 1694. Elérhető a cél. Az első játékos olyan a, b, c racionális számokat választ, amelyekre a + b + c = 0 és a = 0. Ekkor az ax 2 + bx + c = 0 egyenlet gyökei: x1 = 1, c x2 = . a 1695. A kezdő játékos elérheti, hogy az egyenletrendszer megoldása (0, 1, −1) legyen. Ehhez az szükséges, hogy mindegyik egyenletben y és z együtthatója ugyanaz a szám legyen. 1696. Ahogyan az 1689. feladatban, itt is a „helycserés” bizonyítás mutatja, hogy a kezdőnek van nyerő stratégiája. 1697. Itt is a „szerepcsere” segít. A kezdő kilép, majd visszalép a lóval.

28. Matematikai játékok

331

1698. Itt is a „helycsere” segít. A kezdő egy „távoli” mezőre lép. 1699. A kezdő után a második 2 jelet üres sorba helyez, hogy azok egyike se legyen a kezdő jelével egy oszlopban. A következő alkalommal üres sorba, saját oszlopaiba helyezi jeleit. Ekkor a másodiknak 4 vonala van 2–2 jellel, s ezeknek legfeljebb az egyikében van a kezdőnek 1 jele. A második játékos harmadik lépésében két szabad vonalának mindegyikére egy-egy jelet tesz. 1700. Vagy a második nyer, vagy döntetlen lesz. A második minden jel írása előtt megnézi, hogy tud-e harmadik azonos jelet írni. Ha nem, akkor a kezdő utolsónak írt jelével ellentétes jelet ír a tábla középpontjára szimmetrikus mezőre. Így a kezdő a következő lépésében szintén nem tud harmadik jelet sorba tenni, tehát nem győzhet. Arra viszont törekedhet, s el is érheti, hogy ne jöjjön létre sorban egymás mellett három azonos jel. 1701. A másodiknak van nyerő stratégiája. Mindig a kezdő lépésének a tábla középpontjára tükrözésével kapott mezőre lép. 1702. 1 × 2-es dominókkal való lefedés segít. Ha a sarokmező szomszédjáról indul, akkor mindig a kezdőnek van nyerő stratégiája. 1703. Ha m · n páratlan, akkor a kezdőnek; ha páros, akkor a másodiknak van nyerő stratégiája. Ehhez elegendő megmutatni, hogy ha m·n páros, akkor a tábla mezői párba állíthatók úgy, hogy a pár két tagja „huszárlépésre” legyen egymástól. Ha pedig m · n páratlan, akkor egy mező kivételével ez a párosítás elvégezhető. 1704. Képzeljük el, hogy nem sima négyzetrácsos papíron játszunk, hanem egy „sakktáblaszerű” játékmezőn, azaz a négyzetek ki vannak színezve felváltva feketére és fehérre. Kezdetben a fekete bábuk a fekete mezőkön, a fehér bábuk a fehér mezőkön állnak. Mivel minden lépés szomszédos mezőre történik, ezért a lyuk (az a mező, ahol nem áll bábu) minden lépés után más színű mezőre kerül. Ebből következik, hogy a kezdő minden lépése előtt a fekete mezőn lesz a lyuk, a második játékos lépései előtt pedig mindig fehéren. Ebből az is következik, hogy a játék során minden figura legfeljebb egyszer mozoghat; ugyanis pl. a kezdő valamelyik figurája, ha egyszer fehér mezőről feketére lép, akkor többet nem léphet; hiszen csak a kezdő tolhatná tovább, de mint láttuk, ő mindig csak fehér mezőről tolhat feketére bábut. Így kiderült, hogy a játék véges; mivel mindkét játékosnak 12 bábuja van, s minden bábu legfeljebb egyszer mozoghat, így legkésőbb 12 lépéspár után a játék biztosan véget ér. A második játékosnak van nyerő stratégiája. Ehhez képzeletben (2 × 1)-es dominókkal fedjük le a táblának azt a részét, ahol kezdetben a bábuk állnak. A stratégia a következő: a kezdő játékos egyik bábuját elmozdítja valamelyik képzeletbeli dominóról, ekkor a második játékos az abban a dominóban levő bábuját tolja az üresen maradt helyre. Ily módon a második játékos mindig tud válaszolni a kezdő játékos lépésére, s a második játékos lép utoljára.

332

Megoldások, útmutatások

1705. A másodiknak van nyerő stratégiája: mindig a kezdő által választott szám bal szomszédját foglalja el, ha az még választható, különben a jobb szomszédot, ha már az is foglalt, akkor tetszőlegesen választ. 1707. Ha n = 3k, akkor a másodiknak van nyerő stratégiája. Ha a kezdő a (6, 9), (7, 8) párokból az egyiket választotta, akkor ő a másikat írja. Ha n = 3k + 1, akkor a kezdőnek van nyerő stratégiája. Első lépésként egy 9-től különböző számjegyet ír, a továbbiakban pedig úgy játszik, ahogyan a második az előbb leírt esetben. Ha n = 3k + 2, akkor a kezdőnek van nyerő stratégiája. Első lépésében a 9-et írja. 1708. Ha n = 3k, akkor a másodiknak van nyerő stratégiája: ellenfele lépését mindig kiegészíti 6-ra. Ekkor a számjegyek összege 6 · 2n = 36k, ezért a kapott szám osztható 9-cel. Ha n nem osztható 3-mal, akkor a kezdő játékosnak van nyerő stratégiája. Első alkalommal 3-ast ír, majd ezután ellenfele lépését mindig kiegészíti 6-ra. Ekkor az első 2n − 1 számjegy összege 3 + 6(n − 1). Ha n = 3k + 1, akkor 3 + 6(n − 1) = = 18k + 3, amit a második játékos utolsó lépésében nem tud kiegészíteni 9 többszörösére. Ha n = 3k + 2, akkor 3 + 6(n − 1) = 18k + 9, s ezt sem tudja kiegészíteni 9 többszörösére a második játékos az utolsó lépésben. 1709. Tekintsük a számokat 2-től 100-ig! Azt állítjuk, hogy ha a feladatbeli játékot csak erre a 99 számra korlátozzuk, akkor az itteni játékban második játékos el tudja érni, hogy a műveleti jelek beírása után páros legyen az eredmény, bárhogyan is játsszék ellenfele. Ebből már következik, hogy a kezdő meg tudja nyerni az eredeti játékot: első lépésében vagy az összeadás, vagy a szorzás jelét írja az 1 és a 2 közé, s ettől függően páratlan, ill. páros lesz a végeredmény. A fenti, korlátozott játékban a második stratégiája: ha ellenfele felírt egy műveleti jelet, akkor ő ezzel a jellel szomszédos páratlan szám másik oldalára írja fel a szorzás jelét. Ezt mindig megteheti, hiszen a 99 szám közti 98 hely 49 párba sorolható: az egyes párok a páratlan számokat fogják közre. Mivel egy-egy lépésben egy pár két helyére kerülnek műveleti jelek, így a második játékos tud a fenti utasítás szerint játszani. A kitöltés befejezésekor a végeredmény valóban páros lesz, hisz minden egyes páratlan szám egy szomszédjával — ami páros — megszorozva vesz részt a számolásban. 1710. A fehér színével felfelé levő korong helyére írjunk 1-est, a fekete helyére 0-t; így kapunk egy kettes számrendszerbeli számot, nevezzük ezt állapotjelző számnak. Ha egy fehér korongot megfordítunk, akkor az állapotjelző szám értéke csökken. Ez azt jelenti, hogy a játék biztosan véget ér néhány lépés után, hiszen az állapotjelző szám értéke minden lépésben csökken, egyszer csak nulla lesz, azaz mindegyik korong a fekete oldalával felfelé lesz. Nincs nyerő stratégia. Ha az utolsó korong fekete oldalával felfelé van, akkor a második játékos nyer; ha az utolsó korong fehér, akkor a kezdő játékos. Ugyan-

28. Matematikai játékok

333

is minden lépésben egy korong, az utolsó biztosan megfordul. Az lépésenként váltakozva lesz fehér és fekete oldalával felfelé. Ha kezdéskor például a fehér oldalával van felfelé, akkor mindig a kezdő játékos lépését követően fordul fekete oldalára, a játék — mint láttuk — egyszer véget ér, s az utolsó lépés a kezdőé lesz, ő nyer. 1711. Legyen Béla stratégiája az, hogy mindig arra a lépcsőfokra rak követ, ahonnan András az előző lépésben elvett. Ezt Béla mindig megteheti, hiszen a következő lépcsőfokon vagy András lépése előtt is állt már kő, vagy éppen András tett oda követ. Egyszerűen belátható, hogy ha Béla így játszik, akkor András bármely lépése előtt az alsó lépcsőfokon van kő, és két egymás utáni lépcsőfok nem lehet üres (kivéve persze a felső lépcsőfokokat). Így András bármely lépése előtt a legfelső két lépcsőfok üres; hiszen összesen n kő van és közöttük legfeljebb n − 1 üres lépcsőfok. Ezért András soha nem rakhat a lépcső tetejére. (Célszerű játszani a játékot néhány lépésen át, ill. játszunk a megadott stratégiát követve is.) 1712. Vizsgáljuk a következő játékot. Az alábbi 3 × 3-as táblázatban ketten felváltva foglalnak el egy-egy mezőt. Az a játékos nyer, akinek először lesz három saját mezője egy vonalban (egy sorban, egy oszlopban vagy egy átlóban). 2 9 4 7 5 3 6 1 8 Mindkét játékos megakadályozhatja ellenfelét a győzelem elérésében, tehát egyiknek sincs nyerő stratégiája. 1713. A kezdő nyerhet. Először a 47, 48, . . . , 55 számokat veszi el, majd ellenfele lépését úgy egészíti ki választásával, hogy az elvett számok között mindegyiknek a párja is szerepeljen, ha a párbaállítás módja (k, 55 + k). 1715. A kezdő minden alkalommal legalább megkétszerezheti a még megmaradó számok közti legkisebb különbséget (ha minden választása alkalmával minden második számot kihagy). Így elérheti, hogy a játék végén m legalább 32 legyen. A második pedig minden lépése alkalmával a megmaradó számok közti legnagyobb különbséget felezheti; így elérheti, hogy m legfeljebb 32 legyen.

334

Megoldások, útmutatások

29. Különféle kombinatorikai feladatok 1716–1720. Használjuk fel, hogy egy gráfban a páratlan fokszámú csúcsok száma páros. 1721. Nem lehet. Tekintsük például a piros színű éleket! Ha három piros él van, akkor az egyik csúcsból nem indul piros él; ha a piros élek száma négy, akkor valamelyik csúcsból két piros él indul. 1722. Ha lehetne, akkor a 0 szám kilenc másikkal lenne párban. Ha azonban négy csúcsban szerepel a 0, akkor csak 8 szomszédja van; ha pedig öt csúcsban szerepel a 0, akkor 10 másik számmal lesz párban. 1723. Tekintsük az öt vonalon álló számok összegét; s tegyük fel, hogy ezek mindegyike páratlan! Ekkor az öt számot összeadva páratlan összeget kapunk; azonban ez az összeg a körökben álló számok összegének kétszerese, tehát páros. Ellentmondásra jutottunk; ennek oka, hogy hibás volt feltételezésünk. 1724. Több különböző elrendezés is lehetséges. 1725. Nem lehet. 1726. Nem lehet. Adjunk össze minden három szomszédos csúcsban levő számot, s számoljuk ki ezek összegét. Ha a háromtagú összegek mindegyike 13-nál nagyobb, akkor ezek összege legalább 8 · 14 = 112, azonban ez az összeg: 3 · (1 + 2 + 3 + + · · · + 8) = 108. 1727. Nem lehet. Ha összeadjuk a csúcsokba összefutó éleken levő összegeket — s ha azok egyenlők —, akkor a kapott összeg osztható 8-cal; azonban ez az összeg 2 · (1 + 2 + · · · + 12) = 2 · 6 · 13, de ez nem osztható 8-cal. 1730. Ha összeadjuk a 77 kéttagú összeget, akkor a gondolt számok összegének kétszeresét, tehát páros számot kapunk, s ez nem lehet 77 · 1991. 1731–1734. Hasonló az előzőkhöz. 1735. Ha a kiválasztott érme valódi, akkor maradt 5 hamis és 5 valódi. Ebből a 10 érméből 5 db-ot az egyik, 5-öt a másik serpenyőbe teszünk. Ha a bal serpenyőben k db hamis érme volt, akkor a jobb oldaliban 5 − k. A két oldal közti különbség (5 − k) − k = 5 − 2k gramm, s ez a szám páratlan. Ha hamis pénzt választottunk, akkor az előbbi különbség 4 − 2k, azaz páros szám. 1736. a) Nem lehet. A mondott összeg valójában a táblázatban levő számok összegének (ami 625 db páratlan szám összegeként páratlan) kétszerese. b) Nem lehet. 1. megoldás. Legyen a táblázatban mindenhol +1; számoljuk ki a mondott összeget, majd változtassuk a +1-eket −1-ekre, s figyeljük az összeg változását!

29. Különféle kombinatorikai feladatok

335

2. megoldás. Az 50 szám szorzata +1, hiszen ez megegyezik a táblázatban levő számok szorzatának négyzetével. Tehát az 50 szám között páros sok −1 van; míg az 50 szám összege csak akkor lehetne 0, ha közöttük 25 db +1 és 25 db −1 lenne. 1737–1738. Az előző b) feladat mindkét megoldása követhető. 1739. Nem igaz. Bármely n-re (n ≥ 4) adható ellenpélda. Ha n páros, akkor vegyünk egy (n − 1)-oldalú sokszöget, ez lesz egy gúla alaplapja, s írjunk a gúla mindegyik csúcsába −1-et. Ha n páratlan, akkor vegyünk két, egybevágó (n − 2)-szög alapú gúlát, ezeket alaplapjuknál illesszük össze! Az egyik csúcsra +1-et írunk, a másikra és az „alaplapon” levő csúcsokra −1-et. 1740. Kezdetben minden élre +1-et írunk. Egy csúcsot nevezzünk +1 vagy −1 csúcsnak aszerint, hogy a vizsgált szorzat +1 vagy −1. Megadunk egy eljárást, mely kettővel növeli a −1 csúcsok számát: válasszunk ki két +1 csúcsot; jelöljünk ki egy, a két csúcsot összekötő, a poliéder egymáshoz csatlakozó éleiből álló útvonalat, s ezeknek az éleknek mindegyikén változtassuk meg az ott álló szám előjelét! 1741. Követhető az 1736. b) feladat mindkét megoldása. 1742. b) Ha n = 4k, akkor a lefedés megvalósítható. Ha n páratlan, akkor még 1 × 2-es dominókkal sem lehet lefedni a táblát. Ha n páros, de nem osztható 4-gyel, akkor a dominók száma páratlan, így a számok szorzata negatív.

. −1 .. +1 −1 ... +1

−1 ... +1

. +1 .. −1

. +1 +1 .. ... . −1 −1 .. . −1 +1 .. ... . +1 −1 ..

−1 +1 −1 +1

1743. A szorzat kitalálásához legalább 50 kérdést kell feltenni. 1744–1745. Válasszuk ki a számok közül az egyik legkisebbet; látható, hogy ennek szomszédai egyenlők ezzel a számmal. Hasonló okoskodással kiderül, hogy minden szám egyenlő egymással. 1746. Nem lehet. Az 1, 2, 3, 9, 10 számok között nem lehet szomszédos, így ezek minden második helyet foglalnak el valamilyen sorrendben. A 8-nak nincs helye, csak a 3 mellett állhat, a többi nem lehet szomszédja. 1747. Nem lehet. Az 1, 2, 3, 11, 12, 13 számok nem lehetnek szomszédosak. E hat szám közé kell helyezni a maradék hetet. Figyeljük a 4 és 10 számokat! Ezeknek a fenti hat számból csak egy-egy szomszéd jut, ezért ezek ketten szomszédosak lesznek (a hét számból kettőnek szomszédosnak kell lennie). 1748. Igen, lehet. Például: −9, 5, 5, −9, 5, 5, . . . , −9, 5, 5, −9. Egy kör kerületén nem lehetséges ez a felírás, mert akkor a 25 szám összegének egyidejűleg pozitívnak és negatívnak is kellene lennie.

336

Megoldások, útmutatások

1749. Nem lehet. a1 + a2 + · · · + a34 > 0 és a1 + a2 + a3 + · · · + a30 < 0 miatt a31 + a32 + + a33 + a34 > 0. Ugyanígy a35 + a36 + a37 + a38 > 0, . . . , a47 + a48 + a49 + a50 > 0, melyekből a31 + · · · + a50 > 0, azonban az egyik feltétel szerint ez az összeg negatív. 1750. Egy 8-tagú sorozat: −1, −1, −1, 2, 2, −1, −1, −1. Egy 9-tagú sorozat: −1, −1, −1, −1, 5, −1, −1, −1, −1. Egy 10-tagú sorozat: 5, −7, 5, −7, 5, 5, −7, 5, −7, 5. A sorozat nyilván nem lehet 5 · 7 = 35 elemű. Ugyanis ekkor a sorozat elemeit „szakaszoljuk” 5-ösével (az első öt elem, a következő öt, . . . ), és így összeadva a számokat az eredmény pozitív lesz. Ha ugyanezt 7-esével tesszük meg, akkor az összegzésben negatív számokat adunk össze, az eredmény negatív. Ugyanannak a 35 számnak az összege nem lehet pozitív is és negatív is. Tegyük fel, hogy a sorozat 11 tagú és elemei: a1 , a2 , a3 , . . . , a11 . Rendezzük el a sorozat elemeit az alábbi módon: a1 a2 a3 a4 a5 a6 a7 a2 a3 a4 a5 a6 a7 a8 a3 a4 a5 a6 a7 a8 a9 a4 a5 a6 a7 a8 a9 a10 a5 a6 a7 a8 a9 a10 a11 A táblázat elemeit soronként összeadva, a feltétel szerint negatív számokat adunk össze (minden sorban negatív a hét szám összege), a táblázatban levő számok összege emiatt negatív lesz. Ha oszloponként adjuk össze a számokat, akkor pozitív részösszegeket adunk össze, így most a táblázat elemeinek összege pozitív lett. Az ellentmondás oka, hogy feltettük: megadható 11-tagú sorozat a kívánt feltételek szerint. 1751. A hat szám szorzata = −(a11 a12 . . . a33 )2 nempozitív, ezért van közöttük nempozitív szám. 1752. A sokszög csúcsai között 6 különböző távolság mérhető, míg az 5 pont 10 távolságot határoz meg.   10 1753. A 10 számból = 45 kéttagú összeg képezhető. A megadott számokból 2 készíthető legnagyobb kéttagú összeg 39. 1754. Nem lehet. A 100 főből válasszunk ki egyet. Ha a kívánt módon megszervezhető lenne az ügyelet, akkor a megmaradt 99 fő diszjunkt párokba lenne osztható.

29. Különféle kombinatorikai feladatok

337

1755. 1 + 3 + 3 · 2 = 10. Tehát a maximum 10. Ez — ahogyan az ábra mutatja — el is érhető.

vagy

1756. Válasszuk ki a sokszög egyik csúcsát, s innen indulva járjuk végig a sokszög kerületét. Ha vízszintes oldalon indulunk el, akkor egymás után vízszintes–függőleges, vízszintes–függőleges, . . . , vízszintes–függőleges oldalpárokon haladunk. (A kiinduló csúcsba nyilván függőleges állású oldalon érkezünk meg.) 1757. Figyeljük a kocka belsejében levő 1 × 1 × 1-es kockát. Ezt csak úgy tudjuk kifaragni a 3 cm élű kockából, ha az 1 cm élű kocka mindegyik lapja mentén vágunk. Emiatt szükség van a 6 vágásra. 1758. Mivel a kocka konvex, egy vágással két részre tudjuk szétvágni, vagyis minden egyes vágással legfeljebb megkétszerezhetjük az addigi darabok számát. Ez azt jelenti, hogy 6 vágásra mindenképpen szükségünk van, mert 5 vágás után legfeljebb 25 = 32 darabot kaphatunk. A 6 vágás elegendő, ilyen megoldást nem nehéz találni. 1759. A szalagot legfeljebb 21 részre vághatjuk szét. 1760. Az első, a második, . . . , a tizedik lépcsőfokra rendre 1, 2, 3, 5, 8, 13, 21, 34, 55, 89-féleképpen lehet feljutni. Hiszen mondjuk az ötödik lépcsőfokra vagy a negyedikről vagy a harmadikról érkezhetünk, azaz az ötödikre vivő utak száma megegyezik a negyedikre és a harmadikra vivő utak számának összegével. Az 1, 1, 2, 3, 5, . . . sorozatot Fibonacci-sorozatnak nevezik. A sorozat minden eleme — a harmadikkal kezdődően — az előző két elem összege. 1761. Nézzük az egyszerűbb eseteket! A 2 × 1-es téglalapot 2 × 1-es dominókkal 1féleképp, a 2×2-es téglalapot 2-féleképp és a 2×3-as téglalapot 3-féleképp lehet kirakni. A téglalap kirakásainak száma következik a kisebb téglalapok kirakásaiból. Például egy 2×6-os téglalap kirakása származtatható úgy, hogy egy 2×5-ös téglalap mögé állítva tesszük a 2 × 1-es dominót, ill. egy 2 × 4-es téglalap mögé fektetve teszünk 2 db 2 × 1-es dominót. (Ha állítva tesszük, azt már megszámoltuk akkor, amikor a 2 × 5-ös mögé állítva tettünk egy dominót.) Így minden kirakást megszámolunk, és minden kirakást egyszer számolunk. Arra a megállapításra jutunk, hogy a 2 × n-es téglalap kirakásainak száma megegyezik a 2×(n−1)-es és a 2×(n−2)-es téglalap kirakásai számának összegével. Így a 2 × n-es téglalap kirakásainak száma n = 1, 2, 3, 4, 5, 6, 7, 8, 9, 10 esetén rendre 1, 2, 3, 5, 8, 13, 21, 34, 55, 89. (Ez a Fibonacci-sorozat.)

338

Megoldások, útmutatások

Tehát a 2 × 10-es téglalapot 2 × 1-es dominókkal 89-féle módon lehet kirakni. 1762. A kívánt tulajdonságú 0–1 sorozatok számát állapítsuk meg, ha a sorozat 2, 3, . . . számból áll. A 2 számból álló sorozatok: 00, 01, 10; 3 számból álló sorozatok: 000, 100, 010, 001, 101. A 4-jegyű sorozatok mindegyike megkapható úgy, hogy a 2-jegyű sorozatokat 01-gyel egészítjük ki, ill. a 3-jegyű sorozatok végére 0-t írunk. Emiatt a 4-jegyű sorozatok száma: 3 + 5 = 8. Hasonlóan kapjuk meg pl. az 5-jegyű sorozatokat a 3- és 4-jegyű sorozatokból. Így a 2, 3, 4, 5, 6, 7, 8-jegyű sorozatok száma rendre: 3, 5, 8, 13, 21, 34, 55. Tehát 55 kívánt tulajdonságú sorozat van. 1763. Vizsgáljunk egyszerűbb eseteket! 9 kavicsot 3 halomba, 16-ot 4, 25-öt 5 kupacba. Az {1, 3, 5}, {1, 3, 5, 7}, {1, 3, 5, 7, 9} felosztásokhoz jutunk. Hasonló módon kapjuk a keresett felosztást is. Az 1, 3, 5, . . . , 199 szétosztás megfelel a feltételeknek. 1 + 3 + · · · + 199 = (1 + 199) · 100 = 10 000. Ha valamely kupacot két részre osztjuk, akkor az = 2 egyikben páros, a másikban páratlan számú kavics lesz. Mivel az első száz páratlan szám adja a kupacokban levő kavicsok számát, emiatt a két részre osztással keletkezett, páratlan számú kavicsot tartalmazó halom valamely másik halommal egyező számú kavicsot tartalmaz. Így a szétosztás teljesíti a kiírt feltételeket. 1764. Akár a megnyert, akár az elvesztett játszmák számát adjuk össze, eredményül a lejátszott mérkőzések számát kapjuk. Ezért: 0 = (x1 +x2 +· · ·+x10 )−(y1 +y2 +· · · +y10 ) = (x1 −y1 )+(x2 −y2 )+· · ·+(x10 −y10 ). Mindenki egyformán 9 mérkőzést játszott, tehát x1 + y1 = x2 + y2 = · · · = x10 + y10 = 9. Ezekből: 0 = 9(x1 − y1 ) + 9(x2 − y2 ) + · · · + 9(x10 − y10 ) = (x1 + y1 )(x1 − y1 ) + 2 2 −y10 ). +(x2 + y2 )(x2 − y2 )+· · ·+(x10 +y10 )(x10 −y10 ) = (x12 −y12 )+(x22 −y22 )+· · ·+(x10 Ezt kellett igazolnunk. 1765. Tegyük fel az ellenkezőjét. Ez azt jelenti, hogy minden percben minden családban pontosan egy lovagnak volt aranyserlege, hiszen az aranyserlegek és a családok száma megegyezik. 13 perc alatt mindenki kezében minden aranyserleg pontosan egyszer volt, azaz mindenki kezében pontosan k percen át volt aranyserleg. Valamelyik családnak legyen n lovagja. Mivel minden percben pontosan egy lovag kezében volt közülük aranyserleg és minden lovag k percig tartott aranyserleget, így nk = 13. De a feltétel szerint 1 < k < 13, s ez 13 prímszám voltának mond ellent. Ha a lovagok r száma osztható k-val, azaz r = kn, és minden családból pontosan n lovag ül az asztalnál; akkor könnyű olyan elrendezést mutatni, hogy minden percben minden család valamely tagjánál legyen aranyserleg. Ehhez elég pl. az egy családba tartozókat egymás mellé ültetni, az aranyserlegeket pedig egy szabályos k-szög csúcsaiban elhelyezni a lovagok között.

29. Különféle kombinatorikai feladatok

339

1766. Nevezzük V -lovagnak azokat, akiknél éjfél előtt vörös bor volt, és tekintsük a V lovagok jobb oldali harmadszomszédjait. Nem lehet, hogy minden V -lovag jobb oldali harmadszomszédja V -lovag legyen, hiszen ekkor V -lovagtól V -lovagig lépkedve, az asztalt háromszor körüljárva végül is a társaság minden tagjához eljutnánk, és a társaságban csak V -lovag volna. Van tehát legalább egy olyan V -lovag, akinek a jobb oldali harmadszomszédjánál éjfél előtt fehér bor volt. Éjfélkor ez a harmadszomszéd is, a V -lovag is a V -lovag jobb szomszédjának nyújtja serlegét, ez a lovag tehát két serleget kap. Így van olyan tagja is a társaságnak, akinek nem jut serleg. 1767. A sorok és oszlopok közül válasszunk ki egy olyat, amelyikben a számok összege a legkisebb. Legyen ez valamelyik sor, és ebben a számok összege k. Ebben a sorban legalább n − k db 0 áll. Tekintsük azokat az oszlopokat, amelyek ezeket a nullákat tartalmazzák. Egy-egy ilyen oszlopban a számok összege a feladat feltétele miatt legalább n − k. A táblázatban levő számokat oszloponként összegezve n 2 n2 n2 n2 n2 + =2 k− + ≥ . S ≥ (n − k)2 + k 2 = 2 k 2 − kn + 4 2 2 2 2 1768. A 12. sorozatban a k-adik helyen álló számjegyet jelölje xk , melynek értéke vagy → →=x =− →=x = 0, vagy 1. Legyen − xk = 1 − xk . x1996 = x1024+972 = − x972 x204 460 76 − → = x12 = 0. 1769. Megadunk egy eljárást, amely a kártyákat 50 dobozba teszi. Minden kártyáról töröljünk le egy jegyet úgy, hogy a megmaradt 2 számjegy összege páros legyen, majd tegyük a kártyát abba a dobozba, amelynek száma az így kapott kétjegyű szám. Ezt a törlést minden kártya esetén megtehetjük (miért?). Mivel a 00, 01, . . . , 99 számoknak pontosan a felében páros a számjegyek összege (ugyanis egyértelmű megfeleltetés létesíthető a páros és a páratlan számjegyösszegű számok között: (a, b) ↔ (9 − a, b), ahol (a, b) olyan szám, melynek első jegye a, második b), ezért valóban legfeljebb 50 dobozba tettük a kártyákat. Bebizonyítjuk, hogy kevesebb doboz nem elég. Tegyük fel, hogy a kártyákat a feltételeknek megfelelően betettük a dobozokba. Osszuk a 100 db dobozt 10 db 10-es csoportba úgy, hogy minden csoportban az első számjegyek azonosak legyenek (mint pl. 00, 01, . . . , 09). Tekintsük azt az a0, a1, . . . , a9 csoportot, amelyben a legtöbb, k db üres doboz van. Legyenek ezek sorszámai aa1 , aa2 , . . . , aak . Figyeljük meg, hogy ekkor az összes aai aj (i, j = 1, 2, . . . , k és i, j lehet egyenlő is) számú kártya csak az ai aj dobozba tehető, azaz az összes ai aj doboz nem üres. Ilyen ai aj feliratú dobozból nyilván k 2 db van. Tekintsük most azokat a csoportokat, amelyekben a felirat első jegye nem ai (i, j = 1, 2, . . . , k), ilyen csoport (10 − k) db van. Ezekben a csoportokban — mint minden csoportban — legalább (10 − k) db nem üres doboz van, hiszen k megválasztása miatt k-nál több üres doboz nem lehet egy csoportban sem. Tehát ezekben a csoportokban összesen legalább (10 − k)2 db nem üres doboz van.

340

Megoldások, útmutatások

Az előbb talált k 2 és a most talált (10 − k)2 nem üres doboz közt nincs átfedés, hiszen első jegyeik biztosan különböznek. Ezzel beláttuk, hogy összesen legalább k 2 + (10 − k)2 nem üres doboz van. Ez nem lehet kisebb 50-nél, hiszen k 2 + (10 − k)2 = 2(k − 5)2 + 50. 1770. 7 lépésben leszedhetők a kavicsok. Első lépésben a 63-nál több kavicsot tartalmazó halmokból elveszünk 64-et. A kupacokban legfeljebb 63 kavics marad. Másodjára, ahonnan lehet, elveszünk 32 kavicsot. Ekkor egy-egy halomban legfeljebb 31 kavics lehet. Ezután 16 kavicsot vegyünk el a megfelelő halmokból. Ekkor mindenhol legfeljebb 15 maradhat. A következő lépésekben 8-at, 4-et, 2-t, végül 1-et veszünk el. 6 lépésben a leszedés nem valósítható meg. Tegyük fel, hogy mégis. Rendeljünk minden kupachoz egy 6 elemű 0–1 sorozatot: aszerint áll 1 vagy 0 az i-edik helyen, hogy az i-edik elvételkor vettünk-e a kupacból vagy sem. Két különböző kupachoz különböző 0–1 sorozat tartozik. Azonban a 6 elemű 0–1 sorozatok száma 64, miközben 100 kupac van. 1771. (1, 0, 1, 0, 1, 0, 1, 0, 1) + (0, 2, 2, 0, 0, 2, 2, 0, 0) + (0, 0, 0, 4, 4, 4, 4, 0, 0) + +(0, 0, 0, 0, 0, 0, 0, 8, 8) = (1, 2, 3, 4, 5, 6, 7, 8, 9). Tehát 4 sorozattal elvégezhető a kívánt előállítás. Megmutatjuk, hogy 3 sorozattal nem oldható meg a feladat. Ugyanis ekkor az „összeg” elemei csak 8-félék lehetnek, míg a megadott sorozatnak 9 különböző eleme van: 1, 2, . . . , 9. Azért csak 8-féle, mert egy-egy elemet úgy kapunk az összegben, hogy az első sorozat egyik vagy másik elemét vesszük, ehhez hozzáadjuk a második sorozat egyik vagy másik elemét, majd ehhez adjuk a harmadik sorozat egyik vagy másik elemét: ez összesen 2·2·2 = 8-féle lehetőség. 1772. Igen, mindig lehetséges. Állítsuk a 100 számot 50 párba úgy, hogy minden párban 101 legyen a két szám összege. A kiválasztott 25 számmal ezekből a számpárokból k db pár mindkét tagját kivettük; a további 25 − 2k számhoz válasszuk ki a számokból a párokban hozzárendelt társait, és a még háborítatlan párokból válasszunk ki k darabot. Így a 25 szám egy része párokba állt, a többit kiegészítettük a párjával, s újabb számpárokat is választottunk. Ily módon 25 számpárt vettünk ki, s ennek az 50 számnak ugyanannyi az összege, mint a megmaradó 50 számnak. 1773. A 100 mérősúlyt foglaljuk 50 párba úgy, hogy egy-egy párba két szomszédos súly kerüljön. 25 párból a súlyokat rakjuk szét két kupacba: a könnyebbet balra, a nehezebbet jobbra. A megmaradó 25 párban levő mérősúlyokat pedig úgy rakjuk szét, hogy a könnyebbet jobbra, a nehezebbet balra tesszük. Ezzel a mérősúlyok megfelelő szétosztásához jutottunk. 1774. Az alábbi eljárással k mérés után el tudjuk dönteni, hogy vannak-e különböző súlyú érmék; és ez utóbbi esetben ki is tudunk jelölni majd egy könnyebbet és egy nehezebbet. Osszuk el az érméket két egyenlő számú csoportra, és tegyük a két részt a két serpenyőbe. Ha a mérleg egyensúlyban van, akkor az egyik részt tegyük félre, a

29. Különféle kombinatorikai feladatok

341

másikkal pedig ismételjük meg az eljárást, azaz ismét felezzük meg és hasonlítsuk össze a két részt. Folytassuk ezt mindaddig, amíg a mérleg ki nem billen. Ha erre a k-adik mérésben sem kerül sor, akkor az utolsó, k-adik mérésre 2 érme bizonyult egyenlő súlyúnak. Ebben az esetben minden érme egyenlő súlyú. (Miért?) Ha az l-edik mérésben a mérleg kibillen, és l = k; akkor a serpenyőkben 1–1 érme maradt, így a kezünkben van egy könnyebb és egy nehezebb érme. Ha l < k, akkor a következő méréstől kezdve módosítjuk eljárásunkat. Tegyük fel, hogy a bal oldali serpenyőben levő érmék a nehezebbek. Mindegyik serpenyő tartalmát két egyenlő számú érméből álló csoportra osztjuk, s méréssel összehasonlítjuk a bal és a jobb oldali serpenyő tartalmának egyik-egyik felét (az érmék megmaradó részét tegyük félre). Ha most is a bal oldali rész a nehezebb, akkor a most összehasonlított két részen ismételjük meg a leírt eljárást. Ha egyensúly van, vagy a bal oldali a könnyebb, akkor az eljárást az utóbbi mérésnél félretett két részen folytatjuk. Ezen a módon nem veszítjük szem elől a különböző súlyú érméket, az eljárás végén fel tudunk mutatni két különböző súlyú érmét. 1777. 12 golyó közül is kiválasztható a 2 radioaktív golyó hét méréssel. Készítsünk három, 4–4 golyóból álló csoportot, s mindegyiket mérjük meg (3 mérés). Ha csak az egyik csoport radioaktív, abból további 4 méréssel (sőt 3 mérés is elég) kiválasztjuk a keresett 2 golyót. Ha két csoportot is radioaktívnak találunk, akkor az azokban levő 1–1 radioaktív golyót 2–2 méréssel könnyű megtalálni. 1778. Legyen az elsőnek fölrakott, legnehezebb súly tömege M gramm. Az eljárás során a serpenyők eltérése soha nem haladhatja meg az M grammot, hisz ha éppen nincs egyensúly; akkor az újabb, legfeljebb M grammos súlyt a könnyebbik serpenyőbe kell tennünk. Ez igaz akkor is, amikor már valamennyi, 1 grammnál nehezebb súly a mérlegre került. Azt állítjuk, hogy 1 grammos súlyaink száma legalább M, vagyis ezekkel a mérleg kiegyensúlyozható. Valóban, ha az 1 grammos súlyok számát k-val jelöljük, akkor az egy darab M és a k darab 1 grammos súlyon kívül minden további legalább 2 grammos, így 1 · M + k · 1 + (101 − k − 1) · 2 ≤ 200, ahonnan M ≤ k. Miután így alkalmas számú 1 grammos súlyt a könnyebbik serpenyőbe téve egyensúlyba kerül a mérleg, a rajta levő súlyok összege páros, így az esetleg fennmaradó — lehetséges, hogy valamennyi — 1 grammos súlyok száma is páros, a súlyok tömegének összege ugyanis 200 gramm. A megmaradt 1 grammos súlyokat viszont ezután a feladat előírása szerint felváltva tesszük a bal, illetve a jobb oldali serpenyőbe, tehát az eljárás végén a mérleg valóban egyensúlyban lesz. 1779. Ha minden egyes kupacban 2 kavics van, akkor a feladat állítása nyilvánvaló, hisz 25–25 kupacot egy-egy csoportba gyűjtve mindkét csoportban 50 kavics lesz.

342

Megoldások, útmutatások

Ha a kupacok között vannak különböző elemszámúak, akkor jelölje a1 , a2 , . . . , a50 az egyes kupacokban levő kavicsok számát, és tegyük fel, hogy például a1 = a2 . Tekintsük ezután az a1 , a2 , a1 + a2 , a1 + a2 + a3 , . . . , a1 + a2 + · · · + a50 számokat. Ezen 51 szám között feltétlenül van kettő, b1 és b2 , amelyek ugyanazt a maradékot adják 50-nel osztva. Mivel a felsorolt számok különbözők (a1 = a2 és ai > 0), továbbá mindegyikük 1 és 100 között van, ez csak úgy lehetséges, ha |b1 − b2 | = 50. Ez a két szám így nem az a1 és az a2 , hisz az ai -k között nincs 50-nél nagyobb. A felsorolt 51 szám közül viszont bármely további kettőnek a különbsége különböző ai -k összege. Ha pedig ilyen számok összege 50, akkor a megfelelő kupacokat egybegyűjtve, a kapott csoportra — és így a kimaradókból összegyűjtött másikra — teljesül a feladat állítása. 1781. Az általánosság csorbítása nélkül föltehetjük, hogy a kezdetben utolsó helyen álló érme aranyból van. Legyen az ezüst érmék kezdeti sorszámainak összege S. Az átrendezés folyamán minden ezüst érme 1-gyel nagyobb sorszámú helyre kerül (hiszen az utolsó helyen nem ezüst áll), így az ezüst érmék sorszámainak összege (S + k)-ra változik. Ha az átrendezés során ezüst érmét mindig csak ezüsttel cserélnénk fel, akkor a fenti összeg nem változhatna. Az átrendezés folyamán ezért olyan lépésnek is kell lennie, amikor ezüst érmét arannyal cseréltünk fel, és éppen ezt akartuk bizonyítani. 1782. A zárójelezéssel elérhető legkisebb eredmény 1 − (2 + 3 + 4 + · · · + 1999), a legnagyobb pedig 1−(2−3−4−· · ·−1999). Ezek között a határok között lehetnek a végeredmények, továbbá tudhatjuk, hogy az eredmények páros számok (hiszen 999 páros és 1000 páratlan számot adunk össze). Számoljuk ki, hogy hányféle olyan zárójelezés van, ahol a zárójeleken belül pontosan két tag van: minden (2k, 2k + 1) párnál vagy elhelyezünk zárójelet, vagy nem. Mivel 999 ilyen pár van, ez 2999 zárójel elhelyezést jelent, és ezek mind különbözők. A számolások mutatják, hogy 2999 nagyobb, mint a zárójelezésekkel kapható eredmények száma, ezért van két olyan zárójelezés, melyek ugyanazt az eredményt adják. 1783. A sokszög valamely oldalával legfeljebb n − 2 átló párhuzamos. Ezért a valamelyik oldallal párhuzamos átlók száma legfeljebb 2n(n−2), míg a sokszög átlóinak száma n(2n − 3), s n(2n − 3) > 2n(n − 2) = n(2n − 4), tehát valóban van kívánt tulajdonságú átló. 1784. Egy adott színezésből átszínezésekkel nyerhető sakktáblák számára adunk egy felső becslést. Az átszínezési eljárás során az oszlopok, sorok átfestésének sorrendje nem befolyásolja a végeredményt. Másrészt ugyanannak a sornak, ill. oszlopnak kétszeri átfestése is elhagyható; ezért feltehetjük, hogy minden sort és oszlopot legfeljebb egyszer festünk át. Az oszlopok és sorok együttes száma

29. Különféle kombinatorikai feladatok

343

24, így összesen 224 módon jelölhetünk ki átszínezési eljárást (a végeredményül nyert táblaszínezések között persze lehetnek egyformák). Ugyanakkor a sakktábla 144 mezője 2144 -féleképp színezhető két színnel. 2144 > 224 miatt vannak olyan színezések, melyekből a kívánt sakktábla színezést nem lehet elérni. 1785. A táblázatba írt számok mindegyike helyett írjuk be, hogy az adott szám 10-zel osztva milyen maradékot ad. (Feladatunk szempontjából elegendő csak a számok tízes maradékát figyelni.) Így egy mezőbe 10-féle szám kerülhet, a 8 × 8-as táblázat 1064 -féle különböző módon tölthető ki. A megadott műveletekkel hányféle különböző műveletsorrend készíthető? A táblának 36 db 3 × 3-as része van, bármelyik ilyen részben az 1-gyel növelésre 10 (lényegesen) különböző lehetőségünk van. Összesen 1036 -féleképpen változtathatunk meg egy táblázatot. A 4 × 4-es részekből 25 van a táblán, ezekkel 1025 féleképpen változtathatunk. A kétfajta művelettel egy adott táblázatból kiindulva összesen (legfeljebb) 1036 · 1025 = 1061 különböző táblázathoz juthatunk. 1061 < 1064 , azaz az elérhető lehetőségek száma kisebb, mint a kitöltések száma; emiatt a feladat kérdésére tagadó a válasz. 1786. A diákok száma n. Őket a két terembe 2n -féleképp lehet szétosztani. A tanulók két teremben való szétosztásai között vannak „rossz” elhelyezések, amelyekben valamelyik (vagy több) csapatnak minden tagja ugyanabba a terembe kerül. Ha a rossz elhelyezések száma kisebb, mint 2n ; abból következik, hogy van jó elhelyezés is. Tekintsünk egy tetszőleges csapatot, és számítsuk ki, hogy hány olyan rossz elhelyezés van, amikor ennek a csapatnak minden tagja ugyanabba a terembe kerül. Először el kell döntenünk, hogy melyik ez a terem; ez 2 lehetőség. Ezután a többi n − 10 embert kell szétosztanunk, amit 2n−10 -féleképp tehetünk meg. Összesen tehát 2 · 2n−10 = 2n−9 olyan rossz elhelyezés van, amikor egy kiszemelt csapat minden tagját ugyanabban a teremben helyeztük el. 500 csapat lévén az összes rossz elhelyezés száma legfeljebb 500·2n−9 . (Lehetnek esetek, melyeket többször is megszámoltunk.) 500 n Mivel 500 · 2n−9 = · 2 < 2n , ezért a feladat állítása igaz. 512 1787. A bástyák olyan elhelyezéseinek száma, amikor nem ütik egymást (a színezéssel nem törődünk): n!. A bástyák olyan elhelyezéseinek száma, amikor nem ütik egymást, s van legalább n2 két azonos színű mezőn álló bástya: (n − 2)! · (hiszen valamelyik színt kivá2 lasztva, arra a két mezőre két bástyát helyezünk, a többit pedig (n − 2)!-féleképp helyezhetjük el). n2 Az állítást igazoltuk, mivel n! > (n − 2)! · . 2

344

Megoldások, útmutatások

  3n + 1 1788. A társaság 3 fős csoportjainak száma . Az olyan hármasokat számoljuk 3 össze, melyeknek tagjai legfeljebb kétféle játékot játszanak egymás között. A   n társaság egy A tagja n emberrel pingpongozik, így olyan hármasban van 2 benne, amelyikben mind a két másik tagjával   pingpongozik. Ugyanezt kapjuk a n sakk és a tenisz esetén is. Ez összesen 3 · . Ugyanez igaz a társaság bármely 2 tagjára, tehát a „rossz” hármasok   száma legfeljebb (hiszen egyes hármasokat n többször is számolhattunk) 3 · (3n + 1). 2 Számolással ellenőrizhető, hogy az összes hármas száma nagyobb a „rossz” hármasok számánál, tehát van „jó” hármas. Megjegyzés. Ehhez a feladathoz kapcsolódik az 1805. feladat. 1789. Az 1, 2, . . . , 1986 számok 21986 -féleképpen színezhetők két színnel. A „rossz” színezések (amelyekben van egyszínű, 18 elemű számtani sorozat) számára felső becslést adunk. Jelölje S a különböző, 18 elemű számtani sorozatok számát. S kiszámítása: 1 különbségű, 18 elemű számtani sorozat 1986 − 17 van; 2 különbségű ilyen sorozat 1986 − 2 · 17 van; 3 különbségű ilyen sorozat 1986 − − 3 · 17; 4 különbségű ilyen sorozat 1986 − 4 · 17; . . . ; 116 különbségű ilyen sorozat 1986 − 116 · 17 van; 117 különbségű ilyen sorozat nincs, amelynek elemei az 1, 2, . . . , 1986 számok közül valók. Ebből következik, hogy az összes különböző, 18 elemű számtani sorozat száma: (1986 − 17) + (1986 − −2 · 17) + (1986 − 3 · 17) + · · ·+ (1986 − 116 · 17) = 115 014. Egy „rossz” színezést kapunk, ha veszünk egy 18 elemű számtani sorozatot; azt kékre (vagy pirosra) színezzük, a többi 1968 számot pedig tetszőlegesen. Ezért a „rossz” színezések száma legfeljebb 2 · S · 21968 = 21969 · 115 014. Mivel 21969 · 115 014 < 21986 , azaz 115 014 < 217 = 131 072, ezért az összes „rossz” színezés száma kisebb az összes színezés számánál, így van „jó” színezés is. 1790. Itt is határozzuk meg az összes színezés számát, s adjunk felső becslést a „rossz” színezések számára. 1791. Megmutatjuk, hogy már 9 nyelv is kiválasztható a kívánt módon. Bármekkora is a feladatban szereplő n, és bárhogyan veszünk is ki (n + 1)-et a szóban forgó nyelvek közül, biztosan mindenki beszéli ezek valamelyikét, hiszen a választottakon kívül n-nél kevesebb nyelv van, és mindenki legalább n nyelvet beszél. Az n < 9 esetben ebből már következik állításunk, hiszen veszünk (n + 1)-et a szóban forgó nyelvek közül, és ha ez kisebb 9-nél, hozzájuk veszünk tetszés szerint (8 − n) nyelvet. Ha n ≥ 9, akkor tekintsük a 2n nyelvbőlkiválasztható 9 elemű részhalmazokat.  2n Nevezzük ezeket blokkoknak, a számuk . Mondjuk azt, hogy egy dolgozó 9

30. Konstrukciók

345

elront egy blokkot, ha a benne szereplő nyelvek egyikét sem beszéli. Készen vagyunk, ha megmutatjuk, hogy az összes dolgozó együttesen sem tudja elrontani az összes blokkot. egy dolgozó legfeljebb n nyelvet nem beszél, egy dol Mivel  n gozó legfeljebb blokkot ronthat el. Ha mind különböző blokkot rontanak is 9   n el, ez összesen legfeljebb 500 · blokk. 9     n 2n Mivel 500 · < , így igaz az állítás. 9 9 1792. Legyen |X| = t. Ekkor az összes kétszínezések száma 2t . Azoknak a kétszínezéseknek a száma, amelyekre az n elemű Ai ∈ F egyszínűvé válik, nyilván 2t−n+1 . Minden egyes Ai ∈ F esetén kidobva ezeket a rossz kétszínezéseket, összesen legfeljebb |F | · 2t−n+1 kétszínezést dobtunk el. A maradék viszont pontosan a jó kétszínezések halmaza, s ennek van eleme, hiszen 2t > |F | · 2t−n+1 .

30. Konstrukciók 1793. 10 korong nyilván nem tehető fel. 9 korongot fel tudunk helyezni a következő módon: az első korongot tetszőlegesen tesszük fel; ezután a következőt úgy rakjuk fel, hogy oda érkezzen, ahonnan az előző korongot indítottuk; ily módon tesszük fel a többit is. 1794. 1, 3, 9, 27. 1795. „Kieséses versenyt” szervezünk. A mérleg serpenyőibe egy-egy golyót teszünk, s ezek súlyának összehasonlítása után a könnyebbet hagyjuk a mérlegen. Mindig a következő golyót hasonlítjuk össze az addigi legkönnyebbel és a könnyebbet hagyjuk a mérlegen. 99 mérés szükséges. 1796. A 100 golyót párokba állítjuk, s 50 méréssel két csoportba osztjuk: a párokból a nehezebb a „nehezek” csoportjába, a másik a „könnyűek” csoportjába kerül. Az 50 „nehéz” golyóból (az 1795. feladat szerint) 49 méréssel megtaláljuk a legnehezebbet, s 49 méréssel a másik csoportban megtalálható a legkönnyebb. Összesen 148 mérés. 1797. Párokba rendezzük a golyókat, és 50 méréssel a párokból kiválasztjuk a nehezebbiket. Most ezeket rendezzük párokba, és 25 mérés után ezek közül is kiválasztjuk a nehezebbeket. Így folytatva 12, 6, 3, 2, majd végül 1 mérés után megtaláljuk a legnehezebb golyót. (Összesen 99 mérés.) A második legnehezebb golyó csak akkor eshetett ki, amikor a legnehezebb golyóval együtt került mérlegre. A legnehezebb golyót 7 alkalommal tettük fel a mérlegre, az ekkor mért 7 golyó között van a keresett. E hét golyóból a legnehezebbet 6 mérlegeléssel

Megoldások, útmutatások

346

megtalálhatjuk. Összesen 99 + 6 = 105 mérés szükséges a két legnehezebb golyó megtalálásához. 1798. Tegyünk a serpenyőkbe 3–3 érmét! A mérlegelés után tudjuk, hogy mely 3 érme között van a hamis. Ebből a 3 érméből egyet-egyet tegyünk a serpenyőkbe, s most már kiderül, hogy melyik a hamis. 1799.

I. m r s

1 2 3 4

5 6 7 8

9 10 11 12

II. m r s 1 2 5

3 4 6

9 10 11

1 2 5

1 2 3

3 4 6

III. m r s 1

2

7

8

3

4

9

10

12

1

9

10

3

4

7

8

2

1

1 6 2 8 7 3 5 4 9 11 10 12 12 10 11 9 4 5 3 7 8 2 6 1 N K N K K N K N N N N N K K K K K N K N N K N K 1800. Előbb valamelyik 64 utasból álló csoportot vizsgáljuk meg. Ezt követően tudjuk, hogy melyik 64 személy között van a terrorista, ismét a csoport egyik felét vizsgáljuk meg és így tovább. Összesen 7 vizsgálat elegendő. 1801. Tegyünk a mérlegre az első ládikóból 1 db érmét, a másodikból 2 db-ot, a harmadikból 3-at, . . . 1802. Hármuk közül az egyik három részre osztja a zsákmányt, a másik kettő pedig választ. Ha két különböző részt választottak, akkor elteszik a kiválasztott részt, a megmaradó harmadik részt pedig a zsákmányt szétosztó rabló kapja. Ha mindketten ugyanazt a részt választják, akkor a másik két rész egyikét odaadják a zsákmányt szétosztó rablónak, és a megmaradó két részből álló zsákmányt a két rabló egyike újra két részre osztja, a másik rabló pedig választ ezekből. 1803. Elegendő egyszer át- és visszaevezni. A 49 vezetékből sorbakapcsolva egyetlen kábelt lehet készíteni úgy, hogy az egyik drótvéget rákötjük a fázisra, a többit pedig páronként összekötjük, majd átevezünk. Fázisceruza segítségével a túlsó oldalon össze tudjuk fűzni a vezetékeket egyetlen kábellé. Még a túloldalon,

30. Konstrukciók

347

az összefűzés sorrendjében sorszámozzuk a drótvégeket. Visszatérünk a folyó innenső oldalára, megszakítjuk az összeköttetéseket, megtaláljuk az egymáshoz tartozó drótvégeket. 1804. A csapatokat ábrázolják egy szabályos 17-szög csúcsai és középpontja! Egy oldal és a vele párhuzamos összes átló, valamint a középpont és a kiválasztott oldallal szemközti csúcs összekötése jelöl ki egy fordulót. Ezután ismételten forgassuk el a szakaszrendszert. 1805. A 3n + 1 főből válasszunk egyet: X-et, a többieket osszuk szét n–n fős csoportokra, a T , S, P halmazokba. Az X játékos a T halmazba tartozó játékosokkal teniszezik, az S halmazban levőkkel sakkozik, a P -beliekkel pedig pingpongozik. Ha két másik játékost veszünk, s azok ugyanabban a halmazban vannak, akkor egymás közt is ezt a játékot játszák; ha különböző halmazban vannak, akkor a harmadik játékot játszák. (Lássuk be, hogy ez jó konstrukció!) 1806. Van két sor, mely legalább 4 csillagot tartalmaz. A megmaradó két csillagot két oszloppal lefedhetjük. ∗ ∗ ∗ ∗ ∗ ∗ ∗ ∗ ∗ ∗ ∗ ∗ ∗ ∗ ∗ ∗ ∗ ∗ ∗ ∗ ∗ ∗ ∗ ∗ ∗ ∗ ∗ ∗ 1807. Átjuthatnak 12 perc alatt. Adél és Bori átmegy (2 perc), visszajön Adél (1 perc), átadja Csillának és Dórinak a lámpát, átmennek (5 perc), Bori visszajön a lámpával (2 perc), majd Adéllal együtt átmennek az alagúton (2 perc). 1808. A világ körüli útra készülő gépen kívül elég 2 másik gép. Ezek egymást követően úgy fordulnak vissza, hogy üres tartállyal érkezzenek vissza a szigetre, majd később tele tankkal az érkező gép elé mennek, hogy haza segítsék. 1809. A kutató átjuthat a sivatagon. 60 km megtétele után az egyik teherhordó feltölti a kutató és a másik teherhordó készletét 4 napi adagra, és visszafordul. (Ha visszaér a kiinduló állomásra, ott keveset pihen; majd készleteit felfrissíti és társa elé siet, hogy hazasegítse.) A kutató a vele maradó teherhordóval megtesz még 80 kmt; ekkor a teherhordó feltölti a kutató készletét, aki innen már eljut a céljához. A második teherhordó itt fordul vissza, készlete segítségével eljut az első (a 60 km-es) elválási pontra, ahonnan társa segítségével hazatér. 1810. Három segédautóval megoldható az A autó átjuttatása a szomszédos városba. 1811. Lehetséges, ha mindenki az ábra szerint a belül álló két személy valamelyikére lő. ∗



348

Megoldások, útmutatások

31. Teljes indukció 1812. A hiba oka az, hogy az indukciós lépésben felhasználtuk, hogy két egymás utáni értékre igaz az állítás (az (n − 1) és (n − 2) értékekre), de az indukciós lépés igazolásában csak egy n értékre (n = 1-re) ellenőriztük az állítást. 1813. Itt helyes az indukciós lépés igazolása. A hibát az indukciós feltevés ellenőrzésénél követtük el: a bal oldal nem értelmezhető n = 1 esetén, csak n > 1-re. 1814. Az az okoskodás, ami a 3-ról átsegített a 4-re, alkalmazható az n-ről (n + 1)-re való átmenetnél egy kivétellel: nem megy, mint ahogy nem is mehet az 1-ről 2-re való átmenetnél. 1821. Mutassuk meg, hogy an = 4an−1 − an−2 . 1842. Lássuk be teljes indukcióval, hogy 1 1 1 1 + + ···+ =1− . f1 f3 f2 f4 fn fn+2 fn+1 fn+2 1850. Egy kockát könnyű feldarabolni 8, 27, 64, . . . kisebb (egybevágó) kockára. Ha egy kockát feldaraboltunk n kisebb kockára, akkor a kis kockák valamelyikét 8 kockára darabolva, az adott kockát máris n + 7 kockára daraboltuk. Tehát elegendő 7 egymást követő egész szám esetén megvalósítani a darabolást, mert azokról 7-esével haladhatunk felfelé. 1852. Bizonyítsunk teljes indukcióval és használjuk fel, hogy k 2 − (k + 1)2 − (k + 2)2 + (k + 3)2 = 4. 1853. Indukció az egyenesek számára. 1857. Az indukciós feltevés: 2k | (k + 1)(k + 2) . . . (2k). Az indukciós lépés: 2k+1 | (k + 2)(k + 3) . . . (2k + 1)(2k + 2) = 2(k + 1)(k + 2)(k + +3) . . . (2k)(2k+1), s innen a feltevést felhasználva következik az állítás igazsága. 1858. Lásd a Skatulyaelv c. fejezet 1604–1606. feladatait! 1859. n = 1-re 2 jó szám. Az indukciós feltevés szerint az A n-jegyű szám 1 és 2 jegyekből áll, s többszöröse 2n -nek. Az indukciós lépésben megmutatjuk, hogy vagy az 1A, vagy a 2A n + 1-jegyű szám osztható 2n+1 -nel. A két szám másképp írva: 10n +A, ill. 2·10n +A. Ezekből 2n -t kiemelve, 5n +B, ill. 2·5n +B számokhoz jutunk, melyek közül az egyik páros. Más megoldás. Legyen k pozitív egész, és írjuk fel azokat az n-jegyű számokat, amelyeknek minden jegye 1 vagy 2. Azt állítjuk, hogy ezek 2n -nel osztva csupa különböző maradékot adnak (s ebből már következik az állításunk). Legyen x és y két szám az n-jegyű, 1-esekből és 2-esekből álló számok közül. Azt látjuk be, hogy 2n  |x − y. A különbség végén lehet néhány nulla (m db), s az utolsó nemnulla számjegy vagy 1-es, vagy 9-es. Tehát x − y = p · 10m , ahol p páratlan szám. Ezért x − y osztható 2m -nel, és 2m+1  |x − y. Mivel m < n, így a különbség nem osztható 2n -nel.

31. Teljes indukció

349

1860. Az indukciós feltevésben szereplő egyenletnek legyen megoldása u és v. Ekkor az  indukciós lépésbenfelírtegyenlet megoldása  1 1 1 1 (u − v); (7u + v) és (u + v); (7u − v) is. 2 2 2 2 A számpárok egyike páratlan számokból álló pár. 1861. Tegyük fel, hogy az 1, 2, . . . , 2n + 2 számokból ki tudunk választani n + 2 db számot úgy, hogy közülük egyik sem osztója valamely másiknak! Ekkor az indukciós feltevéssel összhangban az 1, 2, . . . , 2n számokból legfeljebb n db számot választottunk ki, tehát a kiválasztott számok között szerepelnie kell a 2n + 1 és a 2n + 2 számoknak; ám az indirekt feltevés miatt n + 1 nem lehet a kiválasztottak között, hiszen akkor osztaná a 2n + 2 számot. Cseréljük ki a 2n + 2 számot az n + 1 számra, s hagyjuk el a számok közül a 2n + 1 számot! Az így kapott n+1 db szám közül egyik sem osztója valamely másiknak, ami ellentmond az indukciós feltevésnek, tehát hamis a kezdeti indirekt feltevés. 1862. Tegyük fel, hogy igaz az állítás n = 2k-ra! Tekintsük az indukciós lépést 2k +1-re (a 2k + 2-re történő indukciós lépés triviális, hiszen ekkor az egyenlőtlenségben csak a jobb oldalon levő szám növekszik):      2k + 1 p= p· p < 4k+1 < 4k+1 · 4k = 42k+1 . k p≤2k+1 k+1 Bn−1 állítás helyett az indukcióval már bizonyítható erősebb An > 2Bn−1 egyenlőtlenséget lássuk be! Az indukciós lépés: An+1 = 3An > 32Bn−1 = 9Bn−1 = 2,25Bn−1 · 4Bn−1 > 2 · 4Bn−1 = 2Bn . 1878. Az egyenlőtlenségre Cauchy adott egy nagyon szép bizonyítást: Mutassuk meg, hogy a pozitív A, B, C, D, . . . számok mértani közepe mindig kisebb a számtani közepüknél, ha a számok között van két különböző. Legyen az A, B, C, D, . . . mennyiségek száma n, s igazoljuk, hogy   A+ B +C + D + ... n . ABCD · · · < n Először, n = 2 esetén nyilvánvalóan igaz, hogy       A+B 2 A−B 2 A+B 2 AB = − < , 2 2 2 amiből sorra feltéve, hogy n = 4, n = 8, . . . , végül n = 2m , ezt kapjuk:      A+B 2 C+D 2 A+B +C+D 4 ABCD < < 2 2 4  4  4 E+F +G+H A+B+C +D ABCDEF GH < < 4 4   A+B +C +D+E+F +G+H 8 < 8 .. .   A+ B +C + D +... n ABCD · · · < (1) n Másodszor, ha n nem tagja a 2, 4, 8, 16, . . . mértani sorozatnak, jelöljük a sorozat egy n-nél nagyobb tagját 2m -mel, és legyen A+ B +C + D + ... K= ; n

31. Teljes indukció

351

feltéve, hogy az (1) képletben az utolsó (2m − n) tag K-val egyenlő, a képlet ezt adja:  m A + B + C + D + · · · + (2m − n)K 2 m , ABCD . . . K 2 −n < 2m m

vagy más szóval ABCD . . . K 2

−n

m

2 < K .  A+B + C +D + ... n n Következésképpen ABCD · · · < K = , amit bizonyín tanunk kellett.

1879. Jelölje Sn a keresett összeget! Lássuk be, hogy Sn+1 = 2Sn + Sn ; s így Sn = 2 · 3n . 1880. Igen, lehet. A k + 1-edik pontot ne kössük össze egyikkel sem, ha azok között van olyan, mely a többivel össze van kötve; különben pedig összekötjük mindegyikkel. Megjegyzés. Lehet úgy is, hogy a pontokat sorszámozzuk az 1, 2, . . . , n szán mokkal, s az i-edik és j -edik pontot összekötjük, ha |i − j | ≤ . Ekkor azonos 2 számú szakasz csak a k-adik és (n − k)-adik pontból indul. 1881. A szorzatok összege nem függ az eljárás során végrehajtott felosztások sorozatától. Esetünkben mindig 500 500 lesz a végeredmény, általában pedig n kavics   n esetén . Indukciós lépésben az eljárás első lépése után, mikor az n elemű 2 kupacot felbontottuk egy k és egy n − k elemű kupacra, az indukciós feltevést felhasználva tudjuk számolni a szorzatok összegét. Ebben az összegben szerepel (az első felosztás miatt) k(n − k), valamint a  k és elemű csoportokhoz  azn − k  k n−k tartozó szorzatok összege, tehát: k(n − k) + + , s ezt kiszámolva 2 2   n kapjuk az eredményt. 2 Megjegyzés. Képzeljük el, hogy a kavicsok páronként zsineggel vannak összekötve! Minden egyes kettéosztásnál a különböző csoportokba tartozó kavicsok között elvágjuk a zsineget. A szétosztásnál számolandó szorzat pontosan az el  n vágandó zsinegek száma. Összesen zsineg van; ezeket mind elvágjuk, tehát 2 ez a keresett végeredmény. 1882. Teljes indukcióval megadunk egy algoritmust, mely a kívánt alsó korlátot biztosítja. k = 2, 2k − 1 = 3, k · 2k−1 = 4. Vegyük pl. az x1 , x2 és az x3 , x4 számpárt! Ezek között egy-egy összehasonlítás megadja a nagyobbat, melyek legyenek most x2 és x4 . Ha ez utóbbi két számot összehasonlítjuk, s x4 a nagyobb, akkor nyertünk egy újabb egyenlőtlenséget is: x1 < x4 . Tehát 4 számot kiválasztva, 3 alkalmas összehasonlítás után 4 egyenlőtlenségünk van; s ráadásul tudjuk, hogy melyik a legnagyobb szám ezek közül.

Megoldások, útmutatások

352

Ez lesz indukciós feltevésünk is: 2k db szám között 2k − 1 alkalmas összehasonlítást végezve, ismerünk legalább k ·2k−1 egyenlőtlenséget; s tudjuk, hogy melyik a legnagyobb szám. Az indukciós lépésben a 2k+1 db számot két, egyaránt 2k db számot tartalmazó részre osztjuk. Ezek között az indukciós feltevésnek megfelelően külön-külön 2k−1 kérdés után tudjuk, hogy egyikben, másikban melyik a legnagyobb szám; s ismerünk mindegyik részben legalább k·2k−1 egyenlőtlenséget. Ez eddig összesen (2k − 1) + (2k − 1) = 2k+1 − 2 kérdés, s legalább 2 · k · 2k−1 egyenlőtlenség. Most hasonlítsuk össze a két halmaz legnagyobb számát! Ezzel a kérdések száma 2k+1 −1 lesz; s az egyenlőtlenségek száma az utolsó kérdés után 2k -nal növekszik, így összesen legalább 2 · k · 2k−1 + 2k = (k + 1) · 2k egyenlőtlenséget ismerünk; s most is tudjuk, hogy a megkérdezett számok közül melyik a legnagyobb. 1883. n = 4 esetén a négy személy: A, B, C, D között a beszélgetések sorrendje: (A, B), (C, D), (A, C), (B, D). Az indukciós lépésben szervezzük a beszélgetéseket úgy, hogy az n + 1-edik személy beszél valakivel, majd az n személy egymás között az indukciós feltevés szerint 2n − 4 hívással kicseréli az összes pletykát (n + 1 pletykát!), s ezután valaki felhívja az n + 1-edik személyt.

32. Kombinatorika a geometriában 1885. b) Vegyük fel a pontpárok közötti szakaszfelező merőlegeseket. Ez véges sok egyenes, ezért van olyan P pontja a síknak, melyen egyik szakaszfelező merőleges sem halad át. Vegyünk fel egy elegendően nagy sugarú kört P középponttal, mely tartalmazza mind a 2000 pontot, majd a kör sugarát folytonosan csökkentsük. Így egymás után, egyesével hagyunk el pontokat a körből. P választása miatt nem eshet meg, hogy egyszerre 2-vel (vagy többel) csökkenne a körben levő pontok száma. Ily módon elérhetjük, hogy a kör belsejében 1000 pont maradjon.

...

...

1895. A nyolcszög egyik oldalától indulva vezessünk egy vonalat az egymáshoz csatlakozó paralelogrammákon keresztül. Amelyik paralelogrammán a vonal áthalad, annak van párhuzamos oldala a kiválasztott nyolcszögoldallal. Ugyanezt tegyük ... meg a nyolcszög egy másik, erre merőleges oldalától indulva is. Ez a két vonal találkozik valamelyik paralelogramma belsejében, s ez a paralelogramma téglalap lesz. A nyolcszög eddig figyelembe nem vett oldalaira is elvégezve az előbbi eljárást, megtaláljuk a másik téglalapot is.

...

32. Kombinatorika a geometriában

353

1897. Tekintsük azokat az eseteket, amikor a ponthalmaz konvex burka hatszög, ötszög, négyszög, ill. háromszög. 1898. Ha az ABC háromszög belsejében van a D pont, akkor az ABC és az ABD háromszögek legkisebb szöge különböző. Ha a 6 pont konvex burka háromszög, négyszög vagy ötszög, akkor az előbbi megállapítás segít. Ha a konvex burok az ABCDEF hatszög, és az ACE háromszög legkisebb szöge A-nál van, akkor a DEA háromszögnek az A-nál levő szöge kisebb. 1901. Tekintsük az öt pont konvex burkát. Ha ez négyszög vagy ötszög, akkor készen vagyunk. Ha háromszög, akkor annak belsejében van a maradék két pont. Ezen a két ponton átmenő egyenes egyik partjára a háromszögnek két csúcsa esik; ez a két csúcs és a háromszögben levő két pont egy konvex négyszög négy csúcsa. Megjegyzés. Azt sejtik, hogy ha adott a síkon 2n−2 + 1 pont úgy, hogy közülük semelyik három sem esik egy egyenesre, akkor kiválasztható közülük n olyan pont, amelyek egy konvex sokszög csúcsait alkotják.   n 1903. n pont közül -féleképpen választhatunk ki öt pontot. Egy korábbi feladat 5 állításaalapján minden pontötösben legalább egy konvex négyszög van. Így leg n alább konvex négyszöget kapnánk. Egy négyszöget azonban esetleg több5 ször is figyelembe vettünk. Mégpedig maximálisan (n − 4)-szer számíthattuk be, hiszen az ötödik pontot (n − 4)-féleképpen választhatjuk.    Tehát a kiválasztható n 1 n 1 = . konvex négyszögek száma legalább n−4 5 5 4 1905. Tegyük fel, hogy k háromszöget sikerült kijelölni az előírt módon. Mivel háromszögeink közül semelyik kettőnek nincs közös oldala (éle), ezért együttvéve 3k élük van. Eza  szám nem nagyobb,   mint az n pontból képezhető pontpárok, élek n n száma, ami ; így 3k ≤ . 2 2 Másrészt nézzük azt az esetet, amikor kijelöltünk k darab háromszöget, s többet  már nem tudtunk. Ez azt jelenti, hogy végigmenve a még föl nem használt n − 3k élen, és mindegyik ilyenhez hozzápróbálva harmadik csúcsnak a vég2 pontjaitól különböző (n − 2) pont mindegyikét, e próbák mindegyike tiltott háromszöghöz vezet; olyanhoz, melynek legalább egy éle a már kijelölt 3k él közül való.   n A ki nem jelölt háromszögek száma − k. Fenti próbáink során mindig ilyen 3 háromszöget kapunk, és ha egy ki nem jelölt háromszögnek 2 olyan oldala van, amely még ki nem jelölt él; azt 2-szer is megkaptuk, mindkét ilyen oldala révén. (3-szor azonban nem kaphatjuk.)

354

Megoldások, útmutatások

      n n Ezek szerint: (n − 2) − 3k ≤ 2 − k . Innen megkapjuk a kívánt 2 3   1 n egyenlőtlenséget. k> 9 2 1906. n kör a síkot legfeljebb n2 − n + 2 részre osztja. Ez teljes indukcióval igazolható. 1907. Nem lehet. A lapok találkozási pontjainál levő szögek 108◦ , 144◦ és 180◦ lehetnek. Ezekből a 360◦ -ot 2 · 108◦ + 144◦ alakban állíthatjuk elő, tehát két ötszög- és egy tízszöglap találkozik. Ezért egy ötszöglap körül felváltva ötszög- és tízszöglapok vannak. Ez a váltakozás nem lehetséges, hiszen 5 páratlan. 1908. A test minden csúcsában q darab p oldalú szabályos sokszög találkozik. A sok 2

szög csúcsaiban levő szög nagysága 1 − π. A test egy-egy csúcsa köp 2

rül a lapokon levő szögek összege kisebb 2π-nél: q 1 − π < 2π, azaz p (p − 2)(q − 2) < 4. (p − 2)(q − 2) = 3: p = 5, q = 3 (dodekaéder); p = 3, q = 5 (ikozaéder). (p − 2)(q − 2) = 2: p = 4, q = 3 (kocka); p = 3, q = 4 (oktaéder). (p − 2)(q − 2) = 1: p = 3, q = 3 (tetraéder). Tehát (legfeljebb) öt szabályos test létezik. 1909. Ha a ponthalmaz konvex burka egy n-oldalú sokszög, akkor a sokszög belsejébe 1000 − n db pont került. Amikor az utolsó szakaszt is berajzoltuk, akkor az noldalú sokszöget olyan háromszögekre daraboltuk fel, amelyek csúcsai az adott 1000 pont közül kerülnek ki. Hány háromszög keletkezett? Legyen a háromszögek száma m. A háromszögek belső szögeinek összege m · 180◦ = (n − 2) · 180◦ + (1000 − n) · 360◦ , tehát m = 1998 − n. A berajzolt szakaszok a háromszögek oldalai, számuk: 3m − n + n = 2997 − n. 2 1910. A válasz: legfeljebb n. Ötlet a megoldáshoz: a vizsgált sokszög az általánosság megszorítása nélkül lehet szabályos is. Soroljuk csoportokba a szabályos n-szög átlóit; egy csoportba tartozzanak az egymással párhuzamos átlók. Lássuk be, hogy a csoportok száma legfeljebb n. 1911. Konvex   sokszög esetén legfeljebb 3, konkáv sokszög esetén pedig legfeljebb 2n + 1 hegyesszöge lehet az n oldalú sokszögnek. 3 Tegyük fel ugyanis, hogy az n-oldalú sokszögnek k számú hegyesszöge van, és ennélfogva n − k szög nem hegyes. Mivel a hegyesszögek kisebbek 90◦ -nál, a többi szög pedig 360◦ -nál, ezért k · 90◦ + (n − k) · 360◦ nagyobb a sokszög

32. Kombinatorika a geometriában

355

szögösszegénél: k · 90◦ + (n − k) · 360◦ > (n − 2) · 180◦ ; ebből 3k < 2n + 4, 2n 3k ≤ 2n + 3, és így k ≤ + 1. Ez az érték elérhető. 3 1912. Igen, lehetséges. Legyen például az i-edik rész mindazoknak az (x, y, z) pontoknak az összessége, melyekre [x] ≡ i (mod 2000). 1913. 1. bizonyítás. (Indirekt.) Legyenek a fellépő távolságok d1 , d2 , . . . , dn . Vegyünk két pontot, P -t és Q-t az adottak közül, s rajzoljunk mindegyik körül d1 , d2 , . . . , dn sugarú köröket. A ponthalmaz bármely pontja rajta van valamelyik P és valamelyik Q körüli körön. Csakhogy a P és Q körüli köröknek legfeljebb 2n2 közös pontja van. 2. bizonyítás. (Indirekt.) Véges sok távolság lép fel; ezek minimuma m, maximuma M. Vegyünk egy négyzetrácsot a síkon, melyben egy négyzet átlója m. Egy négyzetben legfeljebb egy pont lehet. Valamelyik pont körül vegyünk fel egy M sugarú kört. Mindegyik pontnak ebben a körben kell lennie, azonban a körben véges sok négyzet, és így véges sok pont van. 1914. A fellépő háromszögek közül válasszuk ki az egyik legnagyobb területűt. Húzzunk párhuzamosakat ennek csúcsain át a szemközti oldalakkal. Az így kapott háromszög tartalmazza az összes pontot, s a háromszög területe legfeljebb 4 egység. 1915. Tekintsük az 5 piros pontból álló ötszöget. Ezt forgassuk körbe a tízszög csúcsain (összesen 10 állapot van). Ha valamely elforgatásnál az ötszögben van 3 kék csúcs, akkor készen vagyunk. Indirekt feltevéssel igazoljuk, hogy ez mindig teljesül. Ha egyik ötszögben sincs 3 kék csúcs, akkor a 9 elforgatott ötszöggel legfeljebb 9 · 2 = 18 kék csúcsot számoltunk. Minden csúcsot ötször számoltunk (hiszen egy csúcson az elforgatott ötszög mindegyik csúcsa „végigmegy”), így a 18 kék csúcsok száma ≤ , pedig a kék csúcsok száma 5. 5 1916. Válasszunk ki egy sokszöget, melyet a 13 oldalú sokszög átlói határolnak. A sokszög egy oldalához a 13-szög két csúcsa rendelhető, s egy csúcshoz a sokszögnek legfeljebb két oldalegyenese fut be. Ez azt jelenti, hogy a sokszög oldalainak száma legfeljebb 13. 1917. Lássuk be, hogy a piros és fekete pontok konvex burkai diszjunktak. Ezek szétválaszthatók pl. a két burok közti legkisebb távolság szakaszfelező merőlegesével. (Miért?) 1 1918. Tegyük fel, hogy lehetséges az elrendezés. Írjunk mindegyik pont köré r = 2 sugarú köröket. Ezek nem metszik egymást, s rajta vannak a 10,5 sugarú „kibővített” asztallapon; ezért területük összege nem nagyobb ennek az asztallapnak a 1 2 területénél, tehát 450 · π · ≤ (10, 5)2 · π, azaz 450 ≤ 441 lenne. 2

356

Megoldások, útmutatások

1919. Az egységnégyzetek oldalától 1/2 távolságban húzzunk vonalat. Az így kapott π alakzat területe 3 + . A téglalapot is „keretezzük”, mindegyik oldalától 1/2 4 távolságban befele egyenest. Ennek a 19 × 24-es téglalapnak a területe húzunk π

456. Mivel 120 · 3+ < 456, ezért a négyzetek között „van hely” egy körnek. 4 1920. A pénzérmék együttes területe kisebb az asztal területénél: n · π · r 2 < π · R 2 ; √ R innen n < . Most duplázzuk meg mindegyik érme sugarát, így ezek lefedik r

√ 1 R az R −r sugarú asztalt; tehát n·π ·(2r)2 > π ·(R −r)2 , amiből n > −1 . 2 r 1921. A test mindegyik oldallapjához rendeljük hozzá a csúcsok számozásának megfelelő körüljárási irányt. Ekkor egy közös él mentén csatlakozó két háromszög ellentétes körüljárású lesz. Az óramutató járásával megegyező körüljárású lapokat színezzük az egyik, az ellentétes körüljárású lapokat a másik színnel. 1922. Azt kell csak megfigyelnünk, hogy az egyes tartományok hány körön belül vannak. A csupa homorú vonallal határolt tartomány (egy körívet a kör belsejéből szemlélve nevezzük domborúnak, kívülről pedig homorúnak) kívül esik minden körön, tehát 0-val jelölhetjük. A többi tartomány belül esik egy, két, három vagy tetszés szerinti számú körön. Lehet, hogy ez a kör nem is metszi a határát, hanem kívülről foglalja be. Ha a befoglaló körök számát beírjuk a tartományokba, megoldottuk a színezés gondját: a páros számúak kapják az egyik színt, a páratlanok a másikat. (Miért lesz jó a színezés?) 1923. Egy konfiguráció (a körök közül egy és a berajzolt húr) három részre osztja a síkot. Számozzuk e három részt a 0, 1 és 2 számokkal! Minden tartományba (országba) írjuk azt a számot, amelyikkel megjelölt részbe esik a három rész közül. Ezt a számozást minden konfigurációra hajtsuk végre! Minden lépésnél egy-egy újabb szám kerül minden országba. Az egy országon belüli számokat adjuk össze, osszuk el az összeget 3-mal, és írjuk az országba a maradékot (a korábbi számokat töröljük le). Ezután aszerint színezünk egy országot fehérre, feketére vagy pirosra, hogy a beleírt maradék 0, 1 vagy 2. Be kell látnunk, hogy ekkor bármely két szomszédos ország különböző színű (azaz különbözők a maradékaik). Legyen két szomszédos ország L1 és L2 , határaik közös éle pedig h. Vegyük el a térképről a h élt tartalmazó K konfigurációt. Ekkor L1 és L2 között nincs határ. Tehát L1 és L2 a K-tól különböző konfigurációk mindegyikének ugyanabba a részébe esik, a K által felosztott 3 rész közül pedig különbözőbe. Ezért L1 és L2 maradékai nem egyezhetnek meg. Így tehát térképünk 3 színnel jól színezett. 1924. A Diofantoszi egyenletek c. fejezetben már találkoztunk a végtelen leszállás módszerével. Ezt alkalmazzuk itt is. Használjuk fel a négyzetrács következő tulajdonságát: ha egy rácspontot tükrözünk két rácspontot összekötő szakasz felezőpontjára, a tükörkép is rácspont lesz.

32. Kombinatorika a geometriában

357

Ha van egy szabályos rácssokszögünk, abban minden csúcsot tükrözzünk a vele szomszédos két csúcs közti átló felezőpontjára. A szabályos háromszög, hatszög és a négyzet kivételével minden más szabályos sokszög esetén a tükörképek egymástól különböző pontok a sokszög belsejében, melyek maguk is szabályos sokszöget alkotnak. (Miért?) Ily módon újabb és újabb, egyre kisebb szabályos rácssokszöget kapunk; de a kiinduló sokszög belsejében véges sok rácspont volt, így nem létezhet a rácssokszögeknek ez a végtelen sorozata. Az ellentmondás oka a kiinduló feltevés hibás volta. 1925. Az előző feladathoz hasonlóan itt is a végtelen leszállás módszerét alkalmazzuk. Használjuk fel a négyzetrács következő tulajdonságát: ha egy rácspontot egy másik rácspont körül 90◦ -kal elforgatunk, akkor a kapott képpont is rácspont lesz. Tegyük fel, hogy egy szabályos ABCDEF hatszög csúcspontjai a négyzetrács rácspontjai. Ekkor a hatszög A csúcsát B körül, B csúcsát C körül, . . . , F csúcsát A körül elforgatjuk 90◦ -kal úgy, hogy a képpontok a hatszög belsejébe essenek. Ezek a pontok is egy szabályos hatszög csúcsai (miért?), valamint előbbi megjegyzésünk szerint rácspontok is. Ily módon újabb és újabb, egyre kisebb szabályos rácshatszöget kapunk; de a kiinduló hatszög belsejében véges sok rácspont volt, így nem létezhet a rácshatszögeknek ez a végtelen sorozata. Az ellentmondás oka a kiinduló feltevés hibás volta. Mivel nincs szabályos rácshatszög, ezért szabályos rácsháromszög sem lehet. Ha lenne szabályos rácsháromszög, akkor lenne szabályos rácshatszög is. Ugyanis a háromszög oldalainak harmadolópontjai szabályos hatszöget alkotnak, és ha ezt a háromszög egyik csúcsából háromszoros arányban nagyítjuk, szabályos rácshatszöget kapunk. 1926. Vegyünk fel egy, az O szimmetriaközépponton áthaladó húrt, és rá merőlegesen egy másik húrt. A húrok hossza legyen a és b, s tudjuk, hogy O felezi a húrokat. Ezt a merőleges húrpárt forgassuk O körül, s a forgatás szöge folytonosan változzék 90◦ -ig. Forgatás közben figyeljük az a − b különbség változását. Ez a különbség folytonosan változik és előjelet vált, ezért valamely köztes állapotban értéke 0, azaz a = b. Ebben a helyzetben a húrok végpontjai egy négyzet csúcsai. Megjegyzés. Tetszőleges zárt síkgörbén is mindig található 4 olyan pont, amelyek egy négyzet csúcsai. 1927. Válasszuk ki a görbe három tetszőleges pontját, s tekintsük az ezekre illeszkedő síkot; s figyeljük, hogy a görbe ezeken a pontokon áthaladva a síknak mikor melyik oldalán tartózkodik. Könnyen látható, hogy a görbe ezt a síkot egy negyedik pontban is metszeni fogja. Most megmutatjuk, hogy öt pontra nem igaz ez az állítás. Konstruálunk egy olyan térgörbét, melyen nincs öt komplanáris pont. Vegyünk egy gömböt és a gömböt

358

Megoldások, útmutatások

nem metsző S síkot. A gömb felületén vegyünk fel egy olyan L görbét, melynek S-re való merőleges vetülete ellipszis — ez lesz a kívánt tulajdonságú görbe. A vetítéssel megadtunk egy ellipszis alapú hengert. Tegyük fel, hogy L-en van öt komplanáris pont. Nos, az öt pont síkja a gömbből kört, a hengerből ellipszist metsz ki, ami ellentmondásra vezet, mert egy körnek és egy ellipszisnek nem lehet öt közös pontja. 1929. A színeket nevezzük a, b, c, d, e-nek, és A, B, C, D, E jelentsen a továbbiakban egy-egy a, b, c, d, ill. e színű pontot. Egy egyenest, síkot, ha van rajta 3, 4, ill. 5 különböző színű pont, röviden 3-, 4-, ill. 5-színűnek fogunk nevezni. Ha egy 3-színű síknak és egy olyan egyenesnek, amelyiken van a maradó 2 színű pont, van közös pontja, akkor van a térben 4-színű sík. Ugyanis, legyen az S síkon a, b és c színű pont, a v egyenesen d és e színű pont, és P legyen a sík és az egyenes közös pontja. Ha P színe a, b vagy c, akkor v 3-színű, s ekkor v-hez választjuk a térnek valamely negyedik színű O pontját, a v-re és O-ra illeszkedő sík 4-színű; ha pedig P d vagy e színű, akkor S 4-színű. Nos, ezek után igazoljuk a feladat állítását. Ha A, B, C, D, E közül valamely négyen át fektethető sík, akkor igaz a feladat állítása. Ha nem, akkor tekintsük az ABCD tetraéder oldallapjainak a síkját. Ha valamelyiknek, pl. a B, C, D pontokat tartalmazó S síknak ellenkező oldalára esik a tetraéder és E, akkor AE metszi S-t. Ha viszont bármelyik síkot véve, annak ugyanarra az oldalára esik a tetraéder és E, akkor a tetraéder tartalmazza E-t. E különbözik A-tól, mert más színű, így közelebb van S-hez, mint A; tehát ebben az esetben is metszi AE az S síkot. Mindkét esetben az előzetesen bizonyított állítást felhasználva láthatjuk, hogy igaz a feladat állítása. 1930. Igen, például a legkésőbb érkező vendég érkezésének (vagy a legkorábban elmenő távozásának) pillanatában. (Ha egy egyenes véges sok intervalluma közül bármely kettőnek van közös pontja, akkor olyan pont is található, amely az összes intervallumban benne van.) 1931. Előbb n = 4-re lássuk be az állítást. Legyen a négy halmaz közül három-három halmaz közös pontja P123 , P124 , P134 , P234 . Ha e négy pont konvex burka négyszög, akkor átlóinak metszéspontja; ha pedig háromszög, akkor a negyedik pont a négy halmaz közös pontja lesz. Az n > 4 eset — az előbbieket felhasználva — igazolható teljes indukcióval. 1932. Bármely ponttól legfeljebb hat másik lehet minimális távolságra, és a konvex burok pontjaitól még kevesebb. 1933. Lássuk be, hogy a pontpárok között fellépő maximális, d hosszúságú szakaszok közül bármely kettőnek van közös pontja.

32. Kombinatorika a geometriában

359

Mutassuk meg, hogy ha valamelyik pontra legalább három maximális szakasz illeszkedik, akkor van olyan pont is, amelyre legfeljebb egy. Ha az n pont között van olyan, amelyre legfeljebb egy d hosszúságú szakasz illeszkedik, azt hagyjuk el. Ezt az eljárást a maradék pontrendszerre is ismételjük meg, amíg lehet. Ha így m pontot hagyunk el, akkor legfeljebb m db d hosszúságú szakaszt szüntetünk meg. A maradék n − m pont közül bármelyikre legalább két d hosszúságú szakasz illeszkedik; de több a korábbi megjegyzés miatt nem lehet, hiszen akkor lenne olyan, melyre csak egy illeszkedik. Ezért ezek között n − m db d hosszúságú szakasz van. A pontok között összesen legfeljebb m + + (n − m) = n-szer lép fel a maximális távolság. 1934. Legyen A, B, C a ponthalmaz három olyan pontja, amelyek nincsenek egy  egye|P A| − nesen. |AB| = k, |AC| = m, és P egy tetszőleges pont a halmazból.   − |P B| = |q| ≤ |AB| = k, így P rajta van az A, B fókuszú hiperbola valamelyik ágán, q = −k, . . . , −1, 0, 1, . . . , k; azaz a 2k + 1 hiperbola valamelyikén. Ugyanígy illeszkedik a P pont az A, C fókuszú 2m + 1 hiperbola valamelyikére is. Ennek a két hiperbola-seregnek véges sok — legfeljebb 4(2k + 1)(2m + 1) — közös pontja van. Megjegyzés. Létezik olyan végtelen, nem egy egyenesbe eső ponthalmaz, amelyben minden távolság racionális. 1935. Tegyük fel, hogy létezik ilyen pontrendszer. Tekintsük a pontok által kifeszített egyenesek és az egyenesen nem levő pontok távolságait. Ezek közül válasszuk ki a legkisebb távolságot d-t, legyen ez a P pont és az e egyenes távolsága. Bocsássunk P -ből merőlegest e-re; ezzel az egyenest két részre osztottuk, s az egyenesen levő 3 pontból legalább kettő az egyik félegyenesre esik. A két pontból (nevezzük őket A-nak, B-nek) a P -től távolabbi B. Most jutunk ellentmondásra, mert a P B egyenes és az A pont közti távolság kisebb d-nél. Tehát nem létezik a kérdezett tulajdonságú pontrendszer. Megjegyzés. A Sylvester-feladatot Erdős Pál tette ismertté az 1930-as években. Azóta a Sylvester-feladatra újabb és újabb bizonyítások születnek és különböző rokon problémákat fogalmaznak meg. Legyen adva a síkon n pont úgy, hogy nincs mind egy egyenesen, s jelölje f (n) azoknak az egyeneseknek a minimális számát, mely ezekből a pontokból pontosan kettőn megy át. A Sylvester-feladat szerint f (n) ≥ 1. Bebizonyították, hogy f (n) ≥ 3; s ez a becslés pontos is, ha n √ értéke 3, 4, 6 vagy 7. Később megmutatták, hogy f (n) > n; majd azt is, hogy 3 n f (n) ≥ n, sőt n > 7 esetén f (n) ≥ . 7 2 Vizsgáljuk a két-pontú egyenesek helyett a négy-pontúakat. Erdős a következő problémát tűzte ki. Legyen adva n pont a síkon úgy, hogy nincs közte öt, melyek kollineárisak. Jelölje g(n) a négy pontot tartalmazó egyenesek számának maxig(n) mumát. Igaz-e ekkor, hogy 2 → 0, ha n → ∞? A kérdés eldöntésére 100 n

360

Megoldások, útmutatások

dollárt ajánlott fel. Előbb azt igazolták, hogy g(n) > c · n · log n; majd pedig erősebb állítást, mégpedig azt, hogy g(n) > c · n3/2 . Talán ez utóbbi már pontos korlát. A hárompontú egyenesek számának maximuma c · n2 , ahol c pozitív konstans. 1936. Vegyük észre azt, hogy a pontok közülbármely két pontra legfeljebb két egység n sugarú kör illeszkedhet. Az n pontból db pontpár választható, ekkor ezekre 2   n legfeljebb 2 · egységsugarú kört tudunk illeszteni; azonban így minden kört 2 3-szor számoltunk (hiszen az A, B, C pontokon átmenő kört megszámoltuk, mint az A és B, a B és C, valamint a C és A pontokra illeszkedő kört). Ezért az egységsugarú  körökszáma  legfeljebb n 2 n 1 ·2· = . 3 2 3 2 Megadunk egy konstrukciót, melyben az egységkörök száma c · n3/2 . → → Legyenek − a1 , − a2 , . . . , − a→ N egységvektorok a síkban. A ponthalmazt, melyek − → − → köré a köröket  írjuk,  az ai + aj helyvektorok   végpontjai alkotják. Ezért a pontok N N száma m = . E pontok köré n = egységkör írható, ugyanis minden 2 3 → → → → → → i = j = k esetén az − ai + − aj , − aj + − ak , − ak + − ai ponthármas köré írt kör sugara → → → aj + − ak ponttól mindhárom pont egységnyi; hiszen a kör középpontjától, a − ai + − egységnyire van. 1937. Vergilius híres eposzában dolgozta fel Didó, türoszi királylány, Karthágó alapítójáról szóló következő görög-római mondát. Eszerint Didó szellemes furfanggal egy országrészt vásárolt, amely Byrsa vagy Bikabőr néven ismert. Didó valójában csak annyi földre alkudott, amely körülkeríthető egy bikabőrrel. A bikabőrt azonban vékony csíkokra szabdalta, és ezeket egymáshoz illesztve hatalmas területet kerített be, amit ma Byrsanak neveznek, és amely Karthágót magába ölelte. Innen származtatható feladatunk, melynél a keresett síkidom a kör; ezt könnyű megsejteni, de az első bizonyításra sokáig kellett várni. A XIX. században Jacob Steiner (1796–1863), a kor egyik legnagyobb geométere oldotta meg a feladatot. Bizonyítása, a „négycsuklós eljárás”-nak nevezett módszer a következő: belátja, hogy ha a síkidom nem kör, akkor a a területe növelhető úgy, hogy a kerülete nem nő. Ezért, ha létezik az extremális tulajdonsággal rendelkező síkidom, akkor az csak a kör lehet. A bizonyítás állomásai: a síkidom konvex; a síkidomnak van szimmetriatengelye; ha nem kör, akkor (Thalész tétele miatt) van olyan P pontja, melyből a szimmetriatengely A és B végpontjai nem derékszög alatt látszanak. Ekkor a P AP  B négyszöget (a P  pont a P tükörképe a szimmetriatengelyre) mint egy csuklós szerkezetet tekintjük, és ezt a csuklós szerkezetet mozgatjuk (a fölötte levő ívekkel együtt) úgy, hogy P -nél (és így P  -nél is) derékszög legyen — ezzel növeltük a területét, a kerület nem változott.

33. Gráfok

361

Jacob Steiner fenti bizonyítása azonban hiányos. Ő azt bizonyította be, hogy körlemeztől különböző síkidom területe nem lehet legnagyobb a vele egyenlő kerületűek között; de nem bizonyította be, hogy a körlemeznek csakugyan megvan ez a maximum-tulajdonsága. Hogy mennyire hiányos ez a bizonyítás, azt a következő példán mutathatjuk be: Ehhez hasonló gondolatmenettel „bizonyította be” Perron, hogy 1 a legnagyobb természetes szám. Azt mondta ui., hogy a természetes számok 1, 2, 3, 4, . . . sorozatában semelyik, 1-nél nagyobb k szám nem lehet a legnagyobb, mert ebből a feltevésből ellentmondásra jutunk. Ha ti. k > 1 lenne a legnagyobb természetes szám, akkor négyzete: k 2 = = k ·k > k ·1 = k; tehát már találtunk is egy számot, amely nagyobb a feltételezett legnagyobb természetes számnál. Emiatt k > 1 biztosan nem lehet legnagyobb a természetes számok közül. Mivel pedig a négyzetreemeléssel az 1-ből ismét 1 lesz, azaz előbbi okoskodásunk ebben az egyetlen esetben nem vezet ellentmondásra; ebből következik, hogy 1 a legnagyobb természetes szám. Visszatérve az izoperimetrikus problémára, megnyugtatásul közöljük a következőket. Egyrészt a későbbiekben sikerült teljessé tenni Steiner hiányos bizonyítását (a kiegészítés ismertetésétől — annak bonyolult volta miatt — ehelyütt le kell mondanunk), másrészt más helyes, pl. analitikus bizonyítással is igazolták, hogy az egyenlő kerületű síkidomok közül a körlemez területe a legnagyobb.

33. Gráfok 1938. n csapat esetén egy-egy csapatnál a győzelmek száma 1 és n − 1 között lehet. Ezért van két csapat, mondjuk A és B, melyeknek ugyanannyi győzelme van. Vegyük azt, hogy A győzte le B-t. Azok között, amelyeket B legyőzött, van olyan csapat, legyen ez C, amelyet A nem győzött le (hiszen, ha nem lenne ilyen, akkor A több csapattal szemben győzött volna, mint B). 1939. Összesen 28 meccs volt; így van olyan csapat, mely legalább 28/8 = 3, 5, azaz legalább 4 meccset nyert. Legyen ez A, és válasszunk ki 4 csapatot azok közül, amelyeket legyőzött. Ezek egymás között 6 mérkőzést játszottak, valamelyik legalább 6/4 = 1, 5, azaz legalább 2 mérkőzést nyert. Legyen ez B, és az általa legyőzött (amelyeket persze A is legyőzött) csapatok C és D. A C és D jelölést úgy választjuk, hogy C legyőzte D-t. Megjegyzés. Általánosítható az állítás 2n−1 csapatra. Közülük kiválasztható A1 , A2 , . . . , An csapat, hogy i < j esetén Ai legyőzte Aj -t. 1940. Legyen A az első csapat egyik legeredményesebb játékosa. A-nak is volt veresége, kikapott B-től. Mivel B megverte A-t, ezért ő csak egy A-tól különböző

362

Megoldások, útmutatások

C-től kaphatott ki. Mivel A a csapatának legjobb játékosa, van a második csapatban olyan D játékos, akit A megvert, és akitől C kikapott. Ha nem lenne ilyen, akkor C több győzelmet aratott volna, mint A, ami ellentmondás. 1941. Legyen A az (egyik olyan), akinek legtöbb győzelme van. Ha B legyőzte A-t, akkor a B által legyőzöttek száma nem nagyobb, mint az A által legyőzöttek száma (az A választása miatt). Ezért van olyan C, akit A legyőzött, de B nem. 1942. Válasszunk egy tetszőleges csapatot, A-t. A-n kívül 17 csapat van. A eddig 8-cal mérkőzött, 9-cel nem. Ha e között a 9 csapat között van kettő, melyek nem játszottak egymással, akkor készen vagyunk. Ennek így is kell lennie, hiszen különben a 9-es csoport minden csapata a 8 mérkőzést a 9-es csoporton belül játszotta; ami nem lehet, mert ekkor egy-egy fordulóban a 9 csapat valamelyikének nem jutna ellenfél. 1943. Pingpongmérkőzés nem végződhet döntetlenül, így két-résztvevős „körmérkőzés” esetén az állítás nyilvánvalóan igaz. Erre támaszkodva, az állítás helyességét a résztvevők száma szerinti teljes indukcióval bizonyítjuk. Feltesszük, hogy az állítás helyes, ha a résztvevők száma legföljebb n. n + 1 résztvevő esetén szemeljük ki egyiküket, legyen ez B. A többi n játékos feltevésünk szerint sorba állítható a kívánt módon az egymás közt lejátszott mérkőzések alapján. Állítsuk most B-t ebbe a sorba az első olyan elé, akit ő legyőzött — amennyiben ilyen van —, különben a sor végére. Így B helyzete nemcsak a hátsó, hanem elülső szomszédjához viszonyítva is megfelelő, hiszen az előtte állók egyikét sem győzte le, tehát vagy az áll, hogy azok mind legyőzték B-t, vagy pedig senki sem győzte le, akkor pedig őt az első helyre állítottuk. B ilyen beállítása sem az előtte, sem a mögötte állók egymás közti elhelyezését nem változtatta meg, így példát adtunk az n + 1 játékos megfelelő sorbaállítására. Másik bizonyítás. Állítsuk sorba találomra a versenyzőket, majd az első kettőt cseréljük fel, ha az első kikapott a másodiktól, különben hagyjuk őket az eredeti sorrendben; majd sorra a másodikat és harmadikat, harmadikat és negyediket, . . . , cseréljük meg, ha nem megfelelő sorrendben állnak. Előfordulhat, hogy így egy cserével előbbre került versenyző és az előtte álló közt helytelen sorrend keletkezik, ezért a sor végére érve térjünk vissza az elejére és ismételjük meg újra és újra az eljárást. Belátjuk, hogy véges sokszor végighaladva a soron, eljutunk egy olyan elrendezéshez, amely már kielégíti a feladat követelményeit. Valóban, a versenyzők csak véges számú különböző sorrendben állhatnak; így elég sokszor végigmenve a soron, elő kell fordulnia egy olyan sorrendnek, amelyik már korábban is előfordult. Ez azonban csak akkor lehetséges, ha végighaladva már nincs szükség cserére, azaz ha a sorrend kielégíti a feladat feltételeit. Ha ugyanis valakit felcseréltünk a rákövetkezővel, az minden későbbi sorrendben is hátrább marad nála, hiszen őket ellenkező irányban nem cserélhetjük meg, így a korábbi sorrend nem térhet vissza.

33. Gráfok

363

1944. 1. megoldás. Legyen n résztvevő. Jelölje ai az i-edik résztvevő azon ismerőseinek számát, akik előtte állnak, bi pedig legyen az utána álló ismerősök száma. Nyilván a1 + a2 + · · · + an = b1 + b2 + · · · + bn , s mivel 1 < i < n esetén ai = bi , ezért a1 + an = b1 + bn , viszont a1 = bn = 0. Tehát an = b1 . 2. megoldás. Minden résztvevő adjon 1–1 Ft-ot a mögötte álló ismerőseinek. Az első és az utolsó kivételével mindenki ugyanannyit adott, amennyit kapott. Tehát amennyit az első adott, annyit kapott az utolsó, hiszen az összvagyon nem változott. 1945. Lehetséges. 1. megoldás. Számozzuk meg őket az 1, 2, . . . , n számokkal. Az n i-edik és a j -edik ismeri egymást, ha |i − j | ≤ . Ekkor azonos számú ismerőse 2 csak a k-adik és (n − k)-adik személynek van. (Miért?) 2. megoldás. Teljes indukcióval adunk eljárást az ismeretségek szervezésére. Indukciós lépésben (k + 1)-re az új, (k + 1)-edik személy ne ismerje a többit, ha azok között van olyan, aki a többit ismeri; különben pedig ismerjen mindenkit. 1946. Az lenne a kedvező, ha mindenkinek legfeljebb 6 ismerőse lenne, s ez így is van. Az A lakó ismerősei között nem lehet kettő, akik ismernék egymást (az első feltétel szerint), ugyanakkor már 7 ember között vannak ismerősök (második feltétel), tehát A-nak legfeljebb 6 ismerőse lehet. 1947. Belátjuk, hogy legalább 97 tanuló ismeri az összes többit. Tegyük fel, hogy nem ismer mindenki mindenkit. Ekkor legyenek A és B olyanok, akik nem ismerik egymást. A, B-hez válasszunk tetszőlegesen X, Y -t. Ekkor a feltétel szerint pl. X ismeri A, B, Y -t. Most A, B, X, Z-re kapjuk, hogy X mindenkit ismer. A, B-n kívül legfeljebb egy olyan tanuló lehet, aki nem ismer mindenkit. 1948. Legyen A egy tetszőleges diák. Megmutatjuk, hogy van olyan B diák, aki Anak két barátjával is barátkozik; ez együtt négy jó fő. Tegyük fel, hogy nincs ilyen B. Ekkor A mind a 10 barátjának van 8 másik barátja úgy, hogy ezek a második barátok különbözők. (Nézzük meg, miért 8 másik barátot vizsgálunk.) Ha nem lennének különbözők, akkor A, valamint az a két barát, akinek van közös második barátja, megfelelő négy személy lenne. Ez együtt 1+10+80 = 91 diákot jelentene, miközben a társaság 90 főből áll. Tehát kiválasztható a megfelelő négy személy. 1949. 1. megoldás. Legyen A az, akinek legtöbb ismerőse van, összesen n. Az A ismerőseinek a feltétel szerint mind különböző számú ismerőse van. Ezek mindegyikének legalább 1 ismerőse van, hiszen A-t ismerik. Mivel közöttük bármely kettőnek van közös ismerőse (az A), ezért nincs köztük kettő, akinek ugyanannyi ismerőse van. Ez csak úgy lehet, ha ezen n személy ismerőseinek száma valamilyen sorrendben 1, 2, . . . , n. 2. megoldás. Teljes indukció a létszámra.

364

Megoldások, útmutatások

1950. Ha az állítás nem igaz, akkor a társaságban volnának ismerősök; és olyanok is, akik nem ismerik egymást. Mivel az nem lehet, hogy egyesek mindenkit ismerjenek, mások pedig senkit sem; így volna olyan ember is — jelöljük őt E-vel —, akit a társaság bizonyos tagjai ismernek — legyen ezek egyike I —, mások viszont, pl. N, nem. Ha E kimegy, akkor I ismerőseinek száma eggyel csökken, míg N-é nem változik. A feltételek alapján tehát I pontosan eggyel több embert ismer a társaságban, mint N. A társaság akármelyik, E, I , N-től különböző tagjának távozása után I -nek és Nnek csak úgy lehet ugyanannyi ismerőse; ha I ismeri az illetőt, N pedig nem. Ez viszont azt jelenti, hogy N legfeljebb I -t ismeri a társaságból, I pedig legfeljebb N-et nem ismeri. Ha most I és N ismerik egymást, akkor I mindenkit ismer — azaz ismerőseinek száma (n − 1) —, N viszont csak I -t ismeri. Ha pedig I és N nem ismerősök, akkor I ismerőseinek száma n − 2, N-é pedig 0. Tudjuk, hogy I eggyel több embert ismer, mint N, tehát mindkét esetben n = 3 adódik. Ez viszont ellentétben áll az n > 3 feltétellel, s így a feladat állítását igazoltuk. Megjegyzések. Az állítás n = 3-ra nem igaz: ha E, I és N közül csak E és I ismerik egymást, akkor a feltételek teljesülnek. Az állítás érvényben marad akkor is, ha az ismeretségekről nem tesszük fel, hogy kölcsönösek. 1951. A válasz igen. 1. megoldás. Teljes indukció. 2. megoldás. Válasszuk ki a házaspárok legnagyobb csoportját, ahol a férjek ismerik egymást. A többi házaspár megy a másik szobába. Utóbbi szobából választva két házaspárt, és az előbbiből egy harmadik párt; a férjek nem ismerik egymást, így az asszonyok ismerősök. 1952. Teljes indukcióval bizonyítunk. n = 2-re könnyű. 2n + 2 személy esetén válasszuk ki közülük A-t és B-t úgy, hogy egymást nem ismerik. Ha van két közös ismerősük, akkor ez a 4 személy jó. Ellenkező esetben hagyjuk el a 2n + 2 fős társaságból A-t és B-t, s alkalmazhatjuk az indukciós feltevést. 1953. Ha mindenkinek legalább 4 ismerőse van, akkor igaz az állítás. Ha van olyan A, akinek legfeljebb 3 ismerőse van, akkor ő a társaság legalább 5 tagját nem ismeri. Ezek viszont mind ismerik egymást (közülük bármelyik kettőhöz válasszuk A-t, s alkalmazzuk a feladat feltételét). Megjegyzés. Az állítást úgy erősítve, hogy az 5 fő ismeri egymást — már ez nem lesz igaz. Ellenpélda a következő: állítsunk 9 főt körbe, s csak a szomszédok nem ismerik egymást. 1954. Ha van olyan, aki legalább 4 másikat nem ismer, akkor ez utóbbiak közül bármely kettőnek ismernie kell egymást; azaz máris találtunk 4 megfelelő embert.

33. Gráfok

365

Így feltehetjük, hogy mindenkinek legalább 5 ismerőse van. De nem lehet min5·9 denkinek pontosan 5 ismerőse, mert ez ismeretséget jelentene. Kell tehát 6 2 ismerőssel rendelkezőnek is lennie; legyen ez A, és A-nak ismerőse B. Ha B az A ismerősei közül legalább 3-at ismer, akkor ezek közül kettő A-val és B-vel jó négyes. Ha nem, akkor B az A ismerősei közül legalább hármat nem ismer. Ez a három és A jó négyes. Megjegyzés. Teljes indukcióval könnyen be lehet bizonyítani a következő tételt.  n+1 Ha egy szobában ember van, és bármely három között van kettő, akik 2 ismerik egymást; akkor van a szobában n ember, akik közül bármely kettő ismeri   n+1 egymást. Jelöljük f (n)-nel azt a legkisebb egészt, amit helyébe téve még 2 mindig igaz marad az állítás. Könnyű ellenőrizni, hogy f (2) = 3, f (3) = 6 és f (4) = 9. A feladatban f (4) ≤ 9-et kellett bizonyítani. 1956. Eljárást adunk arra, hogyan lehet a társaságból legalább k + 1 személyt leültetni egy kör alakú asztal köré úgy, hogy mindegyikük ismerje a szomszédait. Válasszunk ki valakit a társaságból, és jelöljük 1-gyel. Ezután jelöljük (i +1)-gyel az i-vel jelölt tag egyik, eddig még nem számozott ismerősét, majd ismételjük meg ezt a műveletet addig, amíg el nem akadunk. Legyen a legutolsó sorszám n, az n jelű személy legkisebb sorszámú ismerősének sorszáma m. Ekkor m, m + 1, m + 2, . . . , n leültethető egy kör alakú asztalhoz úgy, hogy mindenki ismerje a szomszédjait. Másrészt n minden ismerősének van sorszáma, hiszen n-nél akadtunk el, és ezek mind az asztalnál ülnek. Így legalább k + 1 személyt ültettünk le, amivel a feladat állításánál többet is bizonyítottunk. 1957. Tekintsünk egy G gráfot, melynek csúcsai a városok, élei pedig kétféle színűek. A kék él jelentse a hajóutat, a piros pedig a repülőutat a két város között. Mivel bármely két csúcs össze van kötve, ezért G teljes gráf, azaz a kék élek által meghatározott részgráfjának komplementere a piros élek által meghatározott részgráf. Mivel egy gráf és komplementere közül legalább az egyik összefüggő, ezért az országot vagy hajóval, vagy repülővel be lehet járni. 1958. Legfeljebb 100 kézfogás lehetséges. 1959. Egy ember legfeljebb hat másikkal fogott kezet, hiszen a társával, ill. saját magával nem fog kezet. Emiatt Barna úr a kérdésére úgy kaphat különböző válaszokat, ha a válaszok rendre: 0, 1, 2, 3, 4, 5, 6. Jelöljük A-val azt az embert, aki 6 másikkal fogott kezet! Ez azt jelenti, hogy csak házastársával nem fogott kezet. Tehát aki a 0-t mondta, az nem lehet más, mint a házastársa. (Mivel Barna úr nem válaszolt a kérdésre, A nem lehet Barna úr!) Ezután ezt a házaspárt küldjük át a szomszéd szobába, és újra tegyük fel a kérdést. Ekkor a 0, 1, 2, 3, 4 válaszokat kapjuk. Ismét látható, hogy a 0-t és 4-et válaszolók házaspárok. Őket átküldjük

Megoldások, útmutatások

366

a szomszédba. Az újabb válaszok 0, 1, 2. A 0-t és 2-t válaszolók házastársak, az 1-et válaszoló Barnáné. Így Barnáné három jelenlevővel fogott kezet. 1960. A vendégségben jelen levő 10 ember közül mindenki ismerte házastársát, ezért nem mutatkozott be neki, és önmagának sem. Így mindenki maximálisan 8 új ismeretséget köthetett. Mivel a feladat szerint a 10 jelenlevő 9 különböző ismeretséget kötött, ezért volt olyan, aki 8, aki 7, 6, 5, 4, 3, 2, 1, 0 ismeretséget kötött. Kovács kézfogásainak száma természetesen ezek közül valamelyikkel megegyezett, hiszen őrá is érvényes, amit az első mondatban mondtunk. A 8 ismeretséget kötő házastársa a 0 ismeretséget kötő volt, hiszen a többiek biztosan megismerkedtek vele. Hasonlóan láthatjuk be lépésről lépésre, hogy a 7, illetve 1; a 6, illetve 2; és az 5, illetve 3 ismeretséget kötők házastársak. Például a 7 ismeretséget kötő önmagán és az x emberrel megismerkedő házastársán kívül a 0 ismeretséget kötőnek nem mutatkozott be, tehát a fennmaradó 7 ember mindegyikét most kellett megismernie. Így a 8 − 0 és 7 − x páron kívül mindenki legalább két emberrel megismerkedett. Emiatt x csak 1-gyel lehet egyenlő. A megmaradó kilencedik jelenlevő, Kovácsné már csak négy ismeretséget köthetett.

34. Halmazrendszerek   n 1961. Az állítás igazolható teljes indukcióval, vagy úgy is, hogy felhasználjuk az + 0       n n n + + + ··· + = 2n összefüggést. Elegáns útja a bizonyításnak a 1 2 n következő: ha az n-elemű halmaz {a1 , a2 , . . . , an }, akkor ennek egy részhalmazát egyértelműen kijelöli egy n hosszúságú 0 − 1 sorozat. Ha a sorozatban a k-adik helyen 1-es áll, akkor ak eleme a részhalmaznak; ha pedig 0 áll a k-adik helyen, akkor nem. Így a részhalmazok száma ugyanannyi, mint az n hosszúságú 0 − 1 sorozatok száma: 2n . 1962. Ha egy részhalmaz a kiválasztottak között van, akkor a komplementere nem lehet ezek között. Így a kiválasztott részhalmazok száma legfeljebb fele az összes részhalmaz számának, azaz legfeljebb 2n−1 . Ez a határ el is érhető: valamely x ∈ H esetén minden Ai olyan, hogy x ∈ Ai . 1963. Soroljuk X részhalmazait párokba úgy, hogy minden részhalmaz a komplementerével legyen egy párban. Mivel a párok száma 2n−1 > m, ezért van olyan pár, amelynek egyik tagja sem szerepel az Ai halmazok között. Legyen egy ilyen pár két tagja B és C. Ha az Ai halmazok mindegyikének van közös eleme B-vel, akkor a B halmaz megfelelő. Feltehetjük tehát, hogy például A1 a B-vel diszjunkt, s így részhalmaza C-nek.

34. Halmazrendszerek

367

Ebben az esetben a C halmaz lesz megfelelő. Valóban, minden egyes Ai halmaznak van közös eleme A1 -gyel, ezért az A1 -et tartalmazó C-vel is. 1964. Legyen S páros számainak halmaza E, a páratlan számok halmaza O. Minden Ai -ből két halmazt készítünk: Ei = Ai ∩E, Oi = Ai ∩O. (Az Ei halmazok között lehetnek megegyezők, s ugyanígy az Oi halmazok között is.) Ekkor Ei ∩ Ej = ∅ 1 és Oi ∩ Oj = ∅. Így az Ei halmazok száma ≤ 2|E| , az Oi halmazok száma 2 1 |O| ≤ 2 . Az Ei és Oi halmazokból tudjuk felépíteni az Ai halmazok rendszerét, 2 1 1 1 ezért m ≤ 2|E| · 2|O| = 2|E|+|O| = 2n−2 . 2 2 4 1966. Ai -hez rendeljünk egy tetszőleges xi ∈ H \ Ai elemet. Ha Ai  = Aj , akkor xi  = xj ; hiszen az előbbi xi minden Aj -nek eleme kell hogy legyen, ahol j  = i. Az így kiválasztott xi -k száma legfeljebb n, ezért m ≤ n. Megjegyzés. Vizsgálhatnánk a következő kérdést: A1 , A2 , . . . , Am ⊂ H , |H | = n, és Ai ∪ Aj ∪ Ak = H , ha i = j = k. Ekkor mekkora lehet m legnagyobb értéke? 1967. A feltétel miatt mindegyik Ai -nek van olyan eleme, amely a többi Aj halmaznak nem eleme. Ezért az Ai halmazok száma legfeljebb n. 1968. Általában igaz, hogy n elemű halmaz esetén a feladatbeli részhalmazok száma legfeljebb 2n−1. Ezt teljes indukcióval igazoljuk. Legyen m elemű a legnagyobb kiválasztott M halmaz, amely nem az alaphalmaz. Ekkor bármely kiválasztott halmaz M-nek vagy komplementerének egy részhalmaza, vagy maga a teljes halmaz. Az indukciós feltevésből ezek száma legfeljebb (2m − 1)+(2(n − m) − 1)+ + 1 = 2n − 1. Legyen a 100 elemű halmaz az 1, 2, . . . , 100 számok halmaza. Válasszuk ki összes egyelemű részhalmazát, valamint az {1, 2, . . . , n} alakú halmazokat, ahol n = 2, 3, . . . , 100. Az így kiválasztott 199 részhalmaz teljesíti a feladat feltételeit. Tehát a feladat kérdésére a válasz: 199. 1969. Ha valamely i-re |Ai | > k + 1, akkor Ai helyett vegyük annak egy k + 1 elemű Ai részhalmazát, s ha |Ai | ≤ k + 1, akkor legyen Ai = Ai . Ha Ai = Aj , akkor Ai = Aj , hiszen |Ai ∩ Aj | ≤ k. Az így kapott A1 , A2 , . . . , Am rendszer halmazai k+1    n legfeljebb k + 1 eleműek, ezért m ≤ . i i=0 Második bizonyítás. Legyen g(n, k) az állításban szereplő m maximális értéke adott n és k mellett. Ekkor teljesül a g(n+1, k +1) ≤ g(n, k +1)+g(n, k) rekurzió. k+1    n Ezt felhasználva teljes indukcióval bizonyítható g(n, k) ≤ . n = 1, n = 2 i i=0 esetén könnyen látható az egyenlőtlenség. Az indukciós lépésben a rekurziót is

Megoldások, útmutatások

368

felhasználva g(n + 1, k + 1) ≤

k+2    n

+

k+1    n

=

    k+2    n+1 n n + + = 0 i i−1 i=1

i i i=0 i=0  k+2   n+1 . = i i=0 Az {Ai : Ai ⊂ H, |H | = n, |Ai | ≤ k + 1} halmazrendszer mutatja, hogy az állításban szereplő felső korlát pontos. 1970. |A1 ∩ A2 ∩ . . . ∩ Ak | ≤ |Ai |. 1971. Az Ai ∩ Aj 1-elemű halmazok, mind különbözők, számuk

  k ; ezért 2

k(k − 1) . 2 1972. A feladat egy más megfogalmazása: Egy n lakosú városban klubokat szerveznek úgy, hogy bármely két klubnak legyen, és bármelyik háromnak már ne legyen közös tagja. Legfeljebb hány klubot lehet így szervezni? Ennek megoldása: Legyen a klubok száma m. A feltétel második része azt jelenti, hogy egy ember legfeljebb két klubba járhat. Rendeljük hozzá minden klub-párhoz valamelyik közös tagjukat. (A feltétel első fele szerint   van közös tagjuk.) Mivel egy ember m nem járhat kettőnél több klubba, az kiválasztott ember mind különböző.  2 m Mivel a városban n ember lakik, így ≤ n. 2   m Megfordítva, ha ≤ n teljesül, akkor lehetséges az m darab klub megszerve2     m m zése: embert szétosztunk a klub-párok között, a maradék n − embert 2 2 pedig például egy klubba sem járatjuk. Ezzel elérjük, hogy bármely két klubnak legyen pontosan egy, de semelyik háromnak nelegyen közös tagja. m(m − 1) m Keressük a legnagyobb m értéket, amelyre ≤ n. ≤ n, 2 2 1 2 1 1 1 m2 − m ≤ 2n, m − ≤ 2n + , m ≤ + 2n + . A legnagyobb ilyen m, 2 2 4 √ 4 1  1   1 + 8n + 1  az m = + 2n + = . 2 4 2 1973. (Megjegyzés. A feladat azonos a következővel: Egy 9 fős társaságban mindenkinek van (legalább) 7 ismerőse. Mutassuk meg, hogy ekkor bármely 4 személynek van közös ismerőse.) |A1 ∪ A2 ∪ . . . ∪ Ak | ≥

34. Halmazrendszerek

369

A négy részhalmaz a 9 elemű halmaznak összesen 4 · 7 = 28 elemét reprezentálja (némelyiket többször). Nem lehet, hogy mindegyik elem legfeljebb 3-szor szerepeljen, mivel 9 · 3 < 28. 1974. (2n+1 − 1)-féleképpen választhatunk ki néhány halmazt, és mindegyik esetben vehetjük ezek unióját. Az uniónak 2n − 1 lehetséges értéke van. A skatulyaelv egyszerű alkalmazásával adódik az állítás. 1977. Bármely két halmaznak egy közös eleme van. Lássuk be, hogy ez a közös elem mindig ugyanaz. A1 -nek van olyan x eleme, melyben 45 másik halmaz metszi, ugyanis 1 + 44 · 45 = 1981 < 1985. Ha van olyan Ai halmaz, mely x-et nem tartalmazza, az előbbi 46 halmaz mindegyikét más-más elemben metszi. Ez azt jelentené, hogy |Ai | ≥ 46. Ezért a feladat kérdésére a válasz: 1985 · 44 + 1. 1980. Ha található a tagok között olyan, aki legalább 7 ülésen volt jelen, akkor az ezeken részt vevő további 7 · 9 ember a feltételek miatt szükségképpen mind különböző. Ez viszont legalább 7 · 9 + 1 = 64 tagot jelent. Ha pedig mindenki legfeljebb 6 ülésen vett részt, akkor az ülések összlétszáma csak úgy lehetett 400, 400 > 66 tagja volt a bizottságnak. ha legalább 6 1981. Az osztály minden tanulója szakköri tag, hiszen mindegyikük még közös szakkörbe is jár valaki mással. Ugyanezért, ha valaki csak egy szakkörbe jár, akkor ennek a szakkörnek mindenki tagja. Ezért feltehetjük, hogy mindenki legalább két szakkörbe jár, azaz — a második feltétel miatt — mindenki pontosan két szakkörbe jár. Tekintsünk valakit az osztályból, s legyen S1 és S2 az a két szakkör, amelynek ő tagja. Ha S1 -ben vagy S2 -ben ott van az egész osztály, akkor készen vagyunk. Különben van olyan g1 és g2 gyerek, hogy g1 ∈ S1 , g1 ∈ S2 és g2 ∈ S2 , g2 ∈ S1 . Legyen S az a szakkör, ahová g1 és g2 is jár. Ha egy g gyerek jár S1 -be is, S2 -be is, akkor más szakkörbe nem jár. Ha g ∈ S1 , g1 ∈ S2 , akkor g ∈ S, hiszen a g2 gyerekkel csak itt járhat közös szakkörbe. Ugyanígy; ha g ∈ S2 , g2 ∈ S1 , akkor g ∈ S, hiszen a g1 gyerekkel csak itt járhat közös szakkörbe. Tehát az S, S1 , S2 szakkörökben az osztály mindegyik diákja szerepel, s mindenki kétszer. Emiatt valamelyik szakkörbe az osztálynak legalább a kétharmada jár. 1982. Két esetet vizsgálunk. Ha bármely két tudós beszél közös nyelvet, akkor bármelyikük 8 másikkal beszél; és mivel legfeljebb 3 nyelven tud, ezért található a többiek között legalább kettő (sőt három is), akikkel azonos nyelven beszél. Ha van két tudós, A és B, akik nem beszélnek közös nyelven; akkor válasszuk harmadiknak sorba a többi hét tudóst. Ezek mindegyike a feltétel szerint vagy A-val, vagy B-vel beszél közös nyelvet. A és B együttvéve legfeljebb 2 · 3 = = 6 nyelven beszél, ezért van legalább két tudós, akik azonos nyelven beszélnek mondjuk A-val.

370

Megoldások, útmutatások

1983. A négy pénztáros közül bármely kettőt választjuk is ki, lesz olyan zár a páncélszekrényen, melyhez nekik nincs kulcsuk; de bármely más pénztárosnak már van (hiszen ketten nem tudják kinyitni, de bármely három pénztáros már igen). Ezért a zárak száma legalább annyi, ahányféleképp a négy pénztáros közül kettőt ki tudunk választani, azaz legalább 6. Ennyi kulccsal megoldható, hogy teljesüljenek a feltételek. Legyenek a pénztárosok A, B, C és D; a kulcsok sorszámai 1., 2., 1. 2. 3. 4. 5. 6. 3., 4., 5. és 6. legyenek. 6-nál több zár esetén is A + + + elvégezhető a kívánt módon a kulcsok megfelelő B + + + szétosztása: az előbbi módon osztjuk szét az első C + + + 6 zár kulcsait, a többi zárhoz pedig mindenkinek D + + + adjunk kulcsot. 1984. A bizottság bármelyik két tagja a többi három távollétében nem nyithatja ki a szekrényt; ezért bármelyik tag-párhoz található legalább egy olyan zár, amelyhez egyiküknek sincs kulcsa. Egy másik tag-párhoz nem tartozhat ugyanez a zár, amit ők sem tudnak kinyitni. Ezért legalább annyi zár kell, ahányféleképp a bizottságból két embert kiválaszthatunk, azaz legalább 10 zárra van szükség. Mindegyik bizottsági tagot legalább annyi kulccsal kell ellátni, ahány különböző tag-pár képezhető a többi négy tagból — vagyis legalább 6 kulccsal —, mert bármelyik tagnál kell kulcsnak lennie a többi négyből képezhető 6 párhoz tartozó 6 különböző nyithatatlan zár 1. 2. 3. 4. 5. 6. 7. 8. 9. 10. mindegyikéhez. A - - - - + + + + + + 10 zár elegendő a feltétel teljesítéB - + + + - - - + + + séhez. Legyenek a bizottság tagjai C + - + + - + + - - + A, B, C, D és E; a kulcsok sorszáD + + - + + - + - + mai 1., 2., 3., . . . , 10. legyenek. E + + + - + + - + -   10 1985. A zárak száma legalább = 120, s ennyi zár elegendő is. 3 1986. Először megmutatjuk, hogy bármely kijelölt R részhalmaznak pontosan m eleme van. Mivel R valódi részhalmaz, azért létezik benne nem lévő p elem. Minden q ∈ R elemnek megfeleltetve az a) szerint egyértelműen létező, p-t és q-t tartalmazó S kijelölt részhalmazt, kölcsönösen egyértelmű megfeleltetést kapunk R elemei és a p-t tartalmazó, R-t metsző kijelölt részhalmazok közt. Tehát minden kijelölt részhalmaz m elemet tartalmaz. A b) feltételből következik, hogy minden p elemet pontosan m+n kijelölt részhalmaz tartalmaz. Rögzítsük most H -nak egy r elemét. Az r-től különböző elemek mindegyike — a) miatt — pontosan egy, r-t tartalmazó részhalmazhoz tartozik hozzá. Az r-t m + n részhalmaz tartalmazza, és ezek mindegyikében m − 1 darab r-től különböző elem van; ezért H -nak összesen h = 1 + (m + n)(m − 1) eleme van.

34. Halmazrendszerek

371

A kijelölt részhalmazok száma legyen k. Számoljuk össze azokat a (p, R) párokat, ahol p a H -nak egy eleme, R egy kijelölt részhalmaz, és p ∈ R. Mivel h elemünk van, és mindegyik m + n részhalmazban van benne, ezért az ilyen párok száma h(m + n). A számolást viszont úgy is végezhetjük, hogy a k kijelölt részhalmaz mindegyike m elemet tartalmaz, ezért a párok száma k · m. A kétféle számolás eredményének meg kell egyeznie, ezért h(m + n) [1 + (m + n)(m − 1)](m + n) = . k · m = h(m + n), ahonnan k = m m Megjegyzés. Érdekes kérdés, hogy léteznek-e a feltételeknek eleget tevő H halmazok. A kérdésre pontos, teljes válasz nem ismert. Ha m = n = 2, akkor a megfelelő H halmaz egy 5-elemű halmaz; a kijelölt részhalmazok pedig ennek az összes 2-elemű részei. 1987. Legyen adva k darab halmaz. Tegyük fel, hogy már kiválasztottuk az A1 , A2 , . . . , An−1 halmazokat (1 ≤ n ≤ k). An -nek válasszunk egy minimálisat a maradékból, azaz olyat, amelynek a még fennmaradó halmazok egyike sem részhalmaza; ezt megtehetjük, hiszen összesen is csak véges sok halmazunk van. Az An kiválasztása után hagyjuk el a maradékból természetesen az An -et, valamint az A1 ∪ An , A2 ∪ An , . . . , An−1 ∪ An közül azokat, amelyek eredetileg ott voltak. Az így megmaradó halmazok közül válasszuk ki hasonlóan An+1 -et stb. Tetszőleges Ai kiválasztásával így összesen legfeljebb 1 + (i − 1) = i darab halmazt „fogyasztottunk el” a készletből, tehát az első n darab halmaz kiválasztása során n(n + 1) összesen legfeljebb 1+2+· · ·+n = darabot. Ez azt jelenti, hogy biztosan 2 n(n + 1) kiválasztható ezen a módon n darab halmaz, ha k > . 2 Az így kiválasztott halmazok eleget tesznek a feladat követelményének: tételezzük fel ugyanis, hogy létezne olyan Ai , Aj , Al , melyre Al = Ai ∪ Aj és például i < j . Mivel ekkor Ai és Aj is részhalmaza Al -nek, azért i, j < l, azaz Al -et az (Ai és) Aj megtalálása után választhattuk csak ki. Ez azonban lehetetlen, mivel Aj kiválasztásakor a maradékból kiselejteztük (Ai ∪ Aj )-t. 1989. Egy n-elemű halmaznak összesen 2n részhalmaza van, ezért a részhalmaz-párok száma 2n · 2n = 4n . A részhalmazpárokat rendezzük 4-es csoportokba: (Ai , Aj ), (Ai , Aj ), (Ai , Aj ), (Ai , Aj ). Ezeknek a csoportoknak a száma: 4n /4 = 4n−1 . Egy ilyen csoportban a metszetek elemszámának összege n. Emiatt a kérdéses összeg: n · 4n−1 . 1990. Álljon H azokból a K részhalmazokból, amelyekre K ⊇ f (K). Ez a H nem üres, mert A eleme, hiszen A ⊇ f (A) biztosan teljesül. Legyen a H -beli halmazok metszete az M halmaz. Mit tudunk az f (M)-ről? Ha K tetszőleges H -beli halmaz, akkor K ⊇ M miatt fönnáll f (K) ⊇ f (M). Ebből pedig K ⊇ f (K) alapján (ez volt a H -beli halmazok definiáló tulajdonsága) K ⊇ f (M). Tehát f (M)-et minden H -beli halmaz tartalmazza, így metszetük,

Megoldások, útmutatások

372

M is: M ⊇ f (M). Ugyanakkor M ⊇ f (M)-ből f (M) ⊇ f (f (M)) következik, tehát (definíció szerint) f (M) H -beli. Az M minden, H -beli halmaznak része, tehát M ⊆ f (M). Ezt az előbbi M ⊇ f (M) eredményünkkel összevetve M = = f (M); ami azt jelenti, hogy megtaláltuk a keresett részhalmazt. 1991. Legyen az n-elemű halmaz {1, 2, 3, . . . , n}. Tekintsük a halmaz egy partícióját, és minden részhalmazban az elemeket rendezzük nagyság szerint csökkenő sorrendbe, majd ezeket a sorozatokat első elemük szerint növekedve írjuk egymás után. Például, ha n = 6 és az egyik partíció {1, 5}, {2, 3}, {4, 6}, akkor ehhez a 325164 sorozat tartozik. Ily módon megadjuk az n elem egy permutációját, különböző partíciókhoz különböző permutációkat. Van olyan permutáció is, melyhez nem tartozik partíció, pl.: 326451. Megjegyzés. Az n-elemű halmaz partícióinak számát a Tn Bell-féle szám jelöli. 1992. Belátjuk, hogy f (n + 2) = f (n + 1) + f (n). Ehhez tekintsük az Mn+2 -nek azokat a kövér részhalmazait, melyek Mn+1 -nek nem kövér részhalmazai, azaz azokat, amelyek az n + 2 elemet tartalmazzák. Ezeknek a halmazoknak a száma f (n + + 2) − f (n + 1). Egy ilyen halmaznak az 1 nyilván nem lehet eleme, mivel akkor nem lenne kövér. Képezzük most egy ilyen T halmazhoz a T  halmazt úgy, hogy előbb minden elemből levonunk 1-et, majd ezután töröljük (n + 1)-et. Ekkor T  kövér részhalmaza Mn -nek, továbbá a T → T  leképezés megfordítható, így f (n + 2) − f (n + 1) = f (n). 1993. A Bs ⊃ Bs−1 ⊃ . . . ⊃ B2 ⊃ B1 sorozatot láncnak nevezzük. A H halmaz n hosszúságú láncainak száma n!. Ha |Ai | = ki , akkor Ai ki ! · (n − ki )! db n hosszúságú láncban szerepel, s ha i = j , akkor Ai és Aj nem lehet ugyanabban a láncban, tehát n! ≥  azaz m ≤

n

[ n2 ]

 .

m 

ki !(n − ki )! ≥ m ·

i=1

Megjegyzés. Az m =



n

[ n2 ]

n  n 

!· n− ! 2 2

 érték elérhető, ha vesszük az n elemű H halmaz

n! elemű részhalmazát. 2 1994. A H halmaz elemeit helyezzük el valamilyen sorrendben egy n oldalú szabályos sokszög csúcsaiba. Számoljuk meg, hogy egy ilyen permutációra hány db Ai jelenik meg úgy, hogy elemei egymást követik a sokszög csúcsaiban. k < n/2 és Ai ∩Aj = ∅, |Ai | = k miatt könnyű látni, hogy legfeljebb k darab. Így leszámlálva Ai -ket multiplicitásukkal együtt, ez a szám M ≤ k · n!. Ezt az M számot másképp megszámlálva egy Ai -t hányszor számolhatunk? Adott k egymást követő helyre Ai elemeit k! módon, a megmaradó n − k helyre az összes

34. Halmazrendszerek

373

elemeket (n − k)!-féleképpen írhatjuk. Ezt a k helyet n módon jelölhetjük ki. Ezért M = m · n · k!(n  −  k)!.   n−1 k n = . Eddigiekből m ≤ n k k−1 Látható, hogy az egyenlőtlenség éles: valamely x ∈ H elemet rögzítve a H \ x halmaz k−1 elemű részeihez hozzávéve az x elemet, egy megfelelő tulajdonságú   n−1 halmazrendszert kapunk; s a halmazrendszer halmazból áll. k−1 1995. Erősebb állítást bizonyítunk: [Bollobás, 1964] Ha A1 , A2 , . . . , Am és B1 , B2 , . . . , Bm olyan, hogy m  1 Ai ∩ Bi = ∅ és Ai ∩ Bj = ∅, ha i = j ; akkor |Ai |+|Bi | ≤ 1. i=1

|Ai |

Következmény. Ha A1 , A2 , . . . , Am és B1 , B2 , . . . , Bm olyan,  hogy  |Ai | ≤ a, a+b |Bi | ≤ b és a tétel metszetfeltételei is teljesülnek, akkor m ≤ . b m m Legyen X = (∪i=1 Ai ) ∪ (∪i=1 Bi ), |Ai | ≤ ai , |Bi | ≤ bi , |X| = n. Tekintsük X egy π permutációját. Azt mondjuk, hogy ez i-típusú, ha Ai minden eleme megelőzi Bi elemeit. Belátjuk, hogy π legfeljebb egy típusú lehet. Tegyük fel, hogy π iés j -típusú. Legyen xi , ill. xj a legnagyobb eleme π-ben Ai -nek, ill. Aj -nek. Ha xi ≤ xj , akkor Ai ∩ Bj = ∅, ami ellentmondás. Számoljuk  meg, hogy hány permutáció i-típusú. Ez a szám  n n! · (n − ai − bi )! · ai ! · bi ! = ai +bi  . Ezeket összegezve a kívánt állítást ai + bi ai kapjuk. 1996. Képzeljük el, hogy sikerült megadni 16 megfelelő H1 , H2 , . . . , H16 részhalmazt. Ekkor az első 10 000 természetes szám bármelyike ezek közül pontosan nyolcnak eleme. Valamennyi i ≤ 10 000 pozitív egészre jegyezzük fel az i számot tartalmazó halmazok sorszámát; megjegyzésünk szerint ezzel a 16-elemű I = {1, 2, . . . , 16} halmaznak egy nyolcelemű Ci részhalmazát kapjuk, minden i-re mást és mást. A feladat állításának igazolásához először ilyen Ci -ket állítunk elő úgy, hogy tetszés szerint kiválasztjuk I -nek 10 000 darab, különböző nyolcelemű  páronként  16 részhalmazát. (Ez megtehető, mert 10 000 < = 12 870.) Definiáljuk ezután 8  a Hj halmazt mindazoknak a k számoknak az összességeként, amelyekre Ck tartalmazza j -t. Mivel Ck -nek nyolc eleme van, ezért mindegyik k pontosan nyolc Hj -ben van benne. Ha pedig k ∗ = k, akkor Ck ∗ = Ck miatt k ∗ és k nem ugyanabban a nyolc Hj halmazban van, tehát a k-t tartalmazó nyolc halmaz egyetlen közös eleme a k. 1997. Legyen H = {x1 , x2 , . . . , xn }, s legyen Xi = {xi }, i = 1, 2, . . . , n. Tegyük fel, hogy A1 , A2 , . . . , Am független metszőrendszer. Megmutatjuk, hogy például X1

Megoldások, útmutatások

374

előállítható az A1 , A2 , . . . , Am halmazokból néhánynak a metszeteként. (Felhasználjuk közben a de Morgan-azonosságokat: A ∪ B = A ∩ B és A ∩ B = A ∪ B.)       X1 = H \ {X2 X3 ··· Xn } = X2 X3 ··· Xn = " " " " " " " " " ··· X3 Xn = ( Ai ) ( Ai ) ··· ( Ai ) = = X2 i∈I2  i∈I 3 " " "   " "i∈In " · · · ( Ai ) = (Ai2 A i3 A in ) = ( Ai ) ( Ai ) ··· i∈I2

i∈I3

i∈In

Mivel X1 egyetlen elemből álló halmazt jelentett, így az unió tagjainak valamelyike egyenlő X1 -gyel. Látható, hogy az A1 , A2 , . . . , Am és az A1 , A2 , . . . , Am rendszerek egyszerre rendelkeznek az (1) tulajdonsággal, és emiatt a (2) tulajdonsággal is. " 1998. Xk = Ai , |Xk | = 1, k = 1, 2, . . . , n. Az I1 , I2 , . . . , In ⊆ {1, 2, . . . , m} rendszer i∈Ik

Sperner-rendszer (egyik halmaz  sem  tartalmazza részként valamely másikat), így m a Sperner-lemma miatt n ≤ m ! , azaz m ≥ c1 · log2 n. Megmutatjuk, hogy az 2  m alsó korlát pontos; vagyis, ha m ! ≥ n, akkor van egy legfeljebb m halmazból 2

álló A1 , A2 , . . . , Am független  metszőrendszer az n elemű H halmazon. m Tekintsük az A m × m ! -es 0 − 1 mátrixot, melynek oszlopai az {1, 2, . . . , m} m 2 halmaz egy-egy -elemű részét reprezentálják. Az A mátrix soraival megadott 2   m ! elemű halmaz bármely elemét m db halmazból néhányat választva az m 2

kimetszhetjük. A felső korlát igazolása. Legyen x ∈ H , s tekintsük azokat az Ai -ket, melyek szükségesek az (1) és (2) szerinti előállításhoz. Ai -kből hagyjuk el az x elemet. Az így kapott halmazokat jelölje Ai . Ezeknek az Ai -knek a száma s. Most bármely s−1 db Ai metszete = ∅, míg az összes Ai metszete = ∅. Így minden Ai -hez hozzárendelhetünk egy olyan elemet a H \ {x} halmazból, mely a többi Aj -nek nem eleme. Ezért s ≤ n − 1. Így leszámolva az A1 , A2 , . . . , Am halmazokat, azt kapjuk, hogy m ≤ n(n − 1). Megadunk olyan független metszőrendszert  n  a H = {x1 , x2 , . . . , xn } halmazon, mely c2 · n2 halmazból áll. Legyen r = , R = {x1 , x2 , . . . , xr }, Ai = R \ {xi }, 2  i = 1, 2, . . . , r; Ai,j = {xi } Aj , i = r + 1, r + 2, . . . , n, j = 1, 2, . . . , r. Az Ai,j halmazok független metszőrendszert alkotnak, s a halmazok száma több, 1 mint n2 − 1. 4 1999. Legyen M egy maximális elemszámú halmaz, mely még egyik Ai -t sem tartalmazza. |M| = r.

34. Halmazrendszerek

375

A maximalitásból adódik, hogy minden x ∈ H \ M-hez van Ax ⊂ M, |Ax | = 2, hogy x ∪ Ax = Ai valamely i-re. Ha x = y, akkor Ax = Ay , különben a metszetfeltétel  nem teljesül. Így n − r ≤  

2 r 1 1 1 1 ≤ , r 2 + r ≥ 2n, r + ≥ 2n + , r + ≥ 2n + ; mivel r egész szám, 2 2 4 2 4 √ így r ≥ [ 2n]. 2000. Jelölje φx mindazoknak az Ai halmazoknak a számát, melyek az x elemet tartalmazzák. Nyilvánvaló, hogy n+1    φx = |Ai ∩ Aj | = n + |Ai ∩ Aj |, x∈Aj

i=j

i=1

innen 

φx ≤ n + nk = n(k + 1)

(1)

x∈Aj

Összegezzünk minden j -re. Ekkor a bal oldalon n+1      φx = φx 1= φx2 j =1 x∈Aj

x∈A

x∈Aj

x∈A

adódik. Ezt a számtani és négyzetes közép közötti egyenlőtlenséggel megbecsülve # 2 2  n+1  1  n2 (n + 1)2 x∈A φx φx2 ≥ |A| = |Aj | = . |A| |A| j =1 |A| x∈A Az (1) egyenlőtlenség jobb oldalából az összegezés után azt kapjuk, hogy n(n + 1)(k + 1). Tehát n2 (n + 1)2 ≤ n(n + 1)(k + 1). |A| Átrendezve: n(n + 1) |A| ≥ . k+1 Mivel az ellenkező irányú egyenlőtlenséget feltettük, itt az egyenlőségnek kell állnia. Ahhoz, hogy ilyen halmazrendszer létezzen, szükséges természetesen k + 1 | n(n + 1).

Tartalom

Bevezető . . . . . . . . . . . . . . . . . . . . . . . . . . . . . . . . . . . . . . . . . . . . . . . . . . . . . . . . . . . . . . . . FELADATOK . . . . . . . . . . . . . . . . . . . . . . . . . . . . . . . . . . . . . . . . . . . . . . . . . . . . . . . . . . 1. Fejtörők . . . . . . . . . . . . . . . . . . . . . . . . . . . . . . . . . . . . . . . . . . . . . . . . . . . . . . . . . . . . 2. Páros vagy páratlan? . . . . . . . . . . . . . . . . . . . . . . . . . . . . . . . . . . . . . . . . . . . . . . . . 3. Párbaállítás . . . . . . . . . . . . . . . . . . . . . . . . . . . . . . . . . . . . . . . . . . . . . . . . . . . . . . . . . 4. Miért nem négyzetszám? . . . . . . . . . . . . . . . . . . . . . . . . . . . . . . . . . . . . . . . . . . . . 5. Négyzetszámok . . . . . . . . . . . . . . . . . . . . . . . . . . . . . . . . . . . . . . . . . . . . . . . . . . . . . 6. Két szomszédos egész szám szorzata . . . . . . . . . . . . . . . . . . . . . . . . . . . . . . . . . 7. Diofantoszi egyenletek . . . . . . . . . . . . . . . . . . . . . . . . . . . . . . . . . . . . . . . . . . . . . . 8. Prímszámok . . . . . . . . . . . . . . . . . . . . . . . . . . . . . . . . . . . . . . . . . . . . . . . . . . . . . . . . 9. Oszthatósági feladatok . . . . . . . . . . . . . . . . . . . . . . . . . . . . . . . . . . . . . . . . . . . . . . . 10. Különféle számelméleti feladatok . . . . . . . . . . . . . . . . . . . . . . . . . . . . . . . . . . . . . 11. Számok reciprokainak összege . . . . . . . . . . . . . . . . . . . . . . . . . . . . . . . . . . . . . . . 12. Számok és számjegyek . . . . . . . . . . . . . . . . . . . . . . . . . . . . . . . . . . . . . . . . . . . . . . 13. Racionális és irracionális számok . . . . . . . . . . . . . . . . . . . . . . . . . . . . . . . . . . . . . 14. Egész együtthatós polinomok . . . . . . . . . . . . . . . . . . . . . . . . . . . . . . . . . . . . . . . . 15. Kombinatorika a számelméletben . . . . . . . . . . . . . . . . . . . . . . . . . . . . . . . . . . . . . 16. Számkonstrukciók . . . . . . . . . . . . . . . . . . . . . . . . . . . . . . . . . . . . . . . . . . . . . . . . . . . 17. Melyik szám a nagyobb? . . . . . . . . . . . . . . . . . . . . . . . . . . . . . . . . . . . . . . . . . . . . 18. Egyenletek és egyenletrendszerek . . . . . . . . . . . . . . . . . . . . . . . . . . . . . . . . . . . . 19. Egyenlőtlenségek . . . . . . . . . . . . . . . . . . . . . . . . . . . . . . . . . . . . . . . . . . . . . . . . . . . 20. Különféle algebrai feladatok . . . . . . . . . . . . . . . . . . . . . . . . . . . . . . . . . . . . . . . . . 21. Függvényegyenletek . . . . . . . . . . . . . . . . . . . . . . . . . . . . . . . . . . . . . . . . . . . . . . . . . 22. Vektorok a geometriában . . . . . . . . . . . . . . . . . . . . . . . . . . . . . . . . . . . . . . . . . . . . 23. Területátalakítások . . . . . . . . . . . . . . . . . . . . . . . . . . . . . . . . . . . . . . . . . . . . . . . . . . 24. Geometriai konstrukciók . . . . . . . . . . . . . . . . . . . . . . . . . . . . . . . . . . . . . . . . . . . . . 25. Invariáns tulajdonságok . . . . . . . . . . . . . . . . . . . . . . . . . . . . . . . . . . . . . . . . . . . . . . 26. Feladatok a sakktáblán . . . . . . . . . . . . . . . . . . . . . . . . . . . . . . . . . . . . . . . . . . . . . . 27. Skatulyaelv . . . . . . . . . . . . . . . . . . . . . . . . . . . . . . . . . . . . . . . . . . . . . . . . . . . . . . . . . 28. Matematikai játékok . . . . . . . . . . . . . . . . . . . . . . . . . . . . . . . . . . . . . . . . . . . . . . . . . 29. Különféle kombinatorikai feladatok . . . . . . . . . . . . . . . . . . . . . . . . . . . . . . . . . . . 30. Konstrukciók . . . . . . . . . . . . . . . . . . . . . . . . . . . . . . . . . . . . . . . . . . . . . . . . . . . . . . . 31. Teljes indukció . . . . . . . . . . . . . . . . . . . . . . . . . . . . . . . . . . . . . . . . . . . . . . . . . . . . . 32. Kombinatorika a geometriában . . . . . . . . . . . . . . . . . . . . . . . . . . . . . . . . . . . . . . .

5 7 7 19 20 23 25 28 29 33 36 43 50 52 54 57 58 60 66 68 72 81 90 95 101 114 118 126 132 141 149 157 159 165

378

Tartalom

33. Gráfok . . . . . . . . . . . . . . . . . . . . . . . . . . . . . . . . . . . . . . . . . . . . . . . . . . . . . . . . . . . . . 170 34. Halmazrendszerek . . . . . . . . . . . . . . . . . . . . . . . . . . . . . . . . . . . . . . . . . . . . . . . . . . . 172 MEGOLDÁSOK . . . . . . . . . . . . . . . . . . . . . . . . . . . . . . . . . . . . . . . . . . . . . . . . . . . . . . . . 1. Fejtörők . . . . . . . . . . . . . . . . . . . . . . . . . . . . . . . . . . . . . . . . . . . . . . . . . . . . . . . . . . . . 2. Páros vagy páratlan? . . . . . . . . . . . . . . . . . . . . . . . . . . . . . . . . . . . . . . . . . . . . . . . . 3. Párbaállítás . . . . . . . . . . . . . . . . . . . . . . . . . . . . . . . . . . . . . . . . . . . . . . . . . . . . . . . . . 4. Miért nem négyzetszám? . . . . . . . . . . . . . . . . . . . . . . . . . . . . . . . . . . . . . . . . . . . . 5. Négyzetszámok . . . . . . . . . . . . . . . . . . . . . . . . . . . . . . . . . . . . . . . . . . . . . . . . . . . . . 6. Két szomszédos egész szám szorzata . . . . . . . . . . . . . . . . . . . . . . . . . . . . . . . . . 7. Diofantoszi egyenletek . . . . . . . . . . . . . . . . . . . . . . . . . . . . . . . . . . . . . . . . . . . . . . 8. Prímszámok . . . . . . . . . . . . . . . . . . . . . . . . . . . . . . . . . . . . . . . . . . . . . . . . . . . . . . . . 9. Oszthatósági feladatok . . . . . . . . . . . . . . . . . . . . . . . . . . . . . . . . . . . . . . . . . . . . . . . 10. Különféle számelméleti feladatok . . . . . . . . . . . . . . . . . . . . . . . . . . . . . . . . . . . . . 11. Számok reciprokainak összege . . . . . . . . . . . . . . . . . . . . . . . . . . . . . . . . . . . . . . . 12. Számok és számjegyek . . . . . . . . . . . . . . . . . . . . . . . . . . . . . . . . . . . . . . . . . . . . . . 13. Racionális és irracionális számok . . . . . . . . . . . . . . . . . . . . . . . . . . . . . . . . . . . . . 14. Egész együtthatós polinomok . . . . . . . . . . . . . . . . . . . . . . . . . . . . . . . . . . . . . . . . 15. Kombinatorika a számelméletben . . . . . . . . . . . . . . . . . . . . . . . . . . . . . . . . . . . . . 16. Számkonstrukciók . . . . . . . . . . . . . . . . . . . . . . . . . . . . . . . . . . . . . . . . . . . . . . . . . . . 17. Melyik szám a nagyobb? . . . . . . . . . . . . . . . . . . . . . . . . . . . . . . . . . . . . . . . . . . . . 18. Egyenletek és egyenletrendszerek . . . . . . . . . . . . . . . . . . . . . . . . . . . . . . . . . . . . 19. Egyenlőtlenségek . . . . . . . . . . . . . . . . . . . . . . . . . . . . . . . . . . . . . . . . . . . . . . . . . . . 20. Különféle algebrai feladatok . . . . . . . . . . . . . . . . . . . . . . . . . . . . . . . . . . . . . . . . . 21. Függvényegyenletek . . . . . . . . . . . . . . . . . . . . . . . . . . . . . . . . . . . . . . . . . . . . . . . . . 22. Vektorok a geometriában . . . . . . . . . . . . . . . . . . . . . . . . . . . . . . . . . . . . . . . . . . . . 23. Területátalakítások . . . . . . . . . . . . . . . . . . . . . . . . . . . . . . . . . . . . . . . . . . . . . . . . . . 24. Geometriai konstrukciók . . . . . . . . . . . . . . . . . . . . . . . . . . . . . . . . . . . . . . . . . . . . . 25. Invariáns tulajdonságok . . . . . . . . . . . . . . . . . . . . . . . . . . . . . . . . . . . . . . . . . . . . . . 26. Feladatok a sakktáblán . . . . . . . . . . . . . . . . . . . . . . . . . . . . . . . . . . . . . . . . . . . . . . 27. Skatulyaelv . . . . . . . . . . . . . . . . . . . . . . . . . . . . . . . . . . . . . . . . . . . . . . . . . . . . . . . . . 28. Matematikai játékok . . . . . . . . . . . . . . . . . . . . . . . . . . . . . . . . . . . . . . . . . . . . . . . . . 29. Különféle kombinatorikai feladatok . . . . . . . . . . . . . . . . . . . . . . . . . . . . . . . . . . . 30. Konstrukciók . . . . . . . . . . . . . . . . . . . . . . . . . . . . . . . . . . . . . . . . . . . . . . . . . . . . . . . 31. Teljes indukció . . . . . . . . . . . . . . . . . . . . . . . . . . . . . . . . . . . . . . . . . . . . . . . . . . . . . 32. Kombinatorika a geometriában . . . . . . . . . . . . . . . . . . . . . . . . . . . . . . . . . . . . . . . 33. Gráfok . . . . . . . . . . . . . . . . . . . . . . . . . . . . . . . . . . . . . . . . . . . . . . . . . . . . . . . . . . . . . 34. Halmazrendszerek . . . . . . . . . . . . . . . . . . . . . . . . . . . . . . . . . . . . . . . . . . . . . . . . . . .

177 177 186 187 191 193 197 198 203 208 215 223 225 227 231 233 237 243 244 250 260 269 278 282 285 293 300 313 325 334 345 348 352 361 366